Back to Filters (/Secure/TestMe/Filter ... - 1 File Download

773
8/12/2016 MyPastest https://mypastest.pastest.com/Secure/TestMe/Browser/429893 1/2 Back to Filters (/Secure/TestMe/Filter/429893/QA) Question 1 of 179 A 24-year-old man is admitted from a local night club after suffering from an epileptic seizure. You understand from other people with him that he was acutely anxious and suffering from paranoid thoughts, and the nurse finds a small amount of what looks like cocaine in his jacket. On examination he is very anxious, and is reluctant even to let you take blood. His temperature is 38.0°C, blood pressure 155/90 mmHg, pulse 120 bpm, regular. He develops generalised seizures that fail to respond to three successive intravenous doses of diazepam 5 mg. His ECG shows 2 mm of ST depression in the anterolateral leads. Which one of the following is the most appropriate next intervention? Explanation The answer is Intubation and ventilation – This man has suffered a further series of seizures, coupled with a rise in blood pressure and heart rate. He is at high risk of myocardial ischaemic damage and hyperthermia, which may lead to rhabdomyolysis and worsening renal failure. Cocaine overdose may cause haemorrhagic stroke. Airway intubation and ventilation may lower his blood pressure and improve ischaemia; subsequent steps would involve measures to control blood pressure and seizure activity. IV esmolol (Option B) is incorrect. Esmolol, a short acting β-blocker may assist in controlling heart rate, but would be of limited value in controlling blood pressure or seizure risk. IV phenytoin (Option C) is incorrect. IV phenytoin may help control seizures, but would not address the underlying agitation, pyrexia and high blood pressure that may be provoking seizures. Intubation and ventilation A IV esmolol B IV phenytoin C IV sodium valproate D IV verapamil E

Transcript of Back to Filters (/Secure/TestMe/Filter ... - 1 File Download

Page 1: Back to Filters (/Secure/TestMe/Filter ... - 1 File Download

8/12/2016 MyPastest

https://mypastest.pastest.com/Secure/TestMe/Browser/429893 1/2

Back to Filters (/Secure/TestMe/Filter/429893/QA)

Question 1 of 179

A 24-year-old man is admitted from a local night club after suffering from an epilepticseizure. You understand from other people with him that he was acutely anxious andsuffering from paranoid thoughts, and the nurse finds a small amount of what looks likecocaine in his jacket. On examination he is very anxious, and is reluctant even to let you takeblood. His temperature is 38.0°C, blood pressure 155/90 mmHg, pulse 120 bpm, regular. Hedevelops generalised seizures that fail to respond to three successive intravenous doses ofdiazepam 5 mg. His ECG shows 2 mm of ST depression in the anterolateral leads.

Which one of the following is the most appropriate next intervention?

Explanation

The answer is Intubation and ventilation –

This man has suffered a further series of seizures, coupled with a rise in blood pressureand heart rate. He is at high risk of myocardial ischaemic damage and hyperthermia,which may lead to rhabdomyolysis and worsening renal failure. Cocaine overdose maycause haemorrhagic stroke. Airway intubation and ventilation may lower his bloodpressure and improve ischaemia; subsequent steps would involve measures to controlblood pressure and seizure activity.

IV esmolol (Option B) is incorrect. Esmolol, a short acting β-blocker may assist in controllingheart rate, but would be of limited value in controlling blood pressure or seizure risk.

IV phenytoin (Option C) is incorrect. IV phenytoin may help control seizures, but would notaddress the underlying agitation, pyrexia and high blood pressure that may be provokingseizures.

Intubation and ventilationA

IV esmololB

IV phenytoinC

IV sodium valproateD

IV verapamilE

Page 2: Back to Filters (/Secure/TestMe/Filter ... - 1 File Download

8/12/2016 MyPastest

https://mypastest.pastest.com/Secure/TestMe/Browser/429893 2/2

46981

IV sodium valproate (Option D) is incorrect. IV sodium valproate may help control seizures,but would not address the underlying agitation, pyrexia and high blood pressure that may beprovoking seizures.

IV verapamil (Option E) is incorrect. Verapamil, a calcium channel blocker, may assist incontrolling heart rate, but would be of limited value in controlling blood pressure or seizurerisk.

Next Question

Previous Question Tag Question Feedback End Review

Difficulty: Average

Peer Responses

Session Progress

0Responses Correct:

179Responses Incorrect:

179Responses Total:

0%Responses - % Correct:

Blog (https://www.pastest.com/blog) About Pastest (https://www.pastest.com/about-us)Contact Us (https://www.pastest.com/contact-us) Help (https://www.pastest.com/help)

© Pastest 2016

Page 3: Back to Filters (/Secure/TestMe/Filter ... - 1 File Download

8/12/2016 MyPastest

https://mypastest.pastest.com/Secure/TestMe/Browser/429893 1/2

Back to Filters (/Secure/TestMe/Filter/429893/QA)

Question 2 of 179

A 16-year-old woman attends the Emergency Department and reports taking around 30 g ofparacetamol and 2 g of dihydrocodeine about 10 h earlier. On examination, she is drowsy witha Glasgow Coma Scale of 14. Heart rate is 100 bpm, blood pressure is 110/66 mmHg, there arepinpoint pupils, and oxygen saturations are 96% on air.

Which one of the following treatments would be most strongly indicated?

Explanation

The answer is N-acetylcysteine intravenously –

N-acetylcysteine is the treatment of choice for paracetamol overdose. At more than 8 hafter ingestion of a potentially toxic quantity, acetylcysteine should be commenced whilethe drug concentrations and blood tests are awaited. Treatment should not be withheldpending the blood test results.

10% dextrose infusion (Option A) is incorrect. There is no indication that the patient issuffering from hypoglycaemia.

Activated charcoal by mouth (Option B) is incorrect. Oral activated charcoal should normallybe given within 1 h of paracetamol ingestion, although may be effective up to 2 h after inpatients that coingest opioids. At 10 h, it is too late to be effective, and in a drowsy patient,there may be a risk of charcoal aspiration into the lungs.

Gastric lavage (Option C) is incorrect. Gastric lavage may be indicated for life-threateningingestions within the previous 1 h, but is rarely indicated, with the exception of certainmedications e.g. lithium.

10% dextrose infusionA

Activated charcoal by mouthB

Gastric lavageC

N-Acetylcysteine intravenouslyD

Naloxone intravenouslyE

Page 4: Back to Filters (/Secure/TestMe/Filter ... - 1 File Download

8/12/2016 MyPastest

https://mypastest.pastest.com/Secure/TestMe/Browser/429893 2/2

46921

Naloxone intravenously (Option E) is incorrect. There are clues in the clinical history for thispatient to suggest that she is not significantly compromised in terms of level ofconsciousness or respiratory function; hence naloxone is not necessary at present.

Next Question

Previous Question Tag Question Feedback End Review

Difficulty: Average

Peer Responses

Session Progress

0Responses Correct:

179Responses Incorrect:

179Responses Total:

0%Responses - % Correct:

Blog (https://www.pastest.com/blog) About Pastest (https://www.pastest.com/about-us)Contact Us (https://www.pastest.com/contact-us) Help (https://www.pastest.com/help)

© Pastest 2016

Page 5: Back to Filters (/Secure/TestMe/Filter ... - 1 File Download

8/12/2016 MyPastest

https://mypastest.pastest.com/Secure/TestMe/Browser/429893 1/2

Back to Filters (/Secure/TestMe/Filter/429893/QA)

Question 3 of 179

A 69-year-old man with chronic atrial fibrillation, controlled on digoxin, is found to have ablood pressure of 178/102 mmHg. He is commenced on modified-release verapamil 240 mgdaily. One week later, he is taken to the hospital Emergency Department having been foundcollapsed. He is tachycardic at 130 bpm, hypotensive with a blood pressure 90/62 mmHg, andan electrocardiogram (ECG) shows him to be in ventricular tachycardia.

What is the most likely explanation for these findings?

Explanation

The answer is He has developed digoxin toxicity –

Digoxin toxicity can result from co-administration of verapamil, nifedipine, quinine orquinidine, all of which inhibit tubular secretion of digoxin, so that the dose of digoxinshould normally be decreased. Digoxin toxicity may cause any one of a number ofdifferent arrhythmias, and may be precipitated by hypokalaemia or hypomagnesaemia.

He has had a myocardial infarction (Option B) is incorrect. Although myocardial infarctionmight explain the haemodynamic profile, this explanation fails to recognise the significanceof the drug interaction.

He has Wolff–Parkinson–White syndrome (Option C) is incorrect. The clinical manifestion ofWolff–Parkinson–White syndrome would perhaps be less rather than more likely to occurafter initiation of verapamil.

The hypotensive effect of verapamil has induced a reflex ventricular tachycardia (Option D) isincorrect. Verapamil would primarily act by reducing resting heart rate, rather than causinghypotension through a direct effect on blood vessel tone.

He has developed digoxin toxicityA

He has had a myocardial infarctionB

He has Wolff–Parkinson–White syndromeC

The hypotensive effect of verapamil has induced a reflex ventricular tachycardiaD

Verapamil has exerted a pro-arrhythmic effectE

Page 6: Back to Filters (/Secure/TestMe/Filter ... - 1 File Download

8/12/2016 MyPastest

https://mypastest.pastest.com/Secure/TestMe/Browser/429893 2/2

46915

Verapamil has exerted a pro-arrhythmic effect (Option E) is incorrect. Verapamil may causebradyarrhythmia and heart block, and would not be expected to cause tachycardia.

Next Question

Previous Question Tag Question Feedback End Review

Difficulty: Average

Peer Responses

Session Progress

0Responses Correct:

179Responses Incorrect:

179Responses Total:

0%Responses - % Correct:

Blog (https://www.pastest.com/blog) About Pastest (https://www.pastest.com/about-us)Contact Us (https://www.pastest.com/contact-us) Help (https://www.pastest.com/help)

© Pastest 2016

Page 7: Back to Filters (/Secure/TestMe/Filter ... - 1 File Download

8/12/2016 MyPastest

https://mypastest.pastest.com/Secure/TestMe/Browser/429893 1/2

Back to Filters (/Secure/TestMe/Filter/429893/QA)

Question 4 of 179

46973

A 78-year-old recently widowed lady is admitted by ambulance to the EmergencyDepartment. She was found unconscious by a neighbour, surrounded by three empty bottlesof propranolol tablets. On examination her pulse was noted to be 38 bpm and she has ablood pressure of 78/50 mmHg. She is given activated charcoal, intravenous fluid loading andatropine, but fails to respond.

Which one of the following is the most appropriate next step in her management?

Explanation

The answer is IV glucagon –

This woman is suffering the effects of an intentional overdose of ß-blockers, probablyrelated to depression at her recent bereavement. Glucagon is a specific antidote forexcessive ß-blockade and is indicated in this situation.

Insertion of temporary pacing wire (Option A) is incorrect. Temporary pacing might beconsidered in patients unresponsive to glucagon therapy, but it is associated with iatrogeniccomplications and normally considered after medical therapies.

IV adrenaline (Option B) is incorrect. IV adrenaline in this situation may precipitatemyocardial ischaemia or actually be proarrhythmogenic.

IV calcium chloride (Option C) is incorrect. IV calcium chloride is useful in calcium channelblocker overdose, but less so after beta-blocker overdose.

IV phenytoin (Option E) is incorrect. Phenytoin would not be expected to offer anysignificant benefit in this situation.

Insertion of temporary pacing wireA

IV adrenalineB

IV calcium chlorideC

IV glucagonD

IV phenytoinE

Page 8: Back to Filters (/Secure/TestMe/Filter ... - 1 File Download

8/12/2016 MyPastest

https://mypastest.pastest.com/Secure/TestMe/Browser/429893 2/2

46973

Next Question

Previous Question Tag Question Feedback End Review

Difficulty: Average

Peer Responses

Session Progress

0Responses Correct:

179Responses Incorrect:

179Responses Total:

0%Responses - % Correct:

Blog (https://www.pastest.com/blog) About Pastest (https://www.pastest.com/about-us)Contact Us (https://www.pastest.com/contact-us) Help (https://www.pastest.com/help)

© Pastest 2016

Page 9: Back to Filters (/Secure/TestMe/Filter ... - 1 File Download

8/12/2016 MyPastest

https://mypastest.pastest.com/Secure/TestMe/Browser/429893 1/2

Back to Filters (/Secure/TestMe/Filter/429893/QA)

Question 5 of 179

A 29-year-old woman with a history of ulcerative colitis is reviewed on the ward 48hrs aftercommencement of IV corticosteroid therapy. She was admitted after failing to respond tohigh dose oral corticosteroids and maximal dose Mesalazine, and continues to pass bloodstained diarrhoeal stool up to 12 times per day. On examination she is pyrexial 37.9°C, her BPis 105/72 mmHg, pulse is 92/min and regular. Her abdomen is generally tender and mildlydistended, with more marked tenderness on the left hand side. Bowel sounds are active.

Investigations;

Hb 9.9 g/dl

WCC 13.1 x10 /l

PLT 191 x10 /l

Na 137 mmol/l

K 3.4 mmol/l

Creatinine 110 micromol/l

CRP 195 mg/l

Albumin 28 g/l

Abdominal X-ray distended colon 5.2cm in diameter

Which of the following is the most appropriate next step?

Explanation

9

9

+

+

Add AzathioprineA

Add InfliximabB

Add rectal corticosteroidsC

Continue IV steroids and MesalazineD

Proceed to colectomyE

Page 10: Back to Filters (/Secure/TestMe/Filter ... - 1 File Download

8/12/2016 MyPastest

https://mypastest.pastest.com/Secure/TestMe/Browser/429893 2/2

40150

The answer is Add Infliximab -

In this situation the choices are proceed to colectomy or move to a trial of Infliximab orCiclosporin. Infliximab can be used to induce rapid remission and for maintenance therapy,although it is unclear whether its use actually reduces the risk of colectomy over the longerterm. Ciclosporin is used to act as a bridge to other treatments, which take time to buildtheir effectiveness such as Azathioprine or 6-mercaptopurine. If there is no response toInfliximab within 72hrs, colectomy is the most appropriate next intervention.

Next Question

Previous Question Tag Question Feedback End Review

Difficulty: Difficult

Peer Responses

Session Progress

0Responses Correct:

179Responses Incorrect:

179Responses Total:

0%Responses - % Correct:

Blog (https://www.pastest.com/blog) About Pastest (https://www.pastest.com/about-us)Contact Us (https://www.pastest.com/contact-us) Help (https://www.pastest.com/help)

© Pastest 2016

Page 11: Back to Filters (/Secure/TestMe/Filter ... - 1 File Download

8/12/2016 MyPastest

https://mypastest.pastest.com/Secure/TestMe/Browser/429893 1/2

Back to Filters (/Secure/TestMe/Filter/429893/QA)

Question 6 of 179

Genetic variations may have a significant impact on the clinical response to various drugs.One example is genetic variation of cytochrome P450 2D6 (CYP-2D6) isoenzyme activitysuch that patients may be considered active or poor metabolisers.

Which of the following drugs is most likely to cause adverse effects in patients with poorCYP-2D6 metabolism?

Explanation

The answer is Fluoxetine –

There are numerous variations in the cytochrome P450 isoenzyme system leading todifferences in how patients respond to certain drugs. Patients who are slow metaboliserswill have increased concentrations of drugs and have a higher rate of occurrence ofadverse effects; rapid metabolisers may have lower drug concentrations and lesstherapeutic response. Drugs that are metabolised via CYP-2D6 include tricyclicantidepressants, β-blockers (especially metoprolol), dihydrocodeine, ecstasy (MDMA)and certain selective serotonin reuptake inhibitors. Dihydrocodeine is metabolised bycytochrome CYP-2D6 to morphine, the active analgesic. Patients who are CYP-2D6 poormetabolisers will have less analgesic response to dihydrocodeine. Other P450isoenzymes are CYP-2C8 (omeprazole, diazepam, barbiturates) and CYP-2C18/19(omeprazole, diazepam, tricyclic antidepressants, proguanil).

Dapsone (Option A) is incorrect. Dapsone is subject to metabolism by CYP-2C8 and CYP-2C9.

DapsoneA

DiazepamB

FluoxetineC

OmeprazoleD

PhenylzineE

Page 12: Back to Filters (/Secure/TestMe/Filter ... - 1 File Download

8/12/2016 MyPastest

https://mypastest.pastest.com/Secure/TestMe/Browser/429893 2/2

46385

Diazepam (Option B) is incorrect. Diazepam is subject to metabolism by CYP-2C8 and CYP-3A4.

Omeprazole (Option D) is incorrect. Omeprazole is subject to metabolism by CYP-2C8, CYP-2C18 and others, but not CYP-2D6.

Phenylzine (Option E) is incorrect. Phenylzine metabolism is not subject to genetic variation.

Next Question

Previous Question Tag Question Feedback End Review

Difficulty: Average

Peer Responses

Session Progress

0Responses Correct:

179Responses Incorrect:

179Responses Total:

0%Responses - % Correct:

Blog (https://www.pastest.com/blog) About Pastest (https://www.pastest.com/about-us)Contact Us (https://www.pastest.com/contact-us) Help (https://www.pastest.com/help)

© Pastest 2016

Page 13: Back to Filters (/Secure/TestMe/Filter ... - 1 File Download

8/12/2016 MyPastest

https://mypastest.pastest.com/Secure/TestMe/Browser/429893 1/2

Back to Filters (/Secure/TestMe/Filter/429893/QA)

Question 7 of 179

46855

You are asked by the ophthalmology registrar to prescribe a course of vitamin A for a youngman who has been suffering from night blindness for the past 3–4 months.

Which of the following substances in vitamin A is most likely to be maximally involved incorrecting the visual disturbance?

Explanation

The answer is Retinaldehyde –

Vitamin A is often used as a collective term for several related biologically activemolecules. Retinaldehyde is derived from the oxidation of retinol, and the cis form isfound in the opsin proteins in the rods (rhodopsin) of the retina. Exposure to light causesretinaldehyde to change to its trans isomer, and the resulting changes in membranepotentials give rise to signals transmitted to the brain.

Beta-carotene (Option A) is incorrect. Beta-carotene is the main carotenoid found in greenvegetables, carrots and other yellow and red fruits. It is partially converted to retinol inhumans, but the process is inefficient: 6 μg of β-carotene give rise to 1 μg of retinol.

Retinoic acid (Option C) is incorrect. Retinol and retinoic acid are involved in the control ofcell proliferation and differentiation, and not directly related to retinal function.

Retinol (Option D) is incorrect. Retinol and retinoic acid are involved in the control of cellproliferation and differentiation, and not directly related to retinal function.

Retinyl phosphate (Option E) is incorrect. Retinyl phosphate is a cofactor in the synthesis ofglycoproteins that contain mannose.

Beta-caroteneA

RetinaldehydeB

Retinoic acidC

RetinolD

Retinyl phosphateE

Page 14: Back to Filters (/Secure/TestMe/Filter ... - 1 File Download

8/12/2016 MyPastest

https://mypastest.pastest.com/Secure/TestMe/Browser/429893 2/2

46855

Next Question

Previous Question Tag Question Feedback End Review

Difficulty: Average

Peer Responses

Session Progress

0Responses Correct:

179Responses Incorrect:

179Responses Total:

0%Responses - % Correct:

Blog (https://www.pastest.com/blog) About Pastest (https://www.pastest.com/about-us)Contact Us (https://www.pastest.com/contact-us) Help (https://www.pastest.com/help)

© Pastest 2016

Page 15: Back to Filters (/Secure/TestMe/Filter ... - 1 File Download

8/12/2016 MyPastest

https://mypastest.pastest.com/Secure/TestMe/Browser/429893 1/2

Back to Filters (/Secure/TestMe/Filter/429893/QA)

Question 8 of 179

46815

You are reviewing a 23-year-old woman on the Acute Medical Unit who has presented tohospital after a mixed overdose. She had been given a single dose of oral activated charcoalin the Emergency Department.

Which of the following circumstances would most strongly indicate a need for repeated oralactivated charcoal administration?

Explanation

The answer is When the drug circulates through the enterohepatic circulation –

Multiple doses of activated charcoal aid the elimination of some drugs that are prone toenterohepatic circulation; charcoal adsorbs drug excreted in bile and preventsreabsorption in the small bowel. Typical examples include aspirin, quinine,carbamazepine and theophylline.

If a bleeding disorder develops (Option A) is incorrect. Charcoal administration is unrelatedto gastrointestinal bleeding.

If it is given within 12 h of ingestion of the poison (Option B) is incorrect. Multiple doses oforal activated charcoal may be effective in enhancing drug clearance even up to several daysafter ingestion.

In cases of heavy-metal poisoning (Option C) is incorrect. Charcoal does not adsorb metalsincluding heavy metals, iron or lithium.

In cases where gastric lavage is contraindicated (Option D) is incorrect. Gastric lavage isirrelevant to multiple dose oral activated charcoal.

If a bleeding disorder developsA

If it is given within 12 h of ingestion of the poisonB

In cases of heavy-metal poisoningC

In cases where gastric lavage is contraindicatedD

When the drug circulates through the enterohepatic circulationE

Page 16: Back to Filters (/Secure/TestMe/Filter ... - 1 File Download

8/12/2016 MyPastest

https://mypastest.pastest.com/Secure/TestMe/Browser/429893 2/2

46815

Next Question

Previous Question Tag Question Feedback End Review

Difficulty: Average

Peer Responses

Session Progress

0Responses Correct:

179Responses Incorrect:

179Responses Total:

0%Responses - % Correct:

Blog (https://www.pastest.com/blog) About Pastest (https://www.pastest.com/about-us)Contact Us (https://www.pastest.com/contact-us) Help (https://www.pastest.com/help)

© Pastest 2016

Page 17: Back to Filters (/Secure/TestMe/Filter ... - 1 File Download

8/12/2016 MyPastest

https://mypastest.pastest.com/Secure/TestMe/Browser/429893 1/2

Back to Filters (/Secure/TestMe/Filter/429893/QA)

Question 9 of 179

You are reviewing a 36-year-old man with hypertension and decide to commence β-blockertherapy; you are contemplating the various agents available.

Which of the following β-blockers has the largest volume of distribution?

Explanation

The answer is Metoprolol –

The volume of distribution is a theoretical volume that expresses the extent to which adrug moves out of the circulating compartment to other tissues. The volume ofdistribution has units of volume (litres). Metoprolol is the most lipid soluble of the β-blockers listed above and therefore has the largest volume of distribution; the clinicalsignificance is that the increased lipid solubility is associated with greater penetrationacross the blood–brain barrier (and also into other tissues), and therefore a greaterincidence of nightmares, headache and other central nervous system adverse effects.Maximal gastrointestinal absorption of drugs occurs when there is intermediate lipid andwater solubility, so that drugs with greater lipid solubility, although allowing greatertissue penetration, may be more poorly absorbed.

Atenolol (Option A) is incorrect. Atenolol has minimal lipid solubility, and comparativelymodest volume of distribution.

Celiprolol (Option B) is incorrect. Celiprolol has minimal lipid solubility, and possessesintrinsic sympathomimetic activity (partial agonist activity) that minimises the occurrence ofresting bradycardia.

AtenololA

CeliprololB

MetoprololC

NadololD

SotalolE

Page 18: Back to Filters (/Secure/TestMe/Filter ... - 1 File Download

8/12/2016 MyPastest

https://mypastest.pastest.com/Secure/TestMe/Browser/429893 2/2

46845

Nadolol (Option D) is incorrect. Nadolol has limited lipid solubility, and comparatively modestvolume of distribution.

Sotalol (Option E) is incorrect. Sotalol has a volume of distribution that approximates theextracellular fluid compartment.

Next Question

Previous Question Tag Question Feedback End Review

Difficulty: Average

Peer Responses

Session Progress

0Responses Correct:

179Responses Incorrect:

179Responses Total:

0%Responses - % Correct:

Blog (https://www.pastest.com/blog) About Pastest (https://www.pastest.com/about-us)Contact Us (https://www.pastest.com/contact-us) Help (https://www.pastest.com/help)

© Pastest 2016

Page 19: Back to Filters (/Secure/TestMe/Filter ... - 1 File Download

8/12/2016 MyPastest

https://mypastest.pastest.com/Secure/TestMe/Browser/429893 1/2

Back to Filters (/Secure/TestMe/Filter/429893/QA)

Question 10 of 179

You are helping to draw up some prescribing guidelines in the Elderly Medicine departmentof your hospital.

Which of the following medications would require greatest caution when prescribed for olderadults?

Explanation

The answer is Triamterene –

The most common drugs associated with adverse effects in older adults are diuretics,digoxin, antihypertensives, anti-inflammatory agents and drugs acting on the centralnervous system (CNS). Age-related changes include declining renal function andlessened capacity for hepatic drug metabolism. Triamterine is a diuretic that blocks theepithelial sodium channels in the distal convoluted tubule. Older adults are moresusceptible to the volume depletion and hyponatraemic effects of diuretics.

Amoxicillin (Option A) is incorrect. Amoxicillin may be used safely in older adults; it is subjectto renal elimination, but has a very wide therapeutic window, such that any increasedconcentrations are unlikely to cause adverse effects.

Low-dose (75 mg daily) aspirin (Option B) is incorrect. Aspirin may be slightly more likely toprovoke gastric irritation in older adults than younger adults and should be used cautiously.However, the impact of age on drug response is much less obvious than for diuretics, whichis why triamterene is the preferred answer.

Omeprazole (Option C) is incorrect. Omeprazole may be used safely in older adults.

AmoxicillinA

Low-dose (75 mg daily) aspirinB

OmeprazoleC

RanitidineD

TriamtereneE

Page 20: Back to Filters (/Secure/TestMe/Filter ... - 1 File Download

8/12/2016 MyPastest

https://mypastest.pastest.com/Secure/TestMe/Browser/429893 2/2

46911

Ranitidine (Option D) is incorrect. Ranitidine may be used safely in older adults.

Next Question

Previous Question Tag Question Feedback End Review

Difficulty: Average

Peer Responses

Session Progress

0Responses Correct:

179Responses Incorrect:

179Responses Total:

0%Responses - % Correct:

Blog (https://www.pastest.com/blog) About Pastest (https://www.pastest.com/about-us)Contact Us (https://www.pastest.com/contact-us) Help (https://www.pastest.com/help)

© Pastest 2016

Page 21: Back to Filters (/Secure/TestMe/Filter ... - 1 File Download

8/12/2016 MyPastest

https://mypastest.pastest.com/Secure/TestMe/Browser/429893 1/2

Back to Filters (/Secure/TestMe/Filter/429893/QA)

Question 11 of 179

A 50-year-man presents to the Acute Medical Unit with severe pain and inflammationaffecting his big toe. There is no history of injury or fever, and you suspect a clinical diagnosisof gout. He has been receiving a number of different medications over many years.

Which of the following long-term treatments is most likely to have contributed to theoccurrence of acute gout?

Explanation

The answer is Chlorthalidone –

Long-term therapy with a thiazide diuretic and low-dose aspirin results in lessened renalurate elimination, so that total body urate burden rises and this increases the risk of anacute attack. The effect of thiazides (eg chlorthalidone) on renal urate elimination isgreater than that of aspirin.

Amlodipine (Option A) is incorrect. Amlodipine has no effect on urate elimination.

Aspirin (Option B) is incorrect. Aspirin interferes with renal tubular secretion of urate so thatthe body urate load increases. The effect is less pronounced than for thiazides, which is whychlorthalidone is the correct answer.

Indometacin (Option D) is incorrect. Indometacin and other non-steroidal anti-inflammatorydrugs (NSAIDs) may be highly effective in treating acute attacks, but do not have anysignificant effect on renal elimination of urate.

Losartan (Option E) is incorrect. Losartan reduces serum urate through a uricosuric renaleffect; this has been demonstrated for ACE inhibitors too, and presumably occurs to some

AmlodipineA

AspirinB

ChlorthalidoneC

IndometacinD

LosartanE

Page 22: Back to Filters (/Secure/TestMe/Filter ... - 1 File Download

8/12/2016 MyPastest

https://mypastest.pastest.com/Secure/TestMe/Browser/429893 2/2

46904

extent for other angiotensin receptor blockers as well.

Next Question

Previous Question Tag Question Feedback End Review

Difficulty: Average

Peer Responses

Session Progress

0Responses Correct:

179Responses Incorrect:

179Responses Total:

0%Responses - % Correct:

Blog (https://www.pastest.com/blog) About Pastest (https://www.pastest.com/about-us)Contact Us (https://www.pastest.com/contact-us) Help (https://www.pastest.com/help)

© Pastest 2016

Page 23: Back to Filters (/Secure/TestMe/Filter ... - 1 File Download

8/12/2016 MyPastest

https://mypastest.pastest.com/Secure/TestMe/Browser/429893 1/2

Back to Filters (/Secure/TestMe/Filter/429893/QA)

Question 12 of 179

The presence of renal failure may alter the pharmacokinetic handling of many drugs.

Which one of the following pharmacokinetic parameters is most likely to occur as a result ofchronic renal failure?

Explanation

The answer is Altered volume of distribution -

Renal failure disturbs virtually every kinetic parameter including:

Gastric absorption

Hepatic metabolism of some drugs

Protein binding

Volume of distribution

Increased drug absorption (Option B) is incorrect. In many instances, the presence of chronickidney disease is associated with gut oedema and reduced drug absorption.

Increased protein binding (Option C) is incorrect. Patients with renal failure often have co-existing hypoalbuminaemia, resulting in lower protein-binding capacity for drugs.

Liver metabolism of drugs (Option D) is incorrect. Renal metabolism and clearance is oftenaltered significantly in renal failure, but liver metabolism is not substantially altered.

Altered volume of distributionA

Increased drug absorptionB

Increased protein bindingC

Liver metabolism of drugsD

Reduced bioavailability immediately after intravenous drug injectionE

Page 24: Back to Filters (/Secure/TestMe/Filter ... - 1 File Download

8/12/2016 MyPastest

https://mypastest.pastest.com/Secure/TestMe/Browser/429893 2/2

46730

Reduced bioavailability immediately after intravenous drug injection (Option E) is incorrect.The bioavailability of an intravenously administered drug is 100% and does not change inrenal failure.

Next Question

Previous Question Tag Question Feedback End Review

Difficulty: Average

Peer Responses

Session Progress

0Responses Correct:

179Responses Incorrect:

179Responses Total:

0%Responses - % Correct:

Blog (https://www.pastest.com/blog) About Pastest (https://www.pastest.com/about-us)Contact Us (https://www.pastest.com/contact-us) Help (https://www.pastest.com/help)

© Pastest 2016

Page 25: Back to Filters (/Secure/TestMe/Filter ... - 1 File Download

8/12/2016 MyPastest

https://mypastest.pastest.com/Secure/TestMe/Browser/429893 1/2

Back to Filters (/Secure/TestMe/Filter/429893/QA)

Question 13 of 179

A 44-year-old man with a body mass index of 34 kg/m was found to have type-2 diabetesmellitus on routine testing 3 months ago. He was advised a low-energy, weight-reducing dietand exercise. He has been unable to comply with this, and recent investigations showpostprandial blood glucose concentration is 14.2 mmol/l.

What would be the best drug to initiate as first-line therapy for type-2 diabetes in thispatient?

Explanation

The answer is option Metformin –

Metformin is the drug of choice in overweight patients. Metformin acts by suppressingappetite, decreasing gluconeogenesis and increasing the peripheral utilisation of glucose.Metformin does not cause hypoglycaemia. Lactic acidosis may occur, particularly inpatients with renal impairment. Metformin is normally contraindicated in patients withacute kidney injury, and chronic renal impairment is a relative contraindication. Inpatients who fail to tolerate metformin, either a sulfonylurea (SU) or dipeptidyl peptidaseIV (DPPIV) inhibitor may be considered in addition.

Gliclazide (Option A) is incorrect. Gliclazide is effective in lowering blood glucose bystimulating pancreatic insulin secretion. Sulfonylureas may promote weight gain, andmetformin is the preferred first-line agent.

Insulin (Option B) is incorrect. Insulin therapy may ultimately be required, but normally this isreserved for patients who fail to achieve adequate glycaemic control through lifestylemeasures and oral antihyperglycaemic agents.

2

GliclazideA

InsulinB

MetforminC

PioglitazoneD

SitagliptinE

Page 26: Back to Filters (/Secure/TestMe/Filter ... - 1 File Download

8/12/2016 MyPastest

https://mypastest.pastest.com/Secure/TestMe/Browser/429893 2/2

46748

Pioglitazone (Option D) is incorrect. Pioglitazone would not be considered a suitable first-line therapy in type-2 diabetes, but may be considered in addition to metformin wheremetformin alone is ineffective.

Sitagliptin (Option E) is incorrrect. Sitagliptin is suitable as a second-line agent, normallygiven in conjunction with metformin. This may promote weight loss.

Next Question

Previous Question Tag Question Feedback End Review

Difficulty: Average

Peer Responses

Session Progress

0Responses Correct:

179Responses Incorrect:

179Responses Total:

0%Responses - % Correct:

Blog (https://www.pastest.com/blog) About Pastest (https://www.pastest.com/about-us)Contact Us (https://www.pastest.com/contact-us) Help (https://www.pastest.com/help)

© Pastest 2016

Page 27: Back to Filters (/Secure/TestMe/Filter ... - 1 File Download

8/12/2016 MyPastest

https://mypastest.pastest.com/Secure/TestMe/Browser/429893#Top 1/2

Back to Filters (/Secure/TestMe/Filter/429893/QA)

Question 14 of 179

A 67-year-old man who has suffered a previous stroke is admitted with collapse. His drughistory includes the use of dipyridamole. On examination he has a regular pulse of 150 bpmdespite having been given a single dose of bisoprolol 2.5 mg in the Emergency Department.You elect to try intravenous adenosine to slow his heart down and assess his cardiac rhythm.

Which of the following is most relevant to the use of adenosine in this patient?

Explanation

The answer is It may be used after ß-blockade –

Adenosine is an ultra-short-acting antiarrhythmic drug that is expected to cause verytemporary AV-nodal blockade for a matter of seconds to minutes. However, dipyridamoleprolongs the half-life of adenosine so that its effects may last for up to several minutes.Adenosine may allow rapid cardioversion of some supraventricular tachycardias,including those associated with Wolff–Parkinson–White syndrome. It has no effect onventricular tachycardia, and therefore adenosine administration can help diagnosetachyarrhythmia. It is contraindicated in patients with second- or third-degree heartblock, sick-sinus syndrome, or patients receiving verapamil owing to the risk ofprecipitating complete heart block. Adverse effects include transient facial flushing andbronchospasm, and a sense of profound anxiety; patients should be warned that this mayoccur in advance. These effects normally last only a matter of seconds, but may be moreprolonged and severe in patients receiving dipyridamole.

It can be used in cases of sick-sinus syndrome (Option A) is incorrect. There is a risk thatadenosine may provoke heart block in some patients with sick-sinus syndrome.

It can be used in cases of sick-sinus syndromeA

It is effective in cardioverting ventricular tachycardiaB

It may be used after ß-blockadeC

Its half-life is decreased by dipyridamoleD

The half-life of adenosine is around 60 sE

Page 28: Back to Filters (/Secure/TestMe/Filter ... - 1 File Download

8/12/2016 MyPastest

https://mypastest.pastest.com/Secure/TestMe/Browser/429893#Top 2/2

46880

It is effective in cardioverting ventricular tachycardia (Option B) is incorrect. Adenosine hasno effect on ventricular tachycardia but is effective for many supraventricular tachycardias,and so can be used to help distinguish these.

Its half-life is decreased by dipyridamole (Option D) is incorrect. Dipyridamole significantlyincreases the half-life and duration of action of adenosine.

The half-life of adenosine is around 60 s (Option E) is incorrect. The half-life of adenosine isaround 3–5 s.

Next Question

Previous Question Tag Question Feedback End Review

Difficulty: Average

Peer Responses

Session Progress

0Responses Correct:

179Responses Incorrect:

179Responses Total:

0%Responses - % Correct:

Blog (https://www.pastest.com/blog) About Pastest (https://www.pastest.com/about-us)Contact Us (https://www.pastest.com/contact-us) Help (https://www.pastest.com/help)

© Pastest 2016

Page 29: Back to Filters (/Secure/TestMe/Filter ... - 1 File Download

8/12/2016 MyPastest

https://mypastest.pastest.com/Secure/TestMe/Browser/429893#Top 1/2

Back to Filters (/Secure/TestMe/Filter/429893/QA)

Question 15 of 179

46839

A 56-year-old woman has been receiving warfarin treatment after recurrent pulmonaryemboli.

Which of the following drugs is most likely to cause an increased INR?

Explanation

The answer is Azathioprine –

Warfarin interferes with the hepatic synthesis of vitamin-K-dependent clotting factors,leading to depression of the activity of factors II, VII, IX, X, protein C and protein S in adose-dependent manner. Commonly used drugs that may lead to an increased INRinclude enzyme inhibitors, including macrolide and quinolone antibiotics, metronidazole,allopurinol and cimetidine. Other drugs capable of increasing INR in patients on warfarintherapy include cephalosporins, azathioprine and testosterone derivatives.

Griseofulvin (Option B) is incorrect. Griseofulvin is an antifungal with powerful enzymeinducing properties that lowers warfarin concentrations and INR.

Phenobarbital (Option C) is incorrect. Phenobarbital is an antiepileptic drug with powerfulenzyme inducer that lowers warfarin concentrations and INR.

Rifampicin (Option D) is incorrect. Rifampicin is an antimicrobial and powerful enzymeinducer that lowers warfarin concentrations and INR.

Vitamin K (Option E) is incorrect. Vitamin K is a competitive inhibitor of warfarin.

AzathioprineA

GriseofulvinB

PhenobarbitalC

RifampicinD

Vitamin KE

Page 30: Back to Filters (/Secure/TestMe/Filter ... - 1 File Download

8/12/2016 MyPastest

https://mypastest.pastest.com/Secure/TestMe/Browser/429893#Top 2/2

Next Question

Previous Question Tag Question Feedback End Review

Difficulty: Average

Peer Responses

Session Progress

0Responses Correct:

179Responses Incorrect:

179Responses Total:

0%Responses - % Correct:

Blog (https://www.pastest.com/blog) About Pastest (https://www.pastest.com/about-us)Contact Us (https://www.pastest.com/contact-us) Help (https://www.pastest.com/help)

© Pastest 2016

Page 31: Back to Filters (/Secure/TestMe/Filter ... - 1 File Download

8/12/2016 MyPastest

https://mypastest.pastest.com/Secure/TestMe/Browser/429893#Top 1/2

Back to Filters (/Secure/TestMe/Filter/429893/QA)

Question 16 of 179

An 18-year-old man is brought to the Emergency Department at 0300 h by two of his friends.He had been in a local nightclub but complained of central chest pain followed by a collapse.On examination he is agitated, heart rate 130 bpm, blood pressure 156/92 mmHg, and pupilsare dilated. You suspect possible recreational drug use.

Which one of the following substances is most likely to explain this patient’s clinical features?

Explanation

The answer is Cocaine –

Cocaine reduces the reuptake of dopamine into neurones by inhibiting the dopamine-reuptake transporter, and provokes intense sympathetic activation and vasoconstrictionmedicated via α-adrenoceptors. Cocaine may provoke a hypertensive crisis, and isassociated with development of non-cardiac chest pain as well as myocardial infarctionand stroke. Cocaine overdose may be rapidly fatal due to arrhythmias, seizures ormyocardial infarction. Some similarities are seen with amphetamines and novelrecreational drugs in terms of agitation, tachycardia, psychosis and dilated pupils;however, vasospasm and risk of hypertensive crises and myocardial infarction are lessthan after cocaine.

Alcohol (Option A) is incorrect. Alcohol intoxication produces typical effects of acutesedative–hypnotic drug overdose, vasodilatation, tachycardia and gastrointestinal irritation.

Amitriptyline (Option B) is incorrect. Amitriptyline may cause tachycardia and dilated pupilsthrough anticholinergic mechanisms. However, significant toxicity is associated with reducedconscious level, and it is not associated with euphoria or prone to abuse.

AlcoholA

AmitriptylineB

CocaineC

Lysergic acid diethylamideD

MorphineE

Page 32: Back to Filters (/Secure/TestMe/Filter ... - 1 File Download

8/12/2016 MyPastest

https://mypastest.pastest.com/Secure/TestMe/Browser/429893#Top 2/2

46784

Lysergic acid diethylamide (LSD) (Option D) is incorrect. LSD produces a series of somatic,perceptual and psychological effects. Dizziness, weakness and tremors occur, along withblurring of vision, hallucinations, impaired memory, poor judgement and altered mood. Itwould not be expected to cause pupillary dilatation.

Morphine (Option E) is incorrect. Opioid overdose causes miosis, slurred speech,disorientation and respiratory depression, and reduced conscious level.

Next Question

Previous Question Tag Question Feedback End Review

Difficulty: Average

Peer Responses

Session Progress

0Responses Correct:

179Responses Incorrect:

179Responses Total:

0%Responses - % Correct:

Blog (https://www.pastest.com/blog) About Pastest (https://www.pastest.com/about-us)Contact Us (https://www.pastest.com/contact-us) Help (https://www.pastest.com/help)

© Pastest 2016

Page 33: Back to Filters (/Secure/TestMe/Filter ... - 1 File Download

8/12/2016 MyPastest

https://mypastest.pastest.com/Secure/TestMe/Browser/429893#Top 1/2

Back to Filters (/Secure/TestMe/Filter/429893/QA)

Question 17 of 179

A 27-year-old woman is taking the oral contraceptive pill and has a history of epilepsy. Shefinds that sodium valproate is causing her to put on weight and she is keen to switch to analternative medication.

Which one of the following would be the most appropriate medication for her?

Explanation

The answer is Lamotrigine –

Lamotrigine is the most appropriate choice here because it does not interfere with theeffectiveness of the oral contraceptive pill. Although limited data in pregnancy areavailable, lamotrigine has a favourable profile in terms of congenital malformationscompared with sodium valproate.

Carbamazepine (Option A) is incorrect. Carbamazepine is a powerful hepatic enzyme inducerand is expected to decrease effectiveness of the oral contraceptive pill and risk unintendedpregnancy.

Clonazepam (Option B) is incorrect. Clonazepam is less likely to be an effective antiepilepticmedication, and may cause sedation.

Phenytoin (Option D) is incorrect. Phenytoin may also cause weight gain, and is a powerfulenzyme inducer so that the oral contraceptive pill will be less effective. Phenytoin is arecognised cause of congenital malformations in around 5%.

Topiramate (Option E) is incorrect. Topiramate may decrease oestrogen levels and thusreduce pill effectiveness. Topiramate, when used as part of a polytherapy regimen, may be

CarbamazepineA

ClonazepamB

LamotrigineC

PhenytoinD

TopiramateE

Page 34: Back to Filters (/Secure/TestMe/Filter ... - 1 File Download

8/12/2016 MyPastest

https://mypastest.pastest.com/Secure/TestMe/Browser/429893#Top 2/2

46997

associated with a ten-fold risk of cleft lip compared to the background population.

Next Question

Previous Question Tag Question Feedback End Review

Difficulty: Average

Peer Responses

Session Progress

0Responses Correct:

179Responses Incorrect:

179Responses Total:

0%Responses - % Correct:

Blog (https://www.pastest.com/blog) About Pastest (https://www.pastest.com/about-us)Contact Us (https://www.pastest.com/contact-us) Help (https://www.pastest.com/help)

© Pastest 2016

Page 35: Back to Filters (/Secure/TestMe/Filter ... - 1 File Download

8/12/2016 MyPastest

https://mypastest.pastest.com/Secure/TestMe/Browser/429893#Top 1/2

Back to Filters (/Secure/TestMe/Filter/429893/QA)

Question 18 of 179

You review a 44-year-old woman in an outpatient clinic for review of her hypertension. Shehad recently been started on hydralazine on the advice of the renal specialist team. Thepatient has read the information leaflet indicating that drug-induced lupus is a recognisedcomplication.

Which of the following factors would most strongly increase the risk of her developinghydralazine-induced lupus?

Explanation

The answer is Caucasian ethnicity –

Systemic lupus erythematosus (SLE) is more common in Afro-Caribbeans and muchmore common in women. In contrast, drug-induced lupus occurs more commonly inCaucasians, with men and women affected equally. The reaction is drug dose- andconcentration-dependent. Other drugs characteristically associated with drug-inducedlupus are hydralazine, procainamide and isoniazid. Most cases resolve spontaneouslyafter removal of the causative drug.

dsDNA antibodies are positive (Option B) is incorrect. These are characteristically increasedin SLE but are normally undetectable in drug-induced lupus. Drug-induced lupus may beassociated with a positive antinuclear factor and antihistone antibody titre.

Family history of systemic lupus erythematosus (Option C) is incorrect. Positive familyhistory of SLE may pose a slightly increased risk of SLE and drug-induced lupus, but this is aless preferred answer owing to the very weak association.

Caucasian ethnicityA

dsDNA antibodies are positiveB

Family history of systemic lupus erythematosusC

HLA-B6 genotypeD

Rapid acetylator status is a risk factorE

Page 36: Back to Filters (/Secure/TestMe/Filter ... - 1 File Download

8/12/2016 MyPastest

https://mypastest.pastest.com/Secure/TestMe/Browser/429893#Top 2/2

45809

HLA-B6 genotype (Option D) is incorrect. Drug-induced lupus occurs more commonly inpatients with HLA-DR4 genotype.

Rapid acetylator status is a risk factor (Option E) is incorrect. Hydralazine is subject toacetylation; slow acetylator phenotype confers a higher risk of drug-induced lupus,presumably because slower metabolism results in higher hydralazine concentrations.

Next Question

Previous Question Tag Question Feedback End Review

Difficulty: Average

Peer Responses

Session Progress

0Responses Correct:

179Responses Incorrect:

179Responses Total:

0%Responses - % Correct:

Blog (https://www.pastest.com/blog) About Pastest (https://www.pastest.com/about-us)Contact Us (https://www.pastest.com/contact-us) Help (https://www.pastest.com/help)

© Pastest 2016

Page 37: Back to Filters (/Secure/TestMe/Filter ... - 1 File Download

8/12/2016 MyPastest

https://mypastest.pastest.com/Secure/TestMe/Browser/429893#Top 1/2

Back to Filters (/Secure/TestMe/Filter/429893/QA)

Question 19 of 179

46611

You are reviewing a 53-year-old female patient with long-standing diabetes and gastricdysmotility. She has read some information about cisapride and has asked you for adviceconcerning this medication.

Which of the following statements is most accurate regarding cisapride?

Explanation

The answer is QT prolongation on the ECG –

Cisapride is a prokinetic drug that reduces gastric emptying time. It interacts withanticoagulants and increases the prothrombin time and bleeding risk. It increases themotility of the entire gastrointestinal tract, and enhances gastric emptying. QTabnormalities are reported with therapy, especially when given with erythromycin orantifungal drugs like ketoconazole (enzyme inhibitor); cisapride has been associated withfatal arrhythmias and its use has been restricted. Anticholinergics antagonise the effectsof cisapride.

Decreases prothrombin time in patients receiving warfarin (Option A) is incorrect. It mayincrease prothrombin time and international normalised ratio (INR).

Delays gastric emptying time (Option B) is incorrect. It reduces gastric emptying time.

Exacerbation of symptoms of heartburn (Option C) is incorrect. It may alleviate heartburn inpatients with delayed gastric emptying.

Relaxation of colonic musculature (Option D) is incorrect. It increases motility and mayincrease colonic tone.

Decreases prothrombin time in patients receiving warfarinA

Delays gastric emptying timeB

Exacerbation of symptoms of heartburnC

Relaxation of colonic musculatureD

QT prolongation on the ECGE

Page 38: Back to Filters (/Secure/TestMe/Filter ... - 1 File Download

8/12/2016 MyPastest

https://mypastest.pastest.com/Secure/TestMe/Browser/429893#Top 2/2

46611

Next Question

Previous Question Tag Question Feedback End Review

Difficulty: Average

Peer Responses

Session Progress

0Responses Correct:

179Responses Incorrect:

179Responses Total:

0%Responses - % Correct:

Blog (https://www.pastest.com/blog) About Pastest (https://www.pastest.com/about-us)Contact Us (https://www.pastest.com/contact-us) Help (https://www.pastest.com/help)

© Pastest 2016

Page 39: Back to Filters (/Secure/TestMe/Filter ... - 1 File Download

8/12/2016 MyPastest

https://mypastest.pastest.com/Secure/TestMe/Browser/429893#Top 1/2

Back to Filters (/Secure/TestMe/Filter/429893/QA)

Question 20 of 179

You review a 78-year-old man who was admitted with hypotension and tachycardia and isfound to have atrial fibrillation. The admitting FY2 diagnosed sepsis and commenced him ongentamicin therapy. He is on multiple drugs, including fluoxetine for depression, bumetanidefor fluid overload, amiodarone for atrial fibrillation and diazepam to help him sleep. You wishto adjust some of his existing medications and add new treatments.

Which of the following medications if stopped or started would take longest for the patientto respond to?

Explanation

The answer is Amiodarone –

Amiodarone has by far the longest half-life of these agents, at around 25–30 days.Initiation of therapy requires a loading regimen over the first few weeks of treatment,and cessation may take several months before drug concentrations have fallen to aninsignificant level. The half-life is dependent upon lipid solubility and wide distribution ofthe drug to extravascular tissues. With wide volume of distribution and extensive tissuebinding there is slow renal or hepatic drug clearance.

Bumetanide (Option B) is incorrect. Bumetanide has a short half-life, around 1 h, so that theeffects are comparatively short-lived.

Diazepam (Option C) is incorrect. Diazepam is highly lipid soluble, which helps it cross theblood–brain barrier; the plasma half-life is around 43 h, although the biological effects maypersist for longer due to persistence in the brain tissue.

AmiodaroneA

BumetanideB

DiazepamC

FluoxetineD

GentamicinE

Page 40: Back to Filters (/Secure/TestMe/Filter ... - 1 File Download

8/12/2016 MyPastest

https://mypastest.pastest.com/Secure/TestMe/Browser/429893#Top 2/2

46887

Fluoxetine (Option D) is incorrect. Fluoxetine is very lipid-soluble and has a comparativelylong half-life of around 50 h. Irrespective of the half-life, the biological response to startingor stopping therapy may take 2–3 weeks (that is, a dissociation between the pharmacokineticprofile and pharmacodynamic actions).

Gentamicin (Option E) is incorrect. Gentamicin is water soluble and clearance is highlydependent upon renal excretion; in patients with normal renal function the half-life ofgentamicin is around 2–3 h.

Next Question

Previous Question Tag Question Feedback End Review

Difficulty: Average

Peer Responses

Session Progress

0Responses Correct:

179Responses Incorrect:

179Responses Total:

0%Responses - % Correct:

Blog (https://www.pastest.com/blog) About Pastest (https://www.pastest.com/about-us)Contact Us (https://www.pastest.com/contact-us) Help (https://www.pastest.com/help)

© Pastest 2016

Page 41: Back to Filters (/Secure/TestMe/Filter ... - 1 File Download

8/12/2016 MyPastest

https://mypastest.pastest.com/Secure/TestMe/Browser/429893#Top 1/2

Back to Filters (/Secure/TestMe/Filter/429893/QA)

Question 21 of 179

A 58-year-old man attends the Nephrology Clinic for review. He admits at the end of hisconsultation that he suffers from erectile dysfunction. You are considering prescribingvardenafil for him. He is taking multiple agents for the treatment of hypertension.

Which one of the following agents would be most likely to provoke adverse effects in apatient receiving vardenafil?

Explanation

The answer is Nicorandil –

Nicorandil is a potassium channel activator recognised to increase nitric oxide (NO), andis contraindicated for prescription in conjunction with vardenafil. Other drugs that arecontraindicated include nitrate donors such as isosorbide dinitrate and isosorbidemononitrate; use of these agents with vardenafil may lead to a precipitous fall in bloodpressure and lead to profound hypotension. Similar adverse effects are likely to occurwith the use of amyl nitrate, which is used primarily for recreational purposes.

Atenolol (Option A) is incorrect. There is no significant interaction between atenolol andvardenafil.

Bendrofluazide (Option B) is incorrect. There is no significant interaction betweenbendrofluazide and vardenafil.

Carbamazepine (Option C) is incorrect. There is no significant interaction betweencarbamazepine and vardenafil.

AtenololA

BendrofluazideB

CarbamazepineC

NicorandilD

RamiprilE

Page 42: Back to Filters (/Secure/TestMe/Filter ... - 1 File Download

8/12/2016 MyPastest

https://mypastest.pastest.com/Secure/TestMe/Browser/429893#Top 2/2

46975

Ramipril (Option E) is incorrect. Both vardenafil and ACE inhibitors may provoke posturalhypotension and should be used with caution; the effect is less severe than that of nicorandilor nitric oxide donors, hence nicorandil is the preferred answer.

Next Question

Previous Question Tag Question Feedback End Review

Difficulty: Average

Peer Responses

Session Progress

0Responses Correct:

179Responses Incorrect:

179Responses Total:

0%Responses - % Correct:

Blog (https://www.pastest.com/blog) About Pastest (https://www.pastest.com/about-us)Contact Us (https://www.pastest.com/contact-us) Help (https://www.pastest.com/help)

© Pastest 2016

Page 43: Back to Filters (/Secure/TestMe/Filter ... - 1 File Download

8/12/2016 MyPastest

https://mypastest.pastest.com/Secure/TestMe/Browser/429893#Top 1/2

Back to Filters (/Secure/TestMe/Filter/429893/QA)

Question 22 of 179

A 19-year-old woman presents to the Emergency Department. She had a row with herboyfriend and took a handful of her mother’s tablets, which she found in a bottle. Her motherhas multiple health problems, including hypertension, reflux oesophagitis, dyslipidaemia andnight cramps. The patient complains of involuntary spasms affecting the left side of her neck,particularly the sternocleidomastoid muscle, and feels very upset. On examination her bloodpressure is 155/90 mmHg.

Which one of the following drugs is she most likely to have taken?

Explanation

The answer is Metoclopramide –

This woman has presented with acute dystonia, a condition known to be associated withuse of antipsychotics and antiemetics, particularly metoclopramide andprochlorperazine. Onset is usually within 1–2 h of ingestion of the causative agent, andprocyclidine is the agent of choice used to treat it.

Omeprazole (Option B) is incorrect. Omeprazole may cause headache and nausea inoverdose.

Quinine (Option C) is incorrect. Quinine may cause blindness, cardiac arrhythmia andneurotoxicity.

Ramipril (Option D) is incorrect. ACE inhibitors may cause profound hypotension, flushingand headache in overdose (in contrast to the notional flat dose response associated withtherapeutic doses).

MetoclopramideA

OmeprazoleB

QuinineC

RamiprilD

SimvastatinE

Page 44: Back to Filters (/Secure/TestMe/Filter ... - 1 File Download

8/12/2016 MyPastest

https://mypastest.pastest.com/Secure/TestMe/Browser/429893#Top 2/2

46977

Simvastatin (Option E) is incorrect. Simvastatin may cause myositis, especially with chronicuse, but overdose is not associated with dystonia.

Next Question

Previous Question Tag Question Feedback End Review

Difficulty: Average

Peer Responses

Session Progress

0Responses Correct:

179Responses Incorrect:

179Responses Total:

0%Responses - % Correct:

Blog (https://www.pastest.com/blog) About Pastest (https://www.pastest.com/about-us)Contact Us (https://www.pastest.com/contact-us) Help (https://www.pastest.com/help)

© Pastest 2016

Page 45: Back to Filters (/Secure/TestMe/Filter ... - 1 File Download

8/12/2016 MyPastest

https://mypastest.pastest.com/Secure/TestMe/Browser/429893#Top 1/2

Back to Filters (/Secure/TestMe/Filter/429893/QA)

Question 23 of 179

A 62-year-old man with a diagnosis of subacute bacterial endocarditis is admitted to theCardiology Ward. He is treated with a combination of IV benzylpenicillin and gentamicin.Other past history of note includes hypertension and chronic renal failure. On examination hisblood pressure is 152/82 mmHg and his pulse is 82 beats per minute and regular.

Investigations:

Hb 10.1 g/dl

White cell count 13.1 × 10 /l

Platelets 202 × 10 /l

Na 139 mmol/l

K 5.2 mmol/l

Creatinine 281 μmol/l

The laboratory tells you that his trough gentamicin level is too high; the 12-hdosing regimenhad been calculated based on his body weight.

Why is the trough level likely to be too high?

Explanation

Altered gentamicin pharmacokinetics

9

9

+

+

Changes in the volume of distributionA

Prolongation of the half-lifeB

Decreased first-pass metabolismC

Decreased protein bindingD

Increased bioavailabilityE

Page 46: Back to Filters (/Secure/TestMe/Filter ... - 1 File Download

8/12/2016 MyPastest

https://mypastest.pastest.com/Secure/TestMe/Browser/429893#Top 2/2

22424

The usual half-life of gentamicin is between 2 and 3 h, although this is considerably

prolonged in patients with renal failure

At steady state around 70% of any given dose of gentamicin is recovered in the urine

24 h later, although some accumulation of gentamicin does occur, particularly in the

kidney

Particular care should be taken when administering gentamicin in conjunction with loop

diuretics because of the risk of exacerbating renal and ototoxicity

The problem of trough levels that are too high can be overcome in this case by

lengthening the dosing interval

Next Question

Previous Question Tag Question Feedback End Review

Difficulty: Average

Peer Responses

Session Progress

0Responses Correct:

179Responses Incorrect:

179Responses Total:

0%Responses - % Correct:

Blog (https://www.pastest.com/blog) About Pastest (https://www.pastest.com/about-us)Contact Us (https://www.pastest.com/contact-us) Help (https://www.pastest.com/help)

© Pastest 2016

Page 47: Back to Filters (/Secure/TestMe/Filter ... - 1 File Download

8/12/2016 MyPastest

https://mypastest.pastest.com/Secure/TestMe/Browser/429893#Top 1/2

Back to Filters (/Secure/TestMe/Filter/429893/QA)

Question 24 of 179

A 79-year-old man is admitted to hospital with palpitations. A resting ECG shows atrialfibrillation with a rate 107/min. You decide to prescribe digoxin therapy.

Which of the following most accurately describes the pharmacological mechanism of actionof digoxin?

Explanation

The answer is Inhibition of sodium-potassium ATPase –

The key pharmacological mechanism is inhibition of Na /K ATPase, which is located inthe sarcolemmal membrane. This causes accumulation of intracellular sodium andcalcium by way of the sodium–calcium exchange mechanism; intracellular potassiumconcentrations decrease whereas extracellular potassium concentrations increase.Digitalis glycosides increase the contractility of the heart (positively inotropic) anddecrease the resting heart rate (negatively chronotropic) and delay A–V nodalconduction.

Opening of calcium channels (Option B) is incorrect. Intracellular calcium concentrationsincrease, but there is no direct action of digoxin on cardiac calcium channels.

Release of calcium from the sarcoplasmic reticulum (Option C) is incorrect. Calciumconcentrations increase due to sodium–calcium exchange, rather than release from thesarcoplasmic reticulum.

Stimulation of membrane phospholipase C (Option D) is incorrect. Digoxin is not known toexert a direct effect on phospholipase C. Many other receptor systems act via phospholipase

Inhibition of sodium-potassium ATPaseA

Opening of calcium channelsB

Release of calcium from the sarcoplasmic reticulumC

Stimulation of membrane phospholipase CD

Stimulation of myosin ATPaseE

+ +

Page 48: Back to Filters (/Secure/TestMe/Filter ... - 1 File Download

8/12/2016 MyPastest

https://mypastest.pastest.com/Secure/TestMe/Browser/429893#Top 2/2

46596

C, including 5-HT2 serotonergic, α1 adrenergic, calcitonin, H1 histamine, glutamate receptors,and M1, M3 and M5 muscarinic receptors.

Stimulation of myosin ATPase (Option E) is incorrect. The effect is inhibition rather thanstimulation of the Na /K ATPase.+ +

Next Question

Previous Question Tag Question Feedback End Review

Difficulty: Average

Peer Responses

Session Progress

0Responses Correct:

179Responses Incorrect:

179Responses Total:

0%Responses - % Correct:

Blog (https://www.pastest.com/blog) About Pastest (https://www.pastest.com/about-us)Contact Us (https://www.pastest.com/contact-us) Help (https://www.pastest.com/help)

© Pastest 2016

Page 49: Back to Filters (/Secure/TestMe/Filter ... - 1 File Download

8/12/2016 MyPastest

https://mypastest.pastest.com/Secure/TestMe/Browser/429893#Top 1/2

Back to Filters (/Secure/TestMe/Filter/429893/QA)

Question 25 of 179

A 65-year-old man has multiple medical disorders including heart failure, transient ischaemicattacks and non-insulin-dependent diabetes mellitus. He has developed profuse and waterydiarrhoea for the past 4 days after eating a sea-food meal. Blood tests reveal urea of 15.2mmol/l and a creatinine of 345 mmol/l.

Which one of his existing medications would be most important to stop?

Explanation

The answer is Irbesartan –

The clinical scenario indicates acute gastroenteritis and renal failure. In healthyindividuals, activation of the renin–angiotensin–aldosterone system preserves glomerularfiltration pressure and renal function. However, in patients receiving ACE inhibitors orangiotensin receptor antagonists this mechanism is blocked so that renal failuredevelops. In cases of dehydration or sepsis, the ACE inhibitor or angiotensin receptorblocker (eg irbesartan) should be temporarily discontinued.

Aspirin (Option A) is incorrect. Aspirin, even in small dosages, can contribute to renalimpairment in severe dehydration owing to its effects on renal prostaglandin synthesis.However, this effect is much smaller than that of an ACE inhibitor or angiotensin receptorblocker, hence irbesartan is the preferred answer.

Bendroflumethiazide (Option B) is incorrect. Diuretic treatment will exacerbate dehydrationand haemodynamic consequences. Although this may also need to be discontinued, it is lesshazardous than an ACE inhibitor or angiotensin receptor blocker, hence irbesartan is thepreferred answer.

AspirinA

BendroflumethiazideB

DigoxinC

GlibenclamideD

IrbesartanE

Page 50: Back to Filters (/Secure/TestMe/Filter ... - 1 File Download

8/12/2016 MyPastest

https://mypastest.pastest.com/Secure/TestMe/Browser/429893#Top 2/2

46966

Digoxin (Option C) is incorrect. Digoxin may accumulate in renal impairment, so that a dosereduction could be anticipated or drug concentrations measured.

Glibenclamide (Option D) is incorrect. Glibenclamide may accumulate in renal impairment sothat dose reduction may be needed. Blood glucose can be monitored and treatment doseadjusted accordingly.

Next Question

Previous Question Tag Question Feedback End Review

Difficulty: Average

Peer Responses

Session Progress

0Responses Correct:

179Responses Incorrect:

179Responses Total:

0%Responses - % Correct:

Blog (https://www.pastest.com/blog) About Pastest (https://www.pastest.com/about-us)Contact Us (https://www.pastest.com/contact-us) Help (https://www.pastest.com/help)

© Pastest 2016

Page 51: Back to Filters (/Secure/TestMe/Filter ... - 1 File Download

8/12/2016 MyPastest

https://mypastest.pastest.com/Secure/TestMe/Browser/429893#Top 1/2

Back to Filters (/Secure/TestMe/Filter/429893/QA)

Question 26 of 179

46989

You review a 72-year-old man with a history of dementia. He is becoming increasingly hard tomanage at home, is agitated and difficult, and is suffering from delusions that the membersof his family who care for him are trying to poison him. You decide to add risperidone to hisexisting medications.

For which one of the following receptors does risperidone have the highest affinity?

Explanation

The answer is 5HT-2 receptors –

Risperidone is a novel antipsychotic belonging to the benzisoxazole derivative class; it isan antagonist with high affinity for D2 and 5HT-2 receptors in the central nervous system.To a lesser extent, risperidone also blocks α1-adrenergic receptors, H1-histaminergic andα2-adrenergic receptors. Common adverse effects include insomnia, agitation andanxiety. Risperidone may also lead to impaired glucose tolerance, although the rate ofoccurrence is similar to other antipsychotics.

5HT-3 receptors (Option B) is incorrect. Ondansetron is a 5HT-3 antagonist.

α1-Adrenergic receptors (Option C) is incorrect. Risperidone is an antagonist at α1-adrenergicreceptors, but this is less important than its effects on 5HT-2 receptors.

D1 receptors (Option D) is incorrect. Blockade of D1 receptors causes parkinsonism.

H2 receptors (Option E) is incorrect. Risperidone is an antagonist at H1-histaminergicreceptors.

5HT-2 receptorsA

5HT-3 receptorsB

α1-Adrenergic receptorsC

D1 receptorsD

H2 receptorsE

Page 52: Back to Filters (/Secure/TestMe/Filter ... - 1 File Download

8/12/2016 MyPastest

https://mypastest.pastest.com/Secure/TestMe/Browser/429893#Top 2/2

Next Question

Previous Question Tag Question Feedback End Review

Difficulty: Average

Peer Responses

Session Progress

0Responses Correct:

179Responses Incorrect:

179Responses Total:

0%Responses - % Correct:

Blog (https://www.pastest.com/blog) About Pastest (https://www.pastest.com/about-us)Contact Us (https://www.pastest.com/contact-us) Help (https://www.pastest.com/help)

© Pastest 2016

Page 53: Back to Filters (/Secure/TestMe/Filter ... - 1 File Download

8/12/2016 MyPastest

https://mypastest.pastest.com/Secure/TestMe/Browser/429893#Top 1/2

Back to Filters (/Secure/TestMe/Filter/429893/QA)

Question 27 of 179

A 25-year-old woman on the oral contraceptive pill is diagnosed with partial epilepsy. Overthe past 3 months, she has had a number of seizures requiring treatment. She advises youthat she wishes to continue the contraceptive pill as she has no plans to become pregnant.

Which of the following measures are appropriate?

Explanation

The answer is Continue oral contraceptive, and commence on lamotrigine –

Lamotrigine is a suitable first-line treatment for partial epilepsy, and does notsignificantly alter oestrogen metabolism.

Change to a progestogen-only contraceptive pill (Option A) is incorrect. This would notaddress the need for epilepsy treatment.

Continue oral contraceptive, and commence on ethosuximide (Option B) is incorrect.Ethosuximide is a suitable treatment for absence and myoclonic attacks in children, butwould be less effective for partial seizures in adults.

Continue oral contraceptive, and commence on phenytoin (Option D) is incorrect. Phenytoininduces liver enzymes, thereby increasing oestrogen breakdown and reducing theeffectiveness of oestrogen-containing contraceptives. Where the combined contraceptive pillis used in conjunction with phenytoin, the contraceptive should contain high-dose oestrogen,50 μg ethinylestradiol or more (carbamazepine is also a potent enzyme inducer and shouldnot be used in combination with the pill).

Change to a progestogen-only contraceptive pillA

Continue oral contraceptive, and commence on ethosuximideB

Continue oral contraceptive, and commence on lamotrigineC

Continue oral contraceptive, and commence on phenytoinD

Stop the oral contraceptive, as this may be precipitating fits, and suggest usingbarrier contraceptive methods

E

Page 54: Back to Filters (/Secure/TestMe/Filter ... - 1 File Download

8/12/2016 MyPastest

https://mypastest.pastest.com/Secure/TestMe/Browser/429893#Top 2/2

46897

Stop the oral contraceptive, as this may be precipitating fits, and suggest using barriercontraceptive methods (Option E) is incorrect. The oral contraceptive pill is not recognisedas a cause of seizures.

Next Question

Previous Question Tag Question Feedback End Review

Difficulty: Average

Peer Responses

Session Progress

0Responses Correct:

179Responses Incorrect:

179Responses Total:

0%Responses - % Correct:

Blog (https://www.pastest.com/blog) About Pastest (https://www.pastest.com/about-us)Contact Us (https://www.pastest.com/contact-us) Help (https://www.pastest.com/help)

© Pastest 2016

Page 55: Back to Filters (/Secure/TestMe/Filter ... - 1 File Download

8/12/2016 MyPastest

https://mypastest.pastest.com/Secure/TestMe/Browser/429893#Top 1/2

Back to Filters (/Secure/TestMe/Filter/429893/QA)

Question 28 of 179

You are considering treatment options in a 36-year-old woman with thyrotoxicosis, and areconsidering the relative merits of propylthiouracil versus carbimazole treatments.

Which of the following factors best characterises the distinguishing characteristics ofpropylthiouracil therapy?

Explanation

The answer is Inhibits conversion of thyroxine to tri-iodothyronine –

Propylthiouracil (PTU) and carbimazole are both derivatives of thiourea with similarchemical structure, mechanisms of action and adverse effects. Both are capable ofinhibiting organification of iodine at the thyroid gland. Neither drug affects iodinetrapping and neither drug inhibits the release of preformed thyroid hormone. A keydistinction is that PTU, but not carbimazole, inhibits iodination of thyroxine to tri-iodothyronine. This may give PTU a modest therapeutic advantage because it reducesthe proportion of active thyroid hormone as well as total T4. Both PTU and carbimazoleare excreted in very small quantities in breast milk, so breast feeding is not advised witheither. Thiourea derivatives have several side-effects, including a maculopapular rash,hepatocellular damage and vasculitis. The most serious side-effect of both agents isagranulocytosis, although it is more common to see a fall in, rather than total absence of,white cells.

Inhibits organification of iodine at the thyroid gland (Option B) is incorrect. Both PTU andcarbimazole are capable of this.

Inhibits conversion of thyroxine to tri-iodothyronineA

Inhibits organification of iodine at the thyroid glandB

Inhibits release of preformed thyroid hormoneC

It is a thiourea derivative, unlike carbimazoleD

It is more potent than carbimazoleE

Page 56: Back to Filters (/Secure/TestMe/Filter ... - 1 File Download

8/12/2016 MyPastest

https://mypastest.pastest.com/Secure/TestMe/Browser/429893#Top 2/2

46631

Inhibits release of preformed thyroid hormone (Option C) is incorrect. Neither drug inhibitsrelease of preformed thyroid hormone.

It is a thiourea derivative, unlike carbimazole (Option D) is incorrect. Both drugs are thioureaderivatives.

It is more potent than carbimazole (Option E) is incorrect. Carbimazole is approximately 15times more potent than PTU.

Next Question

Previous Question Tag Question Feedback End Review

Difficulty: Average

Peer Responses

Session Progress

0Responses Correct:

179Responses Incorrect:

179Responses Total:

0%Responses - % Correct:

Blog (https://www.pastest.com/blog) About Pastest (https://www.pastest.com/about-us)Contact Us (https://www.pastest.com/contact-us) Help (https://www.pastest.com/help)

© Pastest 2016

Page 57: Back to Filters (/Secure/TestMe/Filter ... - 1 File Download

8/12/2016 MyPastest

https://mypastest.pastest.com/Secure/TestMe/Browser/429893#Top 1/2

Back to Filters (/Secure/TestMe/Filter/429893/QA)

Question 29 of 179

You are helping at a pulmonary hypertension follow-up clinic, and are reviewing a 57-year-oldwoman treated with bosentan.

Which of the following statements best represents the pharmacological actions of bosentan?

Explanation

The answer is Patients should be monitored for development of liver toxicity –

Bosentan is a competitive antagonist at both endothelin-A (ETA) and endothelin-B (ETB)receptors, leading to falls in both pulmonary and systemic vascular resistances withoutan increase in heart rate. It is one of the few treatments that show efficacy in patientswith pulmonary arterial hypertension. Common unwanted effects include flushing,hypotension, dyspepsia and fatigue. The most serious adverse effect is dose-dependenthepatotoxicity, and it is therefore contraindicated in patients with moderate to severeliver disease. Hepatotoxicity normally occurs within the first 3–4 months of treatment,and patients should have liver function tests (LFTs) monitored.

Causes a dose-dependent increase in systemic vascular resistance (Option A) is incorrect. Itcauses a dose-dependent fall in systemic vascular resistance.

Haemoglobin concentrations typically rise due to haemoconcentration (Option B) isincorrect. Haemoglobin concentrations can fall by up to 1 g/dl during bosentan treatment.

It is a competitive antagonist of the ETA but not ETB receptor (Option C) is incorrect.Bosentan blocks both ETA and ETB receptor subtypes.

Causes a dose-dependent increase in systemic vascular resistanceA

Haemoglobin concentrations typically rise due to haemoconcentrationB

It is a competitive antagonist of the ETA but not ETB receptorC

May be used safely in pregnancyD

Patients should be monitored for development of liver toxicityE

Page 58: Back to Filters (/Secure/TestMe/Filter ... - 1 File Download

8/12/2016 MyPastest

https://mypastest.pastest.com/Secure/TestMe/Browser/429893#Top 2/2

46494

May be used safely in pregnancy (Option D) is incorrect. Bosentan is teratogenic and it iscontraindicated in pregnancy.

Next Question

Previous Question Tag Question Feedback End Review

Difficulty: Average

Peer Responses

Session Progress

0Responses Correct:

179Responses Incorrect:

179Responses Total:

0%Responses - % Correct:

Blog (https://www.pastest.com/blog) About Pastest (https://www.pastest.com/about-us)Contact Us (https://www.pastest.com/contact-us) Help (https://www.pastest.com/help)

© Pastest 2016

Page 59: Back to Filters (/Secure/TestMe/Filter ... - 1 File Download

8/12/2016 MyPastest

https://mypastest.pastest.com/Secure/TestMe/Browser/429893#Top 1/2

Back to Filters (/Secure/TestMe/Filter/429893/QA)

Question 30 of 179

You see a 27-year-old Asian man in the respiratory clinic who has been receivingantituberculous therapy for the past 3 months. He is complaining of tingling and numbness inhis hands and feet, and on examination you find that he has impaired light touch sensation ina glove and stocking distribution.

Which of the following medications is most likely to account for this problem?

Explanation

The answer is Isoniazid –

Isoniazid is a well-known cause of peripheral neuropathy. Isoniazid is subject toacetylation; slow acetylators are at greater risk of developing peripheral neuropathy.Other risk factors include the presence of malnutrition, alcoholism, diabetes and HIVinfection. Pyridoxine (vitamin B6) is given prophylactically to patients receivingisoniazid, and at a higher dose to treat the occurrence of neuropathy.

Ethambutol (Option A) is incorrect. Ethambutol may cause retinal injury, resulting inpermanent loss of visual acuity, restriction of visual fields and colour blindness. Regularophthalmological review is needed and patients should be advised to urgently report anyvisual symptoms.

Pyrazinamide (Option C) is incorrect. Pyrazinamide may cause hepatotoxicity.

Rifampicin (Option D) is incorrect. Rifampicin may cause hepatotoxicity and gives rise topink-coloured bodily fluids, including urine and tears.

EthambutolA

IsoniazidB

PyrazinamideC

RifampicinD

StreptomycinE

Page 60: Back to Filters (/Secure/TestMe/Filter ... - 1 File Download

8/12/2016 MyPastest

https://mypastest.pastest.com/Secure/TestMe/Browser/429893#Top 2/2

46619

Streptomycin (Option E) is incorrect. Streptomycin causes ototoxicity and nephrotoxicity,especially in elderly patients.

Next Question

Previous Question Tag Question Feedback End Review

Difficulty: Average

Peer Responses

Session Progress

0Responses Correct:

179Responses Incorrect:

179Responses Total:

0%Responses - % Correct:

Blog (https://www.pastest.com/blog) About Pastest (https://www.pastest.com/about-us)Contact Us (https://www.pastest.com/contact-us) Help (https://www.pastest.com/help)

© Pastest 2016

Page 61: Back to Filters (/Secure/TestMe/Filter ... - 1 File Download

8/12/2016 MyPastest

https://mypastest.pastest.com/Secure/TestMe/Browser/429893#Top 1/2

Back to Filters (/Secure/TestMe/Filter/429893/QA)

Question 31 of 179

A 19-year-old woman is admitted to the Acute Medical Unit after an intentional paracetamoloverdose. You have been asked to review some blood test results that have been telephonedto the ward.

Which one of the following metabolic abnormalities might most strongly indicate thedevelopment of severe paracetamol poisoning?

Explanation

The answer is Hypoglycaemia –

In paracetamol overdose, the usual pathway of elimination is overwhelmed due todepletion of glutathione within hepatocytes by a toxic metabolite, N-acetylbenzoquinone imine (NABQI) formed by cytochrome P oxidative metabolism.Features of liver necrosis include hypoglycaemia, coagulation factor depletion, renalimpairment, cerebral oedema and coma. If the patient survives, hepatocyte regenerationoccurs without cirrhosis. Hypoglycaemia is an indicator of impaired hepatic glucosesynthesis.

Hyperkalaemia (Option A) is incorrect. Hyperkalaemia is uncommon unless patients havedeveloped acute tubular necrosis, which normally only becomes detectable several daysafter paracetamol overdose.

Hypocalcaemia (Option B) is incorrect. Hypocalcaemia is not a recognised feature ofparacetamol toxicity; it is a characteristic finding in severe ethylene glycol toxicity.

HyperkalaemiaA

HypocalcaemiaB

HypoglycaemiaC

HypokalaemiaD

HyponatraemiaE

450

Page 62: Back to Filters (/Secure/TestMe/Filter ... - 1 File Download

8/12/2016 MyPastest

https://mypastest.pastest.com/Secure/TestMe/Browser/429893#Top 2/2

46913

Hypokalaemia (Option D) is incorrect. Hypokalaemia is an important feature in paracetamoloverdose, and is caused by dose-dependent renal potassium excretion (perhaps also madeworse if the antidote is administered in dextrose rather than saline solution). However,hypokalaemia does not correlate with the risk or extent of liver damage, hencehypoglycaemia is the preferred answer.

Hyponatraemia (Option E) is incorrect. Hyponatraemia is not a recognised feature ofparacetamol toxicity.

Next Question

Previous Question Tag Question Feedback End Review

Difficulty: Average

Peer Responses

Session Progress

0Responses Correct:

179Responses Incorrect:

179Responses Total:

0%Responses - % Correct:

Blog (https://www.pastest.com/blog) About Pastest (https://www.pastest.com/about-us)Contact Us (https://www.pastest.com/contact-us) Help (https://www.pastest.com/help)

© Pastest 2016

Page 63: Back to Filters (/Secure/TestMe/Filter ... - 1 File Download

8/12/2016 MyPastest

https://mypastest.pastest.com/Secure/TestMe/Browser/429893#Top 1/2

Back to Filters (/Secure/TestMe/Filter/429893/QA)

Question 32 of 179

46608

A 50-year-old female on chemotherapy complains of tingling and numbness in her lowerlimbs and weakness in her left hand. On examination, she is noted to have a wrist drop andloss of ankle jerks. Blood tests show hyponatraemia and a plasma osmolality of 260mosmol/kg. Her urine osmolality is 350 mosmol/kg.

Which of the following chemotherapeutic drugs is most likely to explain these features?

Explanation

The answer is Vincristine –

Peripheral neuropathy is a neurotoxic feature of vincristine. Foot drop, paraesthesias, lossof ankle jerks and wrist drop may also occur. Vincristine causes a biochemical pictureresembling syndrome of inappropriate antidiuretic hormone (SIADH), and hyponatraemiamay provoke seizures. Other adverse effects include mucositis of the entiregastrointestinal tract and bleeding. Vincristine causes less nausea and vomiting thanother chemotherapy agents.

Bleomycin (Option A) is incorrect. Bleomycin may cause interstitial pneumonia.

Cyclophosphamide (Option B) is incorrect. Cyclophosphamide characteristically causeshaemorrhagic cystitis.

Doxorubicin (Option C) is incorrect. Doxorubicin and daunorubicin are known to causecardiomyopathy.

Methotrexate (Option D) is incorrect. Methotrexate characteristically causes hepatic andpulmonary fibrosis.

BleomycinA

CyclophosphamideB

DoxorubicinC

MethotrexateD

VincristineE

Page 64: Back to Filters (/Secure/TestMe/Filter ... - 1 File Download

8/12/2016 MyPastest

https://mypastest.pastest.com/Secure/TestMe/Browser/429893#Top 2/2

46608

Next Question

Previous Question Tag Question Feedback End Review

Difficulty: Average

Peer Responses

Session Progress

0Responses Correct:

179Responses Incorrect:

179Responses Total:

0%Responses - % Correct:

Blog (https://www.pastest.com/blog) About Pastest (https://www.pastest.com/about-us)Contact Us (https://www.pastest.com/contact-us) Help (https://www.pastest.com/help)

© Pastest 2016

Page 65: Back to Filters (/Secure/TestMe/Filter ... - 1 File Download

8/12/2016 MyPastest

https://mypastest.pastest.com/Secure/TestMe/Browser/429893#Top 1/2

Back to Filters (/Secure/TestMe/Filter/429893/QA)

Question 33 of 179

A 64-year-old Asian man attends the Emergency Department for review. He is complaining ofaching pains in his arms and leg muscles and of lethargy; he also has minor symptoms of acold. He is a smoker with hypercholesterolaemia and has recently been prescribed oralrosuvastatin 10 mg daily. He has developed aches and pains around his pelvic girdle, and hisurine is dipstick-positive for blood.

What is the most likely cause of this clinical picture?

Explanation

The answer is Rhabdomyolysis –

Specifically, the risk of rhabdomyolysis may be increased with the use of high-dose statintherapy. Rhabdomyolysis is suggested by the presence of suspected myoglobinuria andmuscle pains; muscle enzymes including creatine kinase (CK) are likely to be abnormallyelevated.

Dermatomyositis (Option A) is incorrect. Dermatomyositis is associated with bronchialcarcinoma; hence, where there are skin changes and muscle aches in association withsmoking history, an underlying carcinoma must be considered.

Influenza (Option B) is incorrect. Influenza may cause generalised aches and myalgia, butrarely causes rhabdomyolysis and therefore this is a less suitable answer.

Polymyositis (Option C) is incorrect. Polymyositis and proximal myopathy would be plausiblediagnoses in this scenario; however, the urine dipstick result makes rhabdomyolysis a morelikely option here.

DermatomyositisA

InfluenzaB

PolymyositisC

Proximal myopathyD

RhabdomyolysisE

Page 66: Back to Filters (/Secure/TestMe/Filter ... - 1 File Download

8/12/2016 MyPastest

https://mypastest.pastest.com/Secure/TestMe/Browser/429893#Top 2/2

46874

Proximal myopathy (Option D) is incorrect. Proximal myopathy might explain the patient’ssymptoms, but would not explain the findings on the dipstick test.

Next Question

Previous Question Tag Question Feedback End Review

Difficulty: Average

Peer Responses

Session Progress

0Responses Correct:

179Responses Incorrect:

179Responses Total:

0%Responses - % Correct:

Blog (https://www.pastest.com/blog) About Pastest (https://www.pastest.com/about-us)Contact Us (https://www.pastest.com/contact-us) Help (https://www.pastest.com/help)

© Pastest 2016

Page 67: Back to Filters (/Secure/TestMe/Filter ... - 1 File Download

8/12/2016 MyPastest

https://mypastest.pastest.com/Secure/TestMe/Browser/429893#Top 1/2

Back to Filters (/Secure/TestMe/Filter/429893/QA)

Question 34 of 179

21363

A 45-year-old woman is seen in the Oncology Clinic with end-stage carcinoma of the breast.She has failed various treatments and you are considering treating her with docetaxel.

What is the mode of action of docetaxel?

Explanation

Docetaxel

Docetaxel reversibly binds to microtubules with high affinity

This leads to a decrease in the availability of free tubulin, thus preventing mitotic cell

activity

Microtubules also accumulate within the cell, increasing apoptosis

Docetaxel has a further action in blocking bcl-2

Side-effects are similar to those seen with other chemotherapeutic agents

Alkylating agents are typically used to disrupt DNA

Specific anti-60S anti-ribosomal antibodies are currently under development for

ribosomal disruption in the treatment of cancer

Vitamin E analogues are currently under development for disruption of mitochondrial

function

Disrupting DNAA

Binding to microtubulesB

Inhibiting mitochondrial energy productionC

Inhibiting RNA productionD

Inhibiting ribosome productionE

Next Question

Page 68: Back to Filters (/Secure/TestMe/Filter ... - 1 File Download

8/12/2016 MyPastest

https://mypastest.pastest.com/Secure/TestMe/Browser/429893#Top 2/2

Previous Question Tag Question Feedback End Review

Difficulty: Average

Peer Responses

Session Progress

0Responses Correct:

179Responses Incorrect:

179Responses Total:

0%Responses - % Correct:

Blog (https://www.pastest.com/blog) About Pastest (https://www.pastest.com/about-us)Contact Us (https://www.pastest.com/contact-us) Help (https://www.pastest.com/help)

© Pastest 2016

Page 69: Back to Filters (/Secure/TestMe/Filter ... - 1 File Download

8/12/2016 MyPastest

https://mypastest.pastest.com/Secure/TestMe/Browser/429893#Top 1/2

Back to Filters (/Secure/TestMe/Filter/429893/QA)

Question 35 of 179

You are reviewing a 46-year-old man in the hypertension clinic who has evidence of leftventricular hypertrophy on his resting ECG.

Which of the following medications is most likely to prevent or allow reversal of myocardialhypertrophy?

Explanation

The answer is Ramipril –

Both β-agonists and angiotensin II promote proto-oncogene expression, stimulate proteinsynthesis and induce the synthesis of fetal forms of actin and myosin, thereby leading tohypertrophy of smooth muscle. Clinical data indicate that ACE inhibitors and angiotensinreceptor antagonists prevent and allow reversal of myocardial hypertrophy; dataconcerning β-blockers are less compelling.

Bendroflumethiazide (Option A) is incorrect. Diuretic treatments reduce circulating volumeand give the appearance of improved left ventricular function on echocardiography, but haveno direct effect on myocardial hypertrophy.

Hydrocortisone (Option B) is incorrect. There is an association between excess cortisol andleft ventricular hypertrophy, although the mechanism underlying this is uncertain.

Propranolol (Option C) is incorrect. Beta-blockers may be capable of preventing andreversing myocardial hypertrophy, but option D is a better answer because there is morecompelling evidence in favour of ACE inhibitors.

BendroflumethiazideA

HydrocortisoneB

PropranololC

RamiprilD

ThyroxineE

Page 70: Back to Filters (/Secure/TestMe/Filter ... - 1 File Download

8/12/2016 MyPastest

https://mypastest.pastest.com/Secure/TestMe/Browser/429893#Top 2/2

46598

Thyroxine (Option E) is incorrect. Thyroxine acts directly via nuclear receptors to regulatemyosin heavy-chain gene transcription, and may promote myocardial hypertrophy.

Next Question

Previous Question Tag Question Feedback End Review

Difficulty: Average

Peer Responses

Session Progress

0Responses Correct:

179Responses Incorrect:

179Responses Total:

0%Responses - % Correct:

Blog (https://www.pastest.com/blog) About Pastest (https://www.pastest.com/about-us)Contact Us (https://www.pastest.com/contact-us) Help (https://www.pastest.com/help)

© Pastest 2016

Page 71: Back to Filters (/Secure/TestMe/Filter ... - 1 File Download

8/12/2016 MyPastest

https://mypastest.pastest.com/Secure/TestMe/Browser/429893#Top 1/2

Back to Filters (/Secure/TestMe/Filter/429893/QA)

Question 36 of 179

A 40-year-old obese female teacher is determined to lose weight. She exercises three times aweek at the local gym and is on a slimming diet. In the last month she managed to lose 3 kgin weight and asked your opinion about initiating orlistat therapy.

On advising her, you would explain that orlistat therapy has which one of the followingeffects?

Explanation

The answer is Reduces fat absorption from the intestine –

Orlistat (Xenical®) therapy effectively promotes weight loss and improves co-morbiditiesby preventing absorption of fat molecules from the intestinal tract. Around 30% ofdietary fat that would otherwise have been absorbed passes straight through the boweland is excreted in the faeces. Therefore, it typically causes ‘fatty stools’, increasedfrequency of defaecation, anal leakage or oily spotting. These adverse effects encouragepeople taking the drug to minimise fat intake. Orlistat itself is not absorbed, except invery small quantities, and thus its side-effects are restricted to the gastrointestinal tract.Patients taking orlistat may require concomitant vitamin supplements because ofmalabsorption of fat-soluble vitamins, such as vitamins A, D, K and E, although theeffects on vitamin absorption are substantially less than is seen with cholestyramine.

Causes dramatic weight loss in the first month (Option A) is incorrect. Orlistat is shown to beclinically efficacious in reducing a person’s weight over a period of a year, when administeredalongside other lifestyle measures intended to promote weight loss.

Causes dramatic weight loss in the first monthA

Improves the bone mineral densityB

Increases the cholesterol level in the first year of therapyC

Increases the risk of clottingD

Reduces fat absorption from the intestineE

Page 72: Back to Filters (/Secure/TestMe/Filter ... - 1 File Download

8/12/2016 MyPastest

https://mypastest.pastest.com/Secure/TestMe/Browser/429893#Top 2/2

46630

Improves the bone mineral density (Option B) is incorrect. No effect on bone mineral densityis expected.

Increases the cholesterol level in the first year of therapy (Option C) is incorrect. Studyresults also showed significant improvement in reducing fasting glucose, total cholesterol,LDL-cholesterol and blood pressure.

Increases the risk of clotting (Option D) is incorrect. No significant effect on clotting isexpected; if there is interference with vitamin K absorption, this might cause coagulopathy,although this is rarely seen in clinical practice.

Next Question

Previous Question Tag Question Feedback End Review

Difficulty: Average

Peer Responses

Session Progress

0Responses Correct:

179Responses Incorrect:

179Responses Total:

0%Responses - % Correct:

Blog (https://www.pastest.com/blog) About Pastest (https://www.pastest.com/about-us)Contact Us (https://www.pastest.com/contact-us) Help (https://www.pastest.com/help)

© Pastest 2016

Page 73: Back to Filters (/Secure/TestMe/Filter ... - 1 File Download

8/12/2016 MyPastest

https://mypastest.pastest.com/Secure/TestMe/Browser/429893#Top 1/2

Back to Filters (/Secure/TestMe/Filter/429893/QA)

Question 37 of 179

20935

A 72-year-old woman who was treated by a locum GP after a urinary tract infection asks tosee the emergency doctor after feeling something ‘go’ in her ankle and then complaining ofright foot drop. She is concerned that it may be related to her antibiotic therapy. Onexamination she appears to have a partial rupture of her right Achilles tendon.

Which one of the following antibiotics is the most likely cause of her symptoms?

Explanation

Tendon rupture as a consequence of antibiotic therapy

Extensive randomised controlled trial data to support the association between

quinolone use and tendon rupture are not easily available

However, a number of case control studies suggest an association between quinolone

use and tendon rupture, particularly in an elderly population and where concomitant

steroid use has occurred

These studies demonstrated that in patients using the recommended daily dose of

quinolone, the risk of rupture may be up to 7 times greater than that for controls

CiprofloxacinA

PenicillinB

ErythromycinC

GentamicinD

ClarithromycinE

Next Question

Previous Question Tag Question Feedback End Review

Page 74: Back to Filters (/Secure/TestMe/Filter ... - 1 File Download

8/12/2016 MyPastest

https://mypastest.pastest.com/Secure/TestMe/Browser/429893#Top 2/2

Difficulty: Average

Peer Responses

Session Progress

0Responses Correct:

179Responses Incorrect:

179Responses Total:

0%Responses - % Correct:

Blog (https://www.pastest.com/blog) About Pastest (https://www.pastest.com/about-us)Contact Us (https://www.pastest.com/contact-us) Help (https://www.pastest.com/help)

© Pastest 2016

Page 75: Back to Filters (/Secure/TestMe/Filter ... - 1 File Download

8/12/2016 MyPastest

https://mypastest.pastest.com/Secure/TestMe/Browser/429893#Top 1/2

Back to Filters (/Secure/TestMe/Filter/429893/QA)

Question 38 of 179

46861

A 14-year-old boy has been given cefuroxime for a respiratory tract infection.

Which of the following characteristics is most likely to increase its efficacy in this condition?

Explanation

The answer is It is more effective than cefaclor against Haemophilus influenzae. –

Cefuroxime is a second-generation cephalosporin that is less susceptible to inactivationby β-lactamase than first-generation cephalosporins, including cefaclor, and thereforeexibits greater efficacy against Haemophillus influenzae and also Escherichia coli,Klebsiella spp and Proteus mirabilis, but is less effective against Gram-positiveorganisms.

It fails to cross the blood–brain barrier (Option A) is incorrect. Cefuroxime readily crosses theblood–brain barrier, unlike first-generation cephalosporins.

It is active against Pseudomonas aeruginosa (Option B) is incorrect. It is minimally activeagainst P. aeruginosa.

It is available only in parenteral form (Option C) is incorrect. Cefuroxime may beadministered orally, intramuscularly, or intravenously.

It is the first-line treatment for infection with Gram-negative organisms (Option E) isincorrect. Cefuroxime has limited activity against Gram-negative organisms; third-generationcephalosporins, including cefotaxime and ceftriaxone, have greater activity against Gram-negative organisms.

It fails to cross the blood–brain barrierA

It is active against Pseudomonas aeruginosaB

It is available only in parenteral formC

It is more effective than cefaclor against Haemophilus influenzaeD

It is the first-line treatment for infection with Gram-negative organismsE

Page 76: Back to Filters (/Secure/TestMe/Filter ... - 1 File Download

8/12/2016 MyPastest

https://mypastest.pastest.com/Secure/TestMe/Browser/429893#Top 2/2

Next Question

Previous Question Tag Question Feedback End Review

Difficulty: Average

Peer Responses

Session Progress

0Responses Correct:

179Responses Incorrect:

179Responses Total:

0%Responses - % Correct:

Blog (https://www.pastest.com/blog) About Pastest (https://www.pastest.com/about-us)Contact Us (https://www.pastest.com/contact-us) Help (https://www.pastest.com/help)

© Pastest 2016

Page 77: Back to Filters (/Secure/TestMe/Filter ... - 1 File Download

8/12/2016 MyPastest

https://mypastest.pastest.com/Secure/TestMe/Browser/429893#Top 1/2

Back to Filters (/Secure/TestMe/Filter/429893/QA)

Question 39 of 179

A 32-year-old glass etcher presents to the Emergency Department after being splashed withchemicals at his work. He brought an information leafelet containing details of the liquid,which contains hydrofluoric acid.

Which of the following statements best describes the toxicity of hydrofluoric acid?

Explanation

The answer is May cause skin coagulative necrosis –

Acids cause injury by coagulative necrosis. Acid effects are mainly topical, with corrosiveburns to the skin, mouth, oropharynx and stomach. Aspiration can lead to inflammationand a chemical pneumonitis. Hydrofluoric acid causes significant hypocalcaemia as itbinds calcium, and exposure to even small amounts (topical or ingested) can producesignificant and fatal hypocalcaemia. Calcium gluconate applied both topically andinjected around the burn may be required.

Clinical features after dermal exposure are confined to the affected area only (Option A) isincorrect. Systemic features are common even after a localised skin contact, includingsystemic hypocalcaemia.

Early application of alkaline solutions to affected areas is helpful (Option B) is incorrect.Neutralisation of acids is not appropriate, since this can generate increased heat and soexacerbate any injury sustained.

Hypercalcaemia is a recognised feature (Option C) is incorrect. Hypocalcaemia occurs, nothypercalcaemia.

Clinical features after dermal exposure are confined to the affected area onlyA

Early application of alkaline solutions to affected areas is helpfulB

Hypercalcaemia is a recognised featureC

May cause skin coagulative necrosisD

The exposed area should be treated with heat packsE

Page 78: Back to Filters (/Secure/TestMe/Filter ... - 1 File Download

8/12/2016 MyPastest

https://mypastest.pastest.com/Secure/TestMe/Browser/429893#Top 2/2

46491

The exposed area should be treated with heat packs (Option E) is incorrect. Heat packsshould be avoided because these may increase the severity of local tissue damage.

Next Question

Previous Question Tag Question Feedback End Review

Difficulty: Average

Peer Responses

Session Progress

0Responses Correct:

179Responses Incorrect:

179Responses Total:

0%Responses - % Correct:

Blog (https://www.pastest.com/blog) About Pastest (https://www.pastest.com/about-us)Contact Us (https://www.pastest.com/contact-us) Help (https://www.pastest.com/help)

© Pastest 2016

Page 79: Back to Filters (/Secure/TestMe/Filter ... - 1 File Download

8/12/2016 MyPastest

https://mypastest.pastest.com/Secure/TestMe/Browser/429893#Top 1/2

Back to Filters (/Secure/TestMe/Filter/429893/QA)

Question 40 of 179

You see a 56-year-old patient with type-2 diabetes in the hypertension clinic. He confidesthat he has been suffering from erectile dysfunction, and you consider prescribing sildenafilcitrate treatment.

Which of the following best describes the mode of pharmacological action of sildenafil?

Explanation

The answer is Inhibition of phosphodiesterase V –

Sildenafil citrate is an oral drug that has been approved for the treatment of erectiledysfunction. It is a potent and selective inhibitor of type-V phosphodiesterase, theprimary form of the enzyme found in human penile erectile tissue. Enzyme inhibitionprevents breakdown of cyclic guanosine monophosphate (cGMP), the intracellularsecond messenger of nitric oxide, and thereby promotes nitric oxide-mediatedvasodilation of the penile arterioles. Efficacy has been reported at 40–85%, dependingupon the aetiology and severity of erectile dysfunction. It may interact with nitrates tocause hypotension and syncope; adverse effects include headache, flushing, dyspepsiaand nasal congestion. A small percentage (2–3%) may experience altered colour vision(blue halo), visual brightness, or blurred vision.

Inhibition of α-glycosidase (Option A) is incorrect. α-glycosidase inhibitors include miglitol,which reduces abdominal fat accumulation and reverses some of the features of metabolicsyndrome.

Inhibition of cyclo-oxygenase II (Option B) is incorrect. Cyclo-oxygenase II may be inhibitedby selective agents including celecoxib, or non-selectively by NSAIDs and aspirin.

Inhibition of α-glycosidaseA

Inhibition of cyclo-oxygenase IIB

Inhibition of H+/K+-ATPaseC

Inhibition of phosphodiesterase VD

Inhibition of topoisomerase IE

Page 80: Back to Filters (/Secure/TestMe/Filter ... - 1 File Download

8/12/2016 MyPastest

https://mypastest.pastest.com/Secure/TestMe/Browser/429893#Top 2/2

46806

Inhibition of H+/K+-ATPase (Option C) is incorrect. This is also known as the proton pump,which may be inhibited by protein pump inhibitors, e.g. omeprazole.

Inhibition of topoisomerase I (Option E) is incorrect. This is the target mechanism of certainanticancer agents and quinolone antibiotics.

Next Question

Previous Question Tag Question Feedback End Review

Difficulty: Average

Peer Responses

Session Progress

0Responses Correct:

179Responses Incorrect:

179Responses Total:

0%Responses - % Correct:

Blog (https://www.pastest.com/blog) About Pastest (https://www.pastest.com/about-us)Contact Us (https://www.pastest.com/contact-us) Help (https://www.pastest.com/help)

© Pastest 2016

Page 81: Back to Filters (/Secure/TestMe/Filter ... - 1 File Download

8/12/2016 MyPastest

https://mypastest.pastest.com/Secure/TestMe/Browser/429893#Top 1/2

Back to Filters (/Secure/TestMe/Filter/429893/QA)

Question 41 of 179

36431

A 72-year-old woman comes to the Rheumatology Clinic for review. She has rheumatoidarthritis and you are planning to start golimumab therapy. She tells you she had tuberculosisas a child which was treated successfully and there have been no symptoms since. She isapyrexial and her chest is clear. There is clear evidence of active synovitis affecting her hands,wrists and elbows in particular. A chest x-ray shows evidence of calcification, presumed to beold, but nil else of note.

Which of the following is the optimal TB screening test in this situation?

Explanation

The answer is Interferon gamma release assay

Golimumab is an anti-TNF therapy used in the treatment of rheumatoid arthritis. As suchspecial care has to be taken before starting this treatment with respect to risk of reactivationof tuberculosis or new infection. There is no reported contact risk from tuberculosis, but shehas had previous active TB as a child. As such tuberculin skin testing is likely to evoke astrong reaction if she has active TB or not, and is therefore not very useful. Interferon gammarelease testing is therefore the preferred option.

Sputum cultureA

Mantoux testB

Interferon gamma release assayC

Repeat CXR in 6 weeksD

Bronchoscopy with washingsE

Next Question

Previous Question Tag Question Feedback End Review

Difficulty: Average

Page 82: Back to Filters (/Secure/TestMe/Filter ... - 1 File Download

8/12/2016 MyPastest

https://mypastest.pastest.com/Secure/TestMe/Browser/429893#Top 2/2

Peer Responses

Session Progress

0Responses Correct:

179Responses Incorrect:

179Responses Total:

0%Responses - % Correct:

Blog (https://www.pastest.com/blog) About Pastest (https://www.pastest.com/about-us)Contact Us (https://www.pastest.com/contact-us) Help (https://www.pastest.com/help)

© Pastest 2016

Page 83: Back to Filters (/Secure/TestMe/Filter ... - 1 File Download

8/12/2016 MyPastest

https://mypastest.pastest.com/Secure/TestMe/Browser/429893#Top 1/2

Back to Filters (/Secure/TestMe/Filter/429893/QA)

Question 42 of 179

You are reviewing some blood results that have been telephoned to the ward from thelaboratory showing lowered serum potassium of 2.7 mmol/l. Serum sodium and creatinine arewithin normal limits. You review the case notes and find out that the patient is a 76-year-oldman with a history of congestive heart failure and atrial fibrillation.

Which of the following drugs would be most likely to have caused hypokalaemia?

Explanation

The answer is Theophylline –

Several drugs can alter serum potassium concentrations. Hypokalaemia may be causedby shifts of potassium from extracellular to intracellular compartments with normal totalbody potassium, e.g. theophylline, salbutamol, caffeine and insulin. Total body potassiummay become deplete with chronic diuretic use. Hyperkalaemia may be caused bypotassium shifting from the intracellular to extracellular compartment (e.g. β-blockers,metabolic acidosis, insulin insufficiency, digoxin), reduced renal excretion (e.g.angiotensin-converting enzyme inhibitors, angiotensin receptor blockers, lithium).

ACE inhibitors (Option A) is incorrect. ACE inhibitors may impair renal excretion ofpotassium and cause hyperkalaemia.

Beta-blockers (Option B) is incorrect. Beta-blockers may cause hyperkalaemia.

Digoxin (Option C) is incorrect. Digoxin inhibits the sodium–potassium ATPase enzyme,thereby increasing serum potassium and decreasing intracellular potassium.

ACE inhibitorsA

Beta-blockersB

DigoxinC

LithiumD

TheophyllineE

Page 84: Back to Filters (/Secure/TestMe/Filter ... - 1 File Download

8/12/2016 MyPastest

https://mypastest.pastest.com/Secure/TestMe/Browser/429893#Top 2/2

46383

Lithium (Option D) is incorrect. Lithium may impair renal excretion of potassium and causehyperkalaemia.

Next Question

Previous Question Tag Question Feedback End Review

Difficulty: Average

Peer Responses

Session Progress

0Responses Correct:

179Responses Incorrect:

179Responses Total:

0%Responses - % Correct:

Blog (https://www.pastest.com/blog) About Pastest (https://www.pastest.com/about-us)Contact Us (https://www.pastest.com/contact-us) Help (https://www.pastest.com/help)

© Pastest 2016

Page 85: Back to Filters (/Secure/TestMe/Filter ... - 1 File Download

8/12/2016 MyPastest

https://mypastest.pastest.com/Secure/TestMe/Browser/429893#Top 1/2

Back to Filters (/Secure/TestMe/Filter/429893/QA)

Question 43 of 179

46942

A 74-year-old woman is admitted via the emergency department with palpitations andbreathlessness and found to have atrial fibrillation with ventricular rate 110 per min. Youdecide to commence an IV infusion of amiodarone.

What is the predominant mode of action of amiodarone?

Explanation

The answer is Potassium channel blockade –

Amiodarone is a class III antiarrhythmic drug; drugs in this class exert theirpharmacological actions through potassium channel blockade. Others include sotalol.They prolong the duration of the action potential with resulting prolongation of theeffective refractory period.

Alpha-adrenoreceptor blockade (Option A) is incorrect. Alpha-adrenoceptors do not conferanti-arrhythmic properties.

Beta-adrenoreceptor blockade (Option B) is incorrect. Beta-blockers possess Class IIantiarrhythmic properties, and reduce adrenergic-mediated tachycardia.

Calcium channel blockade (Option C) is incorrect. Calcium channel blockers exert Class IVantiarrhythmic properties and delay AV nodal conduction.

Sodium channel blockade (Option E) is incorrect. Sodium channel blockers exert Class Iantiarrhythmic effects, e.g. lidocaine.

Alpha-adrenoreceptor blockadeA

Beta-adrenoreceptor blockadeB

Calcium channel blockadeC

Potassium channel blockadeD

Sodium channel blockadeE

Page 86: Back to Filters (/Secure/TestMe/Filter ... - 1 File Download

8/12/2016 MyPastest

https://mypastest.pastest.com/Secure/TestMe/Browser/429893#Top 2/2

Next Question

Previous Question Tag Question Feedback End Review

Difficulty: Average

Peer Responses

Session Progress

0Responses Correct:

179Responses Incorrect:

179Responses Total:

0%Responses - % Correct:

Blog (https://www.pastest.com/blog) About Pastest (https://www.pastest.com/about-us)Contact Us (https://www.pastest.com/contact-us) Help (https://www.pastest.com/help)

© Pastest 2016

Page 87: Back to Filters (/Secure/TestMe/Filter ... - 1 File Download

8/12/2016 MyPastest

https://mypastest.pastest.com/Secure/TestMe/Browser/429893#Top 1/2

Back to Filters (/Secure/TestMe/Filter/429893/QA)

Question 44 of 179

You review a 54-year-old woman who has been referred by her GP with elevated AST andALT to three times the upper limit of normal, accompanied by small rises in bilirubin andalkaline phosphatase. Antinuclear antibodies are negative. She admits to drinking anoccasional glass of wine with her husband. She has a family history of mixed hyperlipidaemiafor which she takes fenofibrate.

What diagnosis fits best with this clinical history?

Explanation

The answer is Drug-induced hepatitis –

Fenofibrate is a recognised cause of drug-induced hepatitis, characterised by raisedtransaminase levels and smaller rise in bilirubin and alkaline phosphatase. Other drugcauses of a chronic hepatitis biochemical picture include methyldopa, carbamazepine,nitrofurantoin and isoniazid. Niacin promotes increased high-density lipoprotein (HDL)cholesterol and may cause flushing and vasodilatation; an acute hepatitis biochemicalpicture is a rare but recognised complication.

Autoimmune chronic active hepatitis (Option A) is incorrect. The negative antinuclearantibody test makes autoimmune hepatitis less likely in this case.

Chronic alcoholism (Option B) is incorrect. Alcoholic liver disease is less likely in the absenceof heavier patterns of drinking and the absence of signs of chronic liver disease.

Haemochromatosis presenting after the menopause (Option D) is incorrect.Haemochromatosis might be associated with other clinical signs such as hyperpigmentation,arthropathy or type-2 diabetes/impaired glucose tolerance.

Autoimmune chronic active hepatitisA

Chronic alcoholismB

Drug-induced hepatitisC

Haemochromatosis presenting after the menopauseD

Wilson’s diseaseE

Page 88: Back to Filters (/Secure/TestMe/Filter ... - 1 File Download

8/12/2016 MyPastest

https://mypastest.pastest.com/Secure/TestMe/Browser/429893#Top 2/2

46879

Wilson’s disease (Option E) is incorrect. Wilson’s disease generally presents at a youngerage.

Next Question

Previous Question Tag Question Feedback End Review

Difficulty: Average

Peer Responses

Session Progress

0Responses Correct:

179Responses Incorrect:

179Responses Total:

0%Responses - % Correct:

Blog (https://www.pastest.com/blog) About Pastest (https://www.pastest.com/about-us)Contact Us (https://www.pastest.com/contact-us) Help (https://www.pastest.com/help)

© Pastest 2016

Page 89: Back to Filters (/Secure/TestMe/Filter ... - 1 File Download

8/12/2016 MyPastest

https://mypastest.pastest.com/Secure/TestMe/Browser/429893#Top 1/2

Back to Filters (/Secure/TestMe/Filter/429893/QA)

Question 45 of 179

A 60-year-old woman with a past history of hypertension has been receiving a number ofdifferent cardiac medications. She presents to the Emergency Department at 0200 h after asuspected drug overdose. She smells strongly of alcohol, and is found to have heart rate 48bpm and blood pressure 90/70 mmHg. An ECG shows sinus bradycardia with first-degreeheart block and prolongation of the PR interval.

What class of drugs is most likely to cause these effects?

Explanation

The answer is ß-blockers –

The patient’s presentation with drowsiness, confusion, low pulse rate (48 bpm), lowsystolic blood pressure (90/70 mmHg), first-degree heart block and prolongation of thePR interval is typical of ß-blocker toxicity.

Class III antiarrhythmic drugs (amiodarone) (Option B) is incorrect. Amiodarone may causeQT interval prolongation, although amiodarone overdose would rarely cause acutebradyarrhythmia.

Digitalis glycosides (Option C) is incorrect. Digoxin toxicity may cause bradyarrhythmia andis capable of prolonging the PR interval, but the clinical picture is usually dominated bysevere nausea and vomiting.

Phenothiazines (Option D) is incorrect. Phenothiazines may cause prolongation of the QTinterval, which may predispose to torsade de pointes arrhythmia (where the ECG looks likeventricular tachycardia with a varying axis).

Beta-blockersA

Class III antiarrhythmic drugs (amiodarone)B

Digitalis glycosidesC

PhenothiazinesD

Tricyclic antidepressantsE

Page 90: Back to Filters (/Secure/TestMe/Filter ... - 1 File Download

8/12/2016 MyPastest

https://mypastest.pastest.com/Secure/TestMe/Browser/429893#Top 2/2

46765

Tricyclic antidepressants (Option E) is incorrect. Tricyclic antidepressants may causearrhythmia caused by sodium channel blockade, which may be detected by QRSprolongation on the ECG.

Next Question

Previous Question Tag Question Feedback End Review

Difficulty: Average

Peer Responses

Session Progress

0Responses Correct:

179Responses Incorrect:

179Responses Total:

0%Responses - % Correct:

Blog (https://www.pastest.com/blog) About Pastest (https://www.pastest.com/about-us)Contact Us (https://www.pastest.com/contact-us) Help (https://www.pastest.com/help)

© Pastest 2016

Page 91: Back to Filters (/Secure/TestMe/Filter ... - 1 File Download

8/12/2016 MyPastest

https://mypastest.pastest.com/Secure/TestMe/Browser/429893#Top 1/2

Back to Filters (/Secure/TestMe/Filter/429893/QA)

Question 46 of 179

46606

You are reviewing a 46-year-old patient in the renal transplant clinic. He has been receivingciclosporin for the past 18 months.

Which of the following is most likely to represent an adverse effect of ciclosporin treatment?

Explanation

The answer is Chronic interstitial nephritis –

The main use of ciclosporin is to reduce T-cell immune function in transplant patients andso reduce the chances of rejection (liver, renal, pancreas, heart and bone marrowtransplants). Ciclosporin has a narrow therapeutic index and requires therapeutic drugmonitoring: renal toxicity is more common with high drug concentrations (> 200 ng/ml),whereas rejection is common with low serum levels. Adverse effects include chronicinterstitial nephritis, hyperkalaemia and hypertrichosis. It is nephrotoxic, but virtuallynon-myelotoxic.

Alopecia (Option A) is incorrect. Hypertrichosis is a common side-effect of ciclosporin,rather than alopecia.

Myelosuppression (Option C) is incorrect. In contrast to other immunosuppressive agents,ciclosporin does not cause significant myelosuppression.

Stomatitis (Option D) is incorrect. Stomatitis is not a recognised adverse effect ofciclosporin.

Urolithiasis (Option E) is incorrect. Urolithiasis is not a recognised adverse effect ofciclosporin.

AlopeciaA

Chronic interstitial nephritisB

MyelosuppressionC

StomatitisD

UrolithiasisE

Page 92: Back to Filters (/Secure/TestMe/Filter ... - 1 File Download

8/12/2016 MyPastest

https://mypastest.pastest.com/Secure/TestMe/Browser/429893#Top 2/2

46606

Next Question

Previous Question Tag Question Feedback End Review

Difficulty: Average

Peer Responses

Session Progress

0Responses Correct:

179Responses Incorrect:

179Responses Total:

0%Responses - % Correct:

Blog (https://www.pastest.com/blog) About Pastest (https://www.pastest.com/about-us)Contact Us (https://www.pastest.com/contact-us) Help (https://www.pastest.com/help)

© Pastest 2016

Page 93: Back to Filters (/Secure/TestMe/Filter ... - 1 File Download

8/12/2016 MyPastest

https://mypastest.pastest.com/Secure/TestMe/Browser/429893#Top 1/2

Back to Filters (/Secure/TestMe/Filter/429893/QA)

Question 47 of 179

You are asked to review a patient on the psychiatric ward, who has been receiving intensivein-patient treatment. He has developed fever, rigidity and altered consciousness, togetherwith tachycardia and intermittent hypotension.

Which one of the following medications would be the most appropriate treatment?

Explanation

The answer is Dantrolene –

The clinical features are indicative of neuroleptic malignant syndrome, a rare butpotentially very serious reaction to antipsychotic medications that is characterised byfever, rigidity and myoclonus, and altered consciousness, often accompanied bytachycardia, autonomic instability and pyrexia. Investigations normally show leukocytosisand markedly elevated creatine kinase. The causative drug should be withdrawn andsupportive care given to manage cardiovascular, respiratory and renal complications.Dantrolene may be helpful in treating agitation, and rigidity, particularly if patients havea high fever. Alternatively, benzodiazepines may be used to reduce agitation and musclerigidity, and high doses are often required.

Atropine (Option A) is incorrect. Atropine would aggravate tachycardia and worsen delirium.

Beta-blocker (Option B) is incorrect. Beta-blockers may be considered if tachycardiapredisposes to a myocardial ischaemia in patients with known coronary artery disease.However, it is rarely required, as heart rate normally resolves after administration of sufficientdoses of benzodiazepines.

AtropineA

Beta-blockerB

DantroleneC

MetoclopramideD

SelegilineE

Page 94: Back to Filters (/Secure/TestMe/Filter ... - 1 File Download

8/12/2016 MyPastest

https://mypastest.pastest.com/Secure/TestMe/Browser/429893#Top 2/2

46796

Metoclopramide (Option D) is incorrect. Metoclopramide should be avoided as this may itselfcause acute dystonia and worsen muscle rigidity.

Selegiline (Option E) is incorrect. Selegeline is unlikely to be of benefit and should beavoided because it is an independent cause of acute dystonia.

Next Question

Previous Question Tag Question Feedback End Review

Difficulty: Average

Peer Responses

Session Progress

0Responses Correct:

179Responses Incorrect:

179Responses Total:

0%Responses - % Correct:

Blog (https://www.pastest.com/blog) About Pastest (https://www.pastest.com/about-us)Contact Us (https://www.pastest.com/contact-us) Help (https://www.pastest.com/help)

© Pastest 2016

Page 95: Back to Filters (/Secure/TestMe/Filter ... - 1 File Download

8/12/2016 MyPastest

https://mypastest.pastest.com/Secure/TestMe/Browser/429893#Top 1/2

Back to Filters (/Secure/TestMe/Filter/429893/QA)

Question 48 of 179

A 62-year-old man attends the clinic complaining of lethargy. He has recently been started onsome medication for control of blood pressure but unfortunately he cannot remember itsname. On examination his blood pressure is 142/86 mmHg. His BMI is 22 kg/m .

Investigations:

Hb 12.1 g/dl

White cell count 4.7 × 10 /l

Platelets 193 × 10 /l

Na 124 mmol/l

K 3.6 mmol/l

Creatinine 90 μmol/l

Which one of the following agents is most likely to be responsible?

Explanation

Drug-induced electrolyte imbalance

Factors associated with thiazide-induced hyponatraemia include increased patient age,

decreased body mass and decreased serum potassium

In this case it would seem most appropriate to switch this patient to a calcium channel

antagonist

2

9

9

+

+

RamiprilA

SpironolactoneB

BendroflumethiazideC

AmilorideD

FurosemideE

Page 96: Back to Filters (/Secure/TestMe/Filter ... - 1 File Download

8/12/2016 MyPastest

https://mypastest.pastest.com/Secure/TestMe/Browser/429893#Top 2/2

20940

Amiloride and spironolactone are associated with hyperkalaemia

both act at receptors in the distal convoluted tubule and promote sodium and

water excretion, with conservation of potassium

Furosemide promotes excretion of sodium and water and is less commonly associated

with hyponatraemia compared with thiazides

Next Question

Previous Question Tag Question Feedback End Review

Difficulty: Average

Peer Responses

Session Progress

0Responses Correct:

179Responses Incorrect:

179Responses Total:

0%Responses - % Correct:

Blog (https://www.pastest.com/blog) About Pastest (https://www.pastest.com/about-us)Contact Us (https://www.pastest.com/contact-us) Help (https://www.pastest.com/help)

© Pastest 2016

Page 97: Back to Filters (/Secure/TestMe/Filter ... - 1 File Download

8/12/2016 MyPastest

https://mypastest.pastest.com/Secure/TestMe/Browser/429893#Top 1/2

Back to Filters (/Secure/TestMe/Filter/429893/QA)

Question 49 of 179

A 67-year-old man on warfarin for atrial fibrillation presents with epistaxis. On examination heis tachycardic with a BP of 95/60 mmHg, and requires emergency treatment from the Ear,Nose and Throat Service, but there has been ongoing nasal bleeding despite attemptedcauterisation. The INR on admission is 10.7 and the last known measurement was more than 3months earlier.

Which of the following would be the most appropriate management?

Explanation

The answer is option Prothrombin complex concentrate and vitamin K –

An INR around 8.7 accompanied by active bleeding indicates that urgent treatment isrequired. Prothrombin complex concentrate (PCC) is an inactivated concentrate offactors II, IX and X, and variable amounts of factor VII. Fresh frozen plasma (FFP) is aplasma-derived blood product containing all clotting factors and fibrinogen; problemsinclude the time taken to thaw specimens prior to treatment (around 1 h) and risk oftransfusion reactions. PCC is lyophilised and can be reconstituted immediately, andcontains around 25-fold higher concentrations of vitamin K-dependent factors than FFP.Treatment results in rapid and effective reversal of INR.

Cryoprecipitate (Option A) is incorrect. Cryoprecipitate is the plasma fraction that isextracted from FFP that is rich in fibrinogen, von Willebrand factor, and factors VIII and XIII.It may be used to restore haemostasis in patients with very low fibrinogen concentrations. Itcontains insufficient vitamin K-dependent factors to effectively reverse warfarin-relatedbleeding.

CryoprecipitateA

DesmopressinB

Prothrombin complex concentrate and vitamin KC

Tranexamic acidD

Vitamin KE

Page 98: Back to Filters (/Secure/TestMe/Filter ... - 1 File Download

8/12/2016 MyPastest

https://mypastest.pastest.com/Secure/TestMe/Browser/429893#Top 2/2

46614

Desmopressin (Option B) is incorrect. Desmopressin stimulates transient release of vonWillebrand factor by endothelial cells by blocking vasopressin V2 receptors, and is used inthe treatment of von Willebrand disease.

Tranexamic acid (Option D) is incorrect. Tranexamic acid is an antifibrinolytic agent and canbe used in the treatment of haemophilia and von Willebrand’s disease.

Vitamin K (Option E) is incorrect. Vitamin K (phytomenadione) is a useful adjunct butreversal of warfarin may take many hours up to several days, and is inadequate alone in thesetting of active bleeding. Reversal by vitamin K may last for several weeks, which can makereintroduction of warfarin more complicated.

Next Question

Previous Question Tag Question Feedback End Review

Difficulty: Average

Peer Responses

Session Progress

0Responses Correct:

179Responses Incorrect:

179Responses Total:

0%Responses - % Correct:

Blog (https://www.pastest.com/blog) About Pastest (https://www.pastest.com/about-us)Contact Us (https://www.pastest.com/contact-us) Help (https://www.pastest.com/help)

© Pastest 2016

Page 99: Back to Filters (/Secure/TestMe/Filter ... - 1 File Download

8/12/2016 MyPastest

https://mypastest.pastest.com/Secure/TestMe/Browser/429893#Top 1/2

Back to Filters (/Secure/TestMe/Filter/429893/QA)

Question 50 of 179

A 43-year-old man with known alcoholic liver disease and cirrhosis is admitted via theEmergency Department after a dramatic decline in his functional status, and is found to haveacute hepatic encephalopathy. His partner tells you that his GP gave him a new medicationjust a few days earlier.

Which of the following drugs is most likely to have precipitated encephalopathy in thispatient?

Explanation

The answer is Diazepam –

The presence of liver disease may influence pharmacodynamic responses to certaindrugs by several mechanisms. Patients with liver cirrhosis are more susceptible to theeffects of various sedative medications on the central nervous system includingbenzodiazepines, opioids, barbiturates and chlorpromazine.

Amlodipine (Option A) is incorrect. Amlodipine and other calcium-channel blockers areextensively metabolised by the liver, and may accumulate in liver disease to causehypotension and peripheral oedema.

Folic acid (Option C) is incorrect. Folate would not be expected to worsen liver disease orprovoke encephalopathy.

Thiamine (Option D) is incorrect. Thiamine is helpful for preventing Wernicke’sencephalopathy in patients with a history of alcohol excess or malnutrition.

AmlodipineA

DiazepamB

Folic acidC

ThiamineD

Vitamin EE

Page 100: Back to Filters (/Secure/TestMe/Filter ... - 1 File Download

8/12/2016 MyPastest

https://mypastest.pastest.com/Secure/TestMe/Browser/429893#Top 2/2

46797

Vitamin E (Option E) is incorrect. Vitamin E is subject to hepatic metabolism to active formsincluding alpha-tocopherol; deficiency may cause ataxia.

Next Question

Previous Question Tag Question Feedback End Review

Difficulty: Average

Peer Responses

Session Progress

0Responses Correct:

179Responses Incorrect:

179Responses Total:

0%Responses - % Correct:

Blog (https://www.pastest.com/blog) About Pastest (https://www.pastest.com/about-us)Contact Us (https://www.pastest.com/contact-us) Help (https://www.pastest.com/help)

© Pastest 2016

Page 101: Back to Filters (/Secure/TestMe/Filter ... - 1 File Download

8/12/2016 MyPastest

https://mypastest.pastest.com/Secure/TestMe/Browser/429893#Top 1/2

Back to Filters (/Secure/TestMe/Filter/429893/QA)

Question 51 of 179

46807

You are asked to prescribe a course of antibiotics for a patient with suspected Chlamydiapneumoniae infection.

What is the mechanism of Chlamydia’s resistance to cephalosporins?

Explanation

The answer is Chlamydia has no peptidoglycan cell wall –

The β-lactams (penicillins, cephalosporins and monobactams, e.g. aztreonam) andglycopeptides (vancomycin and teicoplanin) exert antimicrobial actions via inhibition ofcell wall synthesis. Chlamydia lacks a cell wall and, therefore, these antimicrobials areineffective.

Actively pumps cephalosporins out (Option A) is incorrect. This mechanism is not relevant.

Chlamydia is an extracellular organism (Option C) is incorrect. Chamydia is an intracellularorganism that lacks its own cell wall, and relies upon the host cell for protection.

Forms cephalosporin-binding proteins (Option D) is incorrect. This mechanism is notrelevant.

Hydrolysation of the β-lactam ring (Option E) is incorrect. The β-lactam antibiotics share acommon β-lactam ring. Certain antimicrobials secrete enzymes capable of hydrolysing thisring (penicillinase), which is responsible for treatment resistance.

Actively pumps cephalosporins outA

Chlamydia has no peptidoglycan cell wallB

Chlamydia is an extracellular organismC

Forms cephalosporin-binding proteinsD

Hydrolysation of the β-lactam ringE

Next Question

Page 102: Back to Filters (/Secure/TestMe/Filter ... - 1 File Download

8/12/2016 MyPastest

https://mypastest.pastest.com/Secure/TestMe/Browser/429893#Top 2/2

Previous Question Tag Question Feedback End Review

Difficulty: Average

Peer Responses

Session Progress

0Responses Correct:

179Responses Incorrect:

179Responses Total:

0%Responses - % Correct:

Blog (https://www.pastest.com/blog) About Pastest (https://www.pastest.com/about-us)Contact Us (https://www.pastest.com/contact-us) Help (https://www.pastest.com/help)

© Pastest 2016

Page 103: Back to Filters (/Secure/TestMe/Filter ... - 1 File Download

8/12/2016 MyPastest

https://mypastest.pastest.com/Secure/TestMe/Browser/429893#Top 1/2

Back to Filters (/Secure/TestMe/Filter/429893/QA)

Question 52 of 179

A 32-year-old car mechanic presents to the Emergency Department by paramedicambulance. He was found by his wife to be lying on the floor of his workshop, drowsy,disorientated and hyperventilating. There have apparently been problems with money athome and she is worried that he has taken an overdose. On examination he appearsintoxicated although there is no smell of alcohol, his BP is 90/60 mmHg and his pulse is95/min and regular. His chest is clear although his respiratory rate is elevated at 32/min. IVnormal saline is commenced.

Investigations;

Hb 13.2 g/dl

WCC 10.4 x10 /l

PLT 245 x10 /l

Na 136 mmol/l

K 4.9 mmol/l

Bicarbonate 14 mmol/l

Cl 104 mmol/l

Creatinine 138 micromol/l

Which of the following is the most effective intervention?

9

9

+

+

-

Forced alkaline diuresisA

Gastric lavageB

IV FomepizoleC

IV N-acetylcysteineD

IV Sodium bicarbonateE

Page 104: Back to Filters (/Secure/TestMe/Filter ... - 1 File Download

8/12/2016 MyPastest

https://mypastest.pastest.com/Secure/TestMe/Browser/429893#Top 2/2

38089

Explanation

The answer is IV Fomepizole -

This patient works in a garage and is therefore likely to have ready access to ethylene glycol.The metabolic acidosis with raised anion gap, and the intoxication without an obviousalcoholic odour further support the diagnosis. IV Fomepizole, or 4-methylpyrazole is acompetitive inhibitor of alcohol dehydrogenase and therefore prevents metabolism ofethylene glycol to oxalic acid, responsible for the acidosis and renal failure associated withethylene glycol poisoning. IV ethanol can be used as an alternative although IV ethanol willof course lead to a worsening of intoxication. IV Sodium bicarbonate is only utilised in severeacidosis. IV N-acetylcysteine is used in paracetamol overdose. Forced alkaline diuresis wasformerly used in the treatment of aspirin overdose, but is now not widely recommended.

Next Question

Previous Question Tag Question Feedback End Review

Difficulty: Difficult

Peer Responses

Session Progress

0Responses Correct:

179Responses Incorrect:

179Responses Total:

0%Responses - % Correct:

Blog (https://www.pastest.com/blog) About Pastest (https://www.pastest.com/about-us)Contact Us (https://www.pastest.com/contact-us) Help (https://www.pastest.com/help)

© Pastest 2016

Page 105: Back to Filters (/Secure/TestMe/Filter ... - 1 File Download

8/12/2016 MyPastest

https://mypastest.pastest.com/Secure/TestMe/Browser/429893#Top 1/2

Back to Filters (/Secure/TestMe/Filter/429893/QA)

Question 53 of 179

46824

You are reviewing a 75-year-old woman with mild dementia in the outpatient clinic. Afterdiscussion with the family you decide to administer donepezil.

Which of the following side-effects should be looked out for?

Explanation

The answer is Bradycardia –

Donepezil is a cholinesterase inhibitor, and as such potentiates the actions of cholinergicneurones. Adverse effects include nausea, vomiting and diarrhoea; cardiac side-effectsinclude bradycardia and, rarely, AV block. Less common effects include urinaryincontinence and hepatitis.

Atrial tachycardias (Option A) is incorrect. Bradycardia is a characteristic feature, nottachycardia.

Constipation (Option C) is incorrect. Gastrointestinal disturbance typically results indiarrhoea, not constipation.

Excessive somnolence (Option D) is incorrect. Donepezil does not confer significantsedation.

Nephritis (Option E) is incorrect. Nephritis is not a recognised feature.

Atrial tachycardiasA

BradycardiaB

ConstipationC

Excessive somnolenceD

NephritisE

Next Question

Page 106: Back to Filters (/Secure/TestMe/Filter ... - 1 File Download

8/12/2016 MyPastest

https://mypastest.pastest.com/Secure/TestMe/Browser/429893#Top 2/2

Previous Question Tag Question Feedback End Review

Difficulty: Average

Peer Responses

Session Progress

0Responses Correct:

179Responses Incorrect:

179Responses Total:

0%Responses - % Correct:

Blog (https://www.pastest.com/blog) About Pastest (https://www.pastest.com/about-us)Contact Us (https://www.pastest.com/contact-us) Help (https://www.pastest.com/help)

© Pastest 2016

Page 107: Back to Filters (/Secure/TestMe/Filter ... - 1 File Download

8/12/2016 MyPastest

https://mypastest.pastest.com/Secure/TestMe/Browser/429893#Top 1/2

Back to Filters (/Secure/TestMe/Filter/429893/QA)

Question 54 of 179

46777

A 17-year-old man has been diagnosed with tonsillitis and prescribed an antibiotic by his GP.Three days later he attends the Emergency Department with a blotchy, non-pruritic purpuricrash over his trunk, arms, and legs.

Which of the following antibiotics is most likely to have caused this?

Explanation

The answer is Ampicillin –

Ampicillin, amoxicillin and other penicillins characteristically cause maculopapular skinrashes. These are not allergic in nature, and may be more pronounced in patients withviral illness, including when tonsillitis or pharyngitis is caused by Epstein–Barr infection.Penicillins may also cause allergic reactions with a characteristic urticarial rash, and fixeddrug eruptions.

Benzylpenicillin (Option B) is incorrect. Benzylpenicillin is less likely to cause diffuse rashthan ampicillin or amoxicillin, and it is normally administered only by intramuscular orintravenous routes of administration.

Cefadroxil (Option C) is incorrect. Cefuroxime and cefadroxil may produce an intensely itchy,diffuse, maculopapular rash, but less commonly than after penicillin.

Cefuroxime (Option D) is incorrect. Cefuroxime and cefadroxil may produce an intenselyitchy, diffuse, maculopapular rash, but less commonly than after penicillin.

Erythromycin (Option E) is incorrect. Erythromycin may cause a diffuse maculopapular rashbut less commonly than after penicillins.

AmpicillinA

BenzylpenicillinB

CefadroxilC

CefuroximeD

ErythromycinE

Page 108: Back to Filters (/Secure/TestMe/Filter ... - 1 File Download

8/12/2016 MyPastest

https://mypastest.pastest.com/Secure/TestMe/Browser/429893#Top 2/2

46777

Next Question

Previous Question Tag Question Feedback End Review

Difficulty: Average

Peer Responses

Session Progress

0Responses Correct:

179Responses Incorrect:

179Responses Total:

0%Responses - % Correct:

Blog (https://www.pastest.com/blog) About Pastest (https://www.pastest.com/about-us)Contact Us (https://www.pastest.com/contact-us) Help (https://www.pastest.com/help)

© Pastest 2016

Page 109: Back to Filters (/Secure/TestMe/Filter ... - 1 File Download

8/12/2016 MyPastest

https://mypastest.pastest.com/Secure/TestMe/Browser/429893#Top 1/2

Back to Filters (/Secure/TestMe/Filter/429893/QA)

Question 55 of 179

You are planning a clinical phase-II study in oncology, testing a new chemotherapy in patientswith a malignant tumour. It is a randomised, placebo-controlled, double-blind study. A total of100 patients will receive standard therapy and 100 patients will receive the new substanceover a 5-year treatment duration.

Which statistical test is most appropriate to compare the survival times between the twogroups?

Explanation

The answer is Log-rank test –

The log-rank test is used to examine the null hypothesis that the survival curves in thetwo groups are the same. That is to say that the test can demonstrate the likelihood thatsurvival in the two groups across the study period is similar; if very unlikely (eg a lowprobability or P-value), then the null hypothesis can be rejected and a statisticaldifference in survival claimed between the two groups.

Analysis of variance (ANOVA) (Option A) is incorrect. ANOVA tests examine whether themeans differ between multiple sets of data; it is similar to a t-test, but can consider morethan two groups. This is a less powerful test than the log-rank test.

Chi-square test (Option B) is incorrect. The chi-square test is used to evaluate the proportionof events in one group versus another group; eg this might be used in a simplistic way topresent the proportion of survivors in group A versus group B at a particular time point.However, it may be difficult to define the proportions clearly due to the effect of censoring,

Analysis of variance (ANOVA)A

Chi-square testB

Log-rank testC

Paired t-testD

Unpaired t-testE

Page 110: Back to Filters (/Secure/TestMe/Filter ... - 1 File Download

8/12/2016 MyPastest

https://mypastest.pastest.com/Secure/TestMe/Browser/429893#Top 2/2

46792

and the choice of time point is somewhat arbitrary. The log-rank test gives a comparison ofsurvival across the entire study period.

Paired t-test (Option D) is incorrect. A paired t-test is too simple a measure for comparisonbetween two matched groups, eg a before/after comparison in a cross-over study design.Individual patient matching is used to ensure similar important characteristics between thetwo groups; this might for instance be determined to be age, gender, cancer stage and so on.A t-test might be used to compare a single measure in one group versus the other, but acaveat is that the variable would need to demonstrate normal distribution; many biologicalvariables do not, eg creatinine.

Unpaired t-test (Option E) is incorrect. An unpaired t-test compares only a single variablebetween two unlinked groups, and requires the variable to be distributed normally in bothgroups. As with the paired t-test, this is too simplistic a measure to be useful in comparingsurvival.

Next Question

Previous Question Tag Question Feedback End Review

Difficulty: Average

Peer Responses

Session Progress

0Responses Correct:

179Responses Incorrect:

179Responses Total:

0%Responses - % Correct:

Blog (https://www.pastest.com/blog) About Pastest (https://www.pastest.com/about-us)Contact Us (https://www.pastest.com/contact-us) Help (https://www.pastest.com/help)

© Pastest 2016

Page 111: Back to Filters (/Secure/TestMe/Filter ... - 1 File Download

8/12/2016 MyPastest

https://mypastest.pastest.com/Secure/TestMe/Browser/429893#Top 1/2

Back to Filters (/Secure/TestMe/Filter/429893/QA)

Question 56 of 179

21361

A 26-year-old who has developed a long-standing addiction to heroin which began 4 yearsearlier whilst travelling visits you for advice. He has tried going ‘cold turkey’ on a number ofoccasions but develops unacceptable restlessness, anxiety, vomiting and diarrhoea. He nowhas a child and is determined to stop. There is a place available on the local drug counsellingscheme.

Which one of the following is the most appropriate prescription with respect to medicallymanaging his withdrawal?

Explanation

Heroin substitutes

Both buprenorphine and methadone may be considered for use as heroin replacements

Buprenorphine may be associated with less risk in overdose, but NICE recommends

that unless circumstances dictate otherwise, methadone should be the first-choice

therapy

Co-abuse of alcohol and benzodiazepines may drive preferential use of buprenorphine,

as these agents increase the risk of significant CNS depression

Dihydrocodeine is not indicated for opiate withdrawal in the UK

Diazepam is not indicated for opiate replacement, it is added to other therapies in

some patients to alleviate symptoms of withdrawal, although it may actually broaden

the scope of any addictive behaviour

Atypical anti-psychotics are preferred in this situation to traditional agents such as

chlorpromazine

BuprenorphineA

MethadoneB

DiazepamC

DihydrocodeineD

ChlorpromazineE

Page 112: Back to Filters (/Secure/TestMe/Filter ... - 1 File Download

8/12/2016 MyPastest

https://mypastest.pastest.com/Secure/TestMe/Browser/429893#Top 2/2

21361

Next Question

Previous Question Tag Question Feedback End Review

Difficulty: Easy

Peer Responses

Session Progress

0Responses Correct:

179Responses Incorrect:

179Responses Total:

0%Responses - % Correct:

Blog (https://www.pastest.com/blog) About Pastest (https://www.pastest.com/about-us)Contact Us (https://www.pastest.com/contact-us) Help (https://www.pastest.com/help)

© Pastest 2016

Page 113: Back to Filters (/Secure/TestMe/Filter ... - 1 File Download

8/12/2016 MyPastest

https://mypastest.pastest.com/Secure/TestMe/Browser/429893#Top 1/2

Back to Filters (/Secure/TestMe/Filter/429893/QA)

Question 57 of 179

46901

A 57-year-old presents with a subacute history of breathlessness. Following a CT scan of hischest, a diagnosis of pulmonary fibrosis is made and he is referred to you in the respiratoryclinic. On reviewing his past history, he has taken a number of different medications over thepast few years, and you suspect that one or more of these may have caused his lung disorder.

Which of the following medications would be most likely to cause pulmonary fibrosis?

Explanation

The answer is Methotrexate –

Pulmonary fibrosis is a recognised adverse effect of treatment with a variety of drugs,including busulfan, methotrexate, amiodarone and bleomycin.

Bendrofluazide (Option A) is incorrect. Thiazide diuretics are not a recognised cause ofpulmonary fibrosis.

Captopril (Option B) is incorrect. ACE inhibitors may cause a reversible dry cough in 10–20%of patients, but pulmonary fibrosis is not a recognised complication.

Digoxin (Option C) is incorrect. Digoxin is not a recognised cause of pulmonary fibrosis.

Prednisolone (Option E) is incorrect. Corticosteroids can be used to treat some forms ofinterstitial lung disease, including intrinsic or extrinsic fibrosing alveolitis.

BendrofluazideA

CaptoprilB

DigoxinC

MethotrexateD

PrednisoloneE

Next Question

Page 114: Back to Filters (/Secure/TestMe/Filter ... - 1 File Download

8/12/2016 MyPastest

https://mypastest.pastest.com/Secure/TestMe/Browser/429893#Top 2/2

Previous Question Tag Question Feedback End Review

Difficulty: Average

Peer Responses

Session Progress

0Responses Correct:

179Responses Incorrect:

179Responses Total:

0%Responses - % Correct:

Blog (https://www.pastest.com/blog) About Pastest (https://www.pastest.com/about-us)Contact Us (https://www.pastest.com/contact-us) Help (https://www.pastest.com/help)

© Pastest 2016

Page 115: Back to Filters (/Secure/TestMe/Filter ... - 1 File Download

8/12/2016 MyPastest

https://mypastest.pastest.com/Secure/TestMe/Browser/429893#Top 1/2

Back to Filters (/Secure/TestMe/Filter/429893/QA)

Question 58 of 179

46851

A 54-year-old man with type-2 diabetes and dyslipidaemia has been prescribed metformin,pioglitazone, gliclazide, simvastatin and fenofibrate.

Which of the following drugs has predominantly peroxisome proliferator-activated receptor α(PPAR-α) activity?

Explanation

The answer is Fenofibrate –

The fibrate class of drugs are PPAR-α agonists, their predominant action is in reducingserum triglyceride levels and increasing HDL-cholesterol. PPAR activation results inincreased gene transcription; over 100 different genes are thought to be upregulated byglitazone therapy. This group is distinct from PPAR-γ agonists (the glitazones), andunrelated to PPAR-δ agonists that promote HDL-cholesterol.

Gliclazide (Option B) is incorrect. Sulfonylureas stimulate pancreatic insulin secretion.

Metformin (Option C) is incorrect. Metformin (a biguanide) inhibits lactate dehydrogenasewithin the liver and bowel.

Pioglitazone (Option D) is incorrect. PPAR-γ agonists (the glitazones) act predominantly byreducing free fatty acid levels, therefore improving insulin resistance and hence bloodglucose levels.

Simvastatin (Option E) is incorrect. Simvastatin inhibits activity of HMG-CoA reductase withinthe liver, thereby limiting formation of cholesterol.

FenofibrateA

GliclazideB

MetforminC

PioglitazoneD

SimvastatinE

Page 116: Back to Filters (/Secure/TestMe/Filter ... - 1 File Download

8/12/2016 MyPastest

https://mypastest.pastest.com/Secure/TestMe/Browser/429893#Top 2/2

Next Question

Previous Question Tag Question Feedback End Review

Difficulty: Average

Peer Responses

Session Progress

0Responses Correct:

179Responses Incorrect:

179Responses Total:

0%Responses - % Correct:

Blog (https://www.pastest.com/blog) About Pastest (https://www.pastest.com/about-us)Contact Us (https://www.pastest.com/contact-us) Help (https://www.pastest.com/help)

© Pastest 2016

Page 117: Back to Filters (/Secure/TestMe/Filter ... - 1 File Download

8/12/2016 MyPastest

https://mypastest.pastest.com/Secure/TestMe/Browser/429893#Top 1/2

Back to Filters (/Secure/TestMe/Filter/429893/QA)

Question 59 of 179

You are asked to see a 13-year-old child in the medical clinic because the parents areconcerned that there is a medical cause for poor academic performance at school. Themother has asked been searching the internet and is concerned about the possibility of leadpoisoning.

Which of the following features would most strongly suggest a diagnosis of chronic leadpoisoning?

Explanation

The answer is Peroneal nerve palsy –

Lead poisoning is due to the ingestion of contaminated water or certain traditionalremedies such as ayurvedic medicines. Typical acute effects of lead toxicity occur aboveconcentrations of 450 μg/l, including nausea, abdominal pain and constipation.Neurological effects are predominantly related to motor function including isolated nervelesions. Chronic low-grade exposure (< 450 μg/l) can be associated with mildneurodevelopmental delays. Chelation therapy with oral DMSA (dimercaptosuccinic acid)or intravenous EDTA (ethylenediaminetetraacetic acid) should be considered for patientswith blood lead concentrations over 450 μg/l or signs of severe toxicity. Lead absorptionand toxicity may be greater in patients with iron-deficiency anaemia.

Diarrhoea (Option A) is incorrect. Constipation and abdominal pain are recognised features.

Hypercalcaemia (Option B) is incorrect. Hypocalcaemia may give rise to greater leaddeposition in bones.

DiarrhoeaA

HypercalcaemiaB

Peroneal nerve palsyC

Resting tremor affecting both handsD

ThrombocytopeniaE

Page 118: Back to Filters (/Secure/TestMe/Filter ... - 1 File Download

8/12/2016 MyPastest

https://mypastest.pastest.com/Secure/TestMe/Browser/429893#Top 2/2

45798

Resting tremor affecting both hands (Option D) is incorrect. Parkinsonism is more closelyrelated to manganese toxicity, but it is not a characteristic of lead toxicity.

Thrombocytopenia (Option E) is incorrect. Characteristic haematological effects includechronic anaemia and there may be macrocytosis and basophilic stippling on a blood film.

Next Question

Previous Question Tag Question Feedback End Review

Difficulty: Average

Peer Responses

Session Progress

0Responses Correct:

179Responses Incorrect:

179Responses Total:

0%Responses - % Correct:

Blog (https://www.pastest.com/blog) About Pastest (https://www.pastest.com/about-us)Contact Us (https://www.pastest.com/contact-us) Help (https://www.pastest.com/help)

© Pastest 2016

Page 119: Back to Filters (/Secure/TestMe/Filter ... - 1 File Download

8/12/2016 MyPastest

https://mypastest.pastest.com/Secure/TestMe/Browser/429893#Top 1/2

Back to Filters (/Secure/TestMe/Filter/429893/QA)

Question 60 of 179

A 74-year-old man comes to the clinic some 6 weeks after a stroke for review andmanagement of his risk factors. He is already managed with two antihypertensives,atorvastatin, metformin and gliclazide. He has suffered two myocardial infarctions during thepast 5 years. On examination his BP is 134/78 mmHg, pulse is 78/min and regular, his BMI is28.

Investigations;

Hb 12.9 g/dl

WCC 7.8 x10 /l

PLT 187 x10 /l

Na 138 mmol/l

K 4.3 mmol/l

Creatinine 110 micromol/l

HbA1c 72.68 mmol/mol (8.8%)

You plan to intensify his glucose control.

Which of the following would be an appropriate target with respect to his HbA1c?

Explanation

The answer is 56.28 mmol/mol (7.3%)

9

9

+

+

39.89 mmol/mol (5.8%)A

47.54 mmol/mol (6.5%)B

56.28 mmol/mol (7.3%)C

65.03 mmol/mol (8.1%)D

73.77 mmol/mol (8.9%)E

Page 120: Back to Filters (/Secure/TestMe/Filter ... - 1 File Download

8/12/2016 MyPastest

https://mypastest.pastest.com/Secure/TestMe/Browser/429893#Top 2/2

34348

The clues as to the most appropriate target for glucose control in this patient come from theACCORD and VADT studies, both of which targeted HbA1c of 42.07 mmol/mol (6%) in anattempt to reduce the risk of vascular complications of diabetes. What was seen in thesestudies was a small increase in sudden cardiovascular death in the group targeted forintensive glucose control. The reason for this may have been increased sympathetic drive /electrolyte imbalance at the time of hypoglycaemia, leading to arrhythmia. As such inpatients with Type 2 diabetes and a history of macrovascular disease, a less stringent HbA1ctarget is more appropriate.

Next Question

Previous Question Tag Question Feedback End Review

Difficulty: Difficult

Peer Responses

Session Progress

0Responses Correct:

179Responses Incorrect:

179Responses Total:

0%Responses - % Correct:

Blog (https://www.pastest.com/blog) About Pastest (https://www.pastest.com/about-us)Contact Us (https://www.pastest.com/contact-us) Help (https://www.pastest.com/help)

© Pastest 2016

Page 121: Back to Filters (/Secure/TestMe/Filter ... - 1 File Download

8/12/2016 MyPastest

https://mypastest.pastest.com/Secure/TestMe/Browser/429893#Top 1/2

Back to Filters (/Secure/TestMe/Filter/429893/QA)

Question 61 of 179

46727

You are treating a 56-year-old woman with angina and decide to administer isosorbidemononitrate.

Which one of the following pharmacological mechanisms makes the greatest contribution tothe beneficial effects of nitroglycerin?

Explanation

The answer is Dilation of systemic veins –

Nitroglycerin products are both venous capacitance dilators, and coronary and systemicartery dilators. Nitrates have complex beneficial effects on patients with coronary arterydisease; they dilate systemic veins, decrease myocardial wall tension, decrease oxygendemand, cause vasodilation of large and medium-sized coronary arteries, increasecoronary blood flow to the subendocardium, reduce afterload, reduced preload andincrease ventricular compliance.

Decreases oxygen transport to the myocardium (Option A) is incorrect. Nitroglycerin resultsin increased myocardial oxygen delivery.

Increase of left ventricular afterload (Option C) is incorrect. Nitroglycerin results indecreased left ventricular afterload.

Increase of left ventricular preload (Option D) is incorrect. Nitroglycerin results in decreasedleft ventricular preload.

Reduces sodium–potassium transport in myocardial muscle (Option E) is incorrect.Nitroglycerin has no direct effect on sodium–potassium transport; this is a feature of digoxin.

Decreases oxygen transport to the myocardiumA

Dilation of systemic veinsB

Increase of left ventricular afterloadC

Increase of left ventricular preloadD

Reduces sodium–potassium transport in myocardial muscleE

Page 122: Back to Filters (/Secure/TestMe/Filter ... - 1 File Download

8/12/2016 MyPastest

https://mypastest.pastest.com/Secure/TestMe/Browser/429893#Top 2/2

46727

Next Question

Previous Question Tag Question Feedback End Review

Difficulty: Average

Peer Responses

Session Progress

0Responses Correct:

179Responses Incorrect:

179Responses Total:

0%Responses - % Correct:

Blog (https://www.pastest.com/blog) About Pastest (https://www.pastest.com/about-us)Contact Us (https://www.pastest.com/contact-us) Help (https://www.pastest.com/help)

© Pastest 2016

Page 123: Back to Filters (/Secure/TestMe/Filter ... - 1 File Download

8/12/2016 MyPastest

https://mypastest.pastest.com/Secure/TestMe/Browser/429893#Top 1/2

Back to Filters (/Secure/TestMe/Filter/429893/QA)

Question 62 of 179

46856

A 28-year-old man with a known history of glucose 6-phosphate dehydrogenase (G6PD)deficiency is prescribed an antibiotic for his urinary tract infection. He now presents to theEmergency Department with jaundice and anaemia, and a blood film shows erythrocytefragments.

Which of the following drugs is most likely to have caused his condition?

Explanation

The answer is Co-trimoxazole –

The enzyme glucose 6-phosphate dehydrogenase (G6PD) in red blood cells is needed toreduce nicotinic acid dinucleotide (NAD) to nicotinic acid dinucleotide phosphate(NADPH); NADPH operates via glutathione to protect against oxidative damage. Mostsulfonamides can precipitate an acute haemolytic anaemia in patients with a G6PDdeficiency, including sulfamethoxazole (combined with trimethoprim in co-trimoxazole).

Amoxicillin (Option A) is incorrect. Penicillins are usually safe in this condition, althoughautoimmune haemolytic anaemia is a recognised complication.

Ampicillin (Option B) is incorrect. Penicillins are usually safe in this condition, althoughautoimmune haemolytic anaemia is a recognised complication.

Ceftriaxone (Option C) is incorrect. Cephalosporins are usually safe in this condition.

Erythromycin (Option E) is incorrect. Macrolides are usually safe in this condition.

AmoxicillinA

AmpicillinB

CeftriaxoneC

Co-trimoxazoleD

ErythromycinE

Page 124: Back to Filters (/Secure/TestMe/Filter ... - 1 File Download

8/12/2016 MyPastest

https://mypastest.pastest.com/Secure/TestMe/Browser/429893#Top 2/2

Next Question

Previous Question Tag Question Feedback End Review

Difficulty: Average

Peer Responses

Session Progress

0Responses Correct:

179Responses Incorrect:

179Responses Total:

0%Responses - % Correct:

Blog (https://www.pastest.com/blog) About Pastest (https://www.pastest.com/about-us)Contact Us (https://www.pastest.com/contact-us) Help (https://www.pastest.com/help)

© Pastest 2016

Page 125: Back to Filters (/Secure/TestMe/Filter ... - 1 File Download

8/12/2016 MyPastest

https://mypastest.pastest.com/Secure/TestMe/Browser/429893#Top 1/2

Back to Filters (/Secure/TestMe/Filter/429893/QA)

Question 63 of 179

22522

A 72-year-old woman presents to the Emergency Department with increased drowsiness andlethargy. She is taking long-term extended-release morphine sulphate for an inoperablegynaecological cancer for which she has received radiotherapy. Additional past medicalhistory of note includes type 2 diabetes, for which she is treated with BD insulin. Youunderstand that she has recently started some new medication for a complication of herdiabetes. She has neuropathy and has been seeing the chiropodist for an infected foot ulcer.Her random blood glucose on admission is 5.4 mmol/l.

Which one of the following is most likely to have caused her decreased consciousness level?

Explanation

Out of the drugs listed, it is amitriptyline which is most likely to be responsible for the clinicalpicture seen here;

Interactions with morphine

The summary of product characteristics for morphine does not state any significant

pharmacokinetic interactions that potentiate morphine’s action

It does however state that morphine may reduce the effectiveness of ciprofloxacin

when co-administered and that prescribers should be aware of this interaction

A significant number of drugs exist that result in pharmacodynamic interactions with

morphine, including:

anxiolytics

phenothiazines

barbiturates

tricyclic antidepressants such as amitriptyline

The effect of morphine is markedly potentiated by monoamine oxidase inhibitors, so

co-administration should definitely be avoided

Resourceshttp://www.medicines.org.uk/emc/medicine/4896/SPC/Morphine+Sulphate+Injection+BP+Minijet+1mg+ml/#INTERACTIONS(http://www.medicines.org.uk/emc/medicine/4896/SPC/Morphine+Sulphate+Injection+BP+Minijet+1mg+ml/#INTERACTIONS)

http://www.medicines.org.uk/emc/medicine/13143/SPC/Morphine+Sulphate+10mg+ml+Injection+BP/#INTERACTIONS(http://www.medicines.org.uk/emc/medicine/13143/SPC/Morphine+Sulphate+10mg+ml+Injection+BP/#INTERACTIONS)

AmitriptylineA

GliclazideB

IbuprofenC

CiprofloxacinD

ClarithromycinE

Next Question

Page 126: Back to Filters (/Secure/TestMe/Filter ... - 1 File Download

8/12/2016 MyPastest

https://mypastest.pastest.com/Secure/TestMe/Browser/429893#Top 2/2

Previous QuestionTag Question Feedback End Review

Difficulty: Average

Peer Responses

Session Progress

0Responses Correct:

179Responses Incorrect:

179Responses Total:

0%Responses - % Correct:

Blog (https://www.pastest.com/blog) About Pastest (https://www.pastest.com/about-us)Contact Us (https://www.pastest.com/contact-us) Help (https://www.pastest.com/help)

© Pastest 2016

Page 127: Back to Filters (/Secure/TestMe/Filter ... - 1 File Download

8/12/2016 MyPastest

https://mypastest.pastest.com/Secure/TestMe/Browser/429893#Top 1/2

Back to Filters (/Secure/TestMe/Filter/429893/QA)

Question 64 of 179

46976

A 22-year-old policeman attends the Emergency Department after being bitten on the handby his dog. You clean the wound and apply Steri-Strips and administer an anti-tetanusbooster. You are concerned about the possibility of an early infection.

Which one of the following antibiotics would be most appropriate in this case?

Explanation

The answer is Co-amoxiclav –

Co-amoxiclav may be used to cover a variety of organisms that may be encounteredafter a dog bite. For example is it effective against Pasteurella, Streptococcus,Staphylococcus aureus and Escherichia coli.

Ciprofloxacin (Option A) is incorrect. Ciprofloxaxin would have activity against some of thelikely organisms and may be considered as second-line therapy.

Doxycycline (Option C) is incorrect. Doxycycline and erythromycin would have activityagainst some of the possible infecting organisms, but less than co-amoxiclav.

Penicillin V (Option D) is incorrect. Penicillin would be active against Streptococcus infection,but would have limited efficacy against other organisms.

Trimethoprim (Option E) is incorrect. Trimethoprim would be effective against Escherichiacoli but ineffective against Pasteurella, Streptococcus and Staphylococcus aureus.

CiprofloxacinA

Co-amoxiclavB

DoxycyclineC

Penicillin VD

TrimethoprimE

Next Question

Page 128: Back to Filters (/Secure/TestMe/Filter ... - 1 File Download

8/12/2016 MyPastest

https://mypastest.pastest.com/Secure/TestMe/Browser/429893#Top 2/2

Previous Question Tag Question Feedback End Review

Difficulty: Average

Peer Responses

Session Progress

0Responses Correct:

179Responses Incorrect:

179Responses Total:

0%Responses - % Correct:

Blog (https://www.pastest.com/blog) About Pastest (https://www.pastest.com/about-us)Contact Us (https://www.pastest.com/contact-us) Help (https://www.pastest.com/help)

© Pastest 2016

Page 129: Back to Filters (/Secure/TestMe/Filter ... - 1 File Download

8/12/2016 MyPastest

https://mypastest.pastest.com/Secure/TestMe/Browser/429893#Top 1/2

Back to Filters (/Secure/TestMe/Filter/429893/QA)

Question 65 of 179

You see a 54-year-old man with a history of productive cough and fever, and decide toadminister a 1-week course of clarithromycin. He has been taking a number of regularmedications.

With which one of the following medications would clarithromycin be most likely to cause aclinically important drug interaction?

Explanation

The answer is Ciclosporin –

Macrolide antibiotics including clarithromycin may cause immediate inhibition of thecytochrome P450 system, thereby increasing the circulating concentrations of drugs thatare normally metabolised by this system. This may provoke toxicity, particularly for drugswith a narrow therapeutic index such as ciclosporin. Interaction between macrolides andciclosporin may provoke bone marrow failure and renal failure.

Aspirin (Option A) is incorrect. Aspirin is predominantly cleared by renal elimination.

Atorvastatin (Option B) is incorrect. Macrolides interfere with metabolism of simvastatin, andto a lesser extent other statins, including atorvastatin. As a precaution, statin therapy isusually withheld during treatment with clarithromycin. The interaction betweenclarithromycin and atorvastatin is less hazardous than that involving ciclosporin.

Lithium (Option D) is incorrect. Lithium is a cation, and not subject to metabolism. It iscleared by renal excretion, and lithium toxicity may be caused by drugs that interfere withrenal clearance.

AspirinA

AtorvastatinB

CiclosporinC

LithiumD

ParacetamolE

Page 130: Back to Filters (/Secure/TestMe/Filter ... - 1 File Download

8/12/2016 MyPastest

https://mypastest.pastest.com/Secure/TestMe/Browser/429893#Top 2/2

45807

Paracetamol (Option E) is incorrect. Paracetamol is predominantly metabolised by sulfationand glucuronidation, with only a minor contribution from P450 oxidation.

Next Question

Previous Question Tag Question Feedback End Review

Difficulty: Average

Peer Responses

Session Progress

0Responses Correct:

179Responses Incorrect:

179Responses Total:

0%Responses - % Correct:

Blog (https://www.pastest.com/blog) About Pastest (https://www.pastest.com/about-us)Contact Us (https://www.pastest.com/contact-us) Help (https://www.pastest.com/help)

© Pastest 2016

Page 131: Back to Filters (/Secure/TestMe/Filter ... - 1 File Download

8/12/2016 MyPastest

https://mypastest.pastest.com/Secure/TestMe/Browser/429893#Top 1/2

Back to Filters (/Secure/TestMe/Filter/429893/QA)

Question 66 of 179

A 45-year-old man is admitted via the Emergency Department after intentional overdoseinvolving multiple medications. Your FY1 trainee has asked for advice on whethermeasurements of drug concentrations would be helpful.

For which one of the following would drug concentrations be most helpful in the context ofsuspected drug overdose?

Explanation

The answer is Paraquat –

Measurement of plasma paraquat concentration can help in assessing prognosis and canaid clinical decision-making regarding active versus palliative treatment. Plasmaconcentration measurements are also useful in the management of poisoning todetermine the need for specific antidote treatment, including paracetamol, salicylates,lithium, iron, methanol, ethylene glycol and theophylline.

Chlorpromazine (Option A) is incorrect. Chlorpromazine may cause profound central nervoussystem depression; there is no correlation between circulating blood concentrations andtoxicity.

Diazepam (Option B) is incorrect. Diazepam may cause profound depression of centralnervous system and respiration, but blood concentrations would offer no clinical benefit inassessing toxicity.

Imipramine (Option C) is incorrect. Tricyclic poisoning may cause severe or fatal toxicitythrough sodium channel blockade and anticholinergic mechansism. Toxicity does notcorrelate with blood drug concentrations.

ChlorpromazineA

DiazepamB

ImipramineC

MorphineD

ParaquatE

Page 132: Back to Filters (/Secure/TestMe/Filter ... - 1 File Download

8/12/2016 MyPastest

https://mypastest.pastest.com/Secure/TestMe/Browser/429893#Top 2/2

46912

Morphine (Option D) is incorrect. Opioids may cause profound central nervous systemdepression, hypoventilation and coma. Drug concentrations would add no benefit to theclinical assessment of toxicity.

Next Question

Previous Question Tag Question Feedback End Review

Difficulty: Average

Peer Responses

Session Progress

0Responses Correct:

179Responses Incorrect:

179Responses Total:

0%Responses - % Correct:

Blog (https://www.pastest.com/blog) About Pastest (https://www.pastest.com/about-us)Contact Us (https://www.pastest.com/contact-us) Help (https://www.pastest.com/help)

© Pastest 2016

Page 133: Back to Filters (/Secure/TestMe/Filter ... - 1 File Download

8/12/2016 MyPastest

https://mypastest.pastest.com/Secure/TestMe/Browser/429893#Top 1/2

Back to Filters (/Secure/TestMe/Filter/429893/QA)

Question 67 of 179

46616

A 69-year-old man with long-standing cardiac failure and chronic kidney disease has beenreferred urgently to the general medical clinic for review. Over the past week he hascomplained of nausea, reduced oral intake and visual disturbance characterised by ayellowish-green halo around objects. His cardiorespiratory examination is normal, pupilresponses and eye movements are normal, and fundi appear normal.

Which of the following medications is most likely to account for his present condition?

Explanation

The answer is Digoxin –

Digoxin has a narrow therapeutic index, and the most common features of digoxintoxicity are anorexia, nausea, xanthopsia as a manifestation of neurotoxicity, andbradyarrhythmia. Toxicity is more likely to occur in patients with hypokalaemia andhypercalcaemia.

Enalapril (Option B) is incorrect. Enalapril may cause postural hypotension, dizziness andcough.

Furosemide (Option C) is incorrect. Furosemide may cause postural hypotension, dizziness,renal impairment and metabolic disturbance including hypokalaemia.

Rivaroxaban (Option D) is incorrect. Rivaroxaban may cause bleeding, skin erythema andblistering, sore throat and joint pains.

Salbutamol (Option E) is incorrect. Salbutamol may cause tachycardia, palpitations andhypokalaemia.

DigoxinA

EnalaprilB

FurosemideC

RivaroxabanD

SalbutamolE

Page 134: Back to Filters (/Secure/TestMe/Filter ... - 1 File Download

8/12/2016 MyPastest

https://mypastest.pastest.com/Secure/TestMe/Browser/429893#Top 2/2

46616

Next Question

Previous Question Tag Question Feedback End Review

Difficulty: Average

Peer Responses

Session Progress

0Responses Correct:

179Responses Incorrect:

179Responses Total:

0%Responses - % Correct:

Blog (https://www.pastest.com/blog) About Pastest (https://www.pastest.com/about-us)Contact Us (https://www.pastest.com/contact-us) Help (https://www.pastest.com/help)

© Pastest 2016

Page 135: Back to Filters (/Secure/TestMe/Filter ... - 1 File Download

8/12/2016 MyPastest

https://mypastest.pastest.com/Secure/TestMe/Browser/429893#Top 1/2

Back to Filters (/Secure/TestMe/Filter/429893/QA)

Question 68 of 179

An 18-year-old woman presents to the Emergency Department following an intentionaloverdose of aspirin tablets.

Which of the following treatments would be most effective in reducing aspirin toxicity?

Explanation

The answer is Multi-dose oral activated charcoal –

Initial features include respiratory alkalosis due to direct stimulation of the respiratorycentre and hyperventilation, whereas a metabolic acidosis occurs due to uncoupling ofoxidative phosphorylation and the build-up of lactic acid and fatty acid metabolites.Symptoms of salicylate ingestion include nausea, vomiting, hyperventilation andsweating. Tinnitus typically occurs at plasma salicylate concentrations above 400–500mg/l. Other effects include hypoglycaemia, particularly in children. Severe toxic effectsare central nervous system (CNS) effects, including confusion, coma and seizures.Intravenous bicarbonate administration causes ionisation of circulating aspirin, whichresults in lower tissue concentrations and more extensive renal elimination. Fluidadministration should be directed towards maintaining hydration and correctingelectrolytes; there is no role for forced diuresis or diuretic administration.

Forced alkaline diuresis (Option A) is incorrect. This is hazardous and should be avoided.

Intravenous furosemide (Option B) is incorrect. This does not reduce aspirin toxicity. Fluidsmay be administered to maintain hydration.

Intravenous insulin and glucose administration (Option C) is incorrect. This may be helpful inβ-blocker or calcium channel blocker poisoning, but not aspirin poisoning.

Forced alkaline diuresisA

Intravenous furosemideB

Intravenous insulin and glucose administrationC

Multi-dose oral activated charcoalD

Oral sodium bicarbonateE

Page 136: Back to Filters (/Secure/TestMe/Filter ... - 1 File Download

8/12/2016 MyPastest

https://mypastest.pastest.com/Secure/TestMe/Browser/429893#Top 2/2

45799

Oral sodium bicarbonate (Option E) is incorrect. The urgency of addressing severe aspirinpoisoning means that oral administration is less appropriate than intravenous administration.

Next Question

Previous Question Tag Question Feedback End Review

Difficulty: Average

Peer Responses

Session Progress

0Responses Correct:

179Responses Incorrect:

179Responses Total:

0%Responses - % Correct:

Blog (https://www.pastest.com/blog) About Pastest (https://www.pastest.com/about-us)Contact Us (https://www.pastest.com/contact-us) Help (https://www.pastest.com/help)

© Pastest 2016

Page 137: Back to Filters (/Secure/TestMe/Filter ... - 1 File Download

8/12/2016 MyPastest

https://mypastest.pastest.com/Secure/TestMe/Browser/429893#Top 1/2

Back to Filters (/Secure/TestMe/Filter/429893/QA)

Question 69 of 179

A 65-year-old man has been receiving warfarin for atrial fibrillation. His INR has beenbetween 2.0 and 3.0 for the past 6 months. He attends the Emergency Department due toextensive bruising and is noted to have an INR of 9.6.

Which of the following drugs is most likely to have interacted with warfarin to cause a rise inINR?

Explanation

The answer is Sodium valproate –

Inhibition of P450 enzymes may increase the circulating concentrations of drugs that aremetabolised by the P450 enzymes. Warfarin is an example of a drug that is subject toliver metabolism by the P450 enzyme system. Therefore, concomitant treatment withenzyme inhibitors may give rise to high INR; enzyme inhibitors include: acute alcoholingestion, cimetidine, omeprazole, disulfiram, erythromycin, ciprofloxacin, sodiumvalproate and sulfonamides.

Aspirin (Option A) is incorrect. Aspirin may independently increase the risk of bleeding inpatients receiving anticoagulation, but normally has only a small effect on warfarinconcentrations due to a protein-binding interaction and would not normally cause a largeincrease in INR.

Carbamazepine (Option B) is incorrect. Carbamazepine is an enzyme inducer, likely to lessenwarfarin concentrations and INR.

AspirinA

CarbamazepineB

PhenytoinC

RifampicinD

Sodium valproateE

Page 138: Back to Filters (/Secure/TestMe/Filter ... - 1 File Download

8/12/2016 MyPastest

https://mypastest.pastest.com/Secure/TestMe/Browser/429893#Top 2/2

46488

Phenytoin (Option C) is incorrect. Phenytoin is an enzyme inducer, likely to lessen warfarinconcentrations and INR.

Rifampicin (Option D) is incorrect. Rifampicin is an enzyme inducer, likely to lessen warfarinconcentrations and INR.

Next Question

Previous Question Tag Question Feedback End Review

Difficulty: Average

Peer Responses

Session Progress

0Responses Correct:

179Responses Incorrect:

179Responses Total:

0%Responses - % Correct:

Blog (https://www.pastest.com/blog) About Pastest (https://www.pastest.com/about-us)Contact Us (https://www.pastest.com/contact-us) Help (https://www.pastest.com/help)

© Pastest 2016

Page 139: Back to Filters (/Secure/TestMe/Filter ... - 1 File Download

8/12/2016 MyPastest

https://mypastest.pastest.com/Secure/TestMe/Browser/429893#Top 1/2

Back to Filters (/Secure/TestMe/Filter/429893/QA)

Question 70 of 179

37786

A 72-year-old woman presents to the Emergency Department with palpitations and shortnessof breath. She has a history of COPD, managed with high dose Seretide, and anechocardiogram conducted two months earlier has revealed left atrial enlargement and mildmitral regurgitation. Examination reveals a BP of 115/75 mmHg, pulse is 145/min (atrialfibrillation). Serum creatinine is measured at 90 micromol/l.

You elect to begin loading with digoxin.

What is the pharmacokinetic reason that drives the practice of loading with digoxin?

Explanation

The answer is Volume of distribution -

The volume of distribution for Digoxin is very large (510 litres). This means that administereddoses are rapidly distributed to body tissues. The initial distribution lasts for some 6-8hrs,which drives the typical loading regimen for Digoxin of two larger doses (500mcg) some 6-12hrs apart. Without loading Digoxin typically takes a few days to reach therapeutic effect.

GI absorption is high at approximately 60% in tablet form. The elimination half-life is some30-40hrs in patients with normal renal function (the drug being renally excreted). In patientswith abnormal renal function it can be as long as 100hrs. This drives reduced dosing inpatients with renal impairment.

https://www.medicines.org.uk/emc/medicine/2177/SPC/Lanoxin+PG+Tablets/#PHARMACOKINETIC_PROPS(https://www.medicines.org.uk/emc/medicine/2177/SPC/Lanoxin+PG+Tablets/#PHARMACOKINETIC_PROPS)

Gastrointestinal absorptionA

Half lifeB

Hepatic metabolismC

Renal excretionD

Volume of distributionE

Next Question

Previous Question Tag Question Feedback End Review

Difficulty: Average

Peer Responses

Page 140: Back to Filters (/Secure/TestMe/Filter ... - 1 File Download

8/12/2016 MyPastest

https://mypastest.pastest.com/Secure/TestMe/Browser/429893#Top 2/2

Session Progress

0Responses Correct:

179Responses Incorrect:

179Responses Total:

0%Responses - % Correct:

Blog (https://www.pastest.com/blog) About Pastest (https://www.pastest.com/about-us)Contact Us (https://www.pastest.com/contact-us) Help (https://www.pastest.com/help)

© Pastest 2016

Page 141: Back to Filters (/Secure/TestMe/Filter ... - 1 File Download

8/12/2016 MyPastest

https://mypastest.pastest.com/Secure/TestMe/Browser/429893#Top 1/2

Back to Filters (/Secure/TestMe/Filter/429893/QA)

Question 71 of 179

46618

A 34-year-old man is brought to the Emergency Department by the police. He was found inthe streets with reduced conscious level, and you suspect possible heroin intoxication.

Which of the following signs most strongly supports a diagnosis of opioid toxicity?

Explanation

The answer is Bradycardia –

Heroin and other opioid toxicity is classically associated with reduced conscious level,pinpoint pupils (meiosis), reduced respiratory effort (shallow respirations), andbradycardia and hypotension.

Deep and rapid respiration (Option B) is incorrect. Progressive opioid toxicity causes shallowrespirations and reducing tidal volume; respiratory rate may appear normal until severetoxicity occurs and respirations slow or cease.

Excess sweating and lacrimation (Option C) is incorrect. Excess sweating and lacrimation arerecognised features of opiate withdrawal.

Hypothermia (Option D) is incorrect. Hypothermia is a non-specific finding in patients withimpaired conscious level who have been outdoors for an unknown period of time, commonlydue to alcohol toxicity or other sedative medications including benzodiazepines andbarbiturates.

Mydriasis (Option E) is incorrect. Mydriasis (dilated pupils) may suggest exposure to drugswith anticholinergic or serotonergic properties.

BradycardiaA

Deep and rapid respirationB

Excess sweating and lacrimationC

HypothermiaD

MydriasisE

Page 142: Back to Filters (/Secure/TestMe/Filter ... - 1 File Download

8/12/2016 MyPastest

https://mypastest.pastest.com/Secure/TestMe/Browser/429893#Top 2/2

Next Question

Previous Question Tag Question Feedback End Review

Difficulty: Average

Peer Responses

Session Progress

0Responses Correct:

179Responses Incorrect:

179Responses Total:

0%Responses - % Correct:

Blog (https://www.pastest.com/blog) About Pastest (https://www.pastest.com/about-us)Contact Us (https://www.pastest.com/contact-us) Help (https://www.pastest.com/help)

© Pastest 2016

Page 143: Back to Filters (/Secure/TestMe/Filter ... - 1 File Download

8/12/2016 MyPastest

https://mypastest.pastest.com/Secure/TestMe/Browser/429893#Top 1/2

Back to Filters (/Secure/TestMe/Filter/429893/QA)

Question 72 of 179

A 31-year-old woman with a history of hypertension has a recent blood pressure of 149/78mmHg. She tells you that she has been trying to become pregnant and is keen to avoid anytherapy that might interfere with this.

Which of the following agents would be most suitable for managing hypertension inpregnancy?

Explanation

The answer is Nifedipine –

Regarding calcium antagonists such as nifedipine in pregnancy, it is important to balancepotential risks to the fetus against the risks of uncontrolled hypertension. Where awoman is over 30 years old, has a history of hypertension and has raised blood pressurecurrently, the potential risks of uncontrolled maternal hypertension are likely to begreater than the potential adverse effects of antihypertensive therapy. It has beenreported that the non-dihydropyridine calcium antagonist diltiazem may inhibit the onsetof labour.

Bendrofluazide (Option A) is incorrect. Thiazides or any other diuretics should generally beavoided in pregnancy due to the risk of neonatal thrombocytopaenia, albeit a rare outcome.

Candesartan (Option B) is incorrect. Angiotensin-II-receptor blockers carry broadly the samepotential hazards in pregnancy as ACE inhibitors, namely neonatal hyperkalaemia andoligohydramnios.

Hydrochlorothiazide (Option C) is incorrect. Thiazides or any other diuretics should generallybe avoided in pregnancy due to the risk of neonatal thrombocytopaenia, albeit a rare

BendrofluazideA

CandesartanB

HydrochlorothiazideC

NifedipineD

RamiprilE

Page 144: Back to Filters (/Secure/TestMe/Filter ... - 1 File Download

8/12/2016 MyPastest

https://mypastest.pastest.com/Secure/TestMe/Browser/429893#Top 2/2

46827

outcome.

Ramipril (Option E) is incorrect. Angiotensin converting enzyme (ACE) inhibitors should beavoided in pregnancy as they may adversely affect fetal blood pressure and renal function; inaddition, animal studies have demonstrated a potential association with fetal skull defectsand oligohydramnios.

Next Question

Previous Question Tag Question Feedback End Review

Difficulty: Average

Peer Responses

Session Progress

0Responses Correct:

179Responses Incorrect:

179Responses Total:

0%Responses - % Correct:

Blog (https://www.pastest.com/blog) About Pastest (https://www.pastest.com/about-us)Contact Us (https://www.pastest.com/contact-us) Help (https://www.pastest.com/help)

© Pastest 2016

Page 145: Back to Filters (/Secure/TestMe/Filter ... - 1 File Download

8/12/2016 MyPastest

https://mypastest.pastest.com/Secure/TestMe/Browser/429893#Top 1/2

Back to Filters (/Secure/TestMe/Filter/429893/QA)

Question 73 of 179

A man with advanced cirrhosis attends the clinic for review of his medications. He is taking anumber of drugs, including omeprazole, propranolol and ciprofloxacin.

Which of the following statements best applies to drug metabolism in patients withadvanced liver disease?

Explanation

The answer is Conjugation reactions are affected to a lesser extent by advanced liverdisease and only occur in very late stage disease –

Drug processing via mixed function oxidases is affected early in liver disease, whereasconjugation reactions are affected to a lesser extent and problems only occur much laterin the liver disease process. Clinical trials of drugs are rarely conducted in patients withsevere liver disease, although of course some drugs in particular (e.g. antibiotics) areused in very sick liver patients, and limited studies may take place in these patientgroups.

Conjugation reactions are affected early in cirrhosis (Option A) is incorrect. Conjugationreactions are affected late in cirrhosis.

Drug reactions mediated by mixed function oxidases are affected late in liver disease (OptionC) is incorrect. Mixed function oxidase activity is impaired early in liver cirrhosis.

Intrahepatic cholestasis does not affect drug transport (Option D) is incorrect. Bothintrahepatic and extrahepatic cholestasis may affect the metabolism of drugs that areactively secreted into bile, e.g. ciprofloxacin.

Conjugation reactions are affected early in cirrhosisA

Conjugation reactions are affected to a lesser extent by advanced liver disease andonly occur in very late stage disease

B

Drug reactions mediated by mixed function oxidases are affected late in liver diseaseC

Intrahepatic cholestasis does not affect drug transportD

Plasma proteins increase and affect drug metabolismE

Page 146: Back to Filters (/Secure/TestMe/Filter ... - 1 File Download

8/12/2016 MyPastest

https://mypastest.pastest.com/Secure/TestMe/Browser/429893#Top 2/2

46842

Plasma proteins increase and affect drug metabolism (Option E) is incorrect. Plasma proteinsfall in liver disease and may alter drug distribution.

Next Question

Previous Question Tag Question Feedback End Review

Difficulty: Average

Peer Responses

Session Progress

0Responses Correct:

179Responses Incorrect:

179Responses Total:

0%Responses - % Correct:

Blog (https://www.pastest.com/blog) About Pastest (https://www.pastest.com/about-us)Contact Us (https://www.pastest.com/contact-us) Help (https://www.pastest.com/help)

© Pastest 2016

Page 147: Back to Filters (/Secure/TestMe/Filter ... - 1 File Download

8/12/2016 MyPastest

https://mypastest.pastest.com/Secure/TestMe/Browser/429893#Top 1/2

Back to Filters (/Secure/TestMe/Filter/429893/QA)

Question 74 of 179

A 60-year-old retired surgeon is brought to the Emergency Department by his wife, who hasnoticed his behaviour has become bizarre with paranoid symptoms over the past 4 days. Hehas evidence of coarse tremor, horizontal nystagmus and an ataxic gait. There is a history ofexcessive drinking since retirement.

Which of the following immediate treatments would be most effective?

Explanation

The answer is Intravenous thiamine –

This patient is likely to be suffering from Wernicke’s encephalopathy owing to thiaminedeficiency. Although this is most often seen in alcoholics, it may also occur in severemalnutrition, and in patients on long-term haemodialysis or with AIDS. Wernicke’sencephalopathy presents as a triad of acute mental confusion, ataxia andophthalmoplegia. Nystagmus is usually present. Urgent thiamine should be given toprevent the development of irreversible Korsakoff syndrome, characterised by retrogradeand anterograde amnesia with varying degrees of other cognitive defects.

Acamprosate (Option A) is incorrect. Acamprosate reduces cravings for alcohol and is usefulin the maintenance of abstinence, but is of no value in the setting of acute alcoholwithdrawal.

Disulfiram (Option B) is incorrect. Disulfiram is an aversive agent that creates adversesymptoms when patients consume alcohol; it is helpful in maintaining abstinence but is of novalue in acute alcohol withdrawal.

AcamprosateA

DisulfiramB

Intravenous diazepamC

Intravenous thiamineD

NaltrexoneE

Page 148: Back to Filters (/Secure/TestMe/Filter ... - 1 File Download

8/12/2016 MyPastest

https://mypastest.pastest.com/Secure/TestMe/Browser/429893#Top 2/2

46768

Intravenous diazepam (Option C) is incorrect. Diazepam may be effective in reducing therisks of seizures and reduce symptoms of alcohol withdrawal, but will not correct theneurological features attributable to thiamine deficiency.

Naltrexone (Option E) is incorrect. Naltrexone may be effective in promoting alcoholabstinence and improves quit rates, but it is of no value in the acute setting.

Next Question

Previous Question Tag Question Feedback End Review

Difficulty: Average

Peer Responses

Session Progress

0Responses Correct:

179Responses Incorrect:

179Responses Total:

0%Responses - % Correct:

Blog (https://www.pastest.com/blog) About Pastest (https://www.pastest.com/about-us)Contact Us (https://www.pastest.com/contact-us) Help (https://www.pastest.com/help)

© Pastest 2016

Page 149: Back to Filters (/Secure/TestMe/Filter ... - 1 File Download

8/12/2016 MyPastest

https://mypastest.pastest.com/Secure/TestMe/Browser/429893#Top 1/2

Back to Filters (/Secure/TestMe/Filter/429893/QA)

Question 75 of 179

45802

Your hospital has been alerted to the possibility of a nearby chemical spill, and a number ofpatients are on their way to hospital. There are reports of the chemical having a smell ofrotten eggs.

Which of the following agents would you most strongly suspect?

Explanation

The answer is Hydrogen sulfide –

Certain toxins/poisons have characteristic smells that can assist in the identification ofvarious chemical substances. Not all individuals perceive smells or odours to the sameextent, such that reports may not be very reliable. The smell of rotten eggs ischaracteristic for hydrogen sulfide.

Arsenic (Option A) is incorrect. The smell of garlic is characteristic for arsenic and selenium.

Cyanide (Option B) is incorrect. The smell of almonds is characteristic for cyanide.

Napthalene (Option D) is incorrect. The smell of mothballs is characteristic for naphthalene.

Selenium (Option E) is incorrect. The smell of garlic is characteristic for selenium and arsenic.

ArsenicA

CyanideB

Hydrogen sulfideC

NapthaleneD

SeleniumE

Next Question

Page 150: Back to Filters (/Secure/TestMe/Filter ... - 1 File Download

8/12/2016 MyPastest

https://mypastest.pastest.com/Secure/TestMe/Browser/429893#Top 2/2

Previous QuestionTag Question Feedback End Review

Difficulty: Average

Peer Responses

Session Progress

0Responses Correct:

179Responses Incorrect:

179Responses Total:

0%Responses - % Correct:

Blog (https://www.pastest.com/blog) About Pastest (https://www.pastest.com/about-us)Contact Us (https://www.pastest.com/contact-us) Help (https://www.pastest.com/help)

© Pastest 2016

Page 151: Back to Filters (/Secure/TestMe/Filter ... - 1 File Download

8/12/2016 MyPastest

https://mypastest.pastest.com/Secure/TestMe/Browser/429893#Top 1/2

Back to Filters (/Secure/TestMe/Filter/429893/QA)

Question 76 of 179

A young woman has been taking the combined oral contraceptive pill regularly. Her GP treatsher for an acute skin infection. Seven weeks later she presents with irregular bleeding, and isfound to have a positive pregnancy test.

Which antibiotic medication is most likely to have been prescribed by the GP?

Explanation

The answer is Rifampicin –

Rifampicin is highly effective against Staphylococcus infection, although it is notnormally used as a first-line therapy in view of adverse effects including hepatitis. It is apowerful enzyme inducer that increases the rate and extent of hepatic metabolism of theoral contraceptive pill and other medications, including warfarin. Rifampicin treatmentwill give rise to lower oestrogen concentrations and pill failure.

Amoxicillin (Option A) is incorrect. Amoxicillin is associated with pill failure in patients whodevelop diarrhoea and impaired oestrogen absorption.

Cefalexin (Option B) is incorrect. There is no clear indication that cephalosporins interferewith the oral contraceptive pill, although broad-spectrum antibiotics may provoke diarrhoeaand impaired gut absorption.

Cefuroxime (Option C) is incorrect. There is no clear indication that cephalosporins interferewith the oral contraceptive pill, although broad-spectrum antibiotics may provoke diarrhoeaand impaired gut absorption.

AmoxicillinA

CefalexinB

CefuroximeC

RifampicinD

TetracyclineE

Page 152: Back to Filters (/Secure/TestMe/Filter ... - 1 File Download

8/12/2016 MyPastest

https://mypastest.pastest.com/Secure/TestMe/Browser/429893#Top 2/2

46772

Tetracycline (Option E) is incorrect. Tetracyclines may rarely be associated with oralcontraceptive failure, although the mechanism of interaction is poorly understood.

Next Question

Previous Question Tag Question Feedback End Review

Difficulty: Average

Peer Responses

Session Progress

0Responses Correct:

179Responses Incorrect:

179Responses Total:

0%Responses - % Correct:

Blog (https://www.pastest.com/blog) About Pastest (https://www.pastest.com/about-us)Contact Us (https://www.pastest.com/contact-us) Help (https://www.pastest.com/help)

© Pastest 2016

Page 153: Back to Filters (/Secure/TestMe/Filter ... - 1 File Download

8/12/2016 MyPastest

https://mypastest.pastest.com/Secure/TestMe/Browser/429893#Top 1/2

Back to Filters (/Secure/TestMe/Filter/429893/QA)

Question 77 of 179

46798

You see a 59-year-old man who is about to commence a course of chemotherapy includingdoxorubicin, and you wish to monitor for potential toxicity.

Which would be the most appropriate investigation to carry out before starting thechemotherapy?

Explanation

The answer is Echocardiogram –

The antitumour activity of doxorubicin derives partly from the triggering oftopoisomerase II-dependent DNA breakage. Doxorubicin can cause cardiotoxicity,including cardiomyopathy, acute arrhythmias, congestive heart failure and dose-dependent impairment of left ventricular ejection fraction; myocardial dysfunction ismost likely in patients that receive a cumulative dose of greater than 500 mg/m .

Creatinine clearance (Option A) is incorrect. Nephrotoxicity is a more characteristic featureof cisplatin therapy.

CT brain (Option B) is incorrect. Doxorubicin is not expected to significantly increase risk ofneurological toxicity.

EEG (Option D) is incorrect. Doxorubicin is not expected to increase seizure risk or causeneurological toxicity.

Lung function test (Option E) is incorrect. Pulmonary adverse effects may be more likely tooccur after bleomycin or methotrexate therapy.

Creatinine clearanceA

CT brainB

EchocardiogramC

EEGD

Lung function testE

2

Page 154: Back to Filters (/Secure/TestMe/Filter ... - 1 File Download

8/12/2016 MyPastest

https://mypastest.pastest.com/Secure/TestMe/Browser/429893#Top 2/2

Next Question

Previous Question Tag Question Feedback End Review

Difficulty: Average

Peer Responses

Session Progress

0Responses Correct:

179Responses Incorrect:

179Responses Total:

0%Responses - % Correct:

Blog (https://www.pastest.com/blog) About Pastest (https://www.pastest.com/about-us)Contact Us (https://www.pastest.com/contact-us) Help (https://www.pastest.com/help)

© Pastest 2016

Page 155: Back to Filters (/Secure/TestMe/Filter ... - 1 File Download

8/12/2016 MyPastest

https://mypastest.pastest.com/Secure/TestMe/Browser/429893#Top 1/2

Back to Filters (/Secure/TestMe/Filter/429893/QA)

Question 78 of 179

46620

Antimicrobial drugs possess a variety of different mechanisms of bactericidal andbacteriostatic action.

Which one of the following antimicrobial agents is most likely to exert its pharmacologicalactions through binding to bacterial ribosomes to disrupt protein synthesis?

Explanation

The answer is Azithromycin –

Antimicrobial drugs Macrolides (eg erythromycin, azithromycin and clarithromycin),aminoglycosides and chloramphenicol all effect their pharmacological actions throughbinding to bacterial ribosomes and disrupting protein synthesis.

Ceftriaxone (Option B) is incorrect. Penicillins and cephalosporins interfere with bacterialcell-wall synthesis.

Ciprofloxacin (Option C) is incorrect. Quinolones (eg ciprofloxacin, moxifloxacin) inhibit DNAgyrase.

Penicillin (Option D) is incorrect. Penicillins and cephalosporins interfere with bacterial cell-wall synthesis.

Trimethoprim (Option E) is incorrect. Trimethoprim and sulfonamides inhibit bacterial folatesynthesis.

AzithromycinA

CeftriaxoneB

CiprofloxacinC

PenicillinD

TrimethoprimE

Next Question

Page 156: Back to Filters (/Secure/TestMe/Filter ... - 1 File Download

8/12/2016 MyPastest

https://mypastest.pastest.com/Secure/TestMe/Browser/429893#Top 2/2

Previous Question Tag Question Feedback End Review

Difficulty: Average

Peer Responses

Session Progress

0Responses Correct:

179Responses Incorrect:

179Responses Total:

0%Responses - % Correct:

Blog (https://www.pastest.com/blog) About Pastest (https://www.pastest.com/about-us)Contact Us (https://www.pastest.com/contact-us) Help (https://www.pastest.com/help)

© Pastest 2016

Page 157: Back to Filters (/Secure/TestMe/Filter ... - 1 File Download

8/12/2016 MyPastest

https://mypastest.pastest.com/Secure/TestMe/Browser/429893#Top 1/2

Back to Filters (/Secure/TestMe/Filter/429893/QA)

Question 79 of 179

46857

While prescribing a medication, a doctor tells his patient that its major mode of action isthrough direct drug action, rather than achieving its effects through a metabolite.

Which of the following medications would best fit in this category?

Explanation

The answer is Diazepam –

Diazepam is the only drug listed that does not require metabolism to an activemetabolite in order to exert its therapeutic effects.

Chloral hydrate (Option A) is incorrect. Chloral hydrate is rapidly metabolised totrichloroethanol, which is the active molecule.

Enalapril (Option C) is incorrect. Enalapril is a prodrug that requires metabolism toenalaprilat in order to inhibit ACE.

Oxcarbazepine (Option D) is incorrect. The major metabolite of oxcarbazepine is a 10-monohydroxy derivative that exerts its therapeutic effects.

Vitamin D (Option E) is incorrect. Vitamin D (cholecalciferol) is converted by the liver to 25-hydroxycholecalciferol, and then by the kidney tubule enzyme 1α-hydroxylase to 1,25-dihydrocholecalciferol; this metabolite is much more biologically active than theunhydroxylated or monohydroxylated moieties.

Chloral hydrateA

DiazepamB

EnalaprilC

OxcarbazepineD

Vitamin DE

Next Question

Page 158: Back to Filters (/Secure/TestMe/Filter ... - 1 File Download

8/12/2016 MyPastest

https://mypastest.pastest.com/Secure/TestMe/Browser/429893#Top 2/2

Previous Question Tag Question Feedback End Review

Difficulty: Average

Peer Responses

Session Progress

0Responses Correct:

179Responses Incorrect:

179Responses Total:

0%Responses - % Correct:

Blog (https://www.pastest.com/blog) About Pastest (https://www.pastest.com/about-us)Contact Us (https://www.pastest.com/contact-us) Help (https://www.pastest.com/help)

© Pastest 2016

Page 159: Back to Filters (/Secure/TestMe/Filter ... - 1 File Download

8/12/2016 MyPastest

https://mypastest.pastest.com/Secure/TestMe/Browser/429893#Top 1/2

Back to Filters (/Secure/TestMe/Filter/429893/QA)

Question 80 of 179

A 56-year-old man has recently been diagnosed with type-2 diabetes. His HbA1c after aperiod of diet and exercise is still raised at 8.4%. You are considering initial drug treatment.

Which of the following has a primary mode of action that is most likely to lead to short-terminsulin release, e.g. when taken just before a meal?

Explanation

The answer is Meglitinides –

Meglitinides (nateglinide and repaglinide) specifically increase postprandial insulinrelease. Meglitinides may allow better glycaemic control by addressing short-term fastingor working in shift patterns.

Alpha-glucosidase inhibitor (Option A) is incorrect. Alpha-glucosidase inhibitors, egacarbose, act by inhibiting the intestinal breakdown of long-chain sugars, which leads to areduced peak of glucose after meals. Side-effects include flatulence and diarrhoea due tobacterial metabolism of unabsorbed sugars.

Glitazones (Option B) is incorrect. Glitazones, although having some effect on hepaticglucose output, exert their primary glucose-lowering effects by increasing glucose uptakeinto skeletal muscle and fat.

Metformin (Option D) is incorrect. Metformin is a partial insulin sensitiser, exerting its primaryeffects on blood glucose via a reduction in hepatic glucose output. Metformin iscontraindicated in patients with severe hepatic disease or where the serum creatinine level isabove 130 μmol/l in women and 150 μmol/l in men.

Alpha-glucosidase inhibitorA

GlitazonesB

MeglitinidesC

MetforminD

SulfonylureasE

Page 160: Back to Filters (/Secure/TestMe/Filter ... - 1 File Download

8/12/2016 MyPastest

https://mypastest.pastest.com/Secure/TestMe/Browser/429893#Top 2/2

46832

Sulfonylureas (Option E) is incorrect. Sulfonylureas, eg gliclazide, increase insulin releaseover a prolonged period and may be associated with an increased incidence ofhypoglycaemia compared with the meglitinides.

Next Question

Previous Question Tag Question Feedback End Review

Difficulty: Average

Peer Responses

Session Progress

0Responses Correct:

179Responses Incorrect:

179Responses Total:

0%Responses - % Correct:

Blog (https://www.pastest.com/blog) About Pastest (https://www.pastest.com/about-us)Contact Us (https://www.pastest.com/contact-us) Help (https://www.pastest.com/help)

© Pastest 2016

Page 161: Back to Filters (/Secure/TestMe/Filter ... - 1 File Download

8/12/2016 MyPastest

https://mypastest.pastest.com/Secure/TestMe/Browser/429893#Top 1/2

Back to Filters (/Secure/TestMe/Filter/429893/QA)

Question 81 of 179

46487

Drug metabolism may be subject to genetic variation, for example acetylation status.

Which of the following statements best describes the influence of acetylator status?

Explanation

The answer is Predicts the clinical response to isoniazid –

Drug acetylation is a phase-II reaction, leading to the synthesis of water-solubledrugs/drug metabolites. Genetic variation accounts for slow acetylator or fast acetylatorstatus. In most cases, slow acetylators have higher concentrations of drug due to slowermetabolism and clearance, therefore slow acetylators may be more prone to adversedrug effects. This includes dapsone-induced haemolysis, isoniazid-induced neuropathy,hydralazine-induced lupus and sulfasalazine-induced haemolysis.

Acetylation status is an autosomal-dominant trait (Option A) is incorrect. Acetylation statusis inherited as an autosomal-recessive trait.

Adverse drug effects are normally more severe in rapid acetylators (Option B) is incorrect.Adverse effects are normally greater in slow acetylators. One exception is that rapidacetylators may be more prone to isoniazid-induced hepatitis, which is caused by theacetylation metabolite.

Drug acetylation is an example of drug conjugation (Option C) is incorrect. Acetylation is aphase-II reaction, not drug conjugation.

Explains the variation in adverse effects of penicillin (Option D) is incorrect. Penicillin is notextensively metabolised by acetylation.

Acetylation status is an autosomal-dominant traitA

Adverse drug effects are normally more severe in rapid acetylatorsB

Drug acetylation is an example of drug conjugationC

Explains the variation in adverse effects of penicillinD

Predicts the clinical response to isoniazidE

Page 162: Back to Filters (/Secure/TestMe/Filter ... - 1 File Download

8/12/2016 MyPastest

https://mypastest.pastest.com/Secure/TestMe/Browser/429893#Top 2/2

46487

Next Question

Previous Question Tag Question Feedback End Review

Difficulty: Average

Peer Responses

Session Progress

0Responses Correct:

179Responses Incorrect:

179Responses Total:

0%Responses - % Correct:

Blog (https://www.pastest.com/blog) About Pastest (https://www.pastest.com/about-us)Contact Us (https://www.pastest.com/contact-us) Help (https://www.pastest.com/help)

© Pastest 2016

Page 163: Back to Filters (/Secure/TestMe/Filter ... - 1 File Download

8/12/2016 MyPastest

https://mypastest.pastest.com/Secure/TestMe/Browser/429893#Top 1/2

Back to Filters (/Secure/TestMe/Filter/429893/QA)

Question 82 of 179

A 72-year-old man is admitted unconscious. He has a history of type-2 diabetes and is taking10 mg of glibenclamide. Blood testing reveals a serum creatinine level of 125 μmol/l and ablood glucose of 1.5 mmol/l.

Which treatment regime may be a suitable alternative therapy for his diabetes?

Explanation

The answer is Metformin –

Metformin avoids hypoglycaemia, although it is contraindicated in patients with renalimpairment; discontinuation is recommended when creatinine levels are above 130μmol/l in women and 150 μmol/l in men.

Chlorpropamide (Option A) is incorrect. Chlorpropamide and glibenclamide are long-actingsulfonylureas, and are contraindicated in elderly patients with renal impairment because ofthe risk of hypoglycaemia.

Metformin and insulin combination therapy (Option C) is incorrect. The combination ofmetformin and insulin would be associated with increased complexity and risk ofhypoglycaemia.

Pioglitazone added to glibenclamide (Option D) is incorrect. Pioglitazone would avoidhypoglycaemia, and in patients with creatinine higher than 150 μmol/l, it would be apreferred option to metformin.

Pioglitazone and insulin combination therapy (Option E) is incorrect. The combination ofpioglitazone and insulin would be associated with increased complexity and risk of

ChlorpropamideA

MetforminB

Metformin and insulin combination therapyC

Pioglitazone added to glibenclamideD

Pioglitazone and insulin combination therapyE

Page 164: Back to Filters (/Secure/TestMe/Filter ... - 1 File Download

8/12/2016 MyPastest

https://mypastest.pastest.com/Secure/TestMe/Browser/429893#Top 2/2

46831

hypoglycaemia.

Next Question

Previous Question Tag Question Feedback End Review

Difficulty: Average

Peer Responses

Session Progress

0Responses Correct:

179Responses Incorrect:

179Responses Total:

0%Responses - % Correct:

Blog (https://www.pastest.com/blog) About Pastest (https://www.pastest.com/about-us)Contact Us (https://www.pastest.com/contact-us) Help (https://www.pastest.com/help)

© Pastest 2016

Page 165: Back to Filters (/Secure/TestMe/Filter ... - 1 File Download

8/12/2016 MyPastest

https://mypastest.pastest.com/Secure/TestMe/Browser/429893#Top 1/2

Back to Filters (/Secure/TestMe/Filter/429893/QA)

Question 83 of 179

A 26-year-old woman, who is 9 weeks’ pregnant, has two episodes of convulsions. She had ahistory of generalised seziures 18 months before but has not received any treatment. Youdecide that it would be safest to commence antiepileptic drug treatment. She wants adviceabout the antiepileptic drugs and risks to her pregnancy.

Which treatment would convey the greatest risk of congenital malformations?

Explanation

The answer is Sodium valproate –

It is difficult to define a ‘safe’ medication in pregnancy, but the teratogenic risk of sodiumvalproate is very well characterised and seems to pose at least as great a hazardcompared to other agents. There is an approximately 7% rate of congenital majormalformations, including neural tube defects and craniofacial abnormalities; risks areeven higher for patients taking valproate plus other antiepileptic drugs. The backgroundpopulation rate of congenital malformations is around 1.5%. Counselling should be givento all women of childbearing capacity who are prescribed antiepileptic drugs.

Carbamazepine (Option A) is incorrect. The rate of congenital malformations is around 3%for carbamazepine.

Lamotrigine (Option B) is incorrect. The rate of congenital malformations is around 2% forlamotrigine.

Phenytoin (Option C) is incorrect. The rate of congenital malformations is around 4% forphenytoin.

CarbamazepineA

LamotrigineB

PhenytoinC

Sodium valproateD

TopiramateE

Page 166: Back to Filters (/Secure/TestMe/Filter ... - 1 File Download

8/12/2016 MyPastest

https://mypastest.pastest.com/Secure/TestMe/Browser/429893#Top 2/2

46979

Topiramate (Option E) is incorrect. The rate of congenital malformations is around 5% fortopiramate.

Next Question

Previous Question Tag Question Feedback End Review

Difficulty: Average

Peer Responses

Session Progress

0Responses Correct:

179Responses Incorrect:

179Responses Total:

0%Responses - % Correct:

Blog (https://www.pastest.com/blog) About Pastest (https://www.pastest.com/about-us)Contact Us (https://www.pastest.com/contact-us) Help (https://www.pastest.com/help)

© Pastest 2016

Page 167: Back to Filters (/Secure/TestMe/Filter ... - 1 File Download

8/12/2016 MyPastest

https://mypastest.pastest.com/Secure/TestMe/Browser/429893#Top 1/2

Back to Filters (/Secure/TestMe/Filter/429893/QA)

Question 84 of 179

You are asked to see a 24-year-old patient in the Emergency Department, who has recentlyreturned from a trip to Thailand. He is in police custody and complaining of generalisedabdominal discomfort. You are suspicious that he might be a body packer.

Which of the following statements represents the most appropriate initial management?

Explanation

The answer is Gastric lavage should be avoided –

Management of body packers Gastric lavage may increase the risk of package ruptureand may provoke severe toxicity. Abdominal X-rays may show packages within thegastrointestinal tract, but not always. A CT abdominal scan is better at identifying thepresence of drug packages. Whole-bowel irrigation with polyethylene glycol will clearthe gastrointestinal tract of all the swallowed packages. Upper endoscopy may also beuseful in removing packages that are still in the stomach, but caution is required to avoiddisrupting the packaging and drug release. Urgent surgical intervention may be neededto remove drug packages in patients who develop signs of drug toxicity because ruptureof packages may be fatal, depending upon the quantity and type of drug in eachpackage.

Normal abdominal X-ray appearances exclude the diagnosis (Option B) is incorrect. Packagesmay not be detectable on plain abdominal X-ray.

Patients may be discharged if asymptomatic after 4 hours (Option C) is incorrect. Patientsare at risk of developing toxicity until drug packages are cleared from the body.

Gastric lavage should be avoidedA

Normal abdominal X-ray appearances exclude the diagnosisB

Patients may be discharged if asymptomatic after 4 hoursC

Surgical intervention should be avoided in symptomatic patientsD

Whole-bowel irrigation is contraindicatedE

Page 168: Back to Filters (/Secure/TestMe/Filter ... - 1 File Download

8/12/2016 MyPastest

https://mypastest.pastest.com/Secure/TestMe/Browser/429893#Top 2/2

45805

Surgical intervention should be avoided in symptomatic patients (Option D) is incorrect. Inpatients that develop toxicity, surgical intervention may be needed to prevent severe or fataltoxicity depending upon the type and quantity of drug packages.

Whole-bowel irrigation is contraindicated (Option E) is incorrect. This is often a usefulstrategy for enhancing removal of packages from the body.

Next Question

Previous Question Tag Question Feedback End Review

Difficulty: Average

Peer Responses

Session Progress

0Responses Correct:

179Responses Incorrect:

179Responses Total:

0%Responses - % Correct:

Blog (https://www.pastest.com/blog) About Pastest (https://www.pastest.com/about-us)Contact Us (https://www.pastest.com/contact-us) Help (https://www.pastest.com/help)

© Pastest 2016

Page 169: Back to Filters (/Secure/TestMe/Filter ... - 1 File Download

8/12/2016 MyPastest

https://mypastest.pastest.com/Secure/TestMe/Browser/429893#Top 1/2

Back to Filters (/Secure/TestMe/Filter/429893/QA)

Question 85 of 179

A 44-year-old woman has a fasting total cholesterol level of 6.6 mmol/l and triglycerides of6.5 mmol/l. Dietary intervention has unfortunately shown no benefit and therefore therapy isneeded because of her risk factors.

Which one of the following is the most appropriate therapy?

Explanation

The answer is Bezafibrate –

In this patient, the predominant problem is hypertriglyceridaemia, with modesthypercholesterolaemia. Fibrate drugs diminish serum triglyceride levels by stimulatinglipoprotein lipase and decreasing non-esterified fatty acids (NEFA) in the circulation,probably by stimulating nuclear peroxisome proliferator-activated receptor-α. Fibratedrugs (bezafibrate, ciprofibrate, fenofibrate, gemfibrozil) are first-line therapy; they arealso often highly effective in type-III and primary type-V hyperlipoproteinaemia, and thedyslipoproteinaemia of diabetes mellitus. Fibrates decrease very-low-density lipoproteincholesterol but are less effective than statins in lowering low-density lipoproteincholesterol. In some particularly high-risk patients with combined hyperlipidaemia, statintherapy may be added to fibrate therapy to satisfactorily lower LDL-cholesterol, but thisposes a greater risk of myopathy and close monitoring is required.

Aspirin (Option A) is incorrect. Aspirin has no role in the treatment of hypertriglyceridaemia.

Cholestyramine (Option C) is incorrect. Cholestyramine interferes with dietary fat absorptionand fat-soluble vitamins, but has only a modest effect on circulating triglycerides comparedto fibrates.

AspirinA

BezafibrateB

CholestyramineC

EnalaprilD

PravastatinE

Page 170: Back to Filters (/Secure/TestMe/Filter ... - 1 File Download

8/12/2016 MyPastest

https://mypastest.pastest.com/Secure/TestMe/Browser/429893#Top 2/2

46794

Enalapril (Option D) is incorrect. Enalapril has no significant effect on lipid profile.

Pravastatin (Option E) is incorrect. Pravastatin is effective in lowering LDL-cholesterol buthas only a modest effect on triglyceride concentrations. Other more potent statins includingatorvastatin and rosuvastatin are capable of lowering triglyceride concentrations, but to alesser extent than fibrates.

Next Question

Previous Question Tag Question Feedback End Review

Difficulty: Average

Peer Responses

Session Progress

0Responses Correct:

179Responses Incorrect:

179Responses Total:

0%Responses - % Correct:

Blog (https://www.pastest.com/blog) About Pastest (https://www.pastest.com/about-us)Contact Us (https://www.pastest.com/contact-us) Help (https://www.pastest.com/help)

© Pastest 2016

Page 171: Back to Filters (/Secure/TestMe/Filter ... - 1 File Download

8/12/2016 MyPastest

https://mypastest.pastest.com/Secure/TestMe/Browser/429893#Top 1/2

Back to Filters (/Secure/TestMe/Filter/429893/QA)

Question 86 of 179

A 30-year-old woman was started on carbamazepine for partial complex seizures and wasalso advised to discontinue her moderate alcohol consumption. Therapeutic concentrationsof carbamazepine were achieved within 4 days with a dose of 200 mg daily, but the doseneeded to be increased to 400 mg daily within 2 weeks to achieve a therapeutic plasmaconcentration.

Which one of the following is the most likely reason for needing to increase thecarbamazepine dose?

Explanation

The answer is Auto-induction of carbamazepine metabolism –

Carbamazepine is a hepatic enzyme inducer, and hence the dose of carbamazepine mayneed to be increased after a few weeks of therapy to achieve a steady plasmaconcentration. Its properties as an enzyme inducer mean that carbamazepine oftenlowers plasma concentrations of clonazepam, lamotrigine and phenytoin, so when anti-epileptics are combined it is sensible to be wary of such interactions. Contraindicationsfor carbamazepine include atrioventricular (AV) conduction abnormalities, porphyria andhistory of bone marrow depression. Common adverse effects include nausea andvomiting, drowsiness, dizziness and headache.

Auto-inhibition of carbamazepine metabolism (Option B) is incorrect. Carbamazepineinduces liver enzyme activity.

Cessation of alcohol intake (Option C) is incorrect. Moderate to heavy alcohol consumptionmay increase hepatic enzyme activity; cessation may result in a gradual fall in enzyme

Auto-induction of carbamazepine metabolismA

Auto-inhibition of carbamazepine metabolismB

Cessation of alcohol intakeC

Concomitant prescription of the oral contraceptive pillD

Reduced bioavailability of carbamazepineE

Page 172: Back to Filters (/Secure/TestMe/Filter ... - 1 File Download

8/12/2016 MyPastest

https://mypastest.pastest.com/Secure/TestMe/Browser/429893#Top 2/2

46922

activity so that carbamazepine concentrations may rise.

Concomitant prescription of the oral contraceptive pill (Option D) is incorrect.Carbamazepine should not be used with the contraceptive pill because enzyme inductionmay lower concentrations of the pill so that it may become ineffective. The contraceptive pillwould not be expected to have any direct effect on carbamazepine concentrations.

Reduced bioavailability of carbamazepine (Option E) is incorrect. Carbamazepine absorptionis not likely to alter after continued therapy.

Next Question

Previous Question Tag Question Feedback End Review

Difficulty: Average

Peer Responses

Session Progress

0Responses Correct:

179Responses Incorrect:

179Responses Total:

0%Responses - % Correct:

Blog (https://www.pastest.com/blog) About Pastest (https://www.pastest.com/about-us)Contact Us (https://www.pastest.com/contact-us) Help (https://www.pastest.com/help)

© Pastest 2016

Page 173: Back to Filters (/Secure/TestMe/Filter ... - 1 File Download

8/12/2016 MyPastest

https://mypastest.pastest.com/Secure/TestMe/Browser/429893#Top 1/2

Back to Filters (/Secure/TestMe/Filter/429893/QA)

Question 87 of 179

46752

A 6-month-old child has sensorineural deafness and a ventricular septal defect. Her motherhad received medications during the pregnancy, and asks you for advice about whether thesemight be relevant to her son’s medical problems.

Which of the following drugs taken during pregnancy would be most likely to have causedthese clinical effects?

Explanation

The answer is Isotretinoin –

Isotretinoin is a highly effective treatment for acne but it is highly teratogenic. Womenmust have a negative pregnancy test before treatment and be taking effectivecontraception for at least a month before the course begins, during the course and for amonth after it finishes. Congenital deafness and central nervous system and heartdefects may occur in children exposed to isotretinoin in utero.

Clindamycin (Option A) is incorrect. Clindamycin may be teratogenic, but limited data areavailable.

Cyproterone acetate (Option B) is incorrect. Cyproterone acetate may cause feminisation ofmale fetuses.

Minocycline (Option D) is incorrect. Tetracyclines may cause discoloured teeth in infantsexposed in utero.

Oxytetracyline (Option E) is incorrect. Tetracyclines may cause discoloured teeth in infantsexposed in utero.

ClindamycinA

Cyproterone acetateB

IsotretinoinC

MinocyclineD

OxytetracylineE

Page 174: Back to Filters (/Secure/TestMe/Filter ... - 1 File Download

8/12/2016 MyPastest

https://mypastest.pastest.com/Secure/TestMe/Browser/429893#Top 2/2

46752

Next Question

Previous Question Tag Question Feedback End Review

Difficulty: Average

Peer Responses

Session Progress

0Responses Correct:

179Responses Incorrect:

179Responses Total:

0%Responses - % Correct:

Blog (https://www.pastest.com/blog) About Pastest (https://www.pastest.com/about-us)Contact Us (https://www.pastest.com/contact-us) Help (https://www.pastest.com/help)

© Pastest 2016

Page 175: Back to Filters (/Secure/TestMe/Filter ... - 1 File Download

8/12/2016 MyPastest

https://mypastest.pastest.com/Secure/TestMe/Browser/429893#Top 1/2

Back to Filters (/Secure/TestMe/Filter/429893/QA)

Question 88 of 179

A 65-year-old patient with asthma was started on theophylline due to poor control ofsymptoms, despite other regular inhaler therapy. Other relevant past history includestreatment for heart failure and hepatic steatosis, which appear to be stable, and he isreceiving long-term low-dose erythromycin for treatment of rosacea. He continues to smoke20 cigarettes per day, but after discussion with the clinic nurse today he has decided to quit.

Which of the following factors is most likely to require reduction of theophylline dose?

Explanation

The answer is Smoking cessation –

Theophylline is a bronchodilator used to treat reversible airway obstruction and maycause significant adverse effects, including nausea and vomiting, hypokalaemia andmetabolic acidosis. It has a narrow therapeutic window, and therapeutic drug monitoringis needed to minimise the risk of drug toxicity, typically aiming to achieve concentrationsbetween 10 and 20 mg/l. Regular tobacco use up-regulates hepatic enzyme activity;cessation will be associated with a decrease of hepatic enzyme activity, such thattheophylline concentrations may increase. Other factors that may increase theophyllinedrug concentrations and provoke toxicity include heart failure, liver cirrhosis, acute viralinfection, increased age, drugs that inhibit its metabolism (eg erythromycin), or cessationof enzyme-inducing drugs.

Age above 60 years (Option A) is incorrect. Advancing age may result in a need for lowertheophylline doses due to less extensive liver metabolism. This is unlikely to require adramatic change in dose provided that there is therapeutic drug monitoring.

Age above 60 yearsA

Concomitant use of erythromycinB

Heart failureC

Liver steatosisD

Smoking cessationE

Page 176: Back to Filters (/Secure/TestMe/Filter ... - 1 File Download

8/12/2016 MyPastest

https://mypastest.pastest.com/Secure/TestMe/Browser/429893#Top 2/2

46801

Concomitant use of erythromycin (Option B) is incorrect. Addition of erythromycin wouldcertainly risk an acute rise in theophylline concentrations and may provoke toxicity, but if thepatient was stable on long-term combined treatment this is much less likely to occur.

Heart failure (Option C) is incorrect. Heart failure may result in a need for lower theophyllinedoses, but if clinical features of heart failure are stable, this is unlikely to change dramatically.

Liver steatosis (Option D) is incorrect. Impaired liver metabolism, e.g. in the presence ofdrugs such as erythromycin or levofloxacin, may cause a significant acute rise in theophyllineconcentrations and provoke toxicity. Hepatic steatosis is unlikely to cause dramatic changesin the rate or extent of drug metabolism.

Next Question

Previous Question Tag Question Feedback End Review

Difficulty: Average

Peer Responses

Session Progress

0Responses Correct:

179Responses Incorrect:

179Responses Total:

0%Responses - % Correct:

Blog (https://www.pastest.com/blog) About Pastest (https://www.pastest.com/about-us)Contact Us (https://www.pastest.com/contact-us) Help (https://www.pastest.com/help)

© Pastest 2016

Page 177: Back to Filters (/Secure/TestMe/Filter ... - 1 File Download

8/12/2016 MyPastest

https://mypastest.pastest.com/Secure/TestMe/Browser/429893#Top 1/2

Back to Filters (/Secure/TestMe/Filter/429893/QA)

Question 89 of 179

46732

You are telephoned by the biochemistry laboratory to report a patient has potassiumconcentration 5.9 mmol/l.

Which one of the following factors is most likely to account for hyperkalaemia?

Explanation

The answer is Ciclosporin -

Ciclosporin is used as an immunosuppressant after renal transplant. Hyperkalaemia is acommon adverse effect, and patients treated with ciclosporin may have serum potassiumconcentrations in the range 6.0–7.1 mmol/l. The mechanism is probably a variant ofhyporeninaemic hypoaldosteronism, and is responsive to fludrocortisone.

Bartter syndrome (Option A) is incorrect. Bartter syndrome is associated with hypokalaemiadue to a number of inherited defects of renal function.

Liddle syndrome (Option C) is incorrect. Liddle syndrome is a rare condition characterised byhypokalaemia, hypertension and low aldosterone levels.

Liquorice addiction (Option D) is incorrect. Liquorice inhibits 11-hydroxysteroiddehydrogenase causing potassium wasting from the distal tubule.

Treatment with corticosteroids (Option E) is incorrect. Corticosteroids are associated withhypokalaemia, due to their mineralocorticoid effects.

Bartter syndromeA

CiclosporinB

Liddle syndromeC

Liquorice addictionD

Treatment with corticosteroidsE

Next Question

Page 178: Back to Filters (/Secure/TestMe/Filter ... - 1 File Download

8/12/2016 MyPastest

https://mypastest.pastest.com/Secure/TestMe/Browser/429893#Top 2/2

Previous Question Tag Question Feedback End Review

Difficulty: Average

Peer Responses

Session Progress

0Responses Correct:

179Responses Incorrect:

179Responses Total:

0%Responses - % Correct:

Blog (https://www.pastest.com/blog) About Pastest (https://www.pastest.com/about-us)Contact Us (https://www.pastest.com/contact-us) Help (https://www.pastest.com/help)

© Pastest 2016

Page 179: Back to Filters (/Secure/TestMe/Filter ... - 1 File Download

8/12/2016 MyPastest

https://mypastest.pastest.com/Secure/TestMe/Browser/429893#Top 1/2

Back to Filters (/Secure/TestMe/Filter/429893/QA)

Question 90 of 179

46757

A 29-year-old woman has been taking antiepileptic medications for several years. Shepresents to the Emergency Department with progressive imbalance and clumsiness, and hasfallen twice at home. Speech is slurred and she denies drinking any alcohol.

Which one of the following medications is most likely to be responsible for these symptoms?

Explanation

The answer is Carbamazepine –

Adverse effects of carbamazepine include ataxia, nystagmus and diplopia. These arenormally dose-dependent and reversible after discontinuing or reducing the dose ofregular therapy. Phenytoin may also cause cerebellar ataxia, and other adverse effectsinclude gum hypertrophy, hirsutism, folate deficiency, coarse facial features andosteomalacia.

Lamotrigine (Option B) is incorrect. Lamotrigine may cause significant skin effects includingblistering rashes and Stevens–Johnston syndrome, nightmares, blurred vision and dizziness.

Phenobarbital (Option C) is incorrect. Phenobarbital may cause folate deficiency,megaloblastic anaemia, osteomalacia and neuropathy.

Sodium valproate (Option D) is incorrect. Sodium valproate may cause hair loss and alopecia,liver damage, pancreatitis and weight gain.

Vigabatrin (Option E) is incorrect. Vigabatrin may cause aggression, alopecia, retinal atrophy,reduced peripheral vision, thrombocytopaenia and other blood dyscrasias.

CarbamazepineA

LamotrigineB

PhenobarbitalC

Sodium valproateD

VigabatrinE

Page 180: Back to Filters (/Secure/TestMe/Filter ... - 1 File Download

8/12/2016 MyPastest

https://mypastest.pastest.com/Secure/TestMe/Browser/429893#Top 2/2

Next Question

Previous Question Tag Question Feedback End Review

Difficulty: Average

Peer Responses

Session Progress

0Responses Correct:

179Responses Incorrect:

179Responses Total:

0%Responses - % Correct:

Blog (https://www.pastest.com/blog) About Pastest (https://www.pastest.com/about-us)Contact Us (https://www.pastest.com/contact-us) Help (https://www.pastest.com/help)

© Pastest 2016

Page 181: Back to Filters (/Secure/TestMe/Filter ... - 1 File Download

8/12/2016 MyPastest

https://mypastest.pastest.com/Secure/TestMe/Browser/429893#Top 1/2

Back to Filters (/Secure/TestMe/Filter/429893/QA)

Question 91 of 179

46819

A patient with congestive heart failure requires furosemide therapy for his oedema. On hisfollow-up visit you note that serum potassium concentration has fallen to 3.1 mmol/l.

What would be the most appropriate next step in his management?

Explanation

The answer is Add a small dose of amiloride to furosemide –

When hypokalaemia occurs after loop diuretic therapy, the best choice is to add apotassium sparing diuretic, eg amiloride or spironolactone. Amiloride inhibits sodiumchannels in the distal segment of the distal convoluted tubule.

Add metolazone to his furosemide (Option B) is incorrect. Addition of metolazone willprovoke the worsened hypokalaemia.

Change furosemide to bumetanide (Option C) is incorrect. Loop diuretics furosemide andbumetanide inhibit Na /K /Cl co-transport in the ascending limb of Henle’s loop, and thereis little difference between them.

Change furosemide to hydrochlorothiazide (Option D) is incorrect. Thiazide diuretics inhibitNa /Cl co-transport in the proximal segment of the distal convoluted tubule. They are alsoassociated with the occurrence of hypokalaemia.

Consider a potassium infusion (Option E) is incorrect. Potassium administration fails toaddress the renal potassium loss, so this will offer only temporary correction.

Add a small dose of amiloride to furosemideA

Add metolazone to his furosemideB

Change furosemide to bumetanideC

Change furosemide to hydrochlorothiazideD

Consider a potassium infusionE

+ + –

+ –

Page 182: Back to Filters (/Secure/TestMe/Filter ... - 1 File Download

8/12/2016 MyPastest

https://mypastest.pastest.com/Secure/TestMe/Browser/429893#Top 2/2

Next Question

Previous Question Tag Question Feedback End Review

Difficulty: Average

Peer Responses

Session Progress

0Responses Correct:

179Responses Incorrect:

179Responses Total:

0%Responses - % Correct:

Blog (https://www.pastest.com/blog) About Pastest (https://www.pastest.com/about-us)Contact Us (https://www.pastest.com/contact-us) Help (https://www.pastest.com/help)

© Pastest 2016

Page 183: Back to Filters (/Secure/TestMe/Filter ... - 1 File Download

8/12/2016 MyPastest

https://mypastest.pastest.com/Secure/TestMe/Browser/429893#Top 1/2

Back to Filters (/Secure/TestMe/Filter/429893/QA)

Question 92 of 179

A 35-year-old woman on carbamazepine for epilepsy was found to have Hashimoto’sthyroiditis and has been prescribed thyroxine by her GP. She comes back to the clinic a weeklater complaining of fatigue, depression, weight gain and constipation. Her tri-iodothyronineand thyroxine levels are found to be low despite the proper intake of medications.

What is the most likely cause of her symptoms?

Explanation

The answer is option Increased thyroxine clearance by the action of carbamazepine onliver enzymes –

Hepatic enzyme induction, caused by carbamazepine, phenytoin, rifampicin and otherdrugs, accelerates the metabolism of a number of co-administered drugs. In patients oncarbamazepine who develop Hashimoto’s thyroiditis the dose of thyroxine should beincreased to maintain therapeutic levels. The symptoms in this patient, who hasdeveloped Hashimoto’s thyroiditis while taking carbamazepine, ie fatigue, depression,weight gain and constipation, are due to inadequate treatment of hypothyroidism. Side-effects of carbamazepine include drowsiness, ataxia, nystagmus, diplopia, rash, andthrombocytopaenia and other blood dyscrasias.

Adverse reaction to carbamazepine (Option A) is incorrect. Carbamazepine may causesevere skin reactions, and cerebellar toxicity.

Direct effect of carbamazepine on thyroxine formation (Option B) is incorrect.Carbamazepine does not have a direct impact on thyroxine production (in contrast tocarbimazole!).

Adverse reaction to carbamazepineA

Direct effect of carbamazepine on thyroxine formationB

Increased binding of thyroxine by thyroxine-binding globulinC

Increased thyroxine clearance by the action of carbamazepine on liver enzymesD

Interference with intestinal absorption of thyroxine by carbamazepineE

Page 184: Back to Filters (/Secure/TestMe/Filter ... - 1 File Download

8/12/2016 MyPastest

https://mypastest.pastest.com/Secure/TestMe/Browser/429893#Top 2/2

46737

Increased binding of thyroxine by thyroxine-binding globulin (Option C) is incorrect. Drugsthat compete for binding of thyroxine to circulating thyroxine-binding globulin increase freethyroxine concentrations and may provoke hyperthyroidism.

Interference with intestinal absorption of thyroxine by carbamazepine (Option E) is incorrect.Carbamazepine does not interfere with gut absorption, and thyroxine is formed by thethyroid gland and not absorbed from the gut.

Next Question

Previous Question Tag Question Feedback End Review

Difficulty: Average

Peer Responses

Session Progress

0Responses Correct:

179Responses Incorrect:

179Responses Total:

0%Responses - % Correct:

Blog (https://www.pastest.com/blog) About Pastest (https://www.pastest.com/about-us)Contact Us (https://www.pastest.com/contact-us) Help (https://www.pastest.com/help)

© Pastest 2016

Page 185: Back to Filters (/Secure/TestMe/Filter ... - 1 File Download

8/12/2016 MyPastest

https://mypastest.pastest.com/Secure/TestMe/Browser/429893#Top 1/2

Back to Filters (/Secure/TestMe/Filter/429893/QA)

Question 93 of 179

You are consulted by a 33-year-old woman who is due to go on a cruise. She has been on acruise ship previously, but was kept in her room with nausea and vomiting which she thinkscould be attributed to seasickness. She does not wish to have a similar experience on hernext cruise.

What would you prescribe for her to best avoid such symptoms?

Explanation

The answer is Cinnarizine –

Cinnarizine is thought to be particularly useful for the treatment of motion sickness as ithas a dual action: first, it acts as a depressant of the vestibular system; second, itdampens down smooth muscle contraction in the gut. Patients may experiencedrowsiness and some gastrointestinal disturbance.

Domperidone (Option B) is incorrect. Domperidone may be used to treat motion sickness,but may be less effective than cinnarizine owing to its inability to cross the blood–brainbarrier and its actions being solely peripheral.

Metoclopramide (Option C) is incorrect. Metoclopramide and prochlorperazine may beassociated with an increased risk of acute dystonia.

Ondansetron (Option D) is incorrect. Ondansetron is predominantly centrally acting via 5HT-3 receptors, and may be effective for nausea due to a range of triggers, but is less wellrecognised as a treatment for motion sickness.

CinnarizineA

DomperidoneB

MetoclopramideC

OndansetronD

ProchlorperazineE

Page 186: Back to Filters (/Secure/TestMe/Filter ... - 1 File Download

8/12/2016 MyPastest

https://mypastest.pastest.com/Secure/TestMe/Browser/429893#Top 2/2

46992

Prochlorperazine (Option E) is incorrect. Metoclopramide and prochlorperazine may beassociated with an increased risk of acute dystonia.

Next Question

Previous Question Tag Question Feedback End Review

Difficulty: Average

Peer Responses

Session Progress

0Responses Correct:

179Responses Incorrect:

179Responses Total:

0%Responses - % Correct:

Blog (https://www.pastest.com/blog) About Pastest (https://www.pastest.com/about-us)Contact Us (https://www.pastest.com/contact-us) Help (https://www.pastest.com/help)

© Pastest 2016

Page 187: Back to Filters (/Secure/TestMe/Filter ... - 1 File Download

8/12/2016 MyPastest

https://mypastest.pastest.com/Secure/TestMe/Browser/429893#Top 1/2

Back to Filters (/Secure/TestMe/Filter/429893/QA)

Question 94 of 179

A 65-year-old woman has attended the cardiology outpatient department for review of heratrial fibrillation. She has been taking amiodarone for the past 3 years. Her only symptoms areof lethargy, weight gain over the past 6 months and low mood.

Which of the following investigations would be most useful in this case?

Explanation

The answer is T3, T4 and TSH levels –

Amiodarone is a class III antiarrhythmic drug. It may exert a number of effects on thethyroid, including:

Commonly a rise in free thyroxine (T ) and a fall in free tri-iodothyronine (T )

Some 2% of patients have clinically significant changes – accompanied by

symptoms of hyperthyroidism or hypothyroidism

As amiodarone has a prolonged half-life, the problems may persist for up to 3

months after withdrawal of the drug.

Blood urea and electrolytes (Option A) is incorrect. These tests would perhaps identifysignificant electrolyte disturbance or renal failure; neither is a characteristic adverse effect ofamiodarone.

ECG (Option B) is incorrect. An ECG would be helpful in assessing the patient’s cardiacdisorder but is less likely to identify a cause of her symptoms than thyroid tests.

Blood urea and electrolytesA

ECGB

Full blood countC

Liver function testsD

T3, T4 and TSH levelsE

4 3

Page 188: Back to Filters (/Secure/TestMe/Filter ... - 1 File Download

8/12/2016 MyPastest

https://mypastest.pastest.com/Secure/TestMe/Browser/429893#Top 2/2

46760

Full blood count (Option C) is incorrect. These tests might identify anaemia or other bloodabnormalities, but these are not a characteristic adverse effect of amiodarone.

Liver function tests (Option D) is incorrect. Amiodarone may cause hepatitis but this is muchless common than effects of thyroid function.

Next Question

Previous Question Tag Question Feedback End Review

Difficulty: Average

Peer Responses

Session Progress

0Responses Correct:

179Responses Incorrect:

179Responses Total:

0%Responses - % Correct:

Blog (https://www.pastest.com/blog) About Pastest (https://www.pastest.com/about-us)Contact Us (https://www.pastest.com/contact-us) Help (https://www.pastest.com/help)

© Pastest 2016

Page 189: Back to Filters (/Secure/TestMe/Filter ... - 1 File Download

8/12/2016 MyPastest

https://mypastest.pastest.com/Secure/TestMe/Browser/429893#Top 1/2

Back to Filters (/Secure/TestMe/Filter/429893/QA)

Question 95 of 179

46610

A 34-year-old woman has an acute attack of migraine, and is given a subcutaneous injectionof sumatriptan that brings about prompt relief of her symptoms.

Which of the following statements best explains the mechanism of action of sumatriptan?

Explanation

The answer is Acting on 5-HT receptors in the CNS –

Sumatriptan is a selective 5-HT (5-hydroxytryptamine, serotonin) receptor agonist usedto treat migraine in the acute phase. It may be effective in treating both classical andcommon migraine. It may be administered by the oral, sublingual, subcutaneous route, ornasal spray. It causes vasoconstriction of the cranial arteries and it may also act directlyon neurotransmitters involved in the pain cascade within the trigeminal nerve.

Acting on opioid receptors in the central nervous system (Option B) is incorrect. Opioidsexert their analgesic effects via opioid receptors within the central nervous system, but havelittle efficacy in acute migraine.

Blocking adrenergic receptors (Option C) is incorrect. Sumatriptan has no effect onadrenergic pathways.

Blocking receptors to acetylcholine (Option D) is incorrect. Sumatriptan has no effect onacetylcholine pathways.

Inhibiting cyclo-oxygenase (Option E) is incorrect. NSAIDs exert their effects by inhibitingcyclo-oxygenase, and may be effective in alleviating headache in acute migraine.

Acting on 5-HT receptors in the CNSA

Acting on opioid receptors in the central nervous systemB

Blocking adrenergic receptorsC

Blocking receptors to acetylcholineD

Inhibiting cyclo-oxygenaseE

Page 190: Back to Filters (/Secure/TestMe/Filter ... - 1 File Download

8/12/2016 MyPastest

https://mypastest.pastest.com/Secure/TestMe/Browser/429893#Top 2/2

Next Question

Previous Question Tag Question Feedback End Review

Difficulty: Average

Peer Responses

Session Progress

0Responses Correct:

179Responses Incorrect:

179Responses Total:

0%Responses - % Correct:

Blog (https://www.pastest.com/blog) About Pastest (https://www.pastest.com/about-us)Contact Us (https://www.pastest.com/contact-us) Help (https://www.pastest.com/help)

© Pastest 2016

Page 191: Back to Filters (/Secure/TestMe/Filter ... - 1 File Download

8/12/2016 MyPastest

https://mypastest.pastest.com/Secure/TestMe/Browser/429893#Top 1/2

Back to Filters (/Secure/TestMe/Filter/429893/QA)

Question 96 of 179

45801

A 24-year-old man is brought into the Emergency Department after he was found collapsedin the street following a suspected drug overdose. A BM test shows hypoglycaemia, and asubsequent formal laboratory test shows serum glucose 1.8 mmol/l; the patient is treated byintravenous dextrose administration.

Which of the following drugs is most likely to have been responsible for hypoglycaemia?

Explanation

The answer is Propranolol –

A number of medical disorders may give rise to hypoglycaemia. Few medicines arecapable of causing severe hypoglycaemia, notably insulin and sulfonylureas. Othermedications may cause modest hypoglycaemia, including salicylates, sodium valproate,propranolol and severe iron or paracetamol poisoning. In contrast, a number ofmedicines may cause hyperglycaemia, namely corticosteroids, thiazide diuretics,theophylline, iron (initial period after overdose), caffeine and ß -agonists.

Iron (Option A) is incorrect. Iron causes hyperglycaemia in the initial period after overdose.

Metformin (Option B) is incorrect. Metformin is an antidiabetic drug that does not causehypoglycaemia, even after substantial overdose, but is a recognised cause of severe lacticacidosis.

Prednisolone (Option C) is incorrect. Prednisolone causes hyperglycaemia.

Theophylline (Option E) is incorrect. Theophylline may cause hyperglycaemia.

IronA

MetforminB

PrednisoloneC

PropranololD

TheophyllineE

2

Page 192: Back to Filters (/Secure/TestMe/Filter ... - 1 File Download

8/12/2016 MyPastest

https://mypastest.pastest.com/Secure/TestMe/Browser/429893#Top 2/2

Next Question

Previous Question Tag Question Feedback End Review

Difficulty: Average

Peer Responses

Session Progress

0Responses Correct:

179Responses Incorrect:

179Responses Total:

0%Responses - % Correct:

Blog (https://www.pastest.com/blog) About Pastest (https://www.pastest.com/about-us)Contact Us (https://www.pastest.com/contact-us) Help (https://www.pastest.com/help)

© Pastest 2016

Page 193: Back to Filters (/Secure/TestMe/Filter ... - 1 File Download

8/12/2016 MyPastest

https://mypastest.pastest.com/Secure/TestMe/Browser/429893#Top 1/2

Back to Filters (/Secure/TestMe/Filter/429893/QA)

Question 97 of 179

46766

A 45-year-old man has a past history of hypertension and epilepsy. He has been receivingtreatment with phenytoin, clobazam, lisinopril, cimetidine, sucralfate and allopurinol. He nowpresents to the Emergency Department with progressive ataxia, slurred speech and blurredvision.

Which recently added drug is most likely to be responsible for development of hissymptoms?

Explanation

The answer is Cimetidine –

The symptoms of ataxia, slurred speech and blurred vision are due to phenytoin toxicityin this patient, which has been triggered by addition of cimetidine, an enzyme inhibitorthat reduces hepatic metabolism of certain drugs including phenytoin.

Allopurinol (Option A) is incorrect. Allopurinol is an enzyme inhibitor that may interact withphenytoin, but the interaction is much less significant than for cimetidine, which is whyoption B is the preferred answer.

Clobazam (Option C) is incorrect. There is no apparent interaction between clobazam andphenytoin.

Lisinopril (Option D) is incorrect. There is no apparent interaction between lisinopril andphenytoin.

Sucralfate (Option E) is incorrect. Sucralfate may decrease the pharmacological effects ofphenytoin when administered concurrently.

AllopurinolA

CimetidineB

ClobazamC

LisinoprilD

SucralfateE

Page 194: Back to Filters (/Secure/TestMe/Filter ... - 1 File Download

8/12/2016 MyPastest

https://mypastest.pastest.com/Secure/TestMe/Browser/429893#Top 2/2

46766

Next Question

Previous Question Tag Question Feedback End Review

Difficulty: Average

Peer Responses

Session Progress

0Responses Correct:

179Responses Incorrect:

179Responses Total:

0%Responses - % Correct:

Blog (https://www.pastest.com/blog) About Pastest (https://www.pastest.com/about-us)Contact Us (https://www.pastest.com/contact-us) Help (https://www.pastest.com/help)

© Pastest 2016

Page 195: Back to Filters (/Secure/TestMe/Filter ... - 1 File Download

8/12/2016 MyPastest

https://mypastest.pastest.com/Secure/TestMe/Browser/429893#Top 1/2

Back to Filters (/Secure/TestMe/Filter/429893/QA)

Question 98 of 179

A 34-year-old woman is admitted for management of chemotherapy-induced nausea andvomiting.

What is the most important mode of the antiemetic action of ondansetron?

Explanation

The answer is Serotonin receptor subtype 5-hydroxytryptamine (5-HT ) antagonist –

Ondansetron is a selective antagonist of the serotonin receptor subtype, 5-HT .Cytotoxic chemotherapy and radiotherapy are associated with the release of serotonin(5-HT) from enterochromaffin cells of the small intestine, presumably initiating avomiting reflex through stimulation of 5-HT receptors located on vagal afferents.Ondansetron may block this reflex. Activation of vagal afferents may also cause a centralrelease of serotonin from the chemoreceptor trigger zone of the area postrema, locatedon the floor of the fourth ventricle; the antiemetic effect of ondansetron is probably dueto the selective antagonism of 5-HT receptors on neurones located in either theperipheral or central nervous system.

Dopamine agonist (Option A) is incorrect. Dopamine agonists, such as those used intreatment of parkinsonism, often cause significant nausea.

Dopamine antagonist (Option B) is incorrect. Certain antiemetics exert their effects throughdopamine antagonism, e.g. domperidone.

Histamine agonist (Option C) is incorrect. Antihistamines possess some antiemeticproperties; this may be a relevant mechanism of cyclizine, which possesses centralantihistamine and anticholinertic properties.

Dopamine agonistA

Dopamine antagonistB

Histamine agonistC

Serotonin receptor subtype 5-hydroxytryptamine agonistD

Serotonin receptor subtype 5-hydroxytryptamine antagonistE

3

3

3

3

Page 196: Back to Filters (/Secure/TestMe/Filter ... - 1 File Download

8/12/2016 MyPastest

https://mypastest.pastest.com/Secure/TestMe/Browser/429893#Top 2/2

46943

Serotonin receptor subtype 5-hydroxytryptamine (5-HT ) agonist (Option D) is incorrect.Stimulation of 5-HT receptors is a central mechanism involved in development ofchemotherapy-induced nausea and vomiting.

3

3

Next Question

Previous Question Tag Question Feedback End Review

Difficulty: Average

Peer Responses

Session Progress

0Responses Correct:

179Responses Incorrect:

179Responses Total:

0%Responses - % Correct:

Blog (https://www.pastest.com/blog) About Pastest (https://www.pastest.com/about-us)Contact Us (https://www.pastest.com/contact-us) Help (https://www.pastest.com/help)

© Pastest 2016

Page 197: Back to Filters (/Secure/TestMe/Filter ... - 1 File Download

8/12/2016 MyPastest

https://mypastest.pastest.com/Secure/TestMe/Browser/429893#Top 1/2

Back to Filters (/Secure/TestMe/Filter/429893/QA)

Question 99 of 179

46834

A 59-year-old woman who suffers from arthritis and hypertension visits you and asks you toprescribe increased pain relief.

Which of the following agents is most likely to exert its analgesic effects by specificallyinhibiting COX-2 activity?

Explanation

The answer is Celecoxib –

Both celecoxib and rofecoxib are selective COX-2 inhibitors (coxibs). Prior to thedevelopment of the coxibs it was postulated that the renal toxicity and increasedgastrointestinal bleeding effects of non-steroidal anti-inflammatory drugs (NSAIDs)occur as a result of COX-1 inhibition, and that the major anti-inflammatory effects ofNSAIDs are due to inhibition of COX-2. However, recent data have shown that selectiveinhibition of COX-2 may be associated with increased cardiovascular toxicity.

Aspirin (Option A) is incorrect. Aspirin inhibits cyclo-oxygenase but it is not selective forCOX-2.

Co-proxamol (Option C) is incorrect. Co-proxamol is a fixed dose combination ofparacetamol and dextropropoxyphene.

Diclofenac (Option D) is incorrect. Diclofenac is a non-selective COX inhibitor NSAID.

Nefopam (Option E) is incorrect. Nefopam is a NSAID but it is not selective for COX-2isoenzyme.

AspirinA

CelecoxibB

Co-proxamolC

DiclofenacD

NefopamE

Page 198: Back to Filters (/Secure/TestMe/Filter ... - 1 File Download

8/12/2016 MyPastest

https://mypastest.pastest.com/Secure/TestMe/Browser/429893#Top 2/2

Next Question

Previous Question Tag Question Feedback End Review

Difficulty: Average

Peer Responses

Session Progress

0Responses Correct:

179Responses Incorrect:

179Responses Total:

0%Responses - % Correct:

Blog (https://www.pastest.com/blog) About Pastest (https://www.pastest.com/about-us)Contact Us (https://www.pastest.com/contact-us) Help (https://www.pastest.com/help)

© Pastest 2016

Page 199: Back to Filters (/Secure/TestMe/Filter ... - 1 File Download

8/12/2016 MyPastest

https://mypastest.pastest.com/Secure/TestMe/Browser/429893#Top 1/2

Back to Filters (/Secure/TestMe/Filter/429893/QA)

Question 100 of 179

Two patients are brought in from the same house. They have been undertaking renovationworks recently, and you suspect possible carbon monoxide poisoning.

Which of the following features would most strongly indicate severe carbon monoxidepoisoning?

Explanation

The answer is ST segment depression in leads V4, V5 and V6 on a resting ECG –

Carbon monoxide is a common cause of accidental poisoning fatalities in the UK. Theclinical features are non-specific and the diagnosis can be easily overlooked. Symptomsand signs are attributable to tissue hypoxaemia, and include reduced conscious level,cerebellar toxicity, myocardial ischaemia with corresponding ECG changes, and lacticacidosis. Severe toxicity is often associated with high carboxyhaemoglobinconcentrations, typically > 30% if patients present early, although these may fall within afew hours of removal from exposure. Patients with pre-existing vascular disease are at anincreased risk of morbidity and mortality from carbon monoxide poisoning. Remove thepatient from the source of the carbon monoxide, and treat with high-flow oxygen via atight-fitting mask without a re-breathing circuit. Hyperbaric oxygen therapy may beconsidered if carboxyhaemoglobin concentrations are over 40% and there areneurological features.

Carboxyhaemoglobin concentration of 9% (Option A) is incorrect. Carboxyhaemoglobinconcentrations are typically less than 5% in healthy adults, but as high as 10% in heavysmokers.

Carboxyhaemoglobin concentration of 9%A

HeadacheB

Impairment of fine motor coordination in the upper limbsC

ST segment depression in leads V4, V5 and V6 on a resting ECGD

Systolic blood pressure less than 100 mmHgE

Page 200: Back to Filters (/Secure/TestMe/Filter ... - 1 File Download

8/12/2016 MyPastest

https://mypastest.pastest.com/Secure/TestMe/Browser/429893#Top 2/2

45794

Headache (Option B) is incorrect. Headache is a recognised feature of carbon monoxideexposure, especially chronic exposure, but is a non-specific symptom that does not reliablycorrelate with severity.

Impairment of fine motor coordination in the upper limbs (Option C) is incorrect. Minorneurological impairment may occur, including cerebellar impairment, but this is less specificthan ischaemic ECG changes and a less preferred option in this scenario.

Systolic blood pressure less than 100 mmHg (Option E) is incorrect. Inadequate delivery ofoxygen to tissues is caused by altered binding affinity to haemoglobin, rather than a fall inblood pressure. Patients may have few haemodynamic effects, even after severe carbonmonoxide exposure.

Next Question

Previous Question Tag Question Feedback End Review

Difficulty: Average

Peer Responses

Session Progress

0Responses Correct:

179Responses Incorrect:

179Responses Total:

0%Responses - % Correct:

Blog (https://www.pastest.com/blog) About Pastest (https://www.pastest.com/about-us)Contact Us (https://www.pastest.com/contact-us) Help (https://www.pastest.com/help)

© Pastest 2016

Page 201: Back to Filters (/Secure/TestMe/Filter ... - 1 File Download

8/12/2016 MyPastest

https://mypastest.pastest.com/Secure/TestMe/Browser/429893#Top 1/2

Back to Filters (/Secure/TestMe/Filter/429893/QA)

Question 101 of 179

A 62-year-old woman presents to the Rheumatology Clinic for review. She attended theEmergency Department some 6 weeks earlier following a left Colles’ fracture and isconcerned about osteoporosis. A past history of venous thromboembolism and gastro-oesophageal reflux disease for which she is treated with daily Omeprazole is noted. Onexamination her BP is 132/80 mmHg; pulse is 75/min and regular. Her BMI is 23. The wristfracture is enclosed in a lightweight cast.

Investigations;

Hb 12.9 g/dl

WCC 7.1 x10 /l

PLT 201 x10 /l

Na 137 mmol/l

K 4.3 mmol/l

Creatinine 91 micromol/l

Glucose 5.8 mmol/l

Calcium 2.3 mmol/l

ALP 81 U/l

DEXA – T score -2.7

Which of the following is the most appropriate intervention?

9

9

+

+

Calcium and vitamin DA

DenosumabB

HRTC

RaloxifeneD

RisedronateE

Page 202: Back to Filters (/Secure/TestMe/Filter ... - 1 File Download

8/12/2016 MyPastest

https://mypastest.pastest.com/Secure/TestMe/Browser/429893#Top 2/2

40149

Explanation

The answer is Denosumab -

In this situation with a history of GORD treated with PPI, and given the patient’s young age,bisphosphonates are a suboptimal choice. They run the risk of significantly worseningoesophagitis and leading to adynamic bone disease with long-term use. Whilst SERMs suchas Raloxifene are a reasonable second line choice, they are less effective thanbisphosphonates in reducing risk of fracture. Denosumab, given by 6 monthly subcutaneousinjections, is potentially as effective as bisphosphonates, with a superior adverse eventprofile.

Next Question

Previous Question Tag Question Feedback End Review

Difficulty: Difficult

Peer Responses

Session Progress

0Responses Correct:

179Responses Incorrect:

179Responses Total:

0%Responses - % Correct:

Blog (https://www.pastest.com/blog) About Pastest (https://www.pastest.com/about-us)Contact Us (https://www.pastest.com/contact-us) Help (https://www.pastest.com/help)

© Pastest 2016

Page 203: Back to Filters (/Secure/TestMe/Filter ... - 1 File Download

8/12/2016 MyPastest

https://mypastest.pastest.com/Secure/TestMe/Browser/429893#Top 1/2

Back to Filters (/Secure/TestMe/Filter/429893/QA)

Question 102 of 179

46946

Which of the following drugs is most likely to require plasma level monitoring in routineclinical practice?

Explanation

The answer is Vancomycin –

All patients require plasma vancomycin measurement (after three or four doses if renalfunction is normal, or earlier if renal impairment is present). There is a risk ofnephrotoxicity including renal failure, interstitial nephritis and ototoxicity.

Bleomycin (Option A) is incorrect. Although bleomycin is highly toxic, there is no clearrelationship between blood concentrations and risks of toxicity.

Cefuroxime (Option B) is incorrect. There is a poor correlation between cefuroxime drugconcentrations and therapeutic efficacy or risk of adverse effects, and it has a broadtherapeutic window.

Ciprofloxacin (Option C) is incorrect. Ciprofloxacin has a wide therapeutic window and doesnot require therapeutic monitoring.

Erythromycin (Option D) is incorrect. There is a poor correlation between erythromycin drugconcentrations and therapeutic efficacy or risk of adverse effects.

BleomycinA

CefuroximeB

CiprofloxacinC

ErythromycinD

VancomycinE

Next Question

Page 204: Back to Filters (/Secure/TestMe/Filter ... - 1 File Download

8/12/2016 MyPastest

https://mypastest.pastest.com/Secure/TestMe/Browser/429893#Top 2/2

Previous Question Tag Question Feedback End Review

Difficulty: Average

Peer Responses

Session Progress

0Responses Correct:

179Responses Incorrect:

179Responses Total:

0%Responses - % Correct:

Blog (https://www.pastest.com/blog) About Pastest (https://www.pastest.com/about-us)Contact Us (https://www.pastest.com/contact-us) Help (https://www.pastest.com/help)

© Pastest 2016

Page 205: Back to Filters (/Secure/TestMe/Filter ... - 1 File Download

8/13/2016 MyPastest

https://mypastest.pastest.com/Secure/TestMe/Browser/429893#Top 1/2

Back to Filters (/Secure/TestMe/Filter/429893/QA)

Question 103 of 179

An elderly man on digoxin treatment for atrial fibrillation presented to his GP 3 weeks agocomplaining of pitting oedema affecting both ankles and breathlessness on exertion, and wasprescribed a new medication. He now presents to the Emergency Department with severenausea, vomiting and visual disturbance, and a resting ECG shows complete heart block.

Which of the following new agents is most likely to account for development of hissymptoms and signs?

Explanation

The answer is Furosemide –

The signs and symptoms of complete heart block and profound nausea and vomiting arecharacteristic of digoxin toxicity. Other features typically include xanthopsia (yellowhalo), bradyarrhythmia and hypotension. Administration of a thiazide or loop diureticmay enhance digoxin concentrations and increase cardiac sensitivity to digoxin bycausing hypokalaemia.

Amiodarone (Option A) is incorrect. Amiodarone certainly interacts with digoxin byincreasing drug concentrations and predisposing to digoxin toxicity. However, it is a lesspreferred answer here than option B because the patient is unlikely to have been prescribedamiodarone for treatment of heart failure symptoms (breathlessness and peripheraloedema).

Propranolol (Option C) is incorrect. Propranolol may enhance bradycardia and AV nodalblockade when administered in combination with digoxin, but does not provoke digoxintoxicity.

AmiodaroneA

FurosemideB

PropranololC

SotalolD

WarfarinE

Page 206: Back to Filters (/Secure/TestMe/Filter ... - 1 File Download

8/13/2016 MyPastest

https://mypastest.pastest.com/Secure/TestMe/Browser/429893#Top 2/2

46770

Sotalol (Option D) is incorrect. Sotalol does not provoke digoxin toxicity.

Warfarin (Option E) is incorrect. Warfarin would not be expected provoke digoxin toxicity.

Next Question

Previous Question Tag Question Feedback End Review

Difficulty: Average

Peer Responses

Session Progress

0Responses Correct:

179Responses Incorrect:

179Responses Total:

0%Responses - % Correct:

Blog (https://www.pastest.com/blog) About Pastest (https://www.pastest.com/about-us)Contact Us (https://www.pastest.com/contact-us) Help (https://www.pastest.com/help)

© Pastest 2016

Page 207: Back to Filters (/Secure/TestMe/Filter ... - 1 File Download

8/13/2016 MyPastest

https://mypastest.pastest.com/Secure/TestMe/Browser/429893#Top 1/2

Back to Filters (/Secure/TestMe/Filter/429893/QA)

Question 104 of 179

You review a 17-year-old girl who has taken 20 g of acetaminophen (paracetamol).Acetaminophen is an important cause of acute hepatic failure.

Which of the following statements concerning paracetamol overdose is correct?

Explanation

The answer is N-acetylcysteine is most effective when administered within 8 h ofingestion –

Acetaminophen overdose causes acute liver failure. Liver injury is caused by toxicmetabolites of acetaminophen produced by liver mixed oxidase enzymes. Thismetabolite is usually conjugated with glutathione and excreted via a non-toxic pathway;in overdose, glutathione stores are depleted. N-acetylcysteine is an SH-group donator,providing alternative substrate for conjugation.

Alcoholics are less susceptible to liver injury even with a low dose (Option A) is incorrect.Regular ethanol consumption is associated with liver enzyme induction, therefore severehepatotoxicity can be seen in alcoholics, even with lower dosages of paracetamol.

Haemodialysis is effective in the management of hepatotoxicity (Option B) is incorrect.Haemodialysis may be useful for managing acute renal failure occurring in conjunction withparacetamol-induced liver failure, but is not a treatment for hepatotoxicity.

Significant liver injury rarely occurs with doses of less than 20 g (Option D) is incorrect.Significant liver injury usually occurs with doses of >10 to 15 g, particularly in patients that

Alcoholics are less susceptible to liver injury even with a low doseA

Haemodialysis is effective in the management of hepatotoxicityB

N-acetylcysteine is most effective when administered within 8 h of ingestionC

Significant liver injury rarely occurs with doses of less than 20 gD

Survivors of acetaminophen-induced hepatotoxicity always experience residual liverdamage

E

Page 208: Back to Filters (/Secure/TestMe/Filter ... - 1 File Download

8/13/2016 MyPastest

https://mypastest.pastest.com/Secure/TestMe/Browser/429893#Top 2/2

46933

present late to hospital.

Survivors of acetaminophen-induced hepatotoxicity always experience residual liver damage(Option E) is incorrect. In survivors of acute acetaminophen toxicity, there is usuallycomplete recovery without any cirrhosis or residual liver damage.

Next Question

Previous Question Tag Question Feedback End Review

Difficulty: Average

Peer Responses

Session Progress

0Responses Correct:

179Responses Incorrect:

179Responses Total:

0%Responses - % Correct:

Blog (https://www.pastest.com/blog) About Pastest (https://www.pastest.com/about-us)Contact Us (https://www.pastest.com/contact-us) Help (https://www.pastest.com/help)

© Pastest 2016

Page 209: Back to Filters (/Secure/TestMe/Filter ... - 1 File Download

8/13/2016 MyPastest

https://mypastest.pastest.com/Secure/TestMe/Browser/429893#Top 1/2

Back to Filters (/Secure/TestMe/Filter/429893/QA)

Question 105 of 179

You are seeing a 17-year-old woman in the respiratory clinic for a review of her asthma. Shehas been taking inhaled disodium cromoglicate for several years.

Which of the following mechanisms is most important in the action of disodium cromoglicateto prevent asthma attacks?

Explanation

The answer is option D, inhibition of mast-cell degranulation -

Sodium cromoglicate principally acts by reducing the degranulation of mast cellstriggered by the interaction of antigen and IgE. The inhibitory effect on mast cellsappears to be cell-type specific, since cromoglicate has little inhibitory effect onmediator release from human basophils.

Action on eosinophils to reduce the inflammatory response to inhaled allergens (Option A) isincorrect. Disodium cromoglicate may also act on eosinophils to reduce their inflammatoryresponse to inhaled allergens, but the relevance to asthma is uncertain.

Competitive blockade of histamine at receptor sites (Option B) is incorrect. Anticholinergiceffects are not seen with disodium cromoglicate.

Inhibition of acetylcholine at the synaptic junctions (Option C) is incorrect. Disodiumcromoglicate does not exert anticholinergic effects. Ipratropium and tiotropium exertantimuscarinic effects, and are effective in reducing airway inflammation in COPD.

Potentiation of the effects of β2-receptor agonists (Option E) is incorrect; β2-adrenoceptoragonists also appear to be capable of reducing histamine release through a similar

Action on eosinophils to reduce the inflammatory response to inhaled allergensA

Competitive blockade of histamine at receptor sitesB

Inhibition of acetylcholine at the synaptic junctionsC

Inhibition of mast-cell degranulationD

Potentiation of the effects of β2-receptor agonistsE

Page 210: Back to Filters (/Secure/TestMe/Filter ... - 1 File Download

8/13/2016 MyPastest

https://mypastest.pastest.com/Secure/TestMe/Browser/429893#Top 2/2

46749

mechanism, but their actions are not potentiated by disodium cromoglicate.

Next Question

Previous Question Tag Question Feedback End Review

Difficulty: Average

Peer Responses

Session Progress

0Responses Correct:

179Responses Incorrect:

179Responses Total:

0%Responses - % Correct:

Blog (https://www.pastest.com/blog) About Pastest (https://www.pastest.com/about-us)Contact Us (https://www.pastest.com/contact-us) Help (https://www.pastest.com/help)

© Pastest 2016

Page 211: Back to Filters (/Secure/TestMe/Filter ... - 1 File Download

8/13/2016 MyPastest

https://mypastest.pastest.com/Secure/TestMe/Browser/429893#Top 1/2

Back to Filters (/Secure/TestMe/Filter/429893/QA)

Question 106 of 179

22487

A 44-year-old man comes to the Dermatology Clinic. He has severe psoriasis and has failedconventional therapy. You elect to begin treatment with ciclosporin.

Which one of the following correctly reflects one of the main modes of action of ciclosporin?

Explanation

Ciclosporin

Ciclosporin is thought to have two main actions

arrest of activity of resting lymphocytes in the G0 or G1 phase of the growth

cycle

cytokine inhibition, the major one being inhibition of IL-2

Ciclosporin appears to act specifically on lymphocytes and does not depress red cell

production or the function of phagocytic white blood cells

Ciclosporin levels are particularly affected by drugs metabolised by the CYP3A4 p450

enzyme

For this reason particular caution is advised when prescribing a number of agents such

as statins and macrolide antibiotics

The British National Formulary needs to be consulted when considering prescribing

additional medication in patients taking ciclosporin

https://www.medicines.org.uk/EMC/medicine/1307/SPC/Neoral+Soft+Gelatin+Capsules,+Neoral+Oral+Solution/(https://www.medicines.org.uk/EMC/medicine/1307/SPC/Neoral+Soft+Gelatin+Capsules,+Neoral+Oral+Solution/)

It inhibits IL-6 activityA

It inhibits IL-2 activityB

It inhibits IL-1 activityC

It inhibits TNF-α activityD

It inhibits IL-4 activityE

Next Question

Previous Question Tag Question Feedback End Review

Difficulty: Average

Peer Responses

Page 212: Back to Filters (/Secure/TestMe/Filter ... - 1 File Download

8/13/2016 MyPastest

https://mypastest.pastest.com/Secure/TestMe/Browser/429893#Top 2/2

Session Progress

0Responses Correct:

179Responses Incorrect:

179Responses Total:

0%Responses - % Correct:

Blog (https://www.pastest.com/blog) About Pastest (https://www.pastest.com/about-us)Contact Us (https://www.pastest.com/contact-us) Help (https://www.pastest.com/help)

© Pastest 2016

Page 213: Back to Filters (/Secure/TestMe/Filter ... - 1 File Download

8/13/2016 MyPastest

https://mypastest.pastest.com/Secure/TestMe/Browser/429893#Top 1/2

Back to Filters (/Secure/TestMe/Filter/429893/QA)

Question 107 of 179

A patient with coronary heart disease and high LDL-cholesterol was started on simvastatin 6months ago. His GP recently started him on another medication and now, 2 weeks later, he iscomplaining of muscle pain and weakness. Tests reveal an elevated creatine kinase (CK), tentimes greater than the upper limit of normal.

Which of the following additional drugs is most likely to have resulted in this adverse effect?

Explanation

The answer is Clarithromycin –

Statins inhibit 3-hydroxy-3-methyl-glutaryl-CoA, the rate limiting step in cholesterolsynthesis. These can cause myopathy and rhabdomyolysis, associated with muscle pain,tenderness or weakness. Rhabdomyolysis may cause acute renal failure secondary tomyoglobinuria. The risk of myopathy and rhabdomyolysis is greatest in patients receivinghigh statin doses. The risks of simvastatin-induced myopathy are increased byconcomitant use of potent inhibitors of cytochrome P450 isoenzyme 3A4 inhibitors,such as erythromycin, itraconazole, ketoconazole, clarithromycin, gemfibrozil andciclosporin (the effect of these agents on other statins is recognised but less importantthan with simvastatin).

Amlodipine (Option A) is incorrect. Calcium channel blockers may cause peripheral oedemabut are unlikely to interact significantly with simvastatin.

Aspirin (Option B) is incorrect. Aspirin does not interfere with simvastatin and can be safelyco-prescribed.

AmlodipineA

AspirinB

AtenololC

ClarithromycinD

RifampicinE

Page 214: Back to Filters (/Secure/TestMe/Filter ... - 1 File Download

8/13/2016 MyPastest

https://mypastest.pastest.com/Secure/TestMe/Browser/429893#Top 2/2

46789

Atenolol (Option C) is incorrect. Beta-blockers may reduce peripheral blood flow through acombination of increased peripheral resistance and reduced cardiac output. This oftencauses muscle cramps but myopathy is not a recognised feature.

Rifampicin (Option E) is incorrect. Rifampicin is a powerful enzyme inducer that might withlong-term treatment enhance the metabolism of simvastatin and might lessen itseffectiveness.

Next Question

Previous Question Tag Question Feedback End Review

Difficulty: Average

Peer Responses

Session Progress

0Responses Correct:

179Responses Incorrect:

179Responses Total:

0%Responses - % Correct:

Blog (https://www.pastest.com/blog) About Pastest (https://www.pastest.com/about-us)Contact Us (https://www.pastest.com/contact-us) Help (https://www.pastest.com/help)

© Pastest 2016

Page 215: Back to Filters (/Secure/TestMe/Filter ... - 1 File Download

8/13/2016 MyPastest

https://mypastest.pastest.com/Secure/TestMe/Browser/429893#Top 1/2

Back to Filters (/Secure/TestMe/Filter/429893/QA)

Question 108 of 179

A 22-year-old woman is admitted to the Emergency Department. Her mother suffers frommanic depression. Apparently she had a row with her boyfriend the previous evening and wasfound by her mother that morning. You understand that she had two seizures during theambulance journey to hospital. On examination she is deeply unconscious and her bloodpressure is 148/94 mmHg. She has bilateral increased tone.

Investigations:

Hb 13.1 g/dl

White cell count 5.4 × 10 /l

Platelets 230 × 10 /l

Na 139 mmol/l

K 4.9 mmol/l

Creatinine 155 μmol/l

Lithium 3.7 mmol/l (therapeutic range 0.6–1.5)

Which one of the following is the most appropriate way to manage her?

Explanation

Lithium toxicity

9

9

+

+

Charcoal via nasogastric tubeA

Forced alkaline diuresisB

IV furosemide 120 mgC

HaemodialysisD

IV normal salineE

Page 216: Back to Filters (/Secure/TestMe/Filter ... - 1 File Download

8/13/2016 MyPastest

https://mypastest.pastest.com/Secure/TestMe/Browser/429893#Top 2/2

20938

Haemodialysis is the management of choice for severe lithium toxicity

Both lithium concentration and the change in mental status drive the need for dialysis

Activated charcoal is not very effective in lithium overdose, but charcoal should still be

given if co-ingestion of another agent is suspected

Gastric lavage may be considered in patients presenting very rapidly (within an hour) of

overdose

For patients with mild to moderate lithium overdose, normal saline rehydration may

help drive lithium excretion

For most patients the outcome is favourable, although up to 10% of cases of severe

overdose may carry some form of long-term neurological deficit

Next Question

Previous Question Tag Question Feedback End Review

Difficulty: Average

Peer Responses

Session Progress

0Responses Correct:

179Responses Incorrect:

179Responses Total:

0%Responses - % Correct:

Blog (https://www.pastest.com/blog) About Pastest (https://www.pastest.com/about-us)Contact Us (https://www.pastest.com/contact-us) Help (https://www.pastest.com/help)

© Pastest 2016

Page 217: Back to Filters (/Secure/TestMe/Filter ... - 1 File Download

8/13/2016 MyPastest

https://mypastest.pastest.com/Secure/TestMe/Browser/429893#Top 1/2

Back to Filters (/Secure/TestMe/Filter/429893/QA)

Question 109 of 179

A 67-year-old man with known long-standing asthma presents to the Emergency Departmentfollowing an intentional overdose of his regular theophylline. He reports having taken 20 ofhis theophylline tablets 1 hour earlier to end his life.

Which of the following features would most strongly suggest severe theophylline toxicity?

Explanation

The correct answer is Severe nausea and vomiting –

Management of a theophylline overdose includes multi-dose activated charcoal, whichincreases drug clearance by interfering with enterohepatic recirculation; however, it isoften impossible due to severe vomiting. Other features of severe toxicity includeseizures, myoclonus, hyperthermia, rhabdomyolysis and acute renal failure. Nausea andvomiting are characteristic features of theophylline poisoning.

Hyperkalaemia (Option A) is incorrect. Hypokalaemia is a characteristic feature, and maycorrect rapidly, so frequent monitoring of potassium levels is important to avoid excesspotassium administration.

Hypoglycaemia (Option B) is incorrect. Hyperglycaemia may occur.

Right bundle branch block on a resting ECG (Option C) is incorrect. Conduction defects andbundle branch block may occur but are uncommon. Cardiac arrhythmias typically includesinus tachycardia, atrial and ventricular arrhythmias, and prolongation of the QT and QRSintervals. Arrhythmias are more common in patients with electrolyte disturbances, includinghypokalaemia and metabolic acidosis.

HyperkalaemiaA

HypoglycaemiaB

Right bundle branch block on a resting ECGC

Severe nausea and vomitingD

Sinus bradycardiaE

Page 218: Back to Filters (/Secure/TestMe/Filter ... - 1 File Download

8/13/2016 MyPastest

https://mypastest.pastest.com/Secure/TestMe/Browser/429893#Top 2/2

45795

Sinus bradycardia (Option E) is incorrect. Sinus tachycardia is a common feature.

Next Question

Previous Question Tag Question Feedback End Review

Difficulty: Average

Peer Responses

Session Progress

0Responses Correct:

179Responses Incorrect:

179Responses Total:

0%Responses - % Correct:

Blog (https://www.pastest.com/blog) About Pastest (https://www.pastest.com/about-us)Contact Us (https://www.pastest.com/contact-us) Help (https://www.pastest.com/help)

© Pastest 2016

Page 219: Back to Filters (/Secure/TestMe/Filter ... - 1 File Download

8/13/2016 MyPastest

https://mypastest.pastest.com/Secure/TestMe/Browser/429893#Top 1/2

Back to Filters (/Secure/TestMe/Filter/429893/QA)

Question 110 of 179

A 20-year-old student is admitted to the Emergency Department after an intentionaloverdose involving her grandmother’s analgesic medications. She is pyrexial, pale, sweatingprofusely with laboured breathing. Heart rate is 102 bpm, oxygen saturations 100% on air, andblood pressure 124/68 mmHg.

Which of the following drugs is most likely to cause these findings?

Explanation

The answer is Aspirin –

Aspirin overdose initially causes a respiratory alkalosis, due to direct stimulation of thecentral respiratory centres, which provokes rapid deep breathing ‘air hunger’. Other earlyfeatures include hyperventilation, tinnitus, vertigo, sweating and mild pyrexia. As drugabsorption ensues and toxicity progresses, the later features of aspirin toxicity includemetabolic acidosis, reduced conscious level and hypoglycaemia.

Co-proxamol (Option B) is incorrect. Co-proxamol contains paracetamol anddextropropoxyphene, an opioid that is capable of prolonging QRS duration and a recognisedcause of fatal arrhythmia in overdose. Opioid effects in overdose include respiratorydepression, hypotension, cardiovascular collapse, coma and death.

Diclofenac (Option C) is incorrect. Acute overdose involving diclofenac may cause vomiting,haematemesis, peptic ulceration and renal failure.

Ibuprofen (Option D) is incorrect. Acute overdose involving ibuprofen may cause vomiting,haematemesis, peptic ulceration and renal failure.

AspirinA

Co-proxamolB

DiclofenacC

IbuprofenD

ParacetamolE

Page 220: Back to Filters (/Secure/TestMe/Filter ... - 1 File Download

8/13/2016 MyPastest

https://mypastest.pastest.com/Secure/TestMe/Browser/429893#Top 2/2

46763

Paracetamol (Option E) is incorrect. Paracetamol overdose, even in severe and fatal cases,may have few initial symptoms. Early features may include nausea and abdominal pain.

Next Question

Previous Question Tag Question Feedback End Review

Difficulty: Average

Peer Responses

Session Progress

0Responses Correct:

179Responses Incorrect:

179Responses Total:

0%Responses - % Correct:

Blog (https://www.pastest.com/blog) About Pastest (https://www.pastest.com/about-us)Contact Us (https://www.pastest.com/contact-us) Help (https://www.pastest.com/help)

© Pastest 2016

Page 221: Back to Filters (/Secure/TestMe/Filter ... - 1 File Download

8/13/2016 MyPastest

https://mypastest.pastest.com/Secure/TestMe/Browser/429893#Top 1/2

Back to Filters (/Secure/TestMe/Filter/429893/QA)

Question 111 of 179

46835

You are asked to review a 62-year-old man who takes nifedipine for hypertension. He asks foradvice about eating a healthy diet. His wife is following a homeopathic ‘fruit juice regimen’.

Which of the following fruit juices would he be wisest to avoid?

Explanation

The answer is Grapefruit juice –

Nifedipine is metabolised via the CYP3A4 cytochrome P450 isoenzyme. Grapefruit juiceis capable of powerful inhibition of the CYP3A4 isoenzyme and may significantlyincrease the toxicity of nifedipine, simvastatin and other drugs metabolised by thisisoenzyme.

Apple juice (Option A) is incorrect. Apple juice is unlikely to have any significant effect onCYP3A4 activity.

Blackcurrant juice (Option B) is incorrect. Blackcurrant juice is unlikely to have any significanteffect on CYP3A4 activity.

Orange juice (Option D) is incorrect. Orange juice is unlikely to have any significant effect onCYP3A4 activity.

Tomato juice (Option E) is incorrect. Tomato juice is unlikely to have any significant effect onCYP3A4 activity.

Apple juiceA

Blackcurrant juiceB

Grapefruit juiceC

Orange juiceD

Tomato juiceE

Page 222: Back to Filters (/Secure/TestMe/Filter ... - 1 File Download

8/13/2016 MyPastest

https://mypastest.pastest.com/Secure/TestMe/Browser/429893#Top 2/2

Next Question

Previous Question Tag Question Feedback End Review

Difficulty: Average

Peer Responses

Session Progress

0Responses Correct:

179Responses Incorrect:

179Responses Total:

0%Responses - % Correct:

Blog (https://www.pastest.com/blog) About Pastest (https://www.pastest.com/about-us)Contact Us (https://www.pastest.com/contact-us) Help (https://www.pastest.com/help)

© Pastest 2016

Page 223: Back to Filters (/Secure/TestMe/Filter ... - 1 File Download

8/13/2016 MyPastest

https://mypastest.pastest.com/Secure/TestMe/Browser/429893#Top 1/2

Back to Filters (/Secure/TestMe/Filter/429893/QA)

Question 112 of 179

You are reviewing an 18-year-old young woman who has been brought into the EmergencyDepartment after a row with her boyfriend. The ambulance crew hand you a number ofempty bottles of dihydrocodeine. On examination she is unconscious with a GCS of 6.

Which of the following drugs would be the most appropriate antidote for dihydrocodeine?

Explanation

The answer is Naloxone –

Naloxone is an opiate antagonist indicated for the reversal of opioid-induced respiratorydepression or opioid overdose. Naloxone is relatively short-acting compared with thehalf-life of opioid analgesics; this means there is a real possibility of patients becomingre-sedated as the effects wear off, it should therefore be given as repeated doses or as acontinuous infusion.

Dantrolene (Option A) is incorrect. Dantrolene may be useful in the treatment of malignanthyperthermia in serotonin syndrome.

Dextropropoxyphene (Option B) is incorrect. Dextropropoxyphene is a long-acting opioidused as a constituent of co-proxamol; it is a partial agonist and therefore the effect ofnaloxone in co-proxamol overdose is limited. Dextropropoxyphene is cardiotoxic and isresponsible for sudden cardiac death in overdose.

Flumazenil (Option C) is incorrect. Flumazenil is a benzodiazepine antagonist that should beused with caution, as rapid reversal of benzodiazepines can be associated with convulsions.

DantroleneA

DextropropoxypheneB

FlumazenilC

NaloxoneD

NeostigmineE

Page 224: Back to Filters (/Secure/TestMe/Filter ... - 1 File Download

8/13/2016 MyPastest

https://mypastest.pastest.com/Secure/TestMe/Browser/429893#Top 2/2

46829

Neostigmine (Option E) is incorrect. Neostigmine is useful for reversing the effects ofanaesthetic muscle relaxants.

Next Question

Previous Question Tag Question Feedback End Review

Difficulty: Average

Peer Responses

Session Progress

0Responses Correct:

179Responses Incorrect:

179Responses Total:

0%Responses - % Correct:

Blog (https://www.pastest.com/blog) About Pastest (https://www.pastest.com/about-us)Contact Us (https://www.pastest.com/contact-us) Help (https://www.pastest.com/help)

© Pastest 2016

Page 225: Back to Filters (/Secure/TestMe/Filter ... - 1 File Download

8/13/2016 MyPastest

https://mypastest.pastest.com/Secure/TestMe/Browser/429893#Top 1/2

Back to Filters (/Secure/TestMe/Filter/429893/QA)

Question 113 of 179

37291

You are asked to prescribe an antibiotic for a 72-year-old woman convalescing on the ElderlyCare Ward after a left total hip replacement. She has developed symptoms of a lower UTI.

Which of the following is likely to carry the lowest risk of C difficile infection?

Explanation

The answer is Trimethoprim.

Guidelines suggest that cephalosporins, broad spectrum penicillins such as co-amoxiclav,clindamycin and fluoroquinolones are all associated with increased risk of C difficile infection.As such the best option is trimethoprim. For an upper urinary tract infection the benefits ofusing co-amoxyclav would outweigh the risks of C difficile infection. Guidelines alsorecommend minimising the length of antibiotic therapy as much as possible and avoidingblanket prescribing of antibiotic therapy.

Scottish Antimicrobial Prescribing Group

http://www.scottishmedicines.org.uk/files/sapg/2011_National_prescribing_indicators__Compatibility_Mode_.pdf(http://www.scottishmedicines.org.uk/files/sapg/2011_National_prescribing_indicators__Compatibility_Mode_.pdf)

CeftriaxoneA

CephalexinB

Co-amoxiclavC

LevofloxacinD

TrimethoprimE

Next Question

Previous Question Tag Question Feedback End Review

Difficulty: Average

Peer Responses

Page 226: Back to Filters (/Secure/TestMe/Filter ... - 1 File Download

8/13/2016 MyPastest

https://mypastest.pastest.com/Secure/TestMe/Browser/429893#Top 2/2

Session Progress

0Responses Correct:

179Responses Incorrect:

179Responses Total:

0%Responses - % Correct:

Blog (https://www.pastest.com/blog) About Pastest (https://www.pastest.com/about-us)Contact Us (https://www.pastest.com/contact-us) Help (https://www.pastest.com/help)

© Pastest 2016

Page 227: Back to Filters (/Secure/TestMe/Filter ... - 1 File Download

8/13/2016 MyPastest

https://mypastest.pastest.com/Secure/TestMe/Browser/429893#Top 1/2

Back to Filters (/Secure/TestMe/Filter/429893/QA)

Question 114 of 179

A 43-year-old woman with severe rheumatoid arthritis has experienced marked symptomaticimprovement since the introduction of methotrexate therapy.

What is the most likely mechanism that accounts for the therapeutic action of methotrexatein rheumatoid arthritis?

Explanation

The answer is Inhibition of purine synthesis –

Methotrexate is an antimetabolite that exerts its effects by inhibiting dihydrofolatereductase, which interferes with tetrahydrofolate stores required for synthesis of purinesand thymidylate in inflammatory cells. Clinical trials have demonstrated the superiorefficacy of methotrexate over placebo in controlling the symptoms and signs of early andestablished rheumatoid arthritis in patients previously treated with only non-steroidalanti-inflammatory drugs. In clinical trials lasting 6–12 months, methotrexate slows theprogression of structural damage as assessed by serial radiographs of the hands andfeet.

Inhibition of thymidylate synthase (Option B) is incorrect. The anti-cancer agent 5-fluorouracil is a prodrug of fluorodeoxyuridine monophosphate, which inhibits thymidylatesynthase, an enzyme required for the synthesis of deoxythymidine triphosphate and DNA.

Inhibition of topoisomerase I (Option C) is incorrect. This is a target mechanism of certainanticancer agents and quinolone antibiotics that interfere with DNA synthesis.

Inhibition of tubulin (Option D) is incorrect. Colchicine and vinca alkaloids exert theirtherapeutic effects through inhibition of tubulin.

Inhibition of purine synthesisA

Inhibition of thymidylate synthaseB

Inhibition of topoisomerase IC

Inhibition of tubulinD

Methylation of guanosineE

Page 228: Back to Filters (/Secure/TestMe/Filter ... - 1 File Download

8/13/2016 MyPastest

https://mypastest.pastest.com/Secure/TestMe/Browser/429893#Top 2/2

46805

Methylation of guanosine (Option E) is incorrect. Some novel antiviral treatments forhepatitis C target this mechanism.

Next Question

Previous Question Tag Question Feedback End Review

Difficulty: Average

Peer Responses

Session Progress

0Responses Correct:

179Responses Incorrect:

179Responses Total:

0%Responses - % Correct:

Blog (https://www.pastest.com/blog) About Pastest (https://www.pastest.com/about-us)Contact Us (https://www.pastest.com/contact-us) Help (https://www.pastest.com/help)

© Pastest 2016

Page 229: Back to Filters (/Secure/TestMe/Filter ... - 1 File Download

8/13/2016 MyPastest

https://mypastest.pastest.com/Secure/TestMe/Browser/429893#Top 1/2

Back to Filters (/Secure/TestMe/Filter/429893/QA)

Question 115 of 179

46860

A 35-year-old woman with a bipolar disorder has been prescribed lithium.

Which of the following pharmacological features should best be kept in mind whenprescribing this drug?

Explanation

The answer is Lithium may cause nephrogenic diabetes insipidus –

Long-term treatment with lithium causes insensitivity of ADH receptors causingnephrogenic diabetes insipidus. It is also associated with progressive chronic kidneydisease.

Amiloride is a specific antidote for lithium toxicity (Option A) is incorrect. Amiloride may beeffective in treating lithium-induced nephrogenic diabetes insipidus. It is not an antidote forlithium toxicity; haemodialysis may be required to treat lithium toxicity.

Hyperkalaemia may occur (Option B) is incorrect. Lithium nephropathy is often associatedwith hypokalaemia, and may result in flattening of T-waves on the ECG.

Serum lithium level should not exceed 2 mmol/l (Option D) is incorrect. The therapeuticrange for lithium treatment is 0.5–1.0 mmol/l; levels between 1.0 and 2.0 mmol/l may beassociated with significant clinical toxicity.

Thyroid antibodies may be increased (Option E) is incorrect. Lithium may interfere withthyroid function, often causing hypothyroidism due to recepton insensitivity to thyroidstimulating hormone; the presence of thyroid antibodies may increase the risk ofhypothyroidism.

Amiloride is a specific antidote for lithium toxicityA

Hyperkalaemia may occurB

Lithium may cause nephrogenic diabetes insipidusC

Serum lithium level should not exceed 2 mmol/lD

Thyroid antibodies may be increasedE

Page 230: Back to Filters (/Secure/TestMe/Filter ... - 1 File Download

8/13/2016 MyPastest

https://mypastest.pastest.com/Secure/TestMe/Browser/429893#Top 2/2

46860

Next Question

Previous Question Tag Question Feedback End Review

Difficulty: Average

Peer Responses

Session Progress

0Responses Correct:

179Responses Incorrect:

179Responses Total:

0%Responses - % Correct:

Blog (https://www.pastest.com/blog) About Pastest (https://www.pastest.com/about-us)Contact Us (https://www.pastest.com/contact-us) Help (https://www.pastest.com/help)

© Pastest 2016

Page 231: Back to Filters (/Secure/TestMe/Filter ... - 1 File Download

8/13/2016 MyPastest

https://mypastest.pastest.com/Secure/TestMe/Browser/429893#Top 1/3

Back to Filters (/Secure/TestMe/Filter/429893/QA)

Question 116 of 179

A 52-year-old man with disseminated prostatic carcinoma comes to the EmergencyDepartment after his family called an ambulance. They are very concerned as he has becomeincreasingly drowsy and they are now unable to rouse him from sleep. He is managed withprolonged-release morphine but his dose has remained unchanged for the past 4 weeks. It isonly over the past 3 days that he has deteriorated. On examination he is unconscious andgroans in response to vigorous stimulation. His blood pressure is 100/50 mmHg and hisrespiratory rate is 9 breaths per minute.

Investigations:

Hb 10.2 g/dl

White cell count 6.2 × 10 /l

Platelets 139 × 10 /l

Na 142 mmol/l

K 6.1 mmol/l

Urea 35.2 mmol/l

Creatinine 720 μmol/l

ALT 1024 U/l

Albumin 32 g/l

Alkaline phosphatase 623 U/l

Urine on suprapubic catheterisation blood++

Which one of the following is most likely to be responsible for his impaired conscious level?

9

9

+

+

Your answer was incorrect

Hepatic failureA

Renal failureB

Cerebral metastasesC

Page 232: Back to Filters (/Secure/TestMe/Filter ... - 1 File Download

8/13/2016 MyPastest

https://mypastest.pastest.com/Secure/TestMe/Browser/429893#Top 2/3

21365

Explanation

Drug efficacy in renal failure

It is clear that this unrousable patient with disseminated prostate cancer has severe

renal failure

Whilst his liver function test results are in keeping with hepatic metastases, the relative

preservation of albumin would indicate that his liver function is reasonably intact

Many of the hepatic opiate metabolites are biologically active, which means that dose

adjustment of opiates is recommended in both renal and hepatic failure

Given that this patient has been stable for a prolonged period of time, and there is no

history suggestive of focal neurological impairment, cerebral metastases seem less

likely as the cause of his presentation

StrokeD

Urinary sepsisE

Next Question

Previous Question Tag Question Feedback End Review

Difficulty: Average

Peer Responses

Session Progress

0Responses Correct:

179Responses Incorrect:

179Responses Total:

0%Responses - % Correct:

Page 233: Back to Filters (/Secure/TestMe/Filter ... - 1 File Download

8/13/2016 MyPastest

https://mypastest.pastest.com/Secure/TestMe/Browser/429893#Top 3/3

Blog (https://www.pastest.com/blog) About Pastest (https://www.pastest.com/about-us)Contact Us (https://www.pastest.com/contact-us) Help (https://www.pastest.com/help)

© Pastest 2016

Page 234: Back to Filters (/Secure/TestMe/Filter ... - 1 File Download

8/13/2016 MyPastest

https://mypastest.pastest.com/Secure/TestMe/Browser/429893#Top 1/2

Back to Filters (/Secure/TestMe/Filter/429893/QA)

Question 117 of 179

Renal extraction ratio is a term that may be used to describe drug handling.

Which one of the following terms most accurately describes how renal drug elimination canbe defined?

Explanation

The answer is Decline of drug concentration in the plasma from the arterial to the venousside of the kidney -

The extraction ratio is a measure of how much drug is extracted from the plasma by thekidney, which determines the clearance (clearance = renal plasma flow × extraction ratio).

A measure of the time during which the concentration of drug in the plasma falls by 50%(Option A) is incorrect. The time it takes for the concentration of a drug in the plasma to fallby 50% is the half-life.

The concentration of a drug in the urine divided by the concentration in the plasma (OptionC) is incorrect. The extraction ratio is determined by assessing drug concentration on thearterial and venous sides of the renal circulation rather than assessing plasma versus urineconcentration.

The proportion of an orally administered drug reaching the circulation (Option D) isincorrect. The proportion of orally administered drug reaching the circulation is thebioavailability.

A measure of the time during which the concentration of drug in the plasma falls by50%

A

Decline of drug concentration in the plasma from the arterial to the venous side ofthe kidney

B

The concentration of a drug in the urine divided by the concentration in the plasmaC

The proportion of an orally administered drug reaching the circulationD

The ratio of drug concentration in the urine to drug concentration in the bileE

Page 235: Back to Filters (/Secure/TestMe/Filter ... - 1 File Download

8/13/2016 MyPastest

https://mypastest.pastest.com/Secure/TestMe/Browser/429893#Top 2/2

46731

The ratio of drug concentration in the urine to drug concentration in the bile (Option E) isincorrect. The ratio of drug concentration in the urine to drug concentration in the bile givesan idea of the percent renal excretion versus biliary excretion; this is not the extraction ratio.

Next Question

Previous Question Tag Question Feedback End Review

Difficulty: Average

Peer Responses

Session Progress

0Responses Correct:

179Responses Incorrect:

179Responses Total:

0%Responses - % Correct:

Blog (https://www.pastest.com/blog) About Pastest (https://www.pastest.com/about-us)Contact Us (https://www.pastest.com/contact-us) Help (https://www.pastest.com/help)

© Pastest 2016

Page 236: Back to Filters (/Secure/TestMe/Filter ... - 1 File Download

8/13/2016 MyPastest

https://mypastest.pastest.com/Secure/TestMe/Browser/429893#Top 1/2

Back to Filters (/Secure/TestMe/Filter/429893/QA)

Question 118 of 179

A 32-year-old woman presents to the Emergency Department with jaundice. She has beentaking a 2-week course of antibiotics prescribed by her doctor for recurrent urinary tractinfection, but cannot remember their name. On examination she is apyrexial, her bloodpressure is 132/78 mmHg, her BMI is 24 kg/m2 and she has jaundiced sclerae. There is notenderness on abdominal examination. Investigations show alanine transaminase 102 IU/l,alkaline phosphatase 394 IU/l, and bilirubin 160 μmol/l.

Which one of the following antibiotics is most likely to have caused this biochemical picture?

Explanation

The answer is Co-amoxiclav –

The blood picture seen here is most consistent with obstructive jaundice of cholestasisbecause of the larger rise in alkaline phosphatase compared with the modest elevation inalanine aminotransferase (ALT).

Ciprofloxacin (Option A) is incorrect. Although ciprofloxacin is excreted in bile, it is verymuch less likely to cause cholestatic jaundice than co-amoxiclav.

Erythromycin (Option C) is incorrect. Erythromycin estolate preparations may causecholestatic jaundice, but these formulations are now uncommon.

Rifampicin (Option D) is incorrect. Rifampicin may be associated with hepatic dysfunction,but the picture seen is usually one of hepatocellular dysfunction and a prominentabnormality of transaminases.

CiprofloxacinA

Co-amoxiclavB

ErythromycinC

RifampicinD

TrimethoprimE

Page 237: Back to Filters (/Secure/TestMe/Filter ... - 1 File Download

8/13/2016 MyPastest

https://mypastest.pastest.com/Secure/TestMe/Browser/429893#Top 2/2

46994

Trimethoprim (Option E) is incorrect. Trimethoprim may cause hyperkalaemia and a transientrise in serum creatinine concentrations.

Next Question

Previous Question Tag Question Feedback End Review

Difficulty: Average

Peer Responses

Session Progress

0Responses Correct:

179Responses Incorrect:

179Responses Total:

0%Responses - % Correct:

Blog (https://www.pastest.com/blog) About Pastest (https://www.pastest.com/about-us)Contact Us (https://www.pastest.com/contact-us) Help (https://www.pastest.com/help)

© Pastest 2016

Page 238: Back to Filters (/Secure/TestMe/Filter ... - 1 File Download

8/13/2016 MyPastest

https://mypastest.pastest.com/Secure/TestMe/Browser/429893#Top 1/2

Back to Filters (/Secure/TestMe/Filter/429893/QA)

Question 119 of 179

A 29-year-old woman has been receiving antituberculous treatments for the past 3 monthswith rifampicin, isoniazid, pyrazinamide, ethambutol and pyridoxine. She has been taking theoral contraceptive pill regularly but now suddenly finds she is pregnant.

Which drug is most likely to have resulted in failure of the oral contraceptive treatment?

Explanation

The answer is Rifampicin –

The oral contraceptive pill is highly effective, but may fail if the circulating oestrogenconcentrations are insufficient. Often this is related to inconsistent patient compliance,impaired gastrointestinal absorption, or the co-administration of enzyme inducing drugsthat accelerate hepatic metabolism. Rifampicin is a potent hepatic enzyme inducer thatincreases the metabolism of many drugs, including steroid hormones; barriercontraceptives must be used during rifampicin treatment, and for 4–8 weeks aftercompleting a course of rifampicin.

Ethambutol (Option A) is incorrect. There is no clinically significant interaction betweenethambutol and the oral contraceptive pill.

Isoniazid (Option B) is incorrect. There is no clinically significant interaction betweenisoniazid and the oral contraceptive pill.

Pyrazinamide (Option C) is incorrect. There is no clinically significant interaction betweenpyrazinamide and the oral contraceptive pill.

EthambutolA

IsoniazidB

PyrazinamideC

PyridoxineD

RifampicinE

Page 239: Back to Filters (/Secure/TestMe/Filter ... - 1 File Download

8/13/2016 MyPastest

https://mypastest.pastest.com/Secure/TestMe/Browser/429893#Top 2/2

46621

Pyridoxine (Option D) is incorrect. There is no clinically significant interaction betweenpyridoxine and the oral contraceptive pill.

Next Question

Previous Question Tag Question Feedback End Review

Difficulty: Average

Peer Responses

Session Progress

0Responses Correct:

179Responses Incorrect:

179Responses Total:

0%Responses - % Correct:

Blog (https://www.pastest.com/blog) About Pastest (https://www.pastest.com/about-us)Contact Us (https://www.pastest.com/contact-us) Help (https://www.pastest.com/help)

© Pastest 2016

Page 240: Back to Filters (/Secure/TestMe/Filter ... - 1 File Download

8/13/2016 MyPastest

https://mypastest.pastest.com/Secure/TestMe/Browser/429893#Top 1/2

Back to Filters (/Secure/TestMe/Filter/429893/QA)

Question 120 of 179

A 45-year-old asthmatic patient with a history of myocardial infarction presents to theEmergency Department with palpitations. An ECG shows supraventricular tachycardia.Carotid sinus massage has no effect on symptoms or heart rate. BP is 135/72 mmHg.

What would be the most appropriate treatment to consider administering next?

Explanation

The answer is Verapamil –

Verapamil is the drug of choice in a patient with asthma. Adenosine should normally beavoided in patients with asthma because of the risk of bronchospasm. Note thatverapamil should not be used to treat tachyarrhythmias where the QRS complex is widebecause this may precipitate cardiovascular collapse. Verapamil is also contraindicated inpatients with the Wolff–Parkinson–White syndrome.

Adenosine (Option A) is incorrect. Although adenosine is the drug of choice for terminatingparoxysmal supraventricular tachycardia, it can cause bronchospasm and is thuscontraindicated in patients with asthma.

Digoxin (Option B) is incorrect. Digoxin tends to shorten refractory periods and enhanceexcitability and conduction in other parts of the heart; it may therefore cause more complexatrial and ventricular tachyarrhythmias.

Flecainide (Option C) is incorrect. Flecainide may be hazardous in patients with a history ofmyocardial infarction as it may precipitate ventricular fibrillation.

AdenosineA

DigoxinB

FlecainideC

LignocaineD

VerapamilE

Page 241: Back to Filters (/Secure/TestMe/Filter ... - 1 File Download

8/13/2016 MyPastest

https://mypastest.pastest.com/Secure/TestMe/Browser/429893#Top 2/2

46742

Lignocaine (Option D) is incorrect. Lignocaine is used in patients with recurrent VT, not in thetreatment of paroxysmal SVT.

Next Question

Previous Question Tag Question Feedback End Review

Difficulty: Average

Peer Responses

Session Progress

0Responses Correct:

179Responses Incorrect:

179Responses Total:

0%Responses - % Correct:

Blog (https://www.pastest.com/blog) About Pastest (https://www.pastest.com/about-us)Contact Us (https://www.pastest.com/contact-us) Help (https://www.pastest.com/help)

© Pastest 2016

Page 242: Back to Filters (/Secure/TestMe/Filter ... - 1 File Download

8/13/2016 MyPastest

https://mypastest.pastest.com/Secure/TestMe/Browser/429893#Top 1/2

Back to Filters (/Secure/TestMe/Filter/429893/QA)

Question 121 of 179

46818

A 55-year-old woman has had a history of hypertension over several years. Over the past 3weeks she has experienced symptoms of fever and malaise. Her temperature is 38°C, and sheis noted to have a facial rash that she says is new. There is also swelling and tendernessaffecting her wrists and knees. Blood pressure is normal.

Which of the following drugs is most likely to have caused these findings?

Explanation

The answer is Hydralazine –

Drug-induced lupus is a syndrome mimicking systemic lupus erythematosus, ofteninvolving the joints and generally without renal involvement. It is typically caused bytreatment with hydralazine, procainamide, phenytoin or ethosuximide. Risk is associatedwith high doses, and with hydralazine and procainamide it is more common in slowacetylators. ‘Drug fever’ as an isolated phenomenon can occur with penicillins, phenytoin,hydralazine and quinidine; fever is usually of low grade and the patient is generally well;the fever normally subsides within a few days of stopping the causative drug.

Amlodipine (Option A) is incorrect. Amlodipine may cause peripheral oedema but not feveror the other features of lupus.

Enalapril (Option B) is incorrect. Enalapril does not cause fever or lupus.

Hydrochlorothiazide (Option D) is incorrect. Thiazides typically cause hypokalaemia andincreased risk of gout.

Propranolol (Option E) is incorrect. Propranolol is not a recognised cause of lupus.

AmlodipineA

EnalaprilB

HydralazineC

HydrochlorothiazideD

PropranololE

Page 243: Back to Filters (/Secure/TestMe/Filter ... - 1 File Download

8/13/2016 MyPastest

https://mypastest.pastest.com/Secure/TestMe/Browser/429893#Top 2/2

46818

Next Question

Previous Question Tag Question Feedback End Review

Difficulty: Average

Peer Responses

Session Progress

0Responses Correct:

179Responses Incorrect:

179Responses Total:

0%Responses - % Correct:

Blog (https://www.pastest.com/blog) About Pastest (https://www.pastest.com/about-us)Contact Us (https://www.pastest.com/contact-us) Help (https://www.pastest.com/help)

© Pastest 2016

Page 244: Back to Filters (/Secure/TestMe/Filter ... - 1 File Download

8/13/2016 MyPastest

https://mypastest.pastest.com/Secure/TestMe/Browser/429893#Top 1/2

Back to Filters (/Secure/TestMe/Filter/429893/QA)

Question 122 of 179

You are involved in selecting a candidate drug for further clinical development as anantihypertensive agent. You have been provided with data concerning a number of differentcompounds under investigation.

Which of the following compounds, according to its mode of clearance, is most likely to showstable pharmacokinetic properties when tested between patients?

Explanation

The answer is Compound C is one-third cleared by the kidneys and two-thirds by twodifferent P450 isoforms, neither by CYP2D6 –

To maximise drug absorption, the ideal compound should be small (molecular weight ofless than 300 kDa) and have intermediate lipophilicity and hydrophilicity. Clearance byrenal and hepatic excretion renders the drug less susceptible to variation in renal orhepatic metabolism, particularly with regard to genetic factors that alter liver clearancebetween individuals (eg CYP2D6 or CYP3A4 variations). In addition, the ideal profile ofour candidate drug should be neither an inhibitor nor an inducer of the P450 system toavoid interactions with other medications.

Compound A is predominantly cleared via the kidneys (Option A) is incorrect. Drugs thatdepend solely upon renal elimination are susceptible to inter-individual pharmacokineticvariations due to variable renal function.

Compound B is predominantly cleared via the CYP2D6 route (Option B) is incorrect. CYP2D6is a hepatic enzymatic pathway that shows the greatest genetic variability, and hence there

Compound A is predominantly cleared via the kidneysA

Compound B is predominantly cleared via the CYP2D6 routeB

Compound C is one-third cleared by the kidneys and two-thirds by two differentP450 isoforms, neither by CYP2D6

C

Compound D is 50% cleared by CYP2D6, 50% by another P450 isoformD

Compound E is predominantly cleared via CYP3A4E

Page 245: Back to Filters (/Secure/TestMe/Filter ... - 1 File Download

8/13/2016 MyPastest

https://mypastest.pastest.com/Secure/TestMe/Browser/429893#Top 2/2

46840

are likely to be significant and unpredictable pharmacokinetic differences between patientsin drugs metabolised down this route.

Compound D is 50% cleared by CYP2D6, 50% by another P450 isoform (Option D) isincorrect. Drugs that are eliminated solely by hepatic elimination are prone to inter-individualvariations in liver function, particularly when specific pathways such as the CYP2D6 orCYP3A4 isoenzymes are involved (these are susceptible to genetic variation and altered drugclearance).

Compound E is predominantly cleared via CYP3A4 (Option E) is incorrect. The CYP3A4isoenzyme is subject to genetic variation, and likely to result in significant differences in drughandling between individuals.

Next Question

Previous Question Tag Question Feedback End Review

Difficulty: Average

Peer Responses

Session Progress

0Responses Correct:

179Responses Incorrect:

179Responses Total:

0%Responses - % Correct:

Blog (https://www.pastest.com/blog) About Pastest (https://www.pastest.com/about-us)Contact Us (https://www.pastest.com/contact-us) Help (https://www.pastest.com/help)

© Pastest 2016

Page 246: Back to Filters (/Secure/TestMe/Filter ... - 1 File Download

8/13/2016 MyPastest

https://mypastest.pastest.com/Secure/TestMe/Browser/429893#Top 1/2

Back to Filters (/Secure/TestMe/Filter/429893/QA)

Question 123 of 179

46825

You are asked to review a 72-year-old man who is being treated with warfarin for chronicatrial fibrillation. His cholesterol is 6.5 mmol/l with triglycerides of 3.1 mmol/l. You want tostart him on some lipid-lowering therapy.

Which would be the best drug to prescribe with respect to balancing efficacy versuspotentiating the effects of his warfarin treatment?

Explanation

The answer is Atorvastatin –

Atorvastatin and pravastan do not appreciably interfere with warfarin and may safely beadded with little impact upon warfarin monitoring.

Bezafibrate (Option B) is incorrect. Fibrates can increase the anticoagulant effects ofwarfarin.

Chlolestyramine (Option C) is incorrect. Cholestyramine would be ineffective in loweringcholesterol in a meaningful way, and can significantly increase warfarin effects.

Rosuvastatin (Option D) is incorrect. Rosuvastatin may potentiate the anticoagulant effectsof warfarin, although there is less evidence of this than for simvastatin or fibrates.

Simvastatin (Option E) is incorrect. Simvastatin may potentiate the anticoagulant effects ofwarfarin therapy.

AtorvastatinA

BezafibrateB

ChlolestyramineC

RosuvastatinD

SimvastatinE

Next Question

Page 247: Back to Filters (/Secure/TestMe/Filter ... - 1 File Download

8/13/2016 MyPastest

https://mypastest.pastest.com/Secure/TestMe/Browser/429893#Top 2/2

Previous Question Tag Question Feedback End Review

Difficulty: Average

Peer Responses

Session Progress

0Responses Correct:

179Responses Incorrect:

179Responses Total:

0%Responses - % Correct:

Blog (https://www.pastest.com/blog) About Pastest (https://www.pastest.com/about-us)Contact Us (https://www.pastest.com/contact-us) Help (https://www.pastest.com/help)

© Pastest 2016

Page 248: Back to Filters (/Secure/TestMe/Filter ... - 1 File Download

8/13/2016 MyPastest

https://mypastest.pastest.com/Secure/TestMe/Browser/429893#Top 1/2

Back to Filters (/Secure/TestMe/Filter/429893/QA)

Question 124 of 179

A 48-year-old man is reviewed in the outpatient medical clinic. He has a past history ofangina, hypertension and type-2 diabetes and is receiving a number of regular medications.He mentions that he would like to receive drug therapy for impotence, and you considerprescribing sildenafil.

Which one of the following medications is likely to pose the greatest risk if taken alongsidesildenafil treatment?

Explanation

The answer is Isosorbide mononitrate –

Sildenafil is a phosphodiesterase (PDE-5) inhibitor indicated for the treatment of erectiledysfunction. It is contraindicated in patients with proven coronary artery disease and inpatients who are taking oral nitrate therapy due to an increased risk of systemicvasodilatation and profound hypotension. A common side-effect of sildenafil includesheadache, predominantly through cerebral vasodilation. Alternatives to sildenafil mayinclude prostaglandins given via pessary or injection into the penis; apomorphine mayalso be considered although a number of important adverse effects are recognised.

Aspirin (Option A) is incorrect. There is no significant interaction between sildenafil andaspirin.

Bendroflumethiazide (Option B) is incorrect. There is no significant interaction betweensildenafil and bendrofluazide.

Lisinopril (Option D) is incorrect. There is a potential for the combination of sildenafil andlisinopril to cause significant hypotension. However, the risks are greater for the combination

AspirinA

BendroflumethiazideB

Isosorbide mononitrateC

LisinoprilD

MetforminE

Page 249: Back to Filters (/Secure/TestMe/Filter ... - 1 File Download

8/13/2016 MyPastest

https://mypastest.pastest.com/Secure/TestMe/Browser/429893#Top 2/2

46920

of sildenafil and nitrate therapy, hence option C is the preferred answer.

Metformin (Option E) is incorrect. There is no significant interaction between sildenafil andmetformin.

Next Question

Previous Question Tag Question Feedback End Review

Difficulty: Average

Peer Responses

Session Progress

0Responses Correct:

179Responses Incorrect:

179Responses Total:

0%Responses - % Correct:

Blog (https://www.pastest.com/blog) About Pastest (https://www.pastest.com/about-us)Contact Us (https://www.pastest.com/contact-us) Help (https://www.pastest.com/help)

© Pastest 2016

Page 250: Back to Filters (/Secure/TestMe/Filter ... - 1 File Download

8/13/2016 MyPastest

https://mypastest.pastest.com/Secure/TestMe/Browser/429893#Top 1/2

Back to Filters (/Secure/TestMe/Filter/429893/QA)

Question 125 of 179

46972

A 64-year-old man presents to the Emergency Department complaining of marked muscleaches and tenderness. He has a history of previous myocardial infarction and takes a numberof medications, including simvastatin. Recently he has made a number of changes to his dietto try to improve his health. Plasma creatine kinase in the Emergency Department wasmeasured at 4800 U/l and you suspect rhabdomyolysis.

Which one of the following interactions with simvastatin is most likely to be responsible forhis presentation?

Explanation

The answer is Grapefruit juice –

Grapefruit juice is a potent inhibitor of CYP3A4 metabolism, which affects levels of anumber of drugs, including ciclosporin and simvastatin.

Cod liver oil capsules (Option A) is incorrect. Cod liver oil capsules are not known to interactwith P450 enzymes.

Cranberry juice (Option B) is incorrect. Cranberry juice contains bioflavonoids that inhibit theCYP2C9 isoenzyme, thereby inhibiting metabolism of warfarin.

St John ‘s wort (Option D) is incorrect. St John’s wort is a cytochrome P450 enzyme inducerand as such may reduce the effectiveness of drugs that are metabolised by the P450 route,including simvastatin and warfarin.

Vitamin C (Option E) is incorrect. Vitamin C is not known to affect simvastatin metabolism.

Cod liver oil capsulesA

Cranberry juiceB

Grapefruit juiceC

St John ‘s wortD

Vitamin CE

Page 251: Back to Filters (/Secure/TestMe/Filter ... - 1 File Download

8/13/2016 MyPastest

https://mypastest.pastest.com/Secure/TestMe/Browser/429893#Top 2/2

Next Question

Previous Question Tag Question Feedback End Review

Difficulty: Average

Peer Responses

Session Progress

0Responses Correct:

179Responses Incorrect:

179Responses Total:

0%Responses - % Correct:

Blog (https://www.pastest.com/blog) About Pastest (https://www.pastest.com/about-us)Contact Us (https://www.pastest.com/contact-us) Help (https://www.pastest.com/help)

© Pastest 2016

Page 252: Back to Filters (/Secure/TestMe/Filter ... - 1 File Download

8/13/2016 MyPastest

https://mypastest.pastest.com/Secure/TestMe/Browser/429893#Top 1/2

Back to Filters (/Secure/TestMe/Filter/429893/QA)

Question 126 of 179

37299

A 62-year-old woman is treated with NSAIDs and long term Methotrexate for rheumatoidarthritis. She presents to the clinic for review and complains of increasing nausea andindigestion. An additional finding is increased BP (now 157/72 mmHg), and she is started onnew medications for both problems by her doctor. A recent GFR is measured at 28ml/min.

Which of the following medications should be used with caution in this situation?

Explanation

The answer is Omeprazole

Methotrexate is a substrate for the OAT-1 renal transporter and levels of methotrexate aretherefore affected by decreased renal function. OAT-1 inhibitors include drugs such asProbenecid, and therefore should not be used in conjunction with Methotrexate. Omeprazoleis also known to affect clearance of Methotrexate, this interaction is not thought to be viaOAT-1, but is thought to be related to inhibition of breast cancer resistance protein, which isresponsible for Methotrexate transport.

The other options given are not thought to affect Methotrexate levels. In this situation with aGFR of 28ml/min an alginate preparation or Ranitidine may therefore be better options forinitial therapy for indigestion in this situation.

AmlodipineA

BisoprololB

GavisconC

OmeprazoleD

RanitidineE

Next Question

Previous Question Tag Question Feedback End Review

Page 253: Back to Filters (/Secure/TestMe/Filter ... - 1 File Download

8/13/2016 MyPastest

https://mypastest.pastest.com/Secure/TestMe/Browser/429893#Top 2/2

Difficulty: Difficult

Peer Responses

Session Progress

0Responses Correct:

179Responses Incorrect:

179Responses Total:

0%Responses - % Correct:

Blog (https://www.pastest.com/blog) About Pastest (https://www.pastest.com/about-us)Contact Us (https://www.pastest.com/contact-us) Help (https://www.pastest.com/help)

© Pastest 2016

Page 254: Back to Filters (/Secure/TestMe/Filter ... - 1 File Download

8/13/2016 MyPastest

https://mypastest.pastest.com/Secure/TestMe/Browser/429893#Top 1/2

Back to Filters (/Secure/TestMe/Filter/429893/QA)

Question 127 of 179

A pharmaceutical company wants to bring a generic formulation of candesartanranitidine tothe market after the patent has expired.

What kind of study is needed to obtain approval to market the drug?

Explanation

The answer is Phase-I bioequivalence study –

Generic medicines are produced by companies who are subject to the same tightmanufacturing quality controls as those who make branded products. Often the samecompany makes both branded and generic medicines. A phase-I bioequivalence studyensures that the pharmacokinetic properties of the drug are acceptably similar to theoriginal product, including the maximum concentration (C ), the area under the curve(AUC) and the time of maximum concentration (t ). There is no need to submit theentire body of pre-clinical research required for a new agent.

Phase-II proof-of-concept study (Option B) is incorrect. There is no need to repeat phase-IIproof-of-concept studies.

Phase-III study in patients (Option C) is incorrect. There is no need to repeat phase-III clinicalstudies.

Phase-IV study in patients (Option D) is incorrect. There is no need to repeat phase-IV orpost-marketing studies.

Preclinical toxicology study (Option E) is incorrect. Preclinical toxicology studies areperformed before and during phase-1 clinical research to identify possible adverse effects

Phase-I bioequivalence studyA

Phase-II proof-of-concept studyB

Phase-III study in patientsC

Phase-IV study in patientsD

Preclinical toxicology studyE

max

max

Page 255: Back to Filters (/Secure/TestMe/Filter ... - 1 File Download

8/13/2016 MyPastest

https://mypastest.pastest.com/Secure/TestMe/Browser/429893#Top 2/2

46809

that might be relevant in people.

Next Question

Previous Question Tag Question Feedback End Review

Difficulty: Average

Peer Responses

Session Progress

0Responses Correct:

179Responses Incorrect:

179Responses Total:

0%Responses - % Correct:

Blog (https://www.pastest.com/blog) About Pastest (https://www.pastest.com/about-us)Contact Us (https://www.pastest.com/contact-us) Help (https://www.pastest.com/help)

© Pastest 2016

Page 256: Back to Filters (/Secure/TestMe/Filter ... - 1 File Download

8/13/2016 MyPastest

https://mypastest.pastest.com/Secure/TestMe/Browser/429893#Top 1/2

Back to Filters (/Secure/TestMe/Filter/429893/QA)

Question 128 of 179

Tamoxifen has been prescribed for a woman who is undergoing treatment for breastcarcinoma.

What is the most significant characteristic of this drug?

Explanation

The answer is It functions as a selective oestrogen-receptor modulator –

Tamoxifen is a selective oestrogen-receptor modulator (SERM). It acts both as a potentoestrogen antagonist in some areas and as a weak oestrogen agonist in other places. Inpatients with advanced metastatic breast cancer, oestrogen deprivation causes tumourregression in 30% of unselected women and in more than 60% of those with oestrogen-receptor-positive tumours for a median duration of 20 months. In the adjuvant setting,ovarian ablation or tamoxifen results in a 25% reduction in the relative risk of dying frommetastatic disease.

A yearly PAP smear is recommended for women on tamoxifen (Option A) is incorrect.Tamoxifen does not increase the risk of cervical carcinoma, so there is no need for increasedfrequency of PAP smear screening.

It has no effect on tumours that are oestrogen-receptor negative (Option C) is incorrect.Benefits are greatest in patients with tumours that express the oestrogen receptor, but arenot exclusive to these patients.

It increases low-density lipoprotein cholesterol levels (Option D) is incorrect. Tamoxifendecreases low-density lipoprotein cholesterol with no change in high-density lipoproteincholesterol, and thus decreases the risk of coronary artery disease.

A yearly PAP smear is recommended for women on tamoxifenA

It functions as a selective oestrogen-receptor modulatorB

It has no effect on tumours that are oestrogen-receptor negativeC

It increases low-density lipoprotein cholesterol levelsD

It is recommended mainly in women who have undergone hysterectomyE

Page 257: Back to Filters (/Secure/TestMe/Filter ... - 1 File Download

8/13/2016 MyPastest

https://mypastest.pastest.com/Secure/TestMe/Browser/429893#Top 2/2

46872

It is recommended mainly in women who have undergone hysterectomy (Option E) isincorrect. Endometrial sampling is recommended for women on tamoxifen as the risk ofendometrial cancer is increased 2–3 times with long-term use, but prior hysterectomy is not aprerequisite for treatment with tamoxifen.

Next Question

Previous Question Tag Question Feedback End Review

Difficulty: Average

Peer Responses

Session Progress

0Responses Correct:

179Responses Incorrect:

179Responses Total:

0%Responses - % Correct:

Blog (https://www.pastest.com/blog) About Pastest (https://www.pastest.com/about-us)Contact Us (https://www.pastest.com/contact-us) Help (https://www.pastest.com/help)

© Pastest 2016

Page 258: Back to Filters (/Secure/TestMe/Filter ... - 1 File Download

8/13/2016 MyPastest

https://mypastest.pastest.com/Secure/TestMe/Browser/429893#Top 1/2

Back to Filters (/Secure/TestMe/Filter/429893/QA)

Question 129 of 179

You are asked to review a 36-year-old man who suffers from partial epilepsy. He haspresented to the Emergency Department after a fall, and a visual-field defect has beenidentified. He has been on vigabatrin combination therapy for 3 years and his family statethat his epilepsy had been poorly managed up until then.

What type of adverse drug reaction is most likely to be responsible for his visual-fielddefect?

Explanation

The answer is Type C –

Type C reactions occur in the setting of prolonged drug use (eg analgesic nephropathyor visual-field defects with vigabatrin, as in this case). Visual-field defects due tovigabatrin use are unpredictable and may occur any time between 1 month and severalyears after the commencement of therapy. Visual-field defects are an absolutecontraindication to vigabatrin use.

Type A (Option A) is incorrect. Type A adverse drug reactions occur as an anticipated effectarising from a particular pharmacological property of the drug; they are dose-related, egbradycardia as an adverse effect of β-adrenoceptor blocking drugs.

Type B (Option B) is incorrect. Type B adverse drug reactions are unpredictable and notrelated to a predictable pharmacological effect of the drug. They are ‘idiosyncratic’,unexpected, usually rare, and may be serious.

Type D (Option D) is incorrect. Type D adverse drug reactions emerge late after treatment,e.g. teratogenic or carcinogenic reactions, such as phocomyelia after administration of

Type AA

Type BB

Type CC

Type DD

Type EE

Page 259: Back to Filters (/Secure/TestMe/Filter ... - 1 File Download

8/13/2016 MyPastest

https://mypastest.pastest.com/Secure/TestMe/Browser/429893#Top 2/2

46848

thalidomide in pregnancy.

Type E (Option E) is incorrect. Type E adverse drug reactions are related to drug withdrawalphenomena observed after a drug is discontinued, e.g. SSRI cessation and agitation, oropioid withdrawal syndrome.

Next Question

Previous Question Tag Question Feedback End Review

Difficulty: Average

Peer Responses

Session Progress

0Responses Correct:

179Responses Incorrect:

179Responses Total:

0%Responses - % Correct:

Blog (https://www.pastest.com/blog) About Pastest (https://www.pastest.com/about-us)Contact Us (https://www.pastest.com/contact-us) Help (https://www.pastest.com/help)

© Pastest 2016

Page 260: Back to Filters (/Secure/TestMe/Filter ... - 1 File Download

8/13/2016 MyPastest

https://mypastest.pastest.com/Secure/TestMe/Browser/429893#Top 1/2

Back to Filters (/Secure/TestMe/Filter/429893/QA)

Question 130 of 179

You are asked to give advice to a GP regarding a family with concerns that they may havebeen exposed to high lead concentrations in their local water supply; they are worried thatthey might be suffered from lead poisoning.

Which one of the following features would most strongly support a diagnosis of leadtoxicity?

Explanation

The answer is Basophilic stippling of red blood cells on peripheral blood film –

Lead poisoning may arise from occupational exposure (eg miners, welders, storagebattery workers and pottery glaze workers) or environmental exposure. Accumulationmay occur as a result of lead ingestion, inhalation and direct skin contact. The mostcommon clinical feature is abdominal pain, possibly due to a direct effect of lead onintestinal smooth muscle. Lead interferes with a variety of red cell enzymes and causescharacteristic red cell abnormalities, including punctate basophilic stippling and cloverleaf morphology. Other features include peripheral neuropathy, which is almostexclusively motor. Interstitial nephritis is a characteristic lesion affecting the kidneys, anda gingival blue-black or grey line may be found in up to 20% of adults.

Chronic daily headache (Option B) is incorrect. Headache is not a prominent feature.

Development of membranous glomerulonephritis (Option C) is incorrect. Lead toxicity maycause interstitial nephritis and cause proximal tubular dysfunction.

Posterior uveitis (Option D) is incorrect. Lead poisoning may cause anterior uveitis or iritis,rather than posterior uveitis.

Basophilic stippling of red blood cells on peripheral blood filmA

Chronic daily headacheB

Development of membranous glomerulonephritisC

Posterior uveitisD

Sensory neuropathy in a glove and stocking distributionE

Page 261: Back to Filters (/Secure/TestMe/Filter ... - 1 File Download

8/13/2016 MyPastest

https://mypastest.pastest.com/Secure/TestMe/Browser/429893#Top 2/2

46623

Sensory neuropathy in a glove and stocking distribution (Option E) is incorrect. Leadneuropathy normally involves motor deficits.

Next Question

Previous Question Tag Question Feedback End Review

Difficulty: Average

Peer Responses

Session Progress

0Responses Correct:

179Responses Incorrect:

179Responses Total:

0%Responses - % Correct:

Blog (https://www.pastest.com/blog) About Pastest (https://www.pastest.com/about-us)Contact Us (https://www.pastest.com/contact-us) Help (https://www.pastest.com/help)

© Pastest 2016

Page 262: Back to Filters (/Secure/TestMe/Filter ... - 1 File Download

8/13/2016 MyPastest

https://mypastest.pastest.com/Secure/TestMe/Browser/429893#Top 1/2

Back to Filters (/Secure/TestMe/Filter/429893/QA)

Question 131 of 179

Iron deficiency anaemia has been diagnosed in an 80-year-old woman suffering fromosteoarthritis. She also has a history of cardiovascular disease. You suspect a cause related toher medication.

Which of the following medications is most likely to have caused the iron deficiencyanaemia?

Explanation

The answer is Aspirin –

Chronic, low-volume blood loss is usually subclinical until such time as the patientpresents with iron deficiency anaemia. Equally, the haemoglobin may be low in a patientwith iron deficiency anaemia resulting from chronic haemorrhage, who presents with asmall, acute bleed. Many gastrointestinal lesions can result in haemorrhage but pepticulcers are the most frequent in the UK. Aspirin and non-steroidal anti-inflammatory drugs(NSAIDs) may cause peptic ulceration affecting any part of the bowel.

Celecoxib (Option B) is incorrect. Cyclo-oxygenase-II selective inhibitors produce fewergastrointestinal side-effects compared with NSAIDs. Rofecoxib (Vioxx) was withdrawn fromthe market worldwide in 2004 because of an increased risk of myocardial infarction andcerebrovascular events after long-term use.

Diclofenac–misoprostol combination (Option C) is incorrect. Compared with placebo,misoprostol significantly reduces NSAID-associated gastric and duodenal ulcers found onendoscopy and reduces serious upper gastrointestinal (GI) complications. Bleeding may still

AspirinA

CelecoxibB

Diclofenac–misoprostol combinationC

LactuloseD

MetoclopramideE

Page 263: Back to Filters (/Secure/TestMe/Filter ... - 1 File Download

8/13/2016 MyPastest

https://mypastest.pastest.com/Secure/TestMe/Browser/429893#Top 2/2

46810

occur despite misoprostol but the occurrence is likely to be less than that in the case ofNSAIDs alone or aspirin.

Lactulose (Option D) is incorrect. Lactulose does not increase the risk of peptic ulceration orGI bleeding.

Metoclopramide (Option E) is incorrect. Metoclopramide increases gastric emptying.

Next Question

Previous Question Tag Question Feedback End Review

Difficulty: Average

Peer Responses

Session Progress

0Responses Correct:

179Responses Incorrect:

179Responses Total:

0%Responses - % Correct:

Blog (https://www.pastest.com/blog) About Pastest (https://www.pastest.com/about-us)Contact Us (https://www.pastest.com/contact-us) Help (https://www.pastest.com/help)

© Pastest 2016

Page 264: Back to Filters (/Secure/TestMe/Filter ... - 1 File Download

8/13/2016 MyPastest

https://mypastest.pastest.com/Secure/TestMe/Browser/429893#Top 1/2

Back to Filters (/Secure/TestMe/Filter/429893/QA)

Question 132 of 179

46965

A 31-year-old woman is found unconscious with empty bottles of pills and an empty vodkabottle, and brought to the Emergency Department. An ECG shows rate of 140 bpm with wideQRS (115 ms).

Which one of the following is the best first-line management?

Explanation

The answer is Bicarbonate –

This lady is unconscious and in ventricular tachycardia, and the most likely explanation inthis context is tricyclic overdose. Intravenous sodium bicarbonate reduces sodiumchannel blockade, and makes arrhythmia and seizures less likely to occur. Correction ofacidosis often resolves the arrhythmia.

Atenolol (Option A) is incorrect. Beta-blockers may be used to aid rate control in atrialfibrillation and to control supraventricular tachycardias, but would do little to alter sodiumchannel conductance, which is the main mechanism by which tricyclics cause arrhythmia andseizure.

Lidocaine (Option C) is incorrect. Lidocaine and other class I agents are ineffective intricyclic poisoning, and may worsen the situation by additional sodium channel blockade.

Naloxone (Option D) is incorrect. Naloxone is used as a competitive antagonist to the effectsof opiate toxicity.

Verapamil (Option E) is incorrect. Verapamil should be avoided in tachycardia unless asupraventricular origin is established.

AtenololA

BicarbonateB

LidocaineC

NaloxoneD

VerapamilE

Page 265: Back to Filters (/Secure/TestMe/Filter ... - 1 File Download

8/13/2016 MyPastest

https://mypastest.pastest.com/Secure/TestMe/Browser/429893#Top 2/2

46965

Next Question

Previous Question Tag Question Feedback End Review

Difficulty: Average

Peer Responses

Session Progress

0Responses Correct:

179Responses Incorrect:

179Responses Total:

0%Responses - % Correct:

Blog (https://www.pastest.com/blog) About Pastest (https://www.pastest.com/about-us)Contact Us (https://www.pastest.com/contact-us) Help (https://www.pastest.com/help)

© Pastest 2016

Page 266: Back to Filters (/Secure/TestMe/Filter ... - 1 File Download

8/13/2016 MyPastest

https://mypastest.pastest.com/Secure/TestMe/Browser/429893#Top 1/2

Back to Filters (/Secure/TestMe/Filter/429893/QA)

Question 133 of 179

A 43-year-old woman presents to the Emergency Department after an overdose involving amixture of medications including paracetamol, citalopram, sodium valproate, fluoxetine andferrous sulfate. You are contemplating administering oral activated charcoal.

Which of the following factors would be most strongly influence your decision to administeror avoid oral activated charcoal?

Explanation

The answer is Absence of bowel sounds –

Oral activated charcoal adsorbs many drugs and complex chemicals, thereby reducingdrug absorption from the gastrointestinal tract, and interrupting enterohepaticrecirculation. It is generally safe, but should be administered only in patients who areable to protect their airway. The absence of bowel sounds may indicate a paralytic ileus,which is surprisingly common after overdose, and which is associated with an increasedrisk of charcoal aspiration and pneumonitis.

Administration is less effective after mixed overdose (Option B) is incorrect. Activatedcharcoal is capable of adsorbing around 10% of its own weight, so administration of charcoal50 g might be expected to adsorb around 5 g of drug.

Charcoal minimises toxicity if given within 1 hour of iron overdose (Option C) is incorrect.Iron, lithium and other cations are not adsorbed by charcoal; alcohols including ethanol,methanol and ethylene glycol are not adsorbed either.

Absence of bowel soundsA

Administration is less effective after mixed overdoseB

Charcoal minimises toxicity if given within 1 hour of iron overdoseC

Charcoal should be used only for modified-release preparationsD

The patient is asymptomaticE

Page 267: Back to Filters (/Secure/TestMe/Filter ... - 1 File Download

8/13/2016 MyPastest

https://mypastest.pastest.com/Secure/TestMe/Browser/429893#Top 2/2

45804

Charcoal should be used only for modified-release preparations (Option D) is incorrect.Activated charcoal should normally be administered within 1 hour of drug overdose, but maybe effective when administered after a longer interval, particularly after modified-releasepreparations.

The patient is asymptomatic (Option E) is incorrect. The absence of symptoms does not helpdetermine whether patients are at risk of delayed toxicity, and charcoal should normally beadministered within 1 hour of overdose to minimise absorption and prevent toxicity.

Next Question

Previous Question Tag Question Feedback End Review

Difficulty: Average

Peer Responses

Session Progress

0Responses Correct:

179Responses Incorrect:

179Responses Total:

0%Responses - % Correct:

Blog (https://www.pastest.com/blog) About Pastest (https://www.pastest.com/about-us)Contact Us (https://www.pastest.com/contact-us) Help (https://www.pastest.com/help)

© Pastest 2016

Page 268: Back to Filters (/Secure/TestMe/Filter ... - 1 File Download

8/13/2016 MyPastest

https://mypastest.pastest.com/Secure/TestMe/Browser/429893#Top 1/2

Back to Filters (/Secure/TestMe/Filter/429893/QA)

Question 134 of 179

A 25-year-old man presents to the Emergency Department with an acute exacerbation ofasthma that is failing to respond to increased use of his usual inhaled salbutamol inhaler. Youare asked to review him in the resuscitation area and diagnose acute severe asthma; youdecide to treat him with intravenous magnesium.

Which of the following statements is true?

Explanation

The answer is Magnesium relaxes bronchial smooth muscle –

Intravenous magnesium is now indicated in the management of severe life-threateningacute asthma attacks; the normal adult dose is 1.2–2 g given over 20 min. Its principalpharmacological actions are inhibition of acetylcholine release at the neuromuscularjunction, relaxation of bronchial smooth muscle and stabilisation of mast cells. Adverseeffects are uncommon after a single administration, although a slight decrease in bloodpressure and flushing can occur. Symptoms of chronic hypermagnesaemia may includenausea, diarrhoea, flushing, hypertension, confusion, loss of deep tendon reflexes andcoma.

A rise in systemic blood pressure occurs commonly (Option A) is incorrect. A slight fall inblood pressure may occur.

Drowsiness and coma in severe asthma are normally caused by hypomagnesaemia (OptionB) is incorrect. Drowsiness and coma in severe asthma may be caused by hypercapnoea andrespiratory acidosis, and/or hypoxia.

A rise in systemic blood pressure occurs commonlyA

Drowsiness and coma in severe asthma are normally caused by hypomagnesaemiaB

Magnesium increases bronchial acetylcholine releaseC

Magnesium relaxes bronchial smooth muscleD

The usual dose is 2 mg intravenouslyE

Page 269: Back to Filters (/Secure/TestMe/Filter ... - 1 File Download

8/13/2016 MyPastest

https://mypastest.pastest.com/Secure/TestMe/Browser/429893#Top 2/2

46377

Magnesium increases bronchial acetylcholine release (Option C) is incorrect. Magnesiumreduces bronchial acetylcholine release.

The usual dose is 2 mg intravenously (Option E) is incorrect. The usual dose is 1.2–2 g. It isreasonable to expect candidates to know dosages for medications required in emergencysituations such as this.

Next Question

Previous Question Tag Question Feedback End Review

Difficulty: Average

Peer Responses

Session Progress

0Responses Correct:

179Responses Incorrect:

179Responses Total:

0%Responses - % Correct:

Blog (https://www.pastest.com/blog) About Pastest (https://www.pastest.com/about-us)Contact Us (https://www.pastest.com/contact-us) Help (https://www.pastest.com/help)

© Pastest 2016

Page 270: Back to Filters (/Secure/TestMe/Filter ... - 1 File Download

8/13/2016 MyPastest

https://mypastest.pastest.com/Secure/TestMe/Browser/429893#Top 1/2

Back to Filters (/Secure/TestMe/Filter/429893/QA)

Question 135 of 179

A 25-year-old woman who has a history of asthma attends for review. She is currently takingfluticasone dipropionate 250 μg inhaled twice daily. You plan to add salmeterol.

Which of the following statements best fits the pharmacological characteristics ofsalmeterol?

Explanation

The answer is It may cause paradoxical bronchospasm –

Salmeterol is a long-acting β -agonist indicated for the treatment of bronchial asthma incombination with inhaled corticosteroid therapy. It should not be used as first-linetherapy because clinical studies suggest increased mortality and morbidity in patientstreated with long-acting β -agonists monotherapy; they are normally used after inhaledcorticosteroids. Bronchospasm is a rare but recognised complication of salmeterol andother β -agonist treatments; bronchospasm is a more common complication of inhaleddisodium chromoglycate. Recognised effects of salmeterol and formoterol (long-actingβ -agonists) include hypokalaemia; they should be used with caution in severe livercirrhosis, breast feeding and pregnancy.

It is a long-acting β -agonist (Option B) is incorrect. Although salmeterol is β -selective, it isnot 100% specific and there is some crossover of receptor expression between cardiac andpulmonary tissue so that β - and β -agonist effects are recognised.

It is a short-acting β -agonist (Option C) is incorrect. Although salmeterol is β -selective, it isnot 100% specific and there is some crossover of receptor expression between cardiac andpulmonary tissue so that β - and β -agonist effects are recognised.

It may cause paradoxical bronchospasmA

It is a long-acting β -agonistB 1

It is a short-acting β -agonistC 1

It is a short-acting β -agonistD 2

Tachycardias are not associated with its useE

2

2

2

2

1 2

1 2

1 2

1 2

Page 271: Back to Filters (/Secure/TestMe/Filter ... - 1 File Download

8/13/2016 MyPastest

https://mypastest.pastest.com/Secure/TestMe/Browser/429893#Top 2/2

46882

It is a short-acting β -agonist (Option D) is incorrect. Salmeterol and formoterol are long-acting agents; salbutamol and terbutaline are short-acting β -agonists.

Tachycardias are not associated with its use (option E) is incorrect. Although delivered inhighest concentration to the lungs by inhalation, there may be some systemic effects,including tachycardia due to direct stimulation of β - and β -adrenoceptors in the heart. It is along-acting agent.

2

2

1 2

Next Question

Previous Question Tag Question Feedback End Review

Difficulty: Average

Peer Responses

Session Progress

0Responses Correct:

179Responses Incorrect:

179Responses Total:

0%Responses - % Correct:

Blog (https://www.pastest.com/blog) About Pastest (https://www.pastest.com/about-us)Contact Us (https://www.pastest.com/contact-us) Help (https://www.pastest.com/help)

© Pastest 2016

Page 272: Back to Filters (/Secure/TestMe/Filter ... - 1 File Download

8/13/2016 MyPastest

https://mypastest.pastest.com/Secure/TestMe/Browser/429893#Top 1/2

Back to Filters (/Secure/TestMe/Filter/429893/QA)

Question 136 of 179

A 27-year-old homeless man is admitted having ingested a large quantity of methanol whichhe found on a garage forecourt. He is inebriated with a deteriorating conscious level. He has apH of 7.15 with a serum bicarbonate concentration of 14 mmol/l.

What is the most appropriate next step in his management?

Explanation

The answer is Administration of ethanol –

Methanol is subject to hepatic metabolism, initially by alcohol dehydrogenase andadditional enzymatic steps that result in formation of toxic metabolites. The metabolitesmay cause severe nephrotoxicity and neurotoxicity, and provoke a number of metabolicderangements including profound hypocalcaemia. Administration of fomepizole, acompetitive alcohol dehydrogenase inhibitor, prevents formation of methanolmetabolites and thereby minimises toxicity. An alternative is administration of ethanolwhich competes with methanol for metabolism, thereby lessening but not fullypreventing methanol metabolism. Haemodialysis may be considered when methanollevels are particularly high or when metabolic acidosis cannot be corrected bybicarbonate administration.

Administration of ethylene glycol (Option B) is incorrect. Ethylene glycol would contribute totoxicity.

Administration of folinic acid (Option C) is incorrect. Administration of folinic acid mayprotect against ocular toxicity but is a less good option here because it is an adjunctivetreatment that may be considered in addition to fomepizole or ethanol administration.

Administration of ethanolA

Administration of ethylene glycolB

Administration of folinic acidC

Administration of formaldehydeD

Administration of formateE

Page 273: Back to Filters (/Secure/TestMe/Filter ... - 1 File Download

8/13/2016 MyPastest

https://mypastest.pastest.com/Secure/TestMe/Browser/429893#Top 2/2

46847

Administration of formaldehyde (Option D) is incorrect. Formaldehyde is incorrect; this is oneof the metabolites that contributes to metabolic acidosis.

Administration of formate (Option E) is incorrect. Formate is one of the metabolic substratesarising from metabolism of formaldehyde by formaldehyde dehydrogenase.

Next Question

Previous Question Tag Question Feedback End Review

Difficulty: Average

Peer Responses

Session Progress

0Responses Correct:

179Responses Incorrect:

179Responses Total:

0%Responses - % Correct:

Blog (https://www.pastest.com/blog) About Pastest (https://www.pastest.com/about-us)Contact Us (https://www.pastest.com/contact-us) Help (https://www.pastest.com/help)

© Pastest 2016

Page 274: Back to Filters (/Secure/TestMe/Filter ... - 1 File Download

8/13/2016 MyPastest

https://mypastest.pastest.com/Secure/TestMe/Browser/429893#Top 1/2

Back to Filters (/Secure/TestMe/Filter/429893/QA)

Question 137 of 179

A 32-year-old former heroin user is on a methadone programme. He is admitted to hospitalafter a motor vehicle accident and has sustained multiple pelvic fractures.

Which one of the following is the most appropriate approach to his analgesic medications?

Explanation

The answer is Continue on methadone and add diclofenac –

Continuation of methadone and consideration of analgesics with a different mode ofaction (ie non-steroidals such as parenteral diclofenac) is recommended. Despiteconcerns about cardiovascular risk associated with NSAIDs, the balance of benefit–riskwith respect to short-term use of parenteral diclofenac is favourable in this situation.Methadone can then be titrated over a number of days to relieve the extra pain owing tothe pelvic fractures in this particular patient.

Continue methadone and titrate to pain requirement (Option B) is incorrect. Titration ofmethadone every 6–8 h for acute pain is not recommended in chronic users because there isa significant risk of accumulation.

Discontinue methadone (Option C) is incorrect. Discontinuation of methadone may result insymptoms of acute opiate withdrawal and is not recommended.

Discontinue methadone and start on parenteral morphine (Option D) is incorrect.Discontinuation of methadone and substitution with morphine would be complex anddifficult to manage safely; patients would be at high risk of opioid intoxication or withdrawalfeatures.

Continue on methadone and add diclofenacA

Continue methadone and titrate to pain requirementB

Discontinue methadoneC

Discontinue methadone and start on parenteral morphineD

Discontinue methadone and substitute diclofenacE

Page 275: Back to Filters (/Secure/TestMe/Filter ... - 1 File Download

8/13/2016 MyPastest

https://mypastest.pastest.com/Secure/TestMe/Browser/429893#Top 2/2

46927

Discontinue methadone and substitute diclofenac (Option E) is incorrect. Discontinuation ofmethadone may result in symptoms of acute opioid withdrawal.

Next Question

Previous Question Tag Question Feedback End Review

Difficulty: Average

Peer Responses

Session Progress

0Responses Correct:

179Responses Incorrect:

179Responses Total:

0%Responses - % Correct:

Blog (https://www.pastest.com/blog) About Pastest (https://www.pastest.com/about-us)Contact Us (https://www.pastest.com/contact-us) Help (https://www.pastest.com/help)

© Pastest 2016

Page 276: Back to Filters (/Secure/TestMe/Filter ... - 1 File Download

8/13/2016 MyPastest

https://mypastest.pastest.com/Secure/TestMe/Browser/429893#Top 1/2

Back to Filters (/Secure/TestMe/Filter/429893/QA)

Question 138 of 179

A 35-year-old woman, who has been on antiepileptic medication for many years, presentscomplaining of fatigue, lethargy, bone pain, tingling and numbness in her lower limbs, andswelling of her gums. Investigations reveal Hb 8.4 g/dl, MCV 106 fl, WCC 7.2 × 10 /l, platelets170 × 10 /l, alkaline phosphatase 534 IU/l, parathyroid hormone 10.4 pmol/l.

Which antiepileptic medication is most likely to cause these problems?

Explanation

The answer is Phenytoin –

Both phenytoin and phenobarbital are associated with symptoms of fatigue, lethargy,bone pain, tingling and numbness in the lower limbs; however, phenytoin is the only onethat causes gum hypertrophy. Folate deficiency and megaloblastic anaemia may arisedue to enzyme induction and rapid folate metabolism, and due to phenytoin mediatedinhibition of intestinal conjugase. Osteomalacia and neuropathy are long-term side-effects of both phenytoin and phenobarbital. Primidone is a pro-drug that is converted inthe liver to phenobarbital.

Carbamazepine (Option A) is incorrect. Adverse effects of carbamazepine includedrowsiness, ataxia, nystagmus, diplopia, rash, and thrombocytopaenia and other blooddyscrasias.

Phenobarbital (Option B) is incorrect. Phenobarbital may cause many of the presentedsymptoms, but gum hypertrophy is not a recognised adverse effect.

Primidone (Option D) is incorrect. Primidone is a pro-drug of phenobarbital and may causemany of the presented symptoms, but gum hypertrophy is not a recognised adverse effect.

9

9

CarbamazepineA

PhenobarbitalB

PhenytoinC

PrimidoneD

Sodium valproateE

Page 277: Back to Filters (/Secure/TestMe/Filter ... - 1 File Download

8/13/2016 MyPastest

https://mypastest.pastest.com/Secure/TestMe/Browser/429893#Top 2/2

46738

Sodium valproate (Option E) is incorrect. Sodium valproate is associated with alopecia, rash,blood dyscrasias, liver damage, pancreatitis and hyperammonaemia.

Next Question

Previous Question Tag Question Feedback End Review

Difficulty: Average

Peer Responses

Session Progress

0Responses Correct:

179Responses Incorrect:

179Responses Total:

0%Responses - % Correct:

Blog (https://www.pastest.com/blog) About Pastest (https://www.pastest.com/about-us)Contact Us (https://www.pastest.com/contact-us) Help (https://www.pastest.com/help)

© Pastest 2016

Page 278: Back to Filters (/Secure/TestMe/Filter ... - 1 File Download

8/13/2016 MyPastest

https://mypastest.pastest.com/Secure/TestMe/Browser/429893#Top 1/2

Back to Filters (/Secure/TestMe/Filter/429893/QA)

Question 139 of 179

A 68-year-old male patient with disseminated carcinoma of the prostate is being treated withbuprenorphine, which until recently has controlled his bone pain well. Other significanthistory of note includes chronic renal failure; his creatinine is 205 μmol/l. More recently, hehas complained of increasing pain in the hip, but maintains a good functional status andwants to continue to tend his garden at home.

Which of the following measures would be most appropriate to optimise his pain control?

Explanation

The answer is Change the buprenorphine to morphine elixir –

Buprenorphine is a partial agonist at opioid receptors; it will antagonise the action of afull agonist such as morphine. Therefore, the addition of morphine or codeine tobuprenorphine is illogical. The main options here may be to increase the buprenorphinedose or to switch to an alternative such as morphine or oxycodone. Titrating the dose ofmorphine should be done with short-lived formulations of morphine, and once adequateanalgesia is obtained, then switch to sustained-release morphine.

Add amitriptyline to his therapy (Option A) is incorrect. Amitriptyline is an effectivetreatment for neuropathic pain, but much less effective for bony pain. It may be a helpfuladjunct to allow normal sleep patterns.

Add diclofenac to his therapy (Option B) is incorrect. Non-steroidal anti-inflammatory drugs(NSAIDs) are useful if there is a significant inflammatory component to pain, but wouldprobably be best avoided here in the presence of such a degree of renal impairment.

Add amitriptyline to his therapyA

Add diclofenac to his therapyB

Add morphine elixir to his therapyC

Change the buprenorphine to morphine elixirD

Substitute sustained-relase morphine tablets for buprenorphineE

Page 279: Back to Filters (/Secure/TestMe/Filter ... - 1 File Download

8/13/2016 MyPastest

https://mypastest.pastest.com/Secure/TestMe/Browser/429893#Top 2/2

46895

Add morphine elixir to his therapy (Option C) is incorrect. It would be illogical to addmorphine to buprenorphine, because buprenorphine effectively acts as a morphineantagonist.

Substitute sustained-relase morphine tablets for buprenorphine (Option E) is incorrect. Thismay be a reasonable option if the morphine dose can be estimately safely, but this is a lesspreferred option than administering short-acting morphine first until the daily requirementscan be ascertained.

Next Question

Previous Question Tag Question Feedback End Review

Difficulty: Average

Peer Responses

Session Progress

0Responses Correct:

179Responses Incorrect:

179Responses Total:

0%Responses - % Correct:

Blog (https://www.pastest.com/blog) About Pastest (https://www.pastest.com/about-us)Contact Us (https://www.pastest.com/contact-us) Help (https://www.pastest.com/help)

© Pastest 2016

Page 280: Back to Filters (/Secure/TestMe/Filter ... - 1 File Download

8/13/2016 MyPastest

https://mypastest.pastest.com/Secure/TestMe/Browser/429893#Top 1/2

Back to Filters (/Secure/TestMe/Filter/429893/QA)

Question 140 of 179

46599

You review a 76-year-old man in the cardiovascular clinic. He has a history of congestive heartfailure, and is receiving a number of different medications. He describes feeling dizzy, andblood pressure is 98/62 mmHg. In view of his low blood pressure, you wish to stop one of hisexisting medications.

Which one of the following drugs if stopped would have the least impact on long-termsurvival in this patient?

Explanation

The answer is Digoxin –

All the listed drugs other than digoxin have been shown to improve mortality rates incongestive heart failure. Digoxin may be helpful in reducing cardiac symptoms, and mayreduce heart failure deaths, but has no significant effect on cardiovascular and overallmortality.

Bisoprolol (Option A) is incorrect. Bisoprolol, metoprolol and carvedilol have been shown toimprove survival in heart failure patients.

Candesartan (Option B) is incorrect. Candesartan and other angiotensin receptor antagonistshave been shown to improve survival in heart failure patients.

Enalapril (Option D) is incorrect. Enalapril and other ACE inhibitors have been shown toimprove survival in heart failure patients.

Spironolactone (Option E) is incorrect. Spironolactone has been shown to improve survival inheart failure patients when added to other conventional medications.

BisoprololA

CandesartanB

DigoxinC

EnalaprilD

SpironolactoneE

Page 281: Back to Filters (/Secure/TestMe/Filter ... - 1 File Download

8/13/2016 MyPastest

https://mypastest.pastest.com/Secure/TestMe/Browser/429893#Top 2/2

46599

Next Question

Previous Question Tag Question Feedback End Review

Difficulty: Average

Peer Responses

Session Progress

0Responses Correct:

179Responses Incorrect:

179Responses Total:

0%Responses - % Correct:

Blog (https://www.pastest.com/blog) About Pastest (https://www.pastest.com/about-us)Contact Us (https://www.pastest.com/contact-us) Help (https://www.pastest.com/help)

© Pastest 2016

Page 282: Back to Filters (/Secure/TestMe/Filter ... - 1 File Download

8/13/2016 MyPastest

https://mypastest.pastest.com/Secure/TestMe/Browser/429893#Top 1/2

Back to Filters (/Secure/TestMe/Filter/429893/QA)

Question 141 of 179

An 81-year-old woman is admitted to the Emergency Department with a severe pneumonia,hypotension and signs of sepsis. There is a history of previous GP attendance for a urinarytract infection. According to her husband, she has been prescribed a prolonged course of co-trimoxazole, as this has been her third urinary infection in the past 2 months. She has haddiarrhoea and vomiting for the past 24 h. Her total white blood count is 1.8 × 10 /litre, andneutrophil count is 0.2 × 10 /litre.

What is the most likely explanation for these laboratory findings?

Explanation

The answer is Neutropaenia secondary to co-trimoxazole therapy –

Co-trimoxazole is now only indicated for oral prophylaxis against pneumocystispneumonia, toxoplasmosis and nocardiosis due to the risk of neutropaenia. It is effectiveagainst a number of organisms that cause respiratory and urinary infections, but shouldnormally only be considered where there is no other safer alternative. Adverse effectsinclude nausea, rash, Stephens–Johnson syndrome, toxic epidermal necrolysis,neutropaenia, thrombocytopaenia and photosensitivity. It should be avoided or used withcaution in patients with renal or hepatic impairment.

Chronic myeloid leukaemia (Option A) is incorrect. A hallmark of chronic myeloid leukaemiais production of high numbers of leukocytes, not neutropaenia, albeit that they are oftendysfunctional and render patients susceptible to infection.

Salmonella infection (Option C) is incorrect. Salmonella infection may be associated withleukopaenia and neutropaenia, although it would be very rare for the white count to be

9

9

Chronic myeloid leukaemiaA

Neutropaenia secondary to co-trimoxazole therapyB

Salmonella infectionC

Severe pneumoniaD

Urinary tract infection with sepsisE

Page 283: Back to Filters (/Secure/TestMe/Filter ... - 1 File Download

8/13/2016 MyPastest

https://mypastest.pastest.com/Secure/TestMe/Browser/429893#Top 2/2

46886

suppressed to the low levels reported here.

Severe pneumonia (Option D) is incorrect. Overwhelming bacterial infection may beassociated with leukopaenia and neutropaenia, although it would be very rare for the whitecount to be suppressed to the low levels reported here.

Urinary tract infection with sepsis (Option E) is incorrect. Overwhelming bacterial infectionmay be associated with leukopaenia and neutropaenia, although it would be very rare for thewhite count to be suppressed to the low levels reported here.

Next Question

Previous Question Tag Question Feedback End Review

Difficulty: Average

Peer Responses

Session Progress

0Responses Correct:

179Responses Incorrect:

179Responses Total:

0%Responses - % Correct:

Blog (https://www.pastest.com/blog) About Pastest (https://www.pastest.com/about-us)Contact Us (https://www.pastest.com/contact-us) Help (https://www.pastest.com/help)

© Pastest 2016

Page 284: Back to Filters (/Secure/TestMe/Filter ... - 1 File Download

8/13/2016 MyPastest

https://mypastest.pastest.com/Secure/TestMe/Browser/429893#Top 1/2

Back to Filters (/Secure/TestMe/Filter/429893/QA)

Question 142 of 179

22512

A 72-year-old man is managed on the elderly care ward where he was admitted with terminalbronchial carcinoma because his wife was unable to manage at home. He has known cerebralmetastases and is controlled with regular morphine sulphate (MST) and top-up oralmorphine. You are asked to see him by the on-call FY2 who gave him an injection ofhaloperidol because the nurses were unable to manage his agitation. On examination his eyesare deviated to the left and he seems unable to correct this. His tongue is also protruding andhis neck is in spasm, with his head forcibly rotated to the left-hand side.

Which one of the following is the most appropriate treatment?

Explanation

Oculogyric crisis

The history given here is consistent with an oculogyric crisis, the treatment of which is

with an anti-cholinergic such as trihexiphenidyl hydrochloride

It is important to recognise the condition, which is known to occur with anti-psychotics

and anti-emetics, as prompt intervention can lead to rapid resolution of the condition

Other notes

Because of increased risk of acute dystonia, use of metoclopramide and

prochlorperazine is not recommended, particularly in young women

Further oral morphineA

Oral diazepamB

IV midazolamC

Trihexiphenidyl hydrochloride syrupD

S/c diamorphineE

Next Question

Page 285: Back to Filters (/Secure/TestMe/Filter ... - 1 File Download

8/13/2016 MyPastest

https://mypastest.pastest.com/Secure/TestMe/Browser/429893#Top 2/2

Previous Question Tag Question Feedback End Review

Difficulty: Average

Peer Responses

Session Progress

0Responses Correct:

179Responses Incorrect:

179Responses Total:

0%Responses - % Correct:

Blog (https://www.pastest.com/blog) About Pastest (https://www.pastest.com/about-us)Contact Us (https://www.pastest.com/contact-us) Help (https://www.pastest.com/help)

© Pastest 2016

Page 286: Back to Filters (/Secure/TestMe/Filter ... - 1 File Download

8/13/2016 MyPastest

https://mypastest.pastest.com/Secure/TestMe/Browser/429893#Top 1/2

Back to Filters (/Secure/TestMe/Filter/429893/QA)

Question 143 of 179

46617

A 39-year-old woman underwent excision of a meningioma 9 months ago, and has beenmaintained on regular antiepileptic medication since that time. Over the past few months, shehas complained of facial acne, something she has not suffered from previously, and she hasdeveloped painless swelling around her gums.

Which of the following medicaitons is most likely to account for her symptoms?

Explanation

The answer is Phenytoin –

Patients may be treated with any one of a number of antiepileptic agents afterneurosurgery, including carbamazepine, sodium valproate and lamotrigine. Phenytoinmay be effective, but has many adverse effects associated with long-term use, and is lesspreferred; these include gum hypertrophy, folate deficiency, osteomalacia, coarse facialfeatures, neuropathy and aplastic anaemia (rarely).

Carbamazepine (Option A) is incorrect. Carbamazepine may cause drowsiness, blurred visionand ataxia.

Oxcarbazepine (Option B) is incorrect. Oxcarbazepine may cause drowsiness, blurred visionand ataxia.

Sodium valproate (Option D) is incorrect. Sodium valproate may cause drowsiness, moodchanges, alopecia, thrombocytopaenia, nausea and liver impairment.

Topiramate (Option E) is incorrect. Topiramate may cause drowsiness, speech disturbance,impaired concentration and mood changes.

CarbamazepineA

OxcarbazepineB

PhenytoinC

Sodium valproateD

TopiramateE

Page 287: Back to Filters (/Secure/TestMe/Filter ... - 1 File Download

8/13/2016 MyPastest

https://mypastest.pastest.com/Secure/TestMe/Browser/429893#Top 2/2

46617

Next Question

Previous Question Tag Question Feedback End Review

Difficulty: Average

Peer Responses

Session Progress

0Responses Correct:

179Responses Incorrect:

179Responses Total:

0%Responses - % Correct:

Blog (https://www.pastest.com/blog) About Pastest (https://www.pastest.com/about-us)Contact Us (https://www.pastest.com/contact-us) Help (https://www.pastest.com/help)

© Pastest 2016

Page 288: Back to Filters (/Secure/TestMe/Filter ... - 1 File Download

8/13/2016 MyPastest

https://mypastest.pastest.com/Secure/TestMe/Browser/429893#Top 1/2

Back to Filters (/Secure/TestMe/Filter/429893/QA)

Question 144 of 179

You review a 68-year-old man who attends the Ambulatory Care Unit with a swollen left calfand undergoes a Doppler scan. This confirms a diagnosis of deep vein thrombosis and youdecide to administer warfarin. The patient is receiving a number of other medications.

Which of the following substances is most likely to reduce the effect of warfarin therapy?

Explanation

The answer is Cholestyramine –

Cholestyramine is a resin used for binding cholesterol in the gut and may be used totreat familial hypercholesterolaemia. Cholestyramine reduces absorption of a number ofdrugs including warfarin. Warfarin has a half-life of around 36 h and is eliminated byhepatic metabolism; it is contraindicated in patients with severe hepatic disease. Co-administration of enzyme inducing drugs including phenytoin and carbamazepine mayincrease the extent of warfarin metabolism, thereby reducing INR.

Acute alcohol ingestion (Option A) is incorrect. Acute ethanol intake transiently inhibitshepatic enzyme activity, thereby increasing INR and bleeding risk. In contrast, chronicethanol intake may upregulate hepatic enzyme activity.

Cimetidine (Option C) is incorrect. Cimetidine is an anti-H2 histamine antagonist that inhibitshepatic enzyme activity, thereby increasing INR.

Co-trimoxazole (Option D) is incorrect. Co-trimoxazole may increase warfarin action due to aprotein-binding interaction.

Acute alcohol ingestionA

CholestyramineB

CimetidineC

Co-trimoxazoleD

MetronidazoleE

Page 289: Back to Filters (/Secure/TestMe/Filter ... - 1 File Download

8/13/2016 MyPastest

https://mypastest.pastest.com/Secure/TestMe/Browser/429893#Top 2/2

46892

Metronidazole (Option E) is incorrect. Metronidazole is a hepatic enzyme inhibitor that mayincrease warfarin activity and increase INR.

Next Question

Previous Question Tag Question Feedback End Review

Difficulty: Average

Peer Responses

Session Progress

0Responses Correct:

179Responses Incorrect:

179Responses Total:

0%Responses - % Correct:

Blog (https://www.pastest.com/blog) About Pastest (https://www.pastest.com/about-us)Contact Us (https://www.pastest.com/contact-us) Help (https://www.pastest.com/help)

© Pastest 2016

Page 290: Back to Filters (/Secure/TestMe/Filter ... - 1 File Download

8/13/2016 MyPastest

https://mypastest.pastest.com/Secure/TestMe/Browser/429893#Top 1/2

Back to Filters (/Secure/TestMe/Filter/429893/QA)

Question 145 of 179

46756

A retired salesman attends the cardiology outpatient clinic for review of his hypertension andmoderate heart failure. He is complaining of a persistent dry cough that developed soon afterhis previous clinic appointment 6 months ago.

Which medication is most likely to cause this symptom?

Explanation

The answer is Lisinopril –

Angiotensin-converting enzyme (ACE) inhibitors such as lisinopril interfere with thebreakdown of bradykinin in the lungs, and high bradykinin concentrations appearcapable of causing a persistent dry cough in 10–20% of patients.

Candesartan (Option A) is incorrect. Angiotensin II receptor antagonists (losartan,candesartan) are useful alternatives as they do not potentiate bradykinin, and cough isreported in only 1–2% of patients.

Doxazosin (Option B) is incorrect. Doxazosin typically causes postural hypotension, fluidretention and heart failure; less commonly it may cause rhinitis.

Methyldopa (Option D) is incorrect. Methyldopa is usually associated with nasal congestionand a lupus-like syndrome.

Propranolol (Option E) is incorrect. Propranolol is a non-selective β1- and β2-blocker and mayprecipitate bronchospasm in asthmatic individuals, or prevent β2-agonist bronchodilatation.

CandesartanA

DoxazosinB

LisinoprilC

MethyldopaD

PropranololE

Page 291: Back to Filters (/Secure/TestMe/Filter ... - 1 File Download

8/13/2016 MyPastest

https://mypastest.pastest.com/Secure/TestMe/Browser/429893#Top 2/2

Next Question

Previous Question Tag Question Feedback End Review

Difficulty: Average

Peer Responses

Session Progress

0Responses Correct:

179Responses Incorrect:

179Responses Total:

0%Responses - % Correct:

Blog (https://www.pastest.com/blog) About Pastest (https://www.pastest.com/about-us)Contact Us (https://www.pastest.com/contact-us) Help (https://www.pastest.com/help)

© Pastest 2016

Page 292: Back to Filters (/Secure/TestMe/Filter ... - 1 File Download

8/13/2016 MyPastest

https://mypastest.pastest.com/Secure/TestMe/Browser/429893#Top 1/2

Back to Filters (/Secure/TestMe/Filter/429893/QA)

Question 146 of 179

46937

A 56-year-old man has been admitted with lower abdominal pain and difficulty urinating. Abedside bladder scan indicates a residual bladder volume of around 580 ml. He has beenreceiving a number of different medications.

Which of the following drugs is most likely to cause urinary retention?

Explanation

The answer is Amitriptyline –

Amitriptyline causes a number of effects, including anticholinergic effects. Commonanticholinergic effects include dry mouth, blurred vision, urinary retention, constipation,palpitations and tachycardia.

Diazepam (Option B) is incorrect. Diazepam does not provoke any anticholinergic effects.

Fluoxetine (Option C) is incorrect. Fluoxetine potentiates the effects of serotonin and has nodirect effect on cholinergic pathways.

Venlafaxine (Option D) is incorrect. Venlafaxine possesses a mixture of enhanced serotoninand norepinephrine effects, the latter more prominent after high doses.

Zopiclone (Option E) is incorrect. Zopiclone exerts effects similar to benzodiazepines, andhas no known anticholinergic properties.

AmitriptylineA

DiazepamB

FluoxetineC

VenlafaxineD

ZopicloneE

Next Question

Page 293: Back to Filters (/Secure/TestMe/Filter ... - 1 File Download

8/13/2016 MyPastest

https://mypastest.pastest.com/Secure/TestMe/Browser/429893#Top 2/2

Previous Question Tag Question Feedback End Review

Difficulty: Average

Peer Responses

Session Progress

0Responses Correct:

179Responses Incorrect:

179Responses Total:

0%Responses - % Correct:

Blog (https://www.pastest.com/blog) About Pastest (https://www.pastest.com/about-us)Contact Us (https://www.pastest.com/contact-us) Help (https://www.pastest.com/help)

© Pastest 2016

Page 294: Back to Filters (/Secure/TestMe/Filter ... - 1 File Download

8/13/2016 MyPastest

https://mypastest.pastest.com/Secure/TestMe/Browser/429893#Top 1/2

Back to Filters (/Secure/TestMe/Filter/429893/QA)

Question 147 of 179

A patient with haematemesis is stabilised and undergoes urgent upper gastrointestinalendoscopy. He is started on omeprazole, with good response.

Which of the following best explains the mechanism of action of omeprazole that is likely topromote ulcer healing?

Explanation

The answer is Inhibition of H+/K+-ATPase –

Omeprazole and other members of this class of drugs (proton pump inhibitors (PPIs))act by inhibiting the hydrogen/potassium adenosine triphosphatase enzyme system(H /K -ATPase or ‘proton pump’), which secretes hydrogen ions into the gastric lumen.They are prodrugs that are taken up into the parietal cells and metabolised into an activedrug that interferes with proton pump activity. Acid secretion is only restored when thecell synthesises new proton-pump protein, so that the effects last much longer than thedrugs, which are rapidly cleared from the circulation; they are longer acting than H -receptor antagonists. Once-daily dosing is normally sufficient to allow healing of pepticulcers and prevent gastro-oesophageal acid reflux.

Gastric prostaglandin secretion (Option A) is incorrect. One of the mechanisms of pepticulcer in patients treated with aspirin or non-steroidal anti-inflammatory drugs (NSAIDs) isaltered prostaglandin synthesis. Drugs that specifically target prostaglandin synthesis havenot been proven to improve ulcer healing.

H2-receptor blockade (Option B) is incorrect. Histamine subtype-2 receptor antagonists, egranitidine, are less effective in suppressing gastric acid secretion and not as effective in

Gastric prostaglandin secretionA

H2-receptor blockadeB

Helicobacter pylori eradicationC

Increase in gastric emptyingD

Inhibition of H+/K+-ATPaseE

+ +

2

Page 295: Back to Filters (/Secure/TestMe/Filter ... - 1 File Download

8/13/2016 MyPastest

https://mypastest.pastest.com/Secure/TestMe/Browser/429893#Top 2/2

46804

promoting ulcer healing.

Helicobacter pylori eradication (Option C) is incorrect. PPI therapy is only one part of H.pylori eradication regimens, which require the addition of antibiotics to be effective.Eradication is valuable in accelerating healing of acute and chronic peptic ulcers in patientswith proven infection.

Increase in gastric emptying (Option D) is incorrect. Certain drugs, particularlymetoclopramide or macrolide antibiotics, may promote gastric emptying but this mechanismhas no direct effect on ulcer healing.

Next Question

Previous Question Tag Question Feedback End Review

Difficulty: Average

Peer Responses

Session Progress

0Responses Correct:

179Responses Incorrect:

179Responses Total:

0%Responses - % Correct:

Blog (https://www.pastest.com/blog) About Pastest (https://www.pastest.com/about-us)Contact Us (https://www.pastest.com/contact-us) Help (https://www.pastest.com/help)

© Pastest 2016

Page 296: Back to Filters (/Secure/TestMe/Filter ... - 1 File Download

8/13/2016 MyPastest

https://mypastest.pastest.com/Secure/TestMe/Browser/429893#Top 1/2

Back to Filters (/Secure/TestMe/Filter/429893/QA)

Question 148 of 179

A 55-year-old man is on long-term phenytoin therapy for control of generalised seizures. Heis admitted to hospital after a further generalised seizure, and the dose of his therapy hasbeen increased.

Which of the following adverse effects caused by phenytoin is most likely to be provoked bythe increased drug dose?

Explanation

The answer is Agitation –

Phenytoin may give rise to dose-dependent central nervous system adverse effects suchas agitation, ataxia, nystagmus and drowsiness. These are generally short-term adverseeffects that resolve after drug cessation or dose reduction. Adverse effects related tolong-term therapy include altered vitamin D metabolism and osteomalacia, selectiveimmunoglobulin A deficiency, and Dupuytren’s contracture. Idiosyncratic adverse effectsinclude hepatitis.

Bradycardia (Option B) is incorrect. Parenteral administration has been linked to atrial andventricular conduction depression and ventricular fibrillation; these are not seen after regularoral treatment.

Hemiparesis (Option C) is incorrect. Hemiparesis would suggest a focal neurological defect.

Hypotension (Option D) is incorrect. Hypotension does not normally occur after therapeuticdoses of phenytoin.

AgitationA

BradycardiaB

HemiparesisC

HypotensionD

OsteoporosisE

Page 297: Back to Filters (/Secure/TestMe/Filter ... - 1 File Download

8/13/2016 MyPastest

https://mypastest.pastest.com/Secure/TestMe/Browser/429893#Top 2/2

46914

Osteoporosis (Option E) is incorrect. This is a recognised adverse effect of long-termtreatment and unlikely to be provoked by a short-term dose increase.

Next Question

Previous Question Tag Question Feedback End Review

Difficulty: Average

Peer Responses

Session Progress

0Responses Correct:

179Responses Incorrect:

179Responses Total:

0%Responses - % Correct:

Blog (https://www.pastest.com/blog) About Pastest (https://www.pastest.com/about-us)Contact Us (https://www.pastest.com/contact-us) Help (https://www.pastest.com/help)

© Pastest 2016

Page 298: Back to Filters (/Secure/TestMe/Filter ... - 1 File Download

8/13/2016 MyPastest

https://mypastest.pastest.com/Secure/TestMe/Browser/429893#Top 1/2

Back to Filters (/Secure/TestMe/Filter/429893/QA)

Question 149 of 179

You are reviewing a 78-year-old woman with leg weakness and impaired mobility. Initialinvestigations show significant osteoporosis.

Which one of the following would be most effective in treatment of establishedosteoporosis?

Explanation

The answer is option 1500 mg/day of calcium, 400-800 units/day of vitamin D –

Adequate calcium and vitamin D intake should be part of the prevention and treatmentof osteoporosis. Recommmended dietary intake of calcium should be:

800–1000 mg/day in childhood through early adulthood

1000–1200 mg/day in middle adulthood

1500 mg/day in older adults

If osteoporosis is established, the treatment includes 1500 mg/day of calcium and 400–800 units/day of vitamin D (1000 units is equivalent to 25 micrograms).

800 mg/day of calcium, 100 units/day of vitamin D (Option A) is incorrect. Optimal is 1500mg/day of calcium, 400–800 units/day of vitamin D.

900 mg/day of calcium, 200 units/day of vitamin D (Option B) is incorrect. Optimal is 1500mg/day of calcium, 400–800 units/day of vitamin D.

800 mg/day of calcium, 100 units/day of vitamin DA

900 mg/day of calcium, 200 units/day of vitamin DB

1200 mg/day of calcium, 200–400 units/day of vitamin DC

1500 mg/day of calcium, 400–800 units/day of vitamin DD

1800 mg/day of calcium, 800–1000 units/day of vitamin DE

Page 299: Back to Filters (/Secure/TestMe/Filter ... - 1 File Download

8/13/2016 MyPastest

https://mypastest.pastest.com/Secure/TestMe/Browser/429893#Top 2/2

46734

1200 mg/day of calcium, 200–400 units/day of vitamin D (Option C) is incorrect. Optimal is1500 mg/day of calcium, 400–800 units/day of vitamin D.

1800 mg/day of calcium, 800–1000 units/day of vitamin D (Option E) is incorrect. Optimal is1500 mg/day of calcium, 400–800 units/day of vitamin D.

Next Question

Previous Question Tag Question Feedback End Review

Difficulty: Average

Peer Responses

Session Progress

0Responses Correct:

179Responses Incorrect:

179Responses Total:

0%Responses - % Correct:

Blog (https://www.pastest.com/blog) About Pastest (https://www.pastest.com/about-us)Contact Us (https://www.pastest.com/contact-us) Help (https://www.pastest.com/help)

© Pastest 2016

Page 300: Back to Filters (/Secure/TestMe/Filter ... - 1 File Download

8/13/2016 MyPastest

https://mypastest.pastest.com/Secure/TestMe/Browser/429893#Top 1/2

Back to Filters (/Secure/TestMe/Filter/429893/QA)

Question 150 of 179

46793

A 65-year-old woman has been receiving chemotherapy treatment and asks you what youknow about the possible adverse effects of bleomycin.

What is the most likely serious adverse effect of bleomycin that she is at increased risk ofdeveloping?

Explanation

The answer is Pneumonitis –

Bleomycin causes lung toxicity, including pneumonitis, which can progress to interstitialfibrosis and diminished gas transfer. The risks increase with cumulative bleomycin doses.

Acute kidney injury (Option A) is incorrect. A number of chemotherapy agents may provoketumour lysis syndrome and acute kidney injury, particularly in treatment of leukaemia andother haematological malignancies. Bleomycin is not particularly associated with this.Methotrexate in high doses can cause acute renal failure due to precipitation of the drug inthe renal tubules.

Cardiomyopathy (Option B) is incorrect. Doxorubicin can cause cardiac toxicity andcardiomyopathy.

Neuropathy (Option C) is incorrect. Cisplatin characteristically causes peripheral motor andsensory neuropathy; cytarabine administered in high single doses can cause irreversiblecerebellar damage.

Retinopathy (Option E) is incorrect. Ocular toxicity may characteristically occur afterplatinum-based agents and pyrimidine analogues.

Acute kidney injuryA

CardiomyopathyB

NeuropathyC

PneumonitisD

RetinopathyE

Page 301: Back to Filters (/Secure/TestMe/Filter ... - 1 File Download

8/13/2016 MyPastest

https://mypastest.pastest.com/Secure/TestMe/Browser/429893#Top 2/2

46793

Next Question

Previous Question Tag Question Feedback End Review

Difficulty: Average

Peer Responses

Session Progress

0Responses Correct:

179Responses Incorrect:

179Responses Total:

0%Responses - % Correct:

Blog (https://www.pastest.com/blog) About Pastest (https://www.pastest.com/about-us)Contact Us (https://www.pastest.com/contact-us) Help (https://www.pastest.com/help)

© Pastest 2016

Page 302: Back to Filters (/Secure/TestMe/Filter ... - 1 File Download

8/13/2016 MyPastest

https://mypastest.pastest.com/Secure/TestMe/Browser/429893#Top 1/2

Back to Filters (/Secure/TestMe/Filter/429893/QA)

Question 151 of 179

You are reviewing a patient in the outpatient department who has recently been diagnosedwith tuberculosis and has been started on combination therapy with isoniazid, rifampicin,pyrazinamide and ethambutol. Your consultant asks you to arrange for tests to determineacetylator status.

Which adverse effects would be expected to be more likely to occur if the patient was foundto be a slow acetylator?

Explanation

The answer is Peripheral neuropathy –

The patterns of drug efficacy and adverse effects may be predicted by genetic variation.For example, slow acetylators of isoniazid are more likely to be exposed to highconcentrations of isoniazid and develop adverse effects including peripheral neuropathy,which is a direct drug adverse effect. Rapid acetylators are associated with lower drugconcentrations and a greater risk of treatment failure; rapid acetylators are more proneto isoniazid-induced hepatitis, which is not caused by isoniazid but one of itsmetabolites.

Cranial nerve palsy (Option A) is incorrect. Slow acetylator status increases the likelihood ofisoniazid-induced neuropathy, but this more commonly involves a peripheral sensorimotorneuropathy.

Hepatitis (Option B) is incorrect. Isoniazid-induced hepatitis is less likely to occur in slowacetylators, and more likely in rapid acetylators because it is mediated by an isoniazidmetabolite.

Cranial nerve palsyA

HepatitisB

Peripheral neuropathyC

Renal toxicityD

Visual disturbancesE

Page 303: Back to Filters (/Secure/TestMe/Filter ... - 1 File Download

8/13/2016 MyPastest

https://mypastest.pastest.com/Secure/TestMe/Browser/429893#Top 2/2

46790

Renal toxicity (Option D) is incorrect. The risk of renal toxicity is not influenced acetylatorstatus.

Visual disturbances (Option E) is incorrect. Visual disturbance may represent a seriousadverse effect of ethambutol, but this is not influenced by acetylator status.

Next Question

Previous Question Tag Question Feedback End Review

Difficulty: Average

Peer Responses

Session Progress

0Responses Correct:

179Responses Incorrect:

179Responses Total:

0%Responses - % Correct:

Blog (https://www.pastest.com/blog) About Pastest (https://www.pastest.com/about-us)Contact Us (https://www.pastest.com/contact-us) Help (https://www.pastest.com/help)

© Pastest 2016

Page 304: Back to Filters (/Secure/TestMe/Filter ... - 1 File Download

8/13/2016 MyPastest

https://mypastest.pastest.com/Secure/TestMe/Browser/429893#Top 1/2

Back to Filters (/Secure/TestMe/Filter/429893/QA)

Question 152 of 179

You review a 72-year-old woman who has suffered from agitation and extreme nausea afterprevious general anaesthetics. You are on call for the surgical wards and note that theanaesthetists have prescribed haloperidol for nausea.

Which of the following best describes the main pharmacological site of antiemetic action ofhaloperidol?

Explanation

The answer is Chemoreceptor trigger zone –

Haloperidol is an anti-dopaminergic that is used mainly as an antipsychotic; it causesmore extrapyramidal adverse effects than phenothiazines, but a lower risk ofhypotension. Its main site of antiemetic action is the chemoreceptor trigger zone. Otherdrugs with antiemetic properties through an action on the chemoreceptor trigger zoneinclude phenothiazines (e.g. promethazine) and domperidone. Newer agents, such as the5-hydroxytryptamine (5-HT ) inhibitors (e.g. ondansetron), are associated with fewerside-effects, and act directly on 5-HT receptors in the gut and central nervous system.

Adrenergic receptors (Option A) is incorrect. Adrenergic pathways are not a particularlyimportant target for control of nausea.

Cholinergic receptors (Option C) is incorrect. Haloperidol exerts little if any cholinergiceffects. Cyclizine exerts its antiemetic effects by blocking cholinergic pathways.

It is a prokinetic agent (Option D) is incorrect. Metoclopramide has prokinetic actions andmay be more useful in nausea and vomiting related to gastrointestinal causes.

Adrenergic receptorsA

Chemoreceptor trigger zoneB

Cholinergic receptorsC

It is a prokinetic agentD

Vestibular systemE

3

Page 305: Back to Filters (/Secure/TestMe/Filter ... - 1 File Download

8/13/2016 MyPastest

https://mypastest.pastest.com/Secure/TestMe/Browser/429893#Top 2/2

46917

Vestibular system (Option E) is incorrect. Vestibular suppressants include low-dosebenzodiazepines, anticholinergic agents (eg cyclizine, hyoscine).

Next Question

Previous Question Tag Question Feedback End Review

Difficulty: Average

Peer Responses

Session Progress

0Responses Correct:

179Responses Incorrect:

179Responses Total:

0%Responses - % Correct:

Blog (https://www.pastest.com/blog) About Pastest (https://www.pastest.com/about-us)Contact Us (https://www.pastest.com/contact-us) Help (https://www.pastest.com/help)

© Pastest 2016

Page 306: Back to Filters (/Secure/TestMe/Filter ... - 1 File Download

8/13/2016 MyPastest

https://mypastest.pastest.com/Secure/TestMe/Browser/429893#Top 1/2

Back to Filters (/Secure/TestMe/Filter/429893/QA)

Question 153 of 179

46983

You are reviewing a phase 2 trial of a new agent for treating diabetes. You are aware fromsome phase 1 studies that the half-life of the agent is around 4 h. When deciding what dosingschedule would be appropriate in the next phase 2 study, you consider whether this drugshould be administered once, twice or thrice daily.

Given the half-life of this agent, approximately what percentage of the drug will have beeneliminated after 20 h?

Explanation

The answer is 97% -

The concentration of the drug reduces by 50% over each half-life. Therefore, after 4 h50%, after 8 h 25%, after 12 h 12.5%, after 16 h 6.25%, and after 20 h 3.125%. That is to say,at 20 h, the drug concentration will have fallen by 97%.

3% (Option A) is incorrect. Concentration will have fallen by 97%, so that it would be ataround 3% of original level at five half-lives, 20 h.

50% (Option B) is incorrect. Concentration will have fallen by 50% at one half-life, 4 h.

75% (Option C) is incorrect. Concentration will have fallen by 75% after two half-lives, 8 h.

87.5% (Option D) is incorrect. Concentration will have fallen by 87.5% after three half-lives, 12h.

3%A

50%B

75%C

87.5%D

97%E

Next Question

Page 307: Back to Filters (/Secure/TestMe/Filter ... - 1 File Download

8/13/2016 MyPastest

https://mypastest.pastest.com/Secure/TestMe/Browser/429893#Top 2/2

Previous Question Tag Question Feedback End Review

Difficulty: Average

Peer Responses

Session Progress

0Responses Correct:

179Responses Incorrect:

179Responses Total:

0%Responses - % Correct:

Blog (https://www.pastest.com/blog) About Pastest (https://www.pastest.com/about-us)Contact Us (https://www.pastest.com/contact-us) Help (https://www.pastest.com/help)

© Pastest 2016

Page 308: Back to Filters (/Secure/TestMe/Filter ... - 1 File Download

8/13/2016 MyPastest

https://mypastest.pastest.com/Secure/TestMe/Browser/429893#Top 1/2

Back to Filters (/Secure/TestMe/Filter/429893/QA)

Question 154 of 179

You have investigated a 23-year-old woman with morning headaches and made a diagnosisof benign intracranial hypertension. You have prescribed a course of acetazolamidetreatment.

Which of the following adverse effects would you be most likely to anticipate?

Explanation

The answer is Acute interstitial nephritis –

Acetazolamide is an inhibitor of carbonic anhydrase. It may be used to treat patients withhydrocephalus after intracerebral haemorrhage, to reduce intracranial pressure in benignintracranial hypertension, and to reduce intraocular pressure in glaucoma. It causesmetabolic acidosis, due to bicarbonate loss in the proximal and distal tubules throughinhibition of reabsorption, and hypokalaemia. Recognised adverse effects include acuteinterstitial nephritis (AIN), agranulocytosis and thrombocytopaenia.

Hyperkalaemia (Option B) is incorrect. Hypokalaemia is recognised rather thanhyperkalaemia.

Macrocytic hypochromic anaemia (Option C) is incorrect. Macrocytic hypochromic anaemiais not a recognised feature.

Membranous glomerulonephropathy (Option D) is incorrect. Membranousglomerulonephropathy is not a recognised complication. Acetazolamide has been exploredas a means of preventing contrast-induced nephropathy, although there is insufficientevidence of efficacy.

Acute interstitial nephritisA

HyperkalaemiaB

Macrocytic hypochromic anaemiaC

Membranous glomerulonephropathyD

Metabolic alkalosisE

Page 309: Back to Filters (/Secure/TestMe/Filter ... - 1 File Download

8/13/2016 MyPastest

https://mypastest.pastest.com/Secure/TestMe/Browser/429893#Top 2/2

46605

Metabolic alkalosis (Option E) is incorrect. Metabolic acidosis is a characteristic feature.

Next Question

Previous Question Tag Question Feedback End Review

Difficulty: Average

Peer Responses

Session Progress

0Responses Correct:

179Responses Incorrect:

179Responses Total:

0%Responses - % Correct:

Blog (https://www.pastest.com/blog) About Pastest (https://www.pastest.com/about-us)Contact Us (https://www.pastest.com/contact-us) Help (https://www.pastest.com/help)

© Pastest 2016

Page 310: Back to Filters (/Secure/TestMe/Filter ... - 1 File Download

8/13/2016 MyPastest

https://mypastest.pastest.com/Secure/TestMe/Browser/429893#Top 1/2

Back to Filters (/Secure/TestMe/Filter/429893/QA)

Question 155 of 179

A 19-year-old woman is admitted via the Emergency Department in the early hours of themorning. She presented due to nausea, vomiting and dizziness. Her friend states that she hadtaken some ecstasy tablets (MDMA) while out clubbing a few hours earlier.

Which of the following provides the best explanation for the mechanism of toxicity ofecstasy (MDMA)?

Explanation

The answer is Inappropriate ADH release may cause hyponatraemia –

Ecstasy is an amphetamine derivative (3,4-methylenedioxymethamphetamine (MDMA)).Its principal effects are stimulation of release of serotonin within the central nervoussystem and activation of the sympathetic nervous system. Onset is usually within 1 hourand effects typically last 4–6 hours, although toxic effects may persist for up to severaldays after ingestion. SIADH and hyponatraemia are well characterised features, andhyponatraemia may be made worse by excessive sweating and fluid intake. Adverseeffects include arrhythmia (the main mechanism of MDMA fatalaties), seizures,hypertension and hyponatraemia. Treatment includes benzodiazepines to reduceagitation and minimise seizure risk, active cooling and supportive care.

Alpha-adrenoceptor stimulation within the central nervous system (Option A) is incorrect.This is not a predominant feature of MDMA; the clinical features are more typically related toserotonin and norepinephrine activity.

Alpha-adrenoceptor stimulation within the central nervous systemA

Cerebellar hypoxia causing nystagmusB

Inappropriate ADH release may cause hyponatraemiaC

Stimulation of the parasympathetic nervous systemD

Suppression of serotonergic neurotransmitter activityE

Page 311: Back to Filters (/Secure/TestMe/Filter ... - 1 File Download

8/13/2016 MyPastest

https://mypastest.pastest.com/Secure/TestMe/Browser/429893#Top 2/2

45793

Cerebellar hypoxia causing nystagmus (Option B) is incorrect. This is not typical in MDMAtoxicity. Cerebellar toxicity is a characteristic adverse effect of methoxetamine toxicity, alsoseen after ingestion of ethanol, carbamazepine or phenytoin.

Stimulation of the parasympathetic nervous system (Option D) is incorrect. MDMApredominantly activates the sympathetic nervous system.

Suppression of serotonergic neurotransmitter activity (Option E) is incorrect. One of the keymechanisms of MDMA is activation of serotonergic neurotransmission within the centralnervous system.

Next Question

Previous Question Tag Question Feedback End Review

Difficulty: Average

Peer Responses

Session Progress

0Responses Correct:

179Responses Incorrect:

179Responses Total:

0%Responses - % Correct:

Blog (https://www.pastest.com/blog) About Pastest (https://www.pastest.com/about-us)Contact Us (https://www.pastest.com/contact-us) Help (https://www.pastest.com/help)

© Pastest 2016

Page 312: Back to Filters (/Secure/TestMe/Filter ... - 1 File Download

8/13/2016 MyPastest

https://mypastest.pastest.com/Secure/TestMe/Browser/429893#Top 1/2

Back to Filters (/Secure/TestMe/Filter/429893/QA)

Question 156 of 179

A 67-year-old man presents with weakness and muscle aches. He has a history ofhypertension and dyslipidaemia and is managed with ramipril and simvastatin. He also haschronic obstructive pulmonary disease and is treated with a high-dose seretide inhaler. Youunderstand he was started by his GP on antibiotics a few days earlier for a lower respiratorytract infection.

Investigations:

Hb 12.1 g/dl

White cell count 9.4 × 10 /l

Platelets 272 × 10 /l

Na 141 mmol/l

K 5.9 mmol/l

Creatinine 190 μmol/l

CK 890 U/l (24–195)

Which one of the following is the antibiotic he is most likely to have been prescribed?

Explanation

Drug interactions

Simvastatin is metabolised by CYP3A4

9

9

+

+

DoxycyclineA

CiprofloxacinB

AmoxicillinC

Co-amoxiclavD

ClarithromycinE

Page 313: Back to Filters (/Secure/TestMe/Filter ... - 1 File Download

8/13/2016 MyPastest

https://mypastest.pastest.com/Secure/TestMe/Browser/429893#Top 2/2

21362

Antibiotics in the macrolide class, including clarithromycin and azithromycin, are potent

inhibitors of CYP3

This leads to simvastatin accumulation and possible rhabdomyolysis

The picture seen here, with raised potassium, creatinine and creatine kinase, fits with

that picture

Because of this interaction, caution is recommended when considering macrolides in

conjunction with simvastatin at higher doses, and another antibiotic should be used if

possible

Next Question

Previous Question Tag Question Feedback End Review

Difficulty: Average

Peer Responses

Session Progress

0Responses Correct:

179Responses Incorrect:

179Responses Total:

0%Responses - % Correct:

Blog (https://www.pastest.com/blog) About Pastest (https://www.pastest.com/about-us)Contact Us (https://www.pastest.com/contact-us) Help (https://www.pastest.com/help)

© Pastest 2016

Page 314: Back to Filters (/Secure/TestMe/Filter ... - 1 File Download

8/13/2016 MyPastest

https://mypastest.pastest.com/Secure/TestMe/Browser/429893#Top 1/2

Back to Filters (/Secure/TestMe/Filter/429893/QA)

Question 157 of 179

A 39-year-old woman with a history of bipolar disorder visits her GP for review. She iscurrently treated with lithium therapy. The GP has been monitoring her blood pressure for thelast few months. It is 155/105 mmHg in the clinic and the GP contacts you for adviceregarding the choice of antihypertensive agents available.

Which one of the following statements is most accurate concerning the pharmacologicalinteraction between antihypertensive agents and lithium?

Explanation

The answer is Acetazolamide decreases lithium concentrations –

Osmotic diuretics and carbonic anhydrase inhibitors such as acetazolamide lead toincreased renal excretion of sodium and lithium; lithium concentrations would fall.

Angiotensin converting enzyme (ACE) inhibitors lead to decreased lithium concentration(Option B) is incorrect. ACE inhibitors lead to increased lithium concentration because ofdecreased excretion.

Bendroflumethiazide decreases lithium concentration (Option C) is incorrect. Use of thiazidediuretics may result in paradoxical increased proximal tubular sodium and water retention,and increased lithium reabsorption. Thiazides may cause lithium intoxication.

Calcium channel blockers lessen the risk of lithium neurotoxicity (Option D) is incorrect.Calcium channel blockers combined with lithium may cause a syndrome of ataxia, confusionand sleepiness, which is reversible on stopping the drug.

Acetazolamide decreases lithium concentrationsA

Angiotensin converting enzyme (ACE) inhibitors lead to decreased lithiumconcentration

B

Bendroflumethiazide decreases lithium concentrationC

Calcium channel blockers lessen the risk of lithium neurotoxicityD

Methyldopa leads to decreased neurotoxicityE

Page 315: Back to Filters (/Secure/TestMe/Filter ... - 1 File Download

8/13/2016 MyPastest

https://mypastest.pastest.com/Secure/TestMe/Browser/429893#Top 2/2

46955

Methyldopa leads to decreased neurotoxicity (Option E) is incorrect. Methyldopa also leadsto increased risk of neurotoxicity.

Next Question

Previous Question Tag Question Feedback End Review

Difficulty: Average

Peer Responses

Session Progress

0Responses Correct:

179Responses Incorrect:

179Responses Total:

0%Responses - % Correct:

Blog (https://www.pastest.com/blog) About Pastest (https://www.pastest.com/about-us)Contact Us (https://www.pastest.com/contact-us) Help (https://www.pastest.com/help)

© Pastest 2016

Page 316: Back to Filters (/Secure/TestMe/Filter ... - 1 File Download

8/13/2016 MyPastest

https://mypastest.pastest.com/Secure/TestMe/Browser/429893#Top 1/2

Back to Filters (/Secure/TestMe/Filter/429893/QA)

Question 158 of 179

46849

A 56-year-old man is undergoing acute intervention in the cardiac catheterisation laboratoryfor unstable angina. The cardiologist decides to employ a glycoprotein IIb/IIIa inhibitor.

Which of the following compounds would be most suitable?

Explanation

The answer is Abciximab –

Tirofiban, eptifibatide and abciximab are all glycoprotein IIb/IIIa inhibitors, but onlyeptifibatide and abciximab are licensed as adjunctive therapy in acute coronaryintervention. Glycoprotein IIb/IIIa inhibitors act by blocking the final common pathwayinvolved in platelet aggregation; the main side-effects include bleeding andthrombocytopaenia. Thrombocytopaenia may be particularly severe after abciximab,which contains murine antibody components.

Clopidogrel (Option B) is incorrect. Clopidogrel is an adjunctive therapy in conjunction withheparin and aspirin to prevent early myocardial infarction in patients with unstable angina.

Eptifibatide plus abciximab (Option C) is incorrect. Eptifibatide or abciximab may be usedbut not in combination.

Ticlopidine (Option D) is incorrect. Ticlopidine is licenced on a named patient basis for use ina coronary care unit, and is an adjunctive therapy in conjunction with heparin and aspirin toprevent early myocardial infarction in patients with unstable angina.

Tirofiban (Option E) is incorrect. Tirofiban is used in patients with non-ST elevationmyocardial infarction, but is not licenced for treatment in acute coronary intervention.

AbciximabA

ClopidogrelB

Eptifibatide plus abciximabC

TiclopidineD

TirofibanE

Page 317: Back to Filters (/Secure/TestMe/Filter ... - 1 File Download

8/13/2016 MyPastest

https://mypastest.pastest.com/Secure/TestMe/Browser/429893#Top 2/2

46849

Next Question

Previous Question Tag Question Feedback End Review

Difficulty: Average

Peer Responses

Session Progress

0Responses Correct:

179Responses Incorrect:

179Responses Total:

0%Responses - % Correct:

Blog (https://www.pastest.com/blog) About Pastest (https://www.pastest.com/about-us)Contact Us (https://www.pastest.com/contact-us) Help (https://www.pastest.com/help)

© Pastest 2016

Page 318: Back to Filters (/Secure/TestMe/Filter ... - 1 File Download

8/13/2016 MyPastest

https://mypastest.pastest.com/Secure/TestMe/Browser/429893#Top 1/2

Back to Filters (/Secure/TestMe/Filter/429893/QA)

Question 159 of 179

A 50-year-old truck driver with type-2 diabetes for the past 5 years, BMI 35 kg/m and HbAof 62 mmol/mol (7.8%), is taking 160 mg gliclazide twice a day and metformin 2 g daily.

Which drug would be the most suitable additional therapy in this patient?

Explanation

The answer is Sitagliptin –

There are three key options here: commence insulin, a dipeptidyl peptidase IV (DPPIV)inhibitor, or glitazone such as pioglitazone. Addition of insulin treatment wouldnecessitate rescinding his HGV licence. The HbA1c is close to target, therefore a glitazoneor DPPIV inhibitor would be preferred. Sitagliptin would be the best option because it isassociated with weight maintenance or slight loss, whereas pioglitazone would promoteweight gain. Sitagliptin belongs to the DPPIV inhibitor class, leading to increased plasmalevels of the naturally occurring incretin hormone, GLP-1, by preventing its breakdown.

Acarbose (Option A) is incorrect. Acarbose reduces intestinal absorption of starch, but maybe associated with significant GI upset including diarrhoea and flatulence.

A mixture of short- and medium-acting insulins (Option B) is incorrect. Addition of insulinwould be an effective means of achieving glycaemic control, but this would mean the patientwould lose his HGV licence and livelihood.

Pioglitazone (Option C) is incorrect. Pioglitazone belongs to the thiazolidinedione group ofdrugs, which decrease insulin resistance by decreasing leptin expression and increasingp85α-P1–3K gene expression (a gene that allows insulin to work). The reason that this is a lesspreferred option to sitagliptin is because pioglitazone may cause weight gain. Pioglitazone

21c

AcarboseA

A mixture of short- and medium-acting insulinsB

PioglitazoneC

RepaglinideD

SitagliptinE

Page 319: Back to Filters (/Secure/TestMe/Filter ... - 1 File Download

8/13/2016 MyPastest

https://mypastest.pastest.com/Secure/TestMe/Browser/429893#Top 2/2

46754

leads to weight gain, increased risk of fluid retention, and is possibly associated with bladdercarcinoma, although it is effective at controlling blood glucose.

Repaglinide (Option D) is incorrect. Although repaglinide is shorter acting compared with atraditional SU, it still has at least some potential to cause hypoglycaemia, and is unlikely tooffer significant advantages over and above the existing regimen.

Next Question

Previous Question Tag Question Feedback End Review

Difficulty: Average

Peer Responses

Session Progress

0Responses Correct:

179Responses Incorrect:

179Responses Total:

0%Responses - % Correct:

Blog (https://www.pastest.com/blog) About Pastest (https://www.pastest.com/about-us)Contact Us (https://www.pastest.com/contact-us) Help (https://www.pastest.com/help)

© Pastest 2016

Page 320: Back to Filters (/Secure/TestMe/Filter ... - 1 File Download

8/13/2016 MyPastest

https://mypastest.pastest.com/Secure/TestMe/Browser/429893#Top 1/2

Back to Filters (/Secure/TestMe/Filter/429893/QA)

Question 160 of 179

Which one of the following antihypertensive agents exerts its pharmacological effectspredominantly through blockade of peripheral α1-adrenoceptors?

Explanation

The answer is Doxazosin –

Prazosin, terazosin and doxazosin are peripheral postsynaptic α1-adrenergic blockers thatact on veins and arterioles.

Clonidine (Option A) is incorrect. Clonidine is a centrally acting agent that reducessympathetic outflow.

Losartan (Option C) is incorrect. Losartan blocks angiotensin II receptors and thereforeinterferes with the renin–angiotensin system, perhaps more completely than the angiotensinconverting enzyme inhibitors. Angiotensin II receptor blockers are less likely to provokecough than ace inhibitors, presumably because they do not block the degradation ofbradykinin.

Methyldopa (Option D) is incorrect. Methyldopa, clonidine, guanabenz and guanfacinereduce sympathetic nervous activity by stimulating presynaptic α2-adrenergic receptors inthe brainstem. Methyldopa acts primarily on the brainstem vasomotor centre causing therelease of α-methylnoradrenaline, which enhances central nervous system α2-adrenoceptoractivity (this inhibits sympathetic outflow). Methyldopa reduces peripheral vascularresistance. It causes less peripheral adrenergic blockade and therefore less posturalhypotension.

ClonidineA

DoxazosinB

LosartanC

MethyldopaD

MinoxidilE

Page 321: Back to Filters (/Secure/TestMe/Filter ... - 1 File Download

8/13/2016 MyPastest

https://mypastest.pastest.com/Secure/TestMe/Browser/429893#Top 2/2

46726

Minoxidil (Option E) is incorrect. Minoxidil causes direct relaxation of vascular smooth muscleand affects mainly arterial resistance as opposed to venous capacitance vessels, asevidenced by lack of postural effects. Minoxidil may produce significant hypertrichosis andfluid retention and may provoke congestive heart failure.

Next Question

Previous Question Tag Question Feedback End Review

Difficulty: Average

Peer Responses

Session Progress

0Responses Correct:

179Responses Incorrect:

179Responses Total:

0%Responses - % Correct:

Blog (https://www.pastest.com/blog) About Pastest (https://www.pastest.com/about-us)Contact Us (https://www.pastest.com/contact-us) Help (https://www.pastest.com/help)

© Pastest 2016

Page 322: Back to Filters (/Secure/TestMe/Filter ... - 1 File Download

8/13/2016 MyPastest

https://mypastest.pastest.com/Secure/TestMe/Browser/429893#Top 1/2

Back to Filters (/Secure/TestMe/Filter/429893/QA)

Question 161 of 179

46905

A 40-year-old man with polycystic kidney disease is under follow-up by the Renal Clinic forprogressively declining renal function. His general practitioner has asked for some adviceregarding drug prescribing, given his renal failure.

Which of the following is the single most important factor that determines the effect of renalfunction on drug handling?

Explanation

The answer is Glomerular filtration rate –

In renal failure, there is alteration of drug absorption, distribution, protein binding,metabolism and pharmacodynamic effects. Renal elimination is the most fundamentallyaltered pharmacokinetic factor, and the extent of drug clearance normally depends moreon glomerular filtration rate (GFR) than other aspects of renal elimination; tubularsecretion or reabsorption are more difficult to measure in clinical practice.

Age (Option A) is incorrect. Age is one factor that contributes to GFR.

Blood urea levels (Option B) is incorrect. These provide a less reliable measure of drugelimination than GFR.

Body weight (Option C) is incorrect. Body weight provides a poor estimate of renal functionand GFR.

Molecular weight of the drug (Option E) is incorrect. This will have some impact on whetherthe drug is subject to renal or hepatic elimination, but does not provide a measure of theeffectiveness of elimination.

AgeA

Blood urea levelsB

Body weightC

Glomerular filtration rateD

Molecular weight of the drugE

Page 323: Back to Filters (/Secure/TestMe/Filter ... - 1 File Download

8/13/2016 MyPastest

https://mypastest.pastest.com/Secure/TestMe/Browser/429893#Top 2/2

46905

Next Question

Previous Question Tag Question Feedback End Review

Difficulty: Average

Peer Responses

Session Progress

0Responses Correct:

179Responses Incorrect:

179Responses Total:

0%Responses - % Correct:

Blog (https://www.pastest.com/blog) About Pastest (https://www.pastest.com/about-us)Contact Us (https://www.pastest.com/contact-us) Help (https://www.pastest.com/help)

© Pastest 2016

Page 324: Back to Filters (/Secure/TestMe/Filter ... - 1 File Download

8/13/2016 MyPastest

https://mypastest.pastest.com/Secure/TestMe/Browser/429893#Top 1/2

Back to Filters (/Secure/TestMe/Filter/429893/QA)

Question 162 of 179

46906

A 68-year-old man with known ischaemic heart disease, on regular atenolol and aspirin aswell as glyceryl trinitrate therapy as required, is admitted to hospital with unstable angina.Regular oral clopidogrel and nifedipine are added, and he is placed on regular subcutaneousenoxaparin and an intravenous infusion of glyceryl trinitrate. Three days later, he is noted tobe jaundiced.

What is the most likely explanation?

Explanation

The answer is Nifedipine therapy –

Nifedipine causes jaundice due to cholestasis or, less commonly, drug-induced hepatitis.

Clopidogrel therapy (Option A) is incorrect. Clopidogrel is not expected to cause liverdisease.

Enoxaparin therapy (Option B) is incorrect. Enoxaparin may cause bruising and increasedbleeding risk, but is not normally associated with jaundice.

He has co-existent viral hepatitis (Option C) is incorrect. Viral hepatitis is possible, but fails totake account of all the information provided.

High-dose glyceryl trinitrate therapy (Option D) is incorrect. Glyceryl trinitrate does notcause jaundice.

Clopidogrel therapyA

Enoxaparin therapyB

He has co-existent viral hepatitisC

High-dose glyceryl trinitrate therapyD

Nifedipine therapyE

Next Question

Page 325: Back to Filters (/Secure/TestMe/Filter ... - 1 File Download

8/13/2016 MyPastest

https://mypastest.pastest.com/Secure/TestMe/Browser/429893#Top 2/2

Previous Question Tag Question Feedback End Review

Difficulty: Average

Peer Responses

Session Progress

0Responses Correct:

179Responses Incorrect:

179Responses Total:

0%Responses - % Correct:

Blog (https://www.pastest.com/blog) About Pastest (https://www.pastest.com/about-us)Contact Us (https://www.pastest.com/contact-us) Help (https://www.pastest.com/help)

© Pastest 2016

Page 326: Back to Filters (/Secure/TestMe/Filter ... - 1 File Download

8/13/2016 MyPastest

https://mypastest.pastest.com/Secure/TestMe/Browser/429893#Top 1/2

Back to Filters (/Secure/TestMe/Filter/429893/QA)

Question 163 of 179

46755

A 30-year-old sales executive is admitted for an operative procedure requiring generalanaesthesia. He regularly drinks red wine, equivalent to 60 units of alcohol per week. You areconsidering treatments to prevent the occurrence of alcohol withdrawal symptomspostoperatively.

Which drug would be most appropriate in preventing and alleviating this problem?

Explanation

The answer is Chlordiazepoxide –

Diazepam and chlordiazepoxide are routinely used to prevent seizures in recentlyabstinent chronic alcoholics and, perhaps less importantly, acute withdrawal symptoms.

Chlorpromazine (Option B) is incorrect. Phenothiazines (chlorpromazine) may be a helpfuladjunctive therapy in some patients, but have been associated with significant respiratorydepression.

Clomethiazole (Option C) is incorrect. Clomethiazole readily causes addiction and has beenassociated with respiratory depression; it is rarely used now.

Lorazepam (Option D) is incorrect. Lorazepam may be effective when urgent, intravenousadministration is required. However, it is comparatively short acting and less preferred as oraltherapy compared to chlordiazepoxide or diazepam.

Temazepam (Option E) is incorrect. Temazepam may be effective, but it has a shorterduration of action so that repeated dose administration is needed.

ChlordiazepoxideA

ChlorpromazineB

ClomethiazoleC

LorazepamD

TemazepamE

Page 327: Back to Filters (/Secure/TestMe/Filter ... - 1 File Download

8/13/2016 MyPastest

https://mypastest.pastest.com/Secure/TestMe/Browser/429893#Top 2/2

Next Question

Previous Question Tag Question Feedback End Review

Difficulty: Average

Peer Responses

Session Progress

0Responses Correct:

179Responses Incorrect:

179Responses Total:

0%Responses - % Correct:

Blog (https://www.pastest.com/blog) About Pastest (https://www.pastest.com/about-us)Contact Us (https://www.pastest.com/contact-us) Help (https://www.pastest.com/help)

© Pastest 2016

Page 328: Back to Filters (/Secure/TestMe/Filter ... - 1 File Download

8/13/2016 MyPastest

https://mypastest.pastest.com/Secure/TestMe/Browser/429893#Top 1/2

Back to Filters (/Secure/TestMe/Filter/429893/QA)

Question 164 of 179

A 73-year-old man presents to the Emergency Department with a 2-week history of diarrhoeaand reduced oral fluid intake. He has been taking digoxin for atrial fibrillation for severalyears, and is found to have a serum digoxin concentration more than double the localtherapeutic reference range.

Which of the following statements best describes the management of digoxin poisoning?

Explanation

The answer is Digoxin-specific antibodies may cause serum sickness –

Digoxin-specific antibodies are indicated for severe digoxin toxicity, namely where thereis severe hyperkalaemia that has failed to respond to fluid resuscitation andadministration of insulin and dextrose. They are also indicated in patients with significantarrhythmias and hypotension, most commonly bradycardia that fails to respond toatropine or other measures. Digoxin-specific antibodies offer temporary reversal oftoxicity, but ultimately patients require renal excretion of digoxin and, if there is poorurine output and renal impairment, then haemodialysis may be required. Onceadministered, it is not possible to reliably measure circulating digoxin concentrations.Serum sickness is an uncommon but recognised and potentially life-threatening reaction.

Bradycardia with hypotension is an indication for initial use of digoxin-specific antibodies(Option A) is incorrect. The initial approach would be to administer fluids and correctelectrolyte disturbances.

Bradycardia with hypotension is an indication for initial use of digoxin-specificantibodies

A

Calcium gluconate should be avoided in patients with hyperkalaemiaB

Digoxin-specific antibodies may cause serum sicknessC

Furosemide is often helpful in minimising toxicityD

Oral activated charcoal should be avoidedE

Page 329: Back to Filters (/Secure/TestMe/Filter ... - 1 File Download

8/13/2016 MyPastest

https://mypastest.pastest.com/Secure/TestMe/Browser/429893#Top 2/2

45808

Calcium gluconate should be avoided in patients with hyperkalaemia (Option B) is incorrect.Correction of calcium or magnesium may be required to minimise risk of arrhythmia.

Furosemide is often helpful in minimising toxicity (Option D) is incorrect. Furosemide shouldnot routinely be administered.

Oral activated charcoal should be avoided (Option E) is incorrect. Charcoal may be helpful ifgiven sufficiently early after acute digoxin ingestion. Repeated oral charcoal administrationmay enhance digoxin clearance, although patients with severe toxicity are often vomiting, socharcoal may not be administered.

Next Question

Previous Question Tag Question Feedback End Review

Difficulty: Average

Peer Responses

Session Progress

0Responses Correct:

179Responses Incorrect:

179Responses Total:

0%Responses - % Correct:

Blog (https://www.pastest.com/blog) About Pastest (https://www.pastest.com/about-us)Contact Us (https://www.pastest.com/contact-us) Help (https://www.pastest.com/help)

© Pastest 2016

Page 330: Back to Filters (/Secure/TestMe/Filter ... - 1 File Download

8/13/2016 MyPastest

https://mypastest.pastest.com/Secure/TestMe/Browser/429893#Top 1/2

Back to Filters (/Secure/TestMe/Filter/429893/QA)

Question 165 of 179

A 72-year-old woman with metastatic carcinoma of the breast is admitted to the localhospice for management of her pain. The palliative care team believes she only has a matterof a few weeks left. She is currently managed with 120 mg bd of Morphine slow release(MST), 10–20 mg of oral morphine prn and regular paracetamol. She complains of worseningpain and yet is worried about the morphine making her drowsy and confused.

Investigations:

Hb 10.9 g/dl

White cell count 5.9 × 10 /l

Platelets 191 × 10 /l

Na 139 mmol/l

K 5.0 mmol/l

Creatinine 146 μmol/l

Which one of the following would be the most appropriate plan to manage her pain?

Explanation

Pain relief

9

9

+

+

Reduce her MST and add naproxenA

Keep her MST stable and stop her oral morphineB

Stop her MST and start naproxenC

Keep her MST stable, continue to allow PRN oral morphine and add naproxenD

Transfer her pain relief to fentanyl patchesE

Page 331: Back to Filters (/Secure/TestMe/Filter ... - 1 File Download

8/13/2016 MyPastest

https://mypastest.pastest.com/Secure/TestMe/Browser/429893#Top 2/2

21259

Reducing this patient’s morphine sulphate (MST) is likely to lead to a significant

worsening in her level of pain and agitation

However, a significant degree of confusion and drowsiness is likely to result from the

intermittent dosing of oral morphine

Management of choice is to mix multiple modalities for pain relief, and as such addition

of naproxen is the most appropriate next step, she may then be able to reduce her

intermittent oral morphine

Transition to fentanyl patches, without establishing a stable dose of morphine, is

absolutely not recommended and is associated with a significant risk of over or under

dosing

Next Question

Previous Question Tag Question Feedback End Review

Difficulty: Average

Peer Responses

Session Progress

0Responses Correct:

179Responses Incorrect:

179Responses Total:

0%Responses - % Correct:

Blog (https://www.pastest.com/blog) About Pastest (https://www.pastest.com/about-us)Contact Us (https://www.pastest.com/contact-us) Help (https://www.pastest.com/help)

© Pastest 2016

Page 332: Back to Filters (/Secure/TestMe/Filter ... - 1 File Download

8/13/2016 MyPastest

https://mypastest.pastest.com/Secure/TestMe/Browser/429893#Top 1/2

Back to Filters (/Secure/TestMe/Filter/429893/QA)

Question 166 of 179

46799

A 25-year-old patient presents to the Emergency Department suffering from bloodydiarrhoea and pain in his right abdomen for the past 2 weeks. He just returned from a holidayin the tropics. Entamoeba histolytica is detected in a stool sample.

Which of the following medications would be most appropriate therapy?

Explanation

The answer is Metronidazole –

Metronidazole is active against a number of protozoa including Entamoeba histolytica aswell as anaerobic bacteria. Entamoeba histolytica is by far the most common parasiticcause of dysenteric illness worldwide, and associated with around 50 million cases ofinvasive colitis each year, primarily in developing countries. Within ‘Westernised’ nations,those at highest risk of infection include institutionalised people, recent immigrants andreturning travellers. Infection may be more severe in the context of malnutrition,malignancy, pregnancy and childhood.

Erythromycin (Option A) is incorrect. Erythromycin is inactive against E. histolytica.

Mebendazole (Option B) is incorrect. Mebednazole is active against helminithic infections,including ascariasis, pinworm and hookworm.

Mefloquine (Option C) is incorrect. Mefloquine is active against certain malarial infections.

Vancomycin (Option E) is incorrect. Vancomycin is active against certain Gram-positiveinfections, but is inactive against E. histolytica.

ErythromycinA

MebendazoleB

MefloquineC

MetronidazoleD

VancomycinE

Page 333: Back to Filters (/Secure/TestMe/Filter ... - 1 File Download

8/13/2016 MyPastest

https://mypastest.pastest.com/Secure/TestMe/Browser/429893#Top 2/2

Next Question

Previous Question Tag Question Feedback End Review

Difficulty: Average

Peer Responses

Session Progress

0Responses Correct:

179Responses Incorrect:

179Responses Total:

0%Responses - % Correct:

Blog (https://www.pastest.com/blog) About Pastest (https://www.pastest.com/about-us)Contact Us (https://www.pastest.com/contact-us) Help (https://www.pastest.com/help)

© Pastest 2016

Page 334: Back to Filters (/Secure/TestMe/Filter ... - 1 File Download

8/13/2016 MyPastest

https://mypastest.pastest.com/Secure/TestMe/Browser/429893#Top 1/2

Back to Filters (/Secure/TestMe/Filter/429893/QA)

Question 167 of 179

46823

A diabetic patient is currently being treated with metformin and sulfonylurea. However, hisHbA1C is still suboptimally controlled. You decide to add pioglitazone therapy.

Which of the following best describes the pharmacological action of this drug?

Explanation

The answer is Increases insulin sensitivity –

Thiazolidinediones bind to specific receptors in the nucleus, ‘peroxisome proliferator-activating receptor-γ’ (PPAR-γ). They do not cause hypoglycaemia when used alone, butcan exaggerate the hypoglycaemic effects of insulin or sulfonylureas. Thiazolidinedioneslower glucose by 2–3 mmol/l and HbA1c by 1%, somewhat less than the sulfonylureas.Adverse effects include weight gain, fluid retention and decreased bone mineral density.

Increases insulin secretion (Option A) is incorrect. Thiazolidinediones do not increase insulinsecretion.

Increases glucose elimination (Option C) is incorrect. SGLT2 inhibitors, so-called gliflozins,decrease the renal threshold for glucose elimination.

Inhibits gluconeogenesis (Option D) is incorrect. Thiazolidinediones do not altergluconeogenesis.

Inhibits glucose absorption (Option E) is incorrect. Acarbose delays gastrointestinal glucoseabsorption and reduces post-prandial glucose concentrations.

Increases insulin secretionA

Increases insulin sensitivityB

Increases glucose eliminationC

Inhibits gluconeogenesisD

Inhibits glucose absorptionE

Page 335: Back to Filters (/Secure/TestMe/Filter ... - 1 File Download

8/13/2016 MyPastest

https://mypastest.pastest.com/Secure/TestMe/Browser/429893#Top 2/2

Next Question

Previous Question Tag Question Feedback End Review

Difficulty: Average

Peer Responses

Session Progress

0Responses Correct:

179Responses Incorrect:

179Responses Total:

0%Responses - % Correct:

Blog (https://www.pastest.com/blog) About Pastest (https://www.pastest.com/about-us)Contact Us (https://www.pastest.com/contact-us) Help (https://www.pastest.com/help)

© Pastest 2016

Page 336: Back to Filters (/Secure/TestMe/Filter ... - 1 File Download

8/13/2016 MyPastest

https://mypastest.pastest.com/Secure/TestMe/Browser/429893#Top 1/2

Back to Filters (/Secure/TestMe/Filter/429893/QA)

Question 168 of 179

A 48-year-old man has cirrhosis of the liver secondary to former alcohol abuse.

Which of the following drugs undergoes high first-pass metabolism and should be used withcaution in patients with mild to moderate liver disease?

Explanation

The answer is Verapamil –

Good examples of drugs that undergo high rates of first-pass metabolism (ie have highliver extraction) include propranolol, verapamil and morphine. All should be used withcaution in liver disease, as poor hepatic function may lead to their accumulation becauseof increased bioavailability. The handling of many drugs may be affected by liver diseaseowing to reduced hepatic conjugation, reduced activity of the cytochrome P450 systemand changes in plasma protein binding. It is important to check the liver disease sectionof the British National Formulary before considering therapies in patients with knownchronic liver disease.

Amoxicillin (Option A) is incorrect. Amoxicillin is excreted largely unchanged in the urine.

Atenolol (Option B) is incorrect. Atenolol is subject to renal elimination, and the dose mustbe adjusted in patients with renal impairment.

Ciprofloxacin (Option C) is incorrect. Ciprofloxacin is subject to renal elimination;ciprofloxacin exerts an inhibitory effect on P450 enzyme activity.

Omeprazole (Option D) is incorrect. Omeprazole is completely metabolised by the P450system, but accumulation is less likely to cause toxicity than verapamil, hence it is not the

AmoxicillinA

AtenololB

CiprofloxacinC

OmeprazoleD

VerapamilE

Page 337: Back to Filters (/Secure/TestMe/Filter ... - 1 File Download

8/13/2016 MyPastest

https://mypastest.pastest.com/Secure/TestMe/Browser/429893#Top 2/2

46850

preferred answer here.

Next Question

Previous Question Tag Question Feedback End Review

Difficulty: Average

Peer Responses

Session Progress

0Responses Correct:

179Responses Incorrect:

179Responses Total:

0%Responses - % Correct:

Blog (https://www.pastest.com/blog) About Pastest (https://www.pastest.com/about-us)Contact Us (https://www.pastest.com/contact-us) Help (https://www.pastest.com/help)

© Pastest 2016

Page 338: Back to Filters (/Secure/TestMe/Filter ... - 1 File Download

8/13/2016 MyPastest

https://mypastest.pastest.com/Secure/TestMe/Browser/429893#Top 1/2

Back to Filters (/Secure/TestMe/Filter/429893/QA)

Question 169 of 179

An 81-year-old woman with advanced chronic obstructive pulmonary disease is admitted inan unconscious state. She appears to have an acute lower respiratory tract infection. Theanaesthesiologist feels that she is not a candidate for Intensive Therapy Unit admission due topoor premorbid functional state. After discussing care with the family, you decide toprescribe doxapram therapy.

Which of the following pharmacological properties is most closely associated withdoxapram?

Explanation

The answer is It is contraindicated in hyperthyroidism –

Doxapram is a centrally acting respiratory stimulant, which is generally used in patientswith severe respiratory disease where it may improve respiratory function sufficiently forsurvival. It has a limited clinical role, and may be considered in patients who are deemedunsuitable for invasive ventilatory support. The usual dosing regimen is 1–4 mg/min givenas an intravenous infusion.

It causes hypotension (Option A) is incorrect.Side-effects of therapy include hypertension,exacerbation of apparent dyspnoea, agitation, confusion, sweating, cough, headache,dizziness, nausea, vomiting and urinary retention.

It is safe in phaeochromocytoma (Option C) is incorrect. Doxapram is contraindicated inpatients with heart disease, epilepsy, cerebral oedema, stroke, status asthmaticus,hypertension, hyperthyroidism and phaeochromocytoma.

It causes hypotensionA

It is contraindicated in hyperthyroidismB

It is safe in phaeochromocytomaC

It is compatible in infusion with aminophyllineD

It has a half-life of 12 hE

Page 339: Back to Filters (/Secure/TestMe/Filter ... - 1 File Download

8/13/2016 MyPastest

https://mypastest.pastest.com/Secure/TestMe/Browser/429893#Top 2/2

46893

It is compatible in infusion with aminophylline (Option D) is incorrect. Doxapram solution haspH 3.5–5.0. Infusion is incompatible with alkaline solutions including sodium bicarbonate,furosemide, or aminophylline due to a risk of gas formation or precipitation.

It has a half-life of 12 h (Option E) is incorrect. The half-life is short, and the duration ofaction is between 5 and 10 minutes after intravenous administration.

Next Question

Previous Question Tag Question Feedback End Review

Difficulty: Average

Peer Responses

Session Progress

0Responses Correct:

179Responses Incorrect:

179Responses Total:

0%Responses - % Correct:

Blog (https://www.pastest.com/blog) About Pastest (https://www.pastest.com/about-us)Contact Us (https://www.pastest.com/contact-us) Help (https://www.pastest.com/help)

© Pastest 2016

Page 340: Back to Filters (/Secure/TestMe/Filter ... - 1 File Download

8/13/2016 MyPastest

https://mypastest.pastest.com/Secure/TestMe/Browser/429893#Top 1/2

Back to Filters (/Secure/TestMe/Filter/429893/QA)

Question 170 of 179

You are asked by the FY1 doctor for advice on therapeutic drug monitoring for a patientadmitted to hospital for investigation of abdominal pain.

When would be the best to take a blood level of lithium to provide optimum monitoring?

Explanation

The answer is 12 hours after dose –

Therapeutic drug monitoring is appropriate for drugs with a narrow therapeutic indexand where plasma concentrations give a reliable indicator of likely toxicity. Lithium isused in the treatment and prophylaxis of mania, manic-depressive illness and recurrentdepression, and the ideal therapeutic range is 0.4–1.0 mmol/l on samples taken at 12 hafter the last dose. Toxic effects of lithium include tremor, ataxia and renal impairment,and are more likely to occur at concentrations above the therapeutic range, especially if>1.5 mmol/l.

2 hours after dose (Option A) is incorrect. This would not allow sufficient time for lithiumabsorption.

Any time (Option C) is incorrect. Standardisation of the interval between dose and bloodlevels is important.

Before dose (Option D) is incorrect. The trough levels are a less reliable measure of lithiumexposure and risk of toxicity (trough levels may be used for other drugs, eg vancomycin,because they may correspond with efficacy and risk of renal failure).

2 hours after doseA

12 hours after doseB

Any timeC

Before doseD

Immediately after doseE

Page 341: Back to Filters (/Secure/TestMe/Filter ... - 1 File Download

8/13/2016 MyPastest

https://mypastest.pastest.com/Secure/TestMe/Browser/429893#Top 2/2

46967

Immediately after dose (Option E) is incorrect. This would not allow sufficient time forlithium absorption.

Next Question

Previous Question Tag Question Feedback End Review

Difficulty: Average

Peer Responses

Session Progress

0Responses Correct:

179Responses Incorrect:

179Responses Total:

0%Responses - % Correct:

Blog (https://www.pastest.com/blog) About Pastest (https://www.pastest.com/about-us)Contact Us (https://www.pastest.com/contact-us) Help (https://www.pastest.com/help)

© Pastest 2016

Page 342: Back to Filters (/Secure/TestMe/Filter ... - 1 File Download

8/13/2016 MyPastest

https://mypastest.pastest.com/Secure/TestMe/Browser/429893#Top 1/2

Back to Filters (/Secure/TestMe/Filter/429893/QA)

Question 171 of 179

46844

You are reviewing a 54-year-old man with a phaeochromocytoma.

Which one of the following is most likely to exert α-adrenoreceptor antagonist propertiesthat might be effective in the presurgical management of his hypertension?

Explanation

The answer is Phenoxybenzamine –

Non-competitive antagonists at the α-adrenoceptor (e.g. phenoxybenzamine) cannot bedisplaced or have their effects diminished by an endogenous receptor ligand, namelynorepinephrine. Competitive antagonists bind to the site of action for the endogenousreceptor ligand and can be displaced (e.g. prazosin).

Atenolol (Option A) is incorrect. Atenolol is a β-1 selective β-receptor antagonist.

Nebivolol (Option B) is incorrect. Nebivolol is a racemic compound; one enantiomer is aselective β-1 receptor antagonist, the other enantiomer is a nitric oxide donor that causesvasodilatation.

Propranolol (Option D) is incorrect. Propranolol is a non-selective β-1/β-2 receptor antagonist.

Salbutamol (Option E) is incorrect. Salbutamol is β-2 receptor agonist.

AtenololA

NebivololB

PhenoxybenzamineC

PropranololD

SalbutamolE

Next Question

Page 343: Back to Filters (/Secure/TestMe/Filter ... - 1 File Download

8/13/2016 MyPastest

https://mypastest.pastest.com/Secure/TestMe/Browser/429893#Top 2/2

Previous Question Tag Question Feedback End Review

Difficulty: Average

Peer Responses

Session Progress

0Responses Correct:

179Responses Incorrect:

179Responses Total:

0%Responses - % Correct:

Blog (https://www.pastest.com/blog) About Pastest (https://www.pastest.com/about-us)Contact Us (https://www.pastest.com/contact-us) Help (https://www.pastest.com/help)

© Pastest 2016

Page 344: Back to Filters (/Secure/TestMe/Filter ... - 1 File Download

8/13/2016 MyPastest

https://mypastest.pastest.com/Secure/TestMe/Browser/429893#Top 1/2

Back to Filters (/Secure/TestMe/Filter/429893/QA)

Question 172 of 179

46947

You are asked for advice on how to manage postural hypotension symptoms in a 76-year-oldman who is taking simvastatin 20 mg, ramipril 2.5 mg, nifedipine 20 mg, bendroflumethiazide2.5 mg, doxazocin 8 mg and aspirin 75 mg?

Which drug is most likely responsible for postural hypotension symptoms?

Explanation

The answer is Doxazosin –

Doxazosin and α-blockers often cause postural hypotension, particularly in older adults.

Bendroflumethiazide (Option A) is incorrect. Bendroflumethazide and other diuretics maycause significant volume depletion, especially in chronic use. Symptomatic hypotension ismost likely to occur in patients who have been volume- and/or salt-depleted as a result ofco-existent dietary salt restriction, dialysis, diarrhoea, or vomiting.

Nifedipine (Option C) is incorrect. Nifedipine may cause hypotension, although overall thereis less risk of postural hypotension than ACE inhibitors and α-blockers, which is why option Bis the preferred answer.

Ramipril (Option D) is incorrect. Ramipril can cause symptomatic postural hypotension, aftereither the initial dose or a later dose when the dosage has been increased. This is less likelyto occur than after α-blocker treatment, particularly when low doses of ACE inhibitors areinvolved.

Simvastatin (Option E) is incorrect. Simvastatin is not expected to have any significant effecton blood pressure.

BendroflumethiazideA

DoxazosinB

NifedipineC

RamiprilD

SimvastatinE

Page 345: Back to Filters (/Secure/TestMe/Filter ... - 1 File Download

8/13/2016 MyPastest

https://mypastest.pastest.com/Secure/TestMe/Browser/429893#Top 2/2

46947

Next Question

Previous Question Tag Question Feedback End Review

Difficulty: Average

Peer Responses

Session Progress

0Responses Correct:

179Responses Incorrect:

179Responses Total:

0%Responses - % Correct:

Blog (https://www.pastest.com/blog) About Pastest (https://www.pastest.com/about-us)Contact Us (https://www.pastest.com/contact-us) Help (https://www.pastest.com/help)

© Pastest 2016

Page 346: Back to Filters (/Secure/TestMe/Filter ... - 1 File Download

8/13/2016 MyPastest

https://mypastest.pastest.com/Secure/TestMe/Browser/429893#Top 1/2

Back to Filters (/Secure/TestMe/Filter/429893/QA)

Question 173 of 179

You are reviewing your patients’ blood results before the ward round and notice that a 53-year-old woman has hyponatraemia. You note that she is receiving multiple medications thatare capable of provoking hyponatraemia.

Which one of the following drugs is most likely to cause hyponatraemia brought about bysyndrome of inappropriate antidiuretic hormone (SIADH)?

Explanation

The answer is Amitriptyline –

Hyponatraemia is commonly seen in medical practice and has many causes. SIADH ischaracterised by an increase in total body water with no oedema. There are a number ofdrug causes of SIADH. Some drugs that stimulate the release of ADH from the posteriorpituitary gland include nicotine, phenothiazines and tricyclics. Certain drugs increase therenal response to ADH, including desmopressin, oxytocin and prostaglandin synthesisinhibitors. Drugs that cause SIADH by means of mixed or uncertain mechanism includechlorpropamide, carbamazepine, cyclophosphamide, vincristine and omeprazole.

Hyponatraemia unrelated to SIADH includes circumstances related to decreased totalbody water with oedema (e.g. nephrotic syndrome, cirrhosis, congestive cardiac failure)and decreased body water without oedema (eg diuretics, Addison’s disease, diarrhoea).

Chlorpheniramine (Option B) is incorrect. This is not a recognised adverse effect ofchlorpheniramine.

Furosemide (Option C) is incorrect. Diuretics cause hyponatraemia and total body waterdepletion, not caused by SIADH.

AmitriptylineA

ChlorpheniramineB

FurosemideC

HaloperidolD

ThyroxineE

Page 347: Back to Filters (/Secure/TestMe/Filter ... - 1 File Download

8/13/2016 MyPastest

https://mypastest.pastest.com/Secure/TestMe/Browser/429893#Top 2/2

46384

Haloperidol (Option D) is incorrect. SIADH may occur after phenothiazine antipsychotics, e.g.chlorpromazine.

Thyroxine (Option E) is incorrect. Hypothyroidism may be associated with SIADH, but notthyroxine replacement.

Next Question

Previous Question Tag Question Feedback End Review

Difficulty: Average

Peer Responses

Session Progress

0Responses Correct:

179Responses Incorrect:

179Responses Total:

0%Responses - % Correct:

Blog (https://www.pastest.com/blog) About Pastest (https://www.pastest.com/about-us)Contact Us (https://www.pastest.com/contact-us) Help (https://www.pastest.com/help)

© Pastest 2016

Page 348: Back to Filters (/Secure/TestMe/Filter ... - 1 File Download

8/13/2016 MyPastest

https://mypastest.pastest.com/Secure/TestMe/Browser/429893#Top 1/2

Back to Filters (/Secure/TestMe/Filter/429893/QA)

Question 174 of 179

46828

You are asked to review a 73-year-old man with chronic obstructive pulmonary disease(COPD). He is maintained by his GP on combined inhalers and theophylline tablets, and hasrecently been treated with antibiotics for a COPD exacerbation. He describes feeling intensenausea and vomiting, and you find he has tachycardia. Investigations show serum potassium3.1 mmol/l.

Which of the following antibiotics is most likely to have caused these symptoms andfindings?

Explanation

The answer is Clarithromycin –

Macrolides and quinolones inhibit metabolism of theophylline metabolism and may causetheophylline toxicity. Quinolones may also increase the risk of convulsions if givenconcurrently with theophylline therapy.

Amoxicillin (Option A) is incorrect. Amoxicillin is unlikely to have any significant effect ontheophylline metabolism.

Ceftriaxone (Option B) is incorrect. Ceftriaxone is a long-acting cephalosporin antibiotic thatwould not be expected to interfere with theophylline metabolism.

Penicillin V (Option D) is incorrect. Penicillin V is unlikely to interact with theophylline.

Rifampicin (Option E) is incorrect. Rifampicin leads to the induction of enzymes involved intheophylline metabolism, resulting in a reduced plasma concentration.

AmoxicillinA

CeftriaxoneB

ClarithromycinC

Penicillin VD

RifampicinE

Page 349: Back to Filters (/Secure/TestMe/Filter ... - 1 File Download

8/13/2016 MyPastest

https://mypastest.pastest.com/Secure/TestMe/Browser/429893#Top 2/2

Next Question

Previous Question Tag Question Feedback End Review

Difficulty: Average

Peer Responses

Session Progress

0Responses Correct:

179Responses Incorrect:

179Responses Total:

0%Responses - % Correct:

Blog (https://www.pastest.com/blog) About Pastest (https://www.pastest.com/about-us)Contact Us (https://www.pastest.com/contact-us) Help (https://www.pastest.com/help)

© Pastest 2016

Page 350: Back to Filters (/Secure/TestMe/Filter ... - 1 File Download

8/13/2016 MyPastest

https://mypastest.pastest.com/Secure/TestMe/Browser/429893#Top 1/2

Back to Filters (/Secure/TestMe/Filter/429893/QA)

Question 175 of 179

45811

One of your patients has been referred to clinic for investigation of abnormal liverbiochemistry tests. These show alanine transaminase activity of 340 U/l, bilirubin 32millimoles per litre, and alkaline phosphatase 187 U/l. Hepatitis viral serology tests arenegative. He is taking a number of medications.

Which of the following drugs would be most likely to account for the abnormal liverbiochemistry?

Explanation

The answer is Nifedipine –

Nifedipine is a recognised cause of drug-induced hepatitis and cholestatic jaundice,albeit an uncommon cause. Other more common causes of drug-induced hepatitisinclude phenytoin, carbamazepine and rifampicin.

Bendroflumethiazide (Option A) is incorrect. This is not expected to cause hepatitis.

Gliclazide (Option B) is incorrect. This is not expected to cause hepatitis.

Paracetamol (Option D) is incorrect. Paracetamol toxicity characteristically causes delayedhepatitis, but this is not expected to occur in association with therapeutic paracetamoldoses.

Ramipril (Option E) is incorrect. This is not expected to cause hepatitis.

BendroflumethiazideA

GliclazideB

NifedipineC

ParacetamolD

RamiprilE

Next Question

Page 351: Back to Filters (/Secure/TestMe/Filter ... - 1 File Download

8/13/2016 MyPastest

https://mypastest.pastest.com/Secure/TestMe/Browser/429893#Top 2/2

Previous Question Tag Question Feedback End Review

Difficulty: Average

Peer Responses

Session Progress

0Responses Correct:

179Responses Incorrect:

179Responses Total:

0%Responses - % Correct:

Blog (https://www.pastest.com/blog) About Pastest (https://www.pastest.com/about-us)Contact Us (https://www.pastest.com/contact-us) Help (https://www.pastest.com/help)

© Pastest 2016

Page 352: Back to Filters (/Secure/TestMe/Filter ... - 1 File Download

8/13/2016 MyPastest

https://mypastest.pastest.com/Secure/TestMe/Browser/429893#Top 1/2

Back to Filters (/Secure/TestMe/Filter/429893/QA)

Question 176 of 179

A patient on treatment for a psychiatric disorder was noted to have hypertension and wasprescribed bendroflumethiazide. Within a week she developed tremor and agitation. Now, 4weeks later, she presents with heart block, seizures and a raised creatinine concentration of400 μmol/l.

What psychotropic drug is most likely to have interacted with bendroflumethiazide to causethese clinical features?

Explanation

The answer is Lithium –

Lithium is prescribed for bipolar disorders and schizoaffective psychosis. The patientpresents with heart block, seizures and a raised creatinine concentration of 400 μmol/l,suggestive of lithium toxicity. Other features of toxicity include blurred vision,hypokalaemia, drowsiness, ataxia, tremor, dysarthria, arrhythmias, seizures,hypothyroidism, psychosis and coma. Lithium toxicity may be caused by the concomitantuse of diuretics due to increased renal lithium reabsorption.

Amitriptyline (Option A) is incorrect. Amitriptyline may cause sedation, dry mouth, urinaryretention, blurred vision, acute delirium and postural hypotension.

Chlordiazepoxide (Option B) is incorrect. Adverse effects of chlordiazepoxide includedrowsiness, nausea, headache and skin rash.

Chlorpromazine (Option C) is incorrect. Chlorpromazine may cause cholestatic jaundice,drowsiness, respiratory depression, leukopenia, tardive dyskinesia and QT prolongation.

AmitriptylineA

ChlordiazepoxideB

ChlorpromazineC

FluoxetineD

LithiumE

Page 353: Back to Filters (/Secure/TestMe/Filter ... - 1 File Download

8/13/2016 MyPastest

https://mypastest.pastest.com/Secure/TestMe/Browser/429893#Top 2/2

46771

Fluoxetine (Option D) is incorrect. Fluoxetine is also associated with nausea, dry mouth,blurred vision, seizures, tremor, myoclonus and decreased libido.

Next Question

Previous Question Tag Question Feedback End Review

Difficulty: Average

Peer Responses

Session Progress

0Responses Correct:

179Responses Incorrect:

179Responses Total:

0%Responses - % Correct:

Blog (https://www.pastest.com/blog) About Pastest (https://www.pastest.com/about-us)Contact Us (https://www.pastest.com/contact-us) Help (https://www.pastest.com/help)

© Pastest 2016

Page 354: Back to Filters (/Secure/TestMe/Filter ... - 1 File Download

8/13/2016 MyPastest

https://mypastest.pastest.com/Secure/TestMe/Browser/429893#Top 1/2

Back to Filters (/Secure/TestMe/Filter/429893/QA)

Question 177 of 179

46759

A 30-year-old woman is receiving treatment for secondary generalised tonic–clonic seizuresand complains of anorexia, nausea and general fatigue. On examination, there is no peripheraloedema and respiratory examination is normal. Investigations show Na 124 mmol/l, K 4.0mmol/l and plasma osmolality 220 mosmol/kg.

Which of the following antiepileptic drugs is most likely to explain these abnormalities?

Explanation

The answer is Carbamazepine –

Carbamazepine is a well-recognised cause of hyponatraemia, which occurs in a dose-related manner and is reversible after drug cessation.

Acetazolamide (Option A) is incorrect. Acetazolamide may cause a significant metabolicacidosis.

Lamotrigine (Option C) is incorrect. Lamotrigine would not be expected to causehyponatraemia.

Levetiracetam (Option D) is incorrect. Levetiracetam would not be expected to causehyponatraemia.

Topiramate (Option E) is incorrect. Topirimate would not be expected to causehyponatraemia.

+ +

AcetazolamideA

CarbamazepineB

LamotrigineC

LevetiracetamD

TopiramateE

Next Question

Page 355: Back to Filters (/Secure/TestMe/Filter ... - 1 File Download

8/13/2016 MyPastest

https://mypastest.pastest.com/Secure/TestMe/Browser/429893#Top 2/2

Previous Question Tag Question Feedback End Review

Difficulty: Average

Peer Responses

Session Progress

0Responses Correct:

179Responses Incorrect:

179Responses Total:

0%Responses - % Correct:

Blog (https://www.pastest.com/blog) About Pastest (https://www.pastest.com/about-us)Contact Us (https://www.pastest.com/contact-us) Help (https://www.pastest.com/help)

© Pastest 2016

Page 356: Back to Filters (/Secure/TestMe/Filter ... - 1 File Download

8/13/2016 MyPastest

https://mypastest.pastest.com/Secure/TestMe/Browser/429893#Top 1/2

Back to Filters (/Secure/TestMe/Filter/429893/QA)

Question 178 of 179

An 18-year-old woman presents to the Emergency Department with symptoms of nausea andvomiting. She states that she had been feeling very frustrated and upset and had taken anintentional overdose involving 50 paracetamol tablets 3 h earlier.

Which of the following would be the most effective treatment at this time?

Explanation

The answer is Acetylcysteine –

As little as 10–15 g (20–30 tablets) in an adult or 150 mg/kg of paracetamol taken within24 h may cause severe hepatocellular necrosis and, less frequently, renal tubular necrosis.Early features include nausea and vomiting, and as liver damage progresses over thenext 24 h there may be right subcostal pain and tenderness. In patients that recover, liverdamage is usually maximal 2–3 days after the ingestion. In patients that do not recover,there is progressive liver damage, encephalopathy, haemorrhage, hypoglycaemia,cerebral oedema, coma and death. Acetylcysteine is a highly effective antidote thatprevents fatal liver toxicity if administered within 8 h of overdose, with diminishingeffectiveness after longer intervals between ingestion and treatment.

Methionine (Option B) is incorrect. Although methionine may protect the liver againstparacetamol-induced toxicity, there is a stronger evidence base in support of acetylcysteine.No comparative efficacy data are available for methionine and acetylcysteine, butacetylcysteine is the standard treatment in most international guidelines.

Metoclopramide (Option C) is incorrect. Antiemetics are important in controlling symptomsof nausea and vomiting, but metoclopramide is associated with a high risk of acute dystonia

AcetylcysteineA

MethionineB

MetoclopramideC

NaloxoneD

Oral activated charcoalE

Page 357: Back to Filters (/Secure/TestMe/Filter ... - 1 File Download

8/13/2016 MyPastest

https://mypastest.pastest.com/Secure/TestMe/Browser/429893#Top 2/2

46800

in young female patients, and alternatives such as ondansetron are preferred.

Naloxone (Option D) is incorrect. Naloxone is used for the treatment of opiate overdose, andis normally administered in multiple repeated dosages titrated against clinical response toachieve sufficiently improved conscious level that patients are able to protect their ownairway.

Oral activated charcoal (Option E) is incorrect. Oral activated charcoal is effective inreducing paracetamol absorption, and may lessen the need for acetylcysteine antidote.However, it should be administered within 1 h of ingestion, and is contraindicated if patientsare actively vomiting.

Next Question

Previous Question Tag Question Feedback End Review

Difficulty: Average

Peer Responses

Session Progress

0Responses Correct:

179Responses Incorrect:

179Responses Total:

0%Responses - % Correct:

Blog (https://www.pastest.com/blog) About Pastest (https://www.pastest.com/about-us)Contact Us (https://www.pastest.com/contact-us) Help (https://www.pastest.com/help)

© Pastest 2016

Page 358: Back to Filters (/Secure/TestMe/Filter ... - 1 File Download

8/13/2016 MyPastest

https://mypastest.pastest.com/Secure/TestMe/Browser/429893#Top 1/2

Back to Filters (/Secure/TestMe/Filter/429893/QA)

Question 179 of 179

A 58-year-old man with chronic upper gastrointestinal symptoms has recently beendiagnosed as having a duodenal ulcer at endoscopy. No evidence of oesophageal or gastricpathology was found. Helicobacter pylori testing was positive.

Which of the following treatment regimens would be most appropriate?

Explanation

The answer is Proton-pump inhibitor, clarithromycin and metronidazole, followed by aproton-pump inhibitor long term –

All patients with proven acute or chronic duodenal ulcer disease and those with gastriculcer who are Helicobacter pylori-positive should be offered eradication therapy asprimary treatment. A number of options are available depending on patient factors,including allergy status. The recommended first-line therapy should include a proton-pump inhibitor (PPI) at the standard dose 12 hourly. In addition, antimicrobial treatmentnormally includes clarithromycin 500 mg 12 hourly, plus either amoxicillin 1 g 12 hourly ormetronidazole 400 mg 12 hourly.

H2 receptor antagonist and antacids long term (Option A) is incorrect. PPIs are the mostpowerful inhibitors of gastric acid secretion with maximal inhibition occurring 3–6 h after anoral dose. They have an excellent safety profile, and allow rapid healing of both gastric andduodenal ulcers. Antacids are insufficiently effective in increasing gastric pH to alloweffective healing.

H2 receptor antagonist and antacids long termA

Proton-pump inhibitor and sucralfateB

Proton-pump inhibitor, bismuth, metronidazole and tetracycline, followed byantacids long term

C

Proton-pump inhibitor, clarithromycin and metronidazole, followed by a proton-pump inhibitor long term

D

Proton-pump inhibitor long term and dietary adviceE

Page 359: Back to Filters (/Secure/TestMe/Filter ... - 1 File Download

8/13/2016 MyPastest

https://mypastest.pastest.com/Secure/TestMe/Browser/429893#Top 2/2

46609

Proton-pump inhibitor and sucralfate (Option B) is incorrect. This regimen would not alloweradication of H. pylori.

Proton-pump inhibitor, bismuth, metronidazole and tetracycline, followed by antacids longterm (Option C) is incorrect. Bismuth is no longer routinely recommended.

Proton-pump inhibitor long term and dietary advice (Option E) is incorrect. This regimenwould not allow eradication of H. pylori, and no special dietary advice is required.

End Session

Previous Question Tag Question Feedback End Review

Difficulty: Average

Peer Responses

Session Progress

0Responses Correct:

179Responses Incorrect:

179Responses Total:

0%Responses - % Correct:

Blog (https://www.pastest.com/blog) About Pastest (https://www.pastest.com/about-us)Contact Us (https://www.pastest.com/contact-us) Help (https://www.pastest.com/help)

© Pastest 2016

Page 360: Back to Filters (/Secure/TestMe/Filter ... - 1 File Download

8/11/2016 MyPastest

https://mypastest.pastest.com/Secure/TestMe/Browser/429893 1/2

Back to Filters (/Secure/TestMe/Filter/429893/QA)

Question 1 of 207

A 56-year-old worker is brought into the Emergency Department from a nearbyelectroplating company. Two of his colleagues are present, and advise that he may have beenaccidentally exposed to hydrogen cyanide gas.

Which of the following steps best describes the treatment of suspected cyanide poisoning?

Explanation

The answer is Amyl nitrite may be a useful antidote therapy –

Cyanide can be found in certain plants, such as cyanogenic glycosides (eg bitteralmonds, apricot kernels). It is rapidly absorbed through the respiratory andgastrointestinal tracts and the skin. Effects are more rapid onset after inhalation thaningestion, death may occur within minutes. Immediate management of suspectedcyanide poisoning may consist of oxygen administration and administration of inhalednitrites, eg amyl nitrite.

Await formal laboratory cyanide confirmation before administering antidote (Option B) isincorrect. Although laboratory confirmation may be helpful in confirming the cyanideexposure and extent of poisoning, treatment should not be delayed.

Oral activated charcoal should be administered within 1 h of exposure (Option C) is incorrect.Charcoal is unlikely to be effective.

Patients with generalised headache should be treated with dicobalt edetate (Option D) isincorrect. Intravenous dicobalt edetate is reserved for patients with signs of severe poisoning(coma, metabolic acidosis, respiratory depression). There should be a clear history of

Amyl nitrite may be a useful antidote therapyA

Await formal laboratory cyanide confirmation before administering antidoteB

Oral activated charcoal should be administered within 1 h of exposureC

Patients with generalised headache should be treated with dicobalt edetateD

Supplementary oxygen may increase pulmonary toxicity and should be avoidedE

Page 361: Back to Filters (/Secure/TestMe/Filter ... - 1 File Download

8/11/2016 MyPastest

https://mypastest.pastest.com/Secure/TestMe/Browser/429893 2/2

46492

exposure, since the potential serious adverse effects (bronchospasm, laryngeal oedema) aremore likely if no cyanide ions are present.

Supplementary oxygen may increase pulmonary toxicity and should be avoided (Option E) isincorrect. This is true of Paraquat, but not of cyanide poisoning.

Next Question

Previous Question Tag Question Feedback End Review

Difficulty: Average

Peer Responses

Session Progress

0Responses Correct:

207Responses Incorrect:

207Responses Total:

0%Responses - % Correct:

Blog (https://www.pastest.com/blog) About Pastest (https://www.pastest.com/about-us)Contact Us (https://www.pastest.com/contact-us) Help (https://www.pastest.com/help)

© Pastest 2016

Page 362: Back to Filters (/Secure/TestMe/Filter ... - 1 File Download

8/11/2016 MyPastest

https://mypastest.pastest.com/Secure/TestMe/Browser/429893 1/2

Back to Filters (/Secure/TestMe/Filter/429893/QA)

Question 2 of 207

A 68-year-old woman with severe rheumatoid arthritis presents for review. She complains ofincreasing tiredness and has multiple swollen lymph nodes. There is mild normochromic,normocytic anaemia and her erythrocyte sedimentation rate (ESR) is markedly raised despitequiescent rheumatoid disease at present. She is currently managed with methotrexate andinfliximab.

What diagnosis fits best with this clinical picture?

Explanation

The answer is Lymphoma secondary to immune-modulation therapy –

Various studies have attempted to estimate the long-term risk of lymphoma aftertreatment with antitumour necrosis factor-alpha (TNF-α) immune-modulation therapy(infliximab). The Food and Drug Administration (FDA) estimate a two to six-foldincreased risk of lymphoma, and anti-TNF-α agents carry a warning label concerning therisk of malignancy.

Carcinomatosis (Option A) is incorrect. The possible increased risk of solid tumours is lessstriking than the increased risk of lymphoma, hence this answer is a less preferred optionthan lymphoma secondary to immune-modulation therapy.

Chronic myeloid leukaemia secondary to methotrexate (Option B) is incorrect. Methotrexateis associated with an increased risk of melanoma and other cancers, but less stronglyassociated with lymphoma than anti-TNF-α immune-modulators.

Reactivation of rheumatoid disease (Option D) is incorrect. This cannot explain theoccurrence of lymphadenopathy, and is less likely given the absence of active joint

CarcinomatosisA

Chronic myeloid leukaemia secondary to methotrexateB

Lymphoma secondary to immune-modulation therapyC

Reactivation of rheumatoid diseaseD

SLE-type syndrome related to immune-modulation therapyE

Page 363: Back to Filters (/Secure/TestMe/Filter ... - 1 File Download

8/11/2016 MyPastest

https://mypastest.pastest.com/Secure/TestMe/Browser/429893 2/2

46889

symptoms.

SLE-type syndrome related to immune-modulation therapy (Option E) is incorrect. There isan increased occurrence of antinuclear antibody positivity, although the occurrences ofclinical features of systemic lupus erythematosus (SLE) are rare, and resolve after drugdiscontinuation.

Next Question

Previous Question Tag Question Feedback End Review

Difficulty: Average

Peer Responses

Session Progress

0Responses Correct:

207Responses Incorrect:

207Responses Total:

0%Responses - % Correct:

Blog (https://www.pastest.com/blog) About Pastest (https://www.pastest.com/about-us)Contact Us (https://www.pastest.com/contact-us) Help (https://www.pastest.com/help)

© Pastest 2016

Page 364: Back to Filters (/Secure/TestMe/Filter ... - 1 File Download

8/11/2016 MyPastest

https://mypastest.pastest.com/Secure/TestMe/Browser/429893 1/2

Back to Filters (/Secure/TestMe/Filter/429893/QA)

Question 3 of 207

A 66-year-old woman attends the cardiology outpatient clinic for review. She has beenreceiving a number of medications, including a ß-blocker (ß-adrenergic antagonist) fortreatment of angina, and asks you for information about this medication.

Which of the following statements best describes the therapeutic action of β-blockers inangina?

Explanation

The answer is Decrease the heart rate and myocardial contractility –

The cardiovascular effects of ß-adrenoceptor block depend on the amount ofsympathetic tone present, which normally increases during exercise or activity. Theprimary pharmacological action of β-blockers is to reduce sympathetic drive, whichreduces heart rate (automaticity) and lessens myocardial contractility (rate of rise ofpressure in the ventricle). These effects result in reduced cardiac output and an overallfall in myocardial oxygen consumption.

Increase the cardiac preload (Option B) is incorrect. Beta-blockers tend to increase afterloaddue to increased peripheral vascular resistance, but do not have any significant effect oncardiac preload.

Increase the sinus node automaticity (Option C) is incorrect. Beta-blockers reduce sinusnode automaticity, thereby lowering heart rate.

Increase the left atrial volume and pressure (Option D) is incorrect. Beta-blockers decreaseleft atrial volume and pressure.

Decrease the heart rate and myocardial contractilityA

Increase the cardiac preloadB

Increase the sinus node automaticityC

Increase the left atrial volume and pressureD

Increase the peripheral vascular resistanceE

Page 365: Back to Filters (/Secure/TestMe/Filter ... - 1 File Download

8/11/2016 MyPastest

https://mypastest.pastest.com/Secure/TestMe/Browser/429893 2/2

46629

Increase the peripheral vascular resistance (Option E) is incorrect. It is true that ß-blockerscause a rise in peripheral vascular resistance due to the unopposed action of catecholamineson α-adrenoceptors in the periphery. However, this gives rise to the characteristic adverseeffects (vasoconstriction, muscle cramps and cold peripheries) but not the intendedpharmacological effects.

Next Question

Previous Question Tag Question Feedback End Review

Difficulty: Average

Peer Responses

Session Progress

0Responses Correct:

207Responses Incorrect:

207Responses Total:

0%Responses - % Correct:

Blog (https://www.pastest.com/blog) About Pastest (https://www.pastest.com/about-us)Contact Us (https://www.pastest.com/contact-us) Help (https://www.pastest.com/help)

© Pastest 2016

Page 366: Back to Filters (/Secure/TestMe/Filter ... - 1 File Download

8/11/2016 MyPastest

https://mypastest.pastest.com/Secure/TestMe/Browser/429893 1/2

Back to Filters (/Secure/TestMe/Filter/429893/QA)

Question 4 of 207

A patient is brought into hospital at 0700 h after accidentally drinking 300 ml diethyleneglycol from an unmarked bottle on the previous night. He appears intoxicated with slurredspeech and drowsiness, and initial investigations show metabolic acidosis and acute renalfailure.

What is the most appropriate next step in his management?

Explanation

The answer is Haemodialysis and intravenous ethanol –

Diethylene glycol is used mainly in an industrial setting as a solvent used in manufactureof polyester resins and other chemicals. Several instances of fatal outbreaks haveoccurred where diethylene glycol has been inadvertently added to pharmaceuticalpreparations. Features of toxicity include nausea, vomiting and abdominal pain. Astoxicity develops, there may be jaundice, pulmonary oedema, severe metabolic acidosis,renal failure, coma and death. Management involves supportive measures to correctdehydration and metabolic distrurbances. Toxicity is mediated by the metabolites ofdiethylene glycol, and so fomepizole (4-methylpyrazole) should be administered to blockdiethylene glycol metabolism by alcohol dehydrogenase. Haemodialysis should beconsidered if renal failure develops or there is severe acidosis or metabolic disturbance.An alternative to fomepizole is ethanol, which competes with diethylene glycol formetabolism and minimises formation of toxic metabolites. Fomepizole or ethanol shouldnormally be continued until diethylene glycol is no longer detectable in the blood.

Forced diuresis (Option A) is incorrect. Forced diuresis is unhelpful and potentiallyhazardous.

Forced diuresisA

Gastric lavageB

Haemodialysis and intravenous ethanolC

Intravenous insulinD

Oral methanolE

Page 367: Back to Filters (/Secure/TestMe/Filter ... - 1 File Download

8/11/2016 MyPastest

https://mypastest.pastest.com/Secure/TestMe/Browser/429893 2/2

46791

Gastric lavage (Option B) is incorrect. This is rarely helpful in clinical practice and potentiallyhazardous, and too late to be of any benefit in this case.

Intravenous insulin (Option D) is incorrect. The metabolic acidosis is attributable to build-upof metabolites and renal failure, not diabetic ketoacidosis.

Oral methanol (Option E) is incorrect. Methanol would compete with alcohol dehydrogenasebut would add to the burden of toxic metabolites and worsen the clinical outcome.

Next Question

Previous Question Tag Question Feedback End Review

Difficulty: Average

Peer Responses

Session Progress

0Responses Correct:

207Responses Incorrect:

207Responses Total:

0%Responses - % Correct:

Blog (https://www.pastest.com/blog) About Pastest (https://www.pastest.com/about-us)Contact Us (https://www.pastest.com/contact-us) Help (https://www.pastest.com/help)

© Pastest 2016

Page 368: Back to Filters (/Secure/TestMe/Filter ... - 1 File Download

8/11/2016 MyPastest

https://mypastest.pastest.com/Secure/TestMe/Browser/429893 1/2

Back to Filters (/Secure/TestMe/Filter/429893/QA)

Question 5 of 207

You are reviewing a 63-year-old man in the medical admissions unit who has been given adiagnosis of community-acquired pneumonia. You remember from a recent teaching sessionthat moxifloxacin is licensed for the treatment of community-acquired pneumonia, acuteexacerbation of chronic bronchitis and acute bacterial sinusitis.

Which of the following adverse effects is most strongly associated with moxifloxacin?

Explanation

The answer is Achilles tendon rupture –

Moxifloxacin is a new generation of quinolone antibiotics that is bactericidal byinterfering with DNA synthesis, and is effective against Gram-positive, Gram-negativeand certain atypical respiratory pathogens. Adverse effects of quinolones include risk oftendonitis and tendon rupture. Other adverse effects include gastrointestinaldisturbances, headache and liver dysfunction. Quinolones should not be used incombination with other drugs that prolong the QT interval (eg erythromycin, citalopram)because there is an increased risk of cardiac arrhythmias. Quinolones may reduce cardiacejection fraction and are contraindicated in left heart failure. They should not be given atthe same time as bivalent or trivalent cations (eg aluminium, iron) as these reduceabsorption.

Clostridium difficile associated diarrhoea (Option B) is incorrect. Although all broad-spectrum antibiotics are capable of provoking Clostridium difficile associated diarrhea, this isless specific to quinolones than the occurrence of tendon rupture.

Achilles tendon ruptureA

Clostridium difficile associated diarrhoeaB

NeutropaeniaC

Oesophageal erosions and perforationD

Widening of the QRS durationE

Page 369: Back to Filters (/Secure/TestMe/Filter ... - 1 File Download

8/11/2016 MyPastest

https://mypastest.pastest.com/Secure/TestMe/Browser/429893 2/2

46380

Neutropaenia (Option C) is incorrect. This may be encountered in patients with sepsis, but isnot a commonly recognised adverse effect of quinolones.

Oesophageal erosions and perforation (Option D) is incorrect. This is a recognisedcomplication of bisphosphonates.

Widening of the QRS duration (Option E) is incorrect. Quinolones may cause QTprolongation.

Next Question

Previous Question Tag Question Feedback End Review

Difficulty: Average

Peer Responses

Session Progress

0Responses Correct:

207Responses Incorrect:

207Responses Total:

0%Responses - % Correct:

Blog (https://www.pastest.com/blog) About Pastest (https://www.pastest.com/about-us)Contact Us (https://www.pastest.com/contact-us) Help (https://www.pastest.com/help)

© Pastest 2016

Page 370: Back to Filters (/Secure/TestMe/Filter ... - 1 File Download

8/11/2016 MyPastest

https://mypastest.pastest.com/Secure/TestMe/Browser/429893 1/2

Back to Filters (/Secure/TestMe/Filter/429893/QA)

Question 6 of 207

You review a 75-year-old man in clinic with long-standing Alzheimer’s disease. His familyattends with him and asks you some questions about memantine, a drug that is licensed forthe management of dementia in the UK.

Which of the following statements is most appropriate regarding memantine?

Explanation

The answer is Adverse effects are more likely if co-administered with amantadine -

Excess activation of the NMDA (N-methyl-d-aspartate) receptor may play a role in thepathogenesis of Alzheimer’s disease. Memantine is an antagonist at the NMDA receptor.Other treatment options for Alzheimer’s disease include cholinesterase inhibitors (e.g.tacrine, donepezil). Other drugs with NMDA-receptor antagonist properties includeketamine and amantadine, which increase the risk of psychosis when co-administered.

Concomitant baclofen doses may have to be increased (Option B) is incorrect. Memantinehas some antispasmodic effects, therefore the dose of baclofen may be reduced in somecases.

It inhibits cholinesterase activity within the central nervous system (Option C) is incorrect.Certain other drugs used in Alzheimer’s disease inhibit cholinesterase activity (e.g. tacrine,donepezil).

Its pharmacological action is an NMDA-receptor agonist (Option D) is incorrect. It is anantagonist, not an agonist at the NMDA receptor

Adverse effects are more likely if co-administered with amantadineA

Concomitant baclofen doses may have to be increasedB

It inhibits cholinesterase activity within the central nervous systemC

Its pharmacological action is an NMDA-receptor agonistD

Quinine serum concentrations will fall during memantine useE

Page 371: Back to Filters (/Secure/TestMe/Filter ... - 1 File Download

8/11/2016 MyPastest

https://mypastest.pastest.com/Secure/TestMe/Browser/429893 2/2

46376

Quinine serum concentrations will fall during memantine use (Option E) is incorrect.Memantine interferes with cationic transporters in the kidney so that excretion of some drugsis reduced (eg quinine, cimetidine, ranitidine), therefore higher plasma concentrations arelikely.

Next Question

Previous Question Tag Question Feedback End Review

Difficulty: Average

Peer Responses

Session Progress

0Responses Correct:

207Responses Incorrect:

207Responses Total:

0%Responses - % Correct:

Blog (https://www.pastest.com/blog) About Pastest (https://www.pastest.com/about-us)Contact Us (https://www.pastest.com/contact-us) Help (https://www.pastest.com/help)

© Pastest 2016

Page 372: Back to Filters (/Secure/TestMe/Filter ... - 1 File Download

8/11/2016 MyPastest

https://mypastest.pastest.com/Secure/TestMe/Browser/429893 1/2

Back to Filters (/Secure/TestMe/Filter/429893/QA)

Question 7 of 207

46826

You are reviewing a 74-year-old woman who has been treated with verapamil for chronicatrial tachycardia. You note in clinic that she is hypertensive with a blood pressure of 160/85mmHg. You want to start additional blood-pressure-lowering therapy.

Which of the following would be the most appropriate next agent to add in?

Explanation

The answer is Ramipril –

The combination of a calcium-channel blocker with angiotensin converting enzyme(ACE) inhibitor would be effective in achieving good blood pressure control. Ramipril isan ACE inhibitor with good outcome data for cardiovascular risk reduction.

Atenolol (Option A) is incorrect. The combination of a non-dihydropyridine calciumantagonist with β-blockade is ill advised owing to the risk of severe bradycardia, heart block,severe hypotension and syncope.

Diltiazem (Option B) is incorrect. Diltiazem has a similar mode of action to verapamil and assuch it may provoke heart block and severe bradycardia.

Metoprolol (Option C) is incorrect. The combination of a non-dihydropyridine calciumantagonist with β-blockade is ill advised owing to the risk of severe bradycardia, heart block,severe hypotension and syncope.

Sotalol (Option E) is incorrect. Sotalol is unlikely to confer any meaningful additional bloodpressure control.

AtenololA

DiltiazemB

MetoprololC

RamiprilD

SotalolE

Page 373: Back to Filters (/Secure/TestMe/Filter ... - 1 File Download

8/11/2016 MyPastest

https://mypastest.pastest.com/Secure/TestMe/Browser/429893 2/2

Next Question

Previous Question Tag Question Feedback End Review

Difficulty: Average

Peer Responses

Session Progress

0Responses Correct:

207Responses Incorrect:

207Responses Total:

0%Responses - % Correct:

Blog (https://www.pastest.com/blog) About Pastest (https://www.pastest.com/about-us)Contact Us (https://www.pastest.com/contact-us) Help (https://www.pastest.com/help)

© Pastest 2016

Page 374: Back to Filters (/Secure/TestMe/Filter ... - 1 File Download

8/11/2016 MyPastest

https://mypastest.pastest.com/Secure/TestMe/Browser/429893 1/2

Back to Filters (/Secure/TestMe/Filter/429893/QA)

Question 8 of 207

46595

A patient is referred to the Acute Medical Unit from the local cancer care centre. They havebeen receiving chemotherapy treatment and have presented to hospital with increasedurinary frequency and frank haematuria. Full blood count shows a mild anaemia andleukopenia.

Which of the following drugs is most likely to account for these findings?

Explanation

The answer is Cyclophosphamide –

Cyclophosphamide is a prodrug that is metabolised by the liver to 4-hydroxylcyclophosphamide, which decomposes into alkylating species as well as tochloroacetaldehyde and acrolein. Acrolein is known to cause chemical haemorrhagiccystitis, and therefore excellent hydration must be maintained during cyclophosphamidetherapy. Ifosamide may also cause haemorrhagic cystitis. Co-administration of mesnamay minimise the risk of bladder toxicity by binding to acrolein and the toxic metabolitesof ifosfamide.

Allopurinol (Option A) is incorrect. Allopurinol does not cause bladder irritation.

Cisplatin (Option B) is incorrect. Cisplatin characteristically causes acute kidney injury.

Mithramycin (Option D) is incorrect. Mithramycin may cause renal damage and bone marrowsuppression.

Tamsulosin (Option E) is incorrect. Tamsulosin may cause urinary incontinence.

AllopurinolA

CisplatinB

CyclophosphamideC

MithramycinD

TamsulosinE

Page 375: Back to Filters (/Secure/TestMe/Filter ... - 1 File Download

8/11/2016 MyPastest

https://mypastest.pastest.com/Secure/TestMe/Browser/429893 2/2

Next Question

Previous Question Tag Question Feedback End Review

Difficulty: Average

Peer Responses

Session Progress

0Responses Correct:

207Responses Incorrect:

207Responses Total:

0%Responses - % Correct:

Blog (https://www.pastest.com/blog) About Pastest (https://www.pastest.com/about-us)Contact Us (https://www.pastest.com/contact-us) Help (https://www.pastest.com/help)

© Pastest 2016

Page 376: Back to Filters (/Secure/TestMe/Filter ... - 1 File Download

8/11/2016 MyPastest

https://mypastest.pastest.com/Secure/TestMe/Browser/429893 1/2

Back to Filters (/Secure/TestMe/Filter/429893/QA)

Question 9 of 207

46866

A 57-year-old woman who has had a previous renal transplant is being given long-termazathioprine treatment.

Which of the following statements best describes the main mechanism of action ofazathioprine?

Explanation

The answer is It suppresses lymphocyte numbers and function –

Azathioprine acts as an inhibitor of purine synthase, which is an essential step inproliferation of leukocytes and lymphocytes.

It blocks antibody formation (Option A) is incorrect. Anti-B-cell agents reduce antibodyproduction, including rituximab.

It interferes with T-cell activation mechanisms at an intracellular level (Option B) is incorrect.Ciclosporin and tacrolimus act as immunosuppressants by interfering with T-cell activation atan intracellular level.

It interferes with T-cell–macrophage cooperation (Option C) is incorrect. Corticosteroidsinterfere with T-cell–macrophage cooperation and impair macrophage responses tocytokines.

It reduces the production of cytokines (Option D) is incorrect. Selective antibody agents mayprevent or minimise formation of cytokines, eg anti-IL1 monoclonal antibodies.

It blocks antibody formationA

It interferes with T-cell activation mechanisms at an intracellular levelB

It interferes with T-cell–macrophage cooperationC

It reduces the production of cytokinesD

It suppresses lymphocyte numbers and functionE

Page 377: Back to Filters (/Secure/TestMe/Filter ... - 1 File Download

8/11/2016 MyPastest

https://mypastest.pastest.com/Secure/TestMe/Browser/429893 2/2

Next Question

Previous Question Tag Question Feedback End Review

Difficulty: Average

Peer Responses

Session Progress

0Responses Correct:

207Responses Incorrect:

207Responses Total:

0%Responses - % Correct:

Blog (https://www.pastest.com/blog) About Pastest (https://www.pastest.com/about-us)Contact Us (https://www.pastest.com/contact-us) Help (https://www.pastest.com/help)

© Pastest 2016

Page 378: Back to Filters (/Secure/TestMe/Filter ... - 1 File Download

8/11/2016 MyPastest

https://mypastest.pastest.com/Secure/TestMe/Browser/429893 1/2

Back to Filters (/Secure/TestMe/Filter/429893/QA)

Question 10 of 207

46986

You are examining the local protocols concerning patients treated at the local cancer unit,and the indications for referral for pre-treatment echocardiography.

Which one of the following agents is most likely to cause cardiotoxicity?

Explanation

The answer is Trastuzumab –

Trastuzumab (Herceptin) is used in the treatment of HER2-receptor-positive metastaticbreast cancer. Trastuzumab is associated with cardiomyopathy in between 2% and 7% ofusers, and the risk of cardiomyopathy is increased when given in combination withanthracycline.

Cisplatin (Option A) is incorrect. Cisplatin is associated with neurotoxicity, nephrotoxicityand ototoxicity.

Cyclophosphamide (Option B) is incorrect. Cyclophosphamide is associated withhaemorrhagic cystitis.

Rituximab (Option C) is incorrect. Rituximab is associated with cytokine release.

Sunitinib (Option D) is incorrect. Sunitinib is associated mainly with fatigue, diarrhoea,hypertension and hand/foot syndrome.

CisplatinA

CyclophosphamideB

RituximabC

SunitinibD

TrastuzumabE

Next Question

Page 379: Back to Filters (/Secure/TestMe/Filter ... - 1 File Download

8/11/2016 MyPastest

https://mypastest.pastest.com/Secure/TestMe/Browser/429893 2/2

Previous Question Tag Question Feedback End Review

Difficulty: Average

Peer Responses

Session Progress

0Responses Correct:

207Responses Incorrect:

207Responses Total:

0%Responses - % Correct:

Blog (https://www.pastest.com/blog) About Pastest (https://www.pastest.com/about-us)Contact Us (https://www.pastest.com/contact-us) Help (https://www.pastest.com/help)

© Pastest 2016

Page 380: Back to Filters (/Secure/TestMe/Filter ... - 1 File Download

8/11/2016 MyPastest

https://mypastest.pastest.com/Secure/TestMe/Browser/429893#Top 1/2

Back to Filters (/Secure/TestMe/Filter/429893/QA)

Question 11 of 207

An 82-year-old woman presents to the Acute Medical Unit with a history of vomiting andfever, and has been diagnosed with a lower urinary tract infection. You decide to administeroral trimethoprim.

Which of the following most accurately describes the pharmacological actions oftrimethoprim?

Explanation

The answer is It inhibits dihydrofolate reductase –

Trimethoprim acts by inhibiting dihydrofolate reductase, an enzyme that is important inthe division of bacterial cells. It is bacteriostatic, rather than bactericidal.Sulfamethoxazole also inhibits folate metabolism, and when combined with trimethoprim(co-trimoxazole) this exerts bactericidal effects; co-trimoxazole is associated withsignificant adverse effects including blood dyscrasias and autoimmune phenomena, andis normally reserved for severe infections such as pneumocystis pneumonia.Trimethoprim may cause megaloblastic anaemia due to folate deficiency where used inlong-term treatment.

Hypokalaemia occurs in 20–30% of patients (Option A) is incorrect. Hyperkalaemia and a risein serum creatinine concentrations characteristically occur due to interference with tubularsecretion, and are fully reversible on stopping trimethoprim.

It is bactericidal (Option C) is incorrect. It is bacteriostatic. The combination withsulfamethoxazole does confer bactericidal effects.

Hypokalaemia occurs in 20–30% of patientsA

It inhibits dihydrofolate reductaseB

It is bactericidalC

It may cause marrow depression and microcytic anaemiaD

It requires therapeutic drug monitoring to ensure efficacyE

Page 381: Back to Filters (/Secure/TestMe/Filter ... - 1 File Download

8/11/2016 MyPastest

https://mypastest.pastest.com/Secure/TestMe/Browser/429893#Top 2/2

46603

It may cause marrow depression and microcytic anaemia (Option D) is incorrect. Significantbone marrow depression occurs when used in combination with sulfamethoxazole.Macrocytic anaemia may occur, not microcytic.

It requires therapeutic drug monitoring to ensure efficacy (Option E) is incorrect. Serumlevels are rarely used to monitor trimethoprim treatment.

Next Question

Previous Question Tag Question Feedback End Review

Difficulty: Average

Peer Responses

Session Progress

0Responses Correct:

207Responses Incorrect:

207Responses Total:

0%Responses - % Correct:

Blog (https://www.pastest.com/blog) About Pastest (https://www.pastest.com/about-us)Contact Us (https://www.pastest.com/contact-us) Help (https://www.pastest.com/help)

© Pastest 2016

Page 382: Back to Filters (/Secure/TestMe/Filter ... - 1 File Download

8/11/2016 MyPastest

https://mypastest.pastest.com/Secure/TestMe/Browser/429893 1/2

Back to Filters (/Secure/TestMe/Filter/429893/QA)

Question 12 of 207

You are reviewing a 42-year-old man in the hypertension clinic, and are consideringprescribing doxazosin.

In which circumstances would prescription of an α-blocker be considered valuable in patientswith hypertension?

Explanation

The answer is Hypertension due to phaeochromocytoma –

The mainstay of medical treatment is α-blockade to minimise the effect of excessnorepinephrine concentrations acting on peripheral α-receptors that causevasoconstriction. The α-blocker of choice is phenoxybenzamine, which irreversibly blocksthe α-receptor by alkylation.

Essential hypertension (Option A) is incorrect. Alpha-blockers are second- or third-linetreatments for essential hypertension, after ACE inhibitors, angiotensin-receptor blockers,calcium-channel blockers and diuretics.

Hypertension and asthma (Option B) is incorrect. Beta-blockers should be avoided in asthma.First-line therapies remain ACE inhibitors, angiotensin-receptor blockers and calcium-channelblockers.

Hypertension due to Conn syndrome (Option C) is incorrect. Conn syndrome is characterisedby excess aldosterone. Spironolactone possesses anti-aldosterone properties and may behighly effective in achieving blood pressure control.

Essential hypertensionA

Hypertension and asthmaB

Hypertension due to Conn syndromeC

Hypertension due to phaeochromocytomaD

Hypertension with renal insufficiencyE

Page 383: Back to Filters (/Secure/TestMe/Filter ... - 1 File Download

8/11/2016 MyPastest

https://mypastest.pastest.com/Secure/TestMe/Browser/429893 2/2

46811

Hypertension with renal insufficiency (Option E) is incorrect. Alpha-blockers are often usedin renal failure but are still regarded as second-line to alternative agents including calcium-channel blockers and ACE inhibitors.

Next Question

Previous Question Tag Question Feedback End Review

Difficulty: Average

Peer Responses

Session Progress

0Responses Correct:

207Responses Incorrect:

207Responses Total:

0%Responses - % Correct:

Blog (https://www.pastest.com/blog) About Pastest (https://www.pastest.com/about-us)Contact Us (https://www.pastest.com/contact-us) Help (https://www.pastest.com/help)

© Pastest 2016

Page 384: Back to Filters (/Secure/TestMe/Filter ... - 1 File Download

8/11/2016 MyPastest

https://mypastest.pastest.com/Secure/TestMe/Browser/429893#Top 1/2

Back to Filters (/Secure/TestMe/Filter/429893/QA)

Question 13 of 207

A middle-aged man is rushed in an unconscious state to the Emergency Department. It isstated that he swallowed a large number of unknown tablets. Investigations reveal: Na 137mmol/l; K 3.5 mmol/l; Cl– 96 mmol/l; HCO3– 16 mmol/l; pH 7.25; anion gap 25 mmol/l.

What medicine is he most likely to have swallowed?

Explanation

The answer is Aspirin –

The anion gap represents those negative ions not normally measured in clinical practice,which include phosphate, sulfate, lactate, ketoacids and albumin. A number of formulaeexist for calculating the anion gap, eg:

Anion gap = plasma [Na+ + K+] - (plasma [Cl–] + plasma [HCO3–]) The normal range is8–16 mmol/l.

The anion gap is increased when excessive acid is added to the plasma, either bydisordered metabolism or by the addition of exogenous acid or when there is failure ofacid secretion. Salicylate poisoning is a known cause of an increased anion gap. Othercauses of an increased anion gap include dehydration, lactic acidosis, diabetic andalcoholic ketoacidosis, uraemia, poisoning by ethylene glycol and methanol.

Allopurinol (Option A) is incorrect. Allopurinol, benzylpenicillin and indometacin cause acuteinterstitial nephritis, which may cause metabolic acidosis but the anion gap will be normal.

Benzylpenicillin (Option C) is incorrect. Benzylpenicillin may rarely cause acute interstitialnephritis and metabolic acidosis, but the anion gap will be normal.

+

+

AllopurinolA

AspirinB

BenzylpenicillinC

DiclofenacD

d-PenicillamineE

Page 385: Back to Filters (/Secure/TestMe/Filter ... - 1 File Download

8/11/2016 MyPastest

https://mypastest.pastest.com/Secure/TestMe/Browser/429893#Top 2/2

46745

Diclofenac (Option D) is incorrect. Diclofenac may cause acute interstitial nephritis and acutetubular necrosis, which may be associated with metabolic acidosis, but the anion gap will benormal.

d-Penicillamine (Option E) is incorrect. d-Penicillamine may cause membranous nephropathy,which does not cause metabolic acidosis.

Next Question

Previous Question Tag Question Feedback End Review

Difficulty: Average

Peer Responses

Session Progress

0Responses Correct:

207Responses Incorrect:

207Responses Total:

0%Responses - % Correct:

Blog (https://www.pastest.com/blog) About Pastest (https://www.pastest.com/about-us)Contact Us (https://www.pastest.com/contact-us) Help (https://www.pastest.com/help)

© Pastest 2016

Page 386: Back to Filters (/Secure/TestMe/Filter ... - 1 File Download

8/11/2016 MyPastest

https://mypastest.pastest.com/Secure/TestMe/Browser/429893#Top 1/2

Back to Filters (/Secure/TestMe/Filter/429893/QA)

Question 14 of 207

A 67-year-old man presents with sudden-onset atrial fibrillation (ventricular rate of 150 bpm).His serum creatinine concentration is 250 μmol/l (70–110).

What is the main factor that determines the choice of loading dose of digoxin in this patient?

Explanation

The answer is Renal clearance –

Digoxin is water soluble and, as such, the major determinant of any loading dose is thepatient’s renal function. Effective plasma levels for digoxin are thought to be between 1and 2.6 nmol/l. In a patient with a creatinine of 250 μmol/l, adjustment of both theloading and maintenance doses is necessary because the normal half-life of 36 h isconsiderably lengthened and the drug may accumulate with repeated doseadministration. Digoxin binds to a subunit of the Na+/K+ ATPase pump on themembranes of cardiac myocytes. Adverse effects are increased in patients withconcomitant hypokalaemia.

Bioavailability (Option A) is incorrect. Drug absorption is likely to remain fairly normal, hencebioavailability is unchanged.

First-pass metabolism (Option B) is incorrect. Digoxin is not subject to liver metabolism, sofirst-pass metabolism is not relevant.

Plasma-binding proteins (Option C) is incorrect. Hypoalbuminaemia is a recognised featureof chronic kidney disease, but is a less important determinant of digoxin toxicity thanimpaired elimination.

BioavailabilityA

First-pass metabolismB

Plasma-binding proteinsC

Renal clearanceD

Volume of distributionE

Page 387: Back to Filters (/Secure/TestMe/Filter ... - 1 File Download

8/11/2016 MyPastest

https://mypastest.pastest.com/Secure/TestMe/Browser/429893#Top 2/2

46996

Volume of distribution (Option E) is incorrect. Volume of distribution is unlikely to besignificantly altered.

Next Question

Previous Question Tag Question Feedback End Review

Difficulty: Average

Peer Responses

Session Progress

0Responses Correct:

207Responses Incorrect:

207Responses Total:

0%Responses - % Correct:

Blog (https://www.pastest.com/blog) About Pastest (https://www.pastest.com/about-us)Contact Us (https://www.pastest.com/contact-us) Help (https://www.pastest.com/help)

© Pastest 2016

Page 388: Back to Filters (/Secure/TestMe/Filter ... - 1 File Download

8/11/2016 MyPastest

https://mypastest.pastest.com/Secure/TestMe/Browser/429893#Top 1/2

Back to Filters (/Secure/TestMe/Filter/429893/QA)

Question 15 of 207

20945

A 54-year-old woman with metastatic carcinoma of the breast presents to the clinic with verymuch worsening pain in her back and ribs; she has known bony metastases in the area.Current medication includes 100 mg of MST bd with top-up doses of 30 mg oral morphine asrequired. She also takes regular paracetamol.

Which one of the following additional steps would you take next to control her pain?

Explanation

Pain relief in metastatic disease

Given guidance on the multi-modal approach to pain relief, addition of a NSAID is the

most appropriate next step. They are thought to be particularly effective in managing

pain from bony metastases

Increasing morphine sulphate (MST) or beginning fentanyl patches is an option once

oral morphine top-ups are stable, but should not be attempted until pain is relatively

controlled, this is because of the risk of overdose due to variability in analgesic

requirements from day to day

Equally, a diamorphine syringe driver should only be commenced if the patient is

unable to take oral pain relief to manage pain

Depending on how well localised the metastases are, local radiotherapy may also be

highly effective in controlling pain

Increase dose/frequency of oral morphine top-upsA

Increase dose of MSTB

Continue oral morphine and prescribe diclofenacC

Start diamorphine syringe driverD

Start fentanyl patchesE

Page 389: Back to Filters (/Secure/TestMe/Filter ... - 1 File Download

8/11/2016 MyPastest

https://mypastest.pastest.com/Secure/TestMe/Browser/429893#Top 2/2

Next Question

Previous Question Tag Question Feedback End Review

Difficulty: Difficult

Peer Responses

Session Progress

0Responses Correct:

207Responses Incorrect:

207Responses Total:

0%Responses - % Correct:

Blog (https://www.pastest.com/blog) About Pastest (https://www.pastest.com/about-us)Contact Us (https://www.pastest.com/contact-us) Help (https://www.pastest.com/help)

© Pastest 2016

Page 390: Back to Filters (/Secure/TestMe/Filter ... - 1 File Download

8/11/2016 MyPastest

https://mypastest.pastest.com/Secure/TestMe/Browser/429893#Top 1/2

Back to Filters (/Secure/TestMe/Filter/429893/QA)

Question 16 of 207

A 12-year-old boy presents to an ‘Out of hours’ service with headache, drowsiness, fever andneck stiffness. The fundi appear normal. There is a purpuric rash on his body. Previousmedical history and family history are unremarkable. The GP phones you for advice as towhat he should do while awaiting urgent ambulance transfer to hospital.

What advice should be given?

Explanation

The answer is Give intramuscular benzylpenicillin –

The scenario is suggestive of meningitis caused by Neisseria meningitides. Initial ‘blind’treatment of suspected bacterial meningitis (especially meningococcal disease, as islikely given the rash) involves urgent transfer to hospital and parenteral benzylpenicillinbefore transfer (IV being preferable to intramuscular, if possible). Cefotaxime may be analternative in penicillin allergy, and chloramphenicol may be used if there is a history ofanaphylaxis to penicillin or cephalosporins.

In hospital the current guideline is to give IV cefotaxime pending confirmation of theorganism and antibiotic sensitivities; thereafter the regimen of choice for confirmedmeningococcal meningitis is intravenous benzylpenicillin, 2.4 g every 4 h for 5–7 days orIV cefotaxime.

Do nothing pending blood cultures and CSF analysis (Option A) is incorrect. Where there is anon-blanching rash, current guidelines advocate pre-hospital immediate antibioticadministration.

Do nothing pending blood cultures and CSF analysisA

Give bolus IV cefotaximeB

Give intramuscular benzylpenicillinC

Give single high-dose oral benzylpenicillinD

Give single high-dose oral cephalosporinE

Page 391: Back to Filters (/Secure/TestMe/Filter ... - 1 File Download

8/11/2016 MyPastest

https://mypastest.pastest.com/Secure/TestMe/Browser/429893#Top 2/2

46735

Give bolus IV cefotaxime (Option B) is incorrect. Although this may be effective, currentguidelines advocate benzylpenicillin as first-line therapy pre-hospital.

Give single high-dose oral benzylpenicillin (Option D) is incorrect. Benzylpenicillin isineffective when administered orally.

Give single high-dose oral cephalosporin (Option E) is incorrect. Oral antibiotics would beinappropriate owing to slower onset and lower bioavailability.

Next Question

Previous Question Tag Question Feedback End Review

Difficulty: Average

Peer Responses

Session Progress

0Responses Correct:

207Responses Incorrect:

207Responses Total:

0%Responses - % Correct:

Blog (https://www.pastest.com/blog) About Pastest (https://www.pastest.com/about-us)Contact Us (https://www.pastest.com/contact-us) Help (https://www.pastest.com/help)

© Pastest 2016

Page 392: Back to Filters (/Secure/TestMe/Filter ... - 1 File Download

8/11/2016 MyPastest

https://mypastest.pastest.com/Secure/TestMe/Browser/429893#Top 1/2

Back to Filters (/Secure/TestMe/Filter/429893/QA)

Question 17 of 207

46957

You are asked to add a new medication to the medication chart of a patient receivingwarfarin and you are concerned about a possible drug interaction.

Which one of the following drugs is least likely to induce cytochrome P450?

Explanation

The answer is Paracetamol –

Paracetamol is partly metabolised by the P450 system but it has no significant effect onenzyme activity.

Carbamazepine (Option A) is incorrect. Carbamazepine is a powerful inducer of hepaticenzyme activity.

Ethanol (Option B) is incorrect. Chronic alcohol intake causes upregulation of hepaticenzyme activity; acute intake may inhibit enzyme activity.

Phenytoin (Option D) is incorrect. Phenytoin is a hepatic enzyme inducer.

Rifampicin (Option E) is incorrect. Rifampicin is a powerful inducer of the P450 enzymesystem.

CarbamazepineA

EthanolB

ParacetamolC

PhenytoinD

RifampicinE

Next Question

Page 393: Back to Filters (/Secure/TestMe/Filter ... - 1 File Download

8/11/2016 MyPastest

https://mypastest.pastest.com/Secure/TestMe/Browser/429893#Top 2/2

Previous QuestionTag Question Feedback End Review

Difficulty: Average

Peer Responses

Session Progress

0Responses Correct:

207Responses Incorrect:

207Responses Total:

0%Responses - % Correct:

Blog (https://www.pastest.com/blog) About Pastest (https://www.pastest.com/about-us)Contact Us (https://www.pastest.com/contact-us) Help (https://www.pastest.com/help)

© Pastest 2016

Page 394: Back to Filters (/Secure/TestMe/Filter ... - 1 File Download

8/11/2016 MyPastest

https://mypastest.pastest.com/Secure/TestMe/Browser/429893#Top 1/2

Back to Filters (/Secure/TestMe/Filter/429893/QA)

Question 18 of 207

A 56-year-old man is entering his sixth year of combination antiretroviral treatment for HIVinfection. Over the past few years he has noticed increasing gynaecomastia and abdominalfat, and his partner has complained that he appears to be acquiring a ‘buffalo hump’. Routineclinical examination reveals a blood pressure of 160/85 mmHg, and a glucose concentrationof 16.1 mmol/l (3–6).

What diagnosis best fits with this clinical picture?

Explanation

The answer is Antiretroviral insulin-resistance syndrome –

Long-term use of combination antiretroviral therapy including protease inhibitorregimens is associated with a redistribution of body fat in some patients. HIVlipodystrophy follows an insulin-resistance pattern, with a loss of fat on the face,increasing abdominal fat and deposition of subcutaneous fat on the back. Other featuresof the insulin-resistance (metabolic syndrome) also occur, including low high-densitylipoprotein (HDL) cholesterol and high triglyceride levels, hypertension and impairedglucose tolerance, often preceding the development of type-2 diabetes. There is someevidence to suggest that insulin sensitisers (glitazones) may be an effective treatmentfor HIV lipodystrophy.

Cushing’s disease (Option B) is incorrect. The clinical features are strongly suggestive ofCushing syndrome but the history makes antiretroviral insulin-resistance syndrome a betteranswer. Cushing’s disease refers to functional adrenal adenoma.

Antiretroviral insulin-resistance syndromeA

Cushing’s diseaseB

Impaired glucose toleranceC

Metabolic syndromeD

Type-2 diabetesE

Page 395: Back to Filters (/Secure/TestMe/Filter ... - 1 File Download

8/11/2016 MyPastest

https://mypastest.pastest.com/Secure/TestMe/Browser/429893#Top 2/2

46884

Impaired glucose tolerance (Option C) is incorrect. The high blood glucose is more likely toindicate underlying type-2 diabetes than impaired glucose tolerance.

Metabolic syndrome (Option D) is incorrect. Although the patient’s features are consistentwith metabolic syndrome, this diagnosis alone would not explain the changes to breast andfat tissue.

Type-2 diabetes (Option E) is incorrect. Although the high blood glucose concentrationestablishes a diagnosis of type-2 diabetes, this only explains part of the clinical features andoffers a less complete answer than antiretroviral insulin-resistance syndrome.

Next Question

Previous Question Tag Question Feedback End Review

Difficulty: Average

Peer Responses

Session Progress

0Responses Correct:

207Responses Incorrect:

207Responses Total:

0%Responses - % Correct:

Blog (https://www.pastest.com/blog) About Pastest (https://www.pastest.com/about-us)Contact Us (https://www.pastest.com/contact-us) Help (https://www.pastest.com/help)

© Pastest 2016

Page 396: Back to Filters (/Secure/TestMe/Filter ... - 1 File Download

8/11/2016 MyPastest

https://mypastest.pastest.com/Secure/TestMe/Browser/429893#Top 1/2

Back to Filters (/Secure/TestMe/Filter/429893/QA)

Question 19 of 207

A 61-year-old man comes to the clinic for a check-up some 6 weeks after his inferiormyocardial infarction. Current medication includes ramipril, bisoprolol, aspirin andsimvastatin. He asks for advice about when the best time is to take his simvastatin.

At what time would simvastatin therapy be expected to give the greatest cholesterol-lowering effect?

Explanation

The answer is Last thing at night –

Simvastatin inhibits 3-hydroxy-3-methyl-glutaryl coA reductase (HMG-coA reductase).HMG-coA reductase activity is greatest at night in the fasting state and when liver bloodflow increases owing to resting in the supine position. Simvastatin works by inhibitingproduction of low-density lipoprotein (LDL) cholesterol by the liver.

After breakfast (Option A) is incorrect. Simvastatin may still inhibit cholesterol synthesis, butsynthetic activity is generally lower in the daytime in any case, suppressed by dietarycholesterol intake. Simvastatin is metabolised by CYP3A4, which is competitively inhibited bya number of substances, including grapefruit juice.

After evening meal (Option B) is incorrect. Simvastatin may still inhibit cholesterol synthesis,but synthetic activity is generally suppressed by dietary cholesterol intake.

First thing in the morning (Option C) is incorrect. Simvastatin may still inhibit cholesterolsynthesis, but synthetic activity is generally lower in the daytime in any case, suppressed bydietary cholesterol intake.

After breakfastA

After evening mealB

First thing in the morningC

Last thing at nightD

Timing doesn’t matterE

Page 397: Back to Filters (/Secure/TestMe/Filter ... - 1 File Download

8/11/2016 MyPastest

https://mypastest.pastest.com/Secure/TestMe/Browser/429893#Top 2/2

46995

Timing doesn’t matter (Option E) is incorrect. Timing at bedtime is likely to give a betteroverall reduction in circulating cholesterol concentrations; night-time simvastatin dosing wasused in the clinical trials that demonstrated efficacy in reducing cardiovascular events.

Next Question

Previous Question Tag Question Feedback End Review

Difficulty: Average

Peer Responses

Session Progress

0Responses Correct:

207Responses Incorrect:

207Responses Total:

0%Responses - % Correct:

Blog (https://www.pastest.com/blog) About Pastest (https://www.pastest.com/about-us)Contact Us (https://www.pastest.com/contact-us) Help (https://www.pastest.com/help)

© Pastest 2016

Page 398: Back to Filters (/Secure/TestMe/Filter ... - 1 File Download

8/11/2016 MyPastest

https://mypastest.pastest.com/Secure/TestMe/Browser/429893#Top 1/2

Back to Filters (/Secure/TestMe/Filter/429893/QA)

Question 20 of 207

A 67-year-old man is admitted to the Acute Medical Unit with breathlessness due topulmonary oedema. He has a background history of metastatic cancer and has previouslybeen treated with several cycles of chemotherapy.

Which one of the following cytotoxic agents is most frequently associated with thedevelopment of cardiotoxicity?

Explanation

The answer is Doxorubicin –

Anthracyclines (doxorubicin, daunorubicin) are particularly known to producecumulative, dose-dependent cardiotoxicity, which manifests as impaired left ventricularfunction and congestive cardiac failure.

Bleomycin (Option A) is incorrect. Bleomycin characteristically causes progressive interstitialpulmonary fibrosis with bibasal pulmonary infiltrate seen on chest radiography.

Cisplatin (Option B) is incorrect. cis-Platinum (cisplatin) is a heavy metal compound oftenused for testicular and ovarian tumours. It causes nephrotoxicity in 30% of patients due todamage to distal tubules and collecting ducts, which may lead to persistent magnesiumwasting.

Cyclophosphamide (Option C) is incorrect. Cyclophosphamide is a prodrug of an alkylatingagent acrolein; it is metabolised in the liver to acrolein, which causes haemorrhagic cystitis.

Vincristine (Option E) is incorrect. Vincristine is a vinca alkaloid which, when used on aweekly basis, carries a risk of progressive peripheral neuropathy. After treatment

BleomycinA

CisplatinB

CyclophosphamideC

DoxorubicinD

VincristineE

Page 399: Back to Filters (/Secure/TestMe/Filter ... - 1 File Download

8/11/2016 MyPastest

https://mypastest.pastest.com/Secure/TestMe/Browser/429893#Top 2/2

46632

discontinuation, there may be some degree of recovery, especially improvement of sensorychanges rather than motor neuropathy.

Next Question

Previous Question Tag Question Feedback End Review

Difficulty: Average

Peer Responses

Session Progress

0Responses Correct:

207Responses Incorrect:

207Responses Total:

0%Responses - % Correct:

Blog (https://www.pastest.com/blog) About Pastest (https://www.pastest.com/about-us)Contact Us (https://www.pastest.com/contact-us) Help (https://www.pastest.com/help)

© Pastest 2016

Page 400: Back to Filters (/Secure/TestMe/Filter ... - 1 File Download

8/11/2016 MyPastest

https://mypastest.pastest.com/Secure/TestMe/Browser/429893#Top 1/2

Back to Filters (/Secure/TestMe/Filter/429893/QA)

Question 21 of 207

A 17-year-old male is admitted to the Emergency Department. He volunteers that he hastaken some 40 paracetamol tablets around 5 h earlier, after splitting up from his girlfriend.His paracetamol levels are in the range for N-acetylcysteine treatment and you commencethis.

Which one of the following is the main mechanism by which N-acetylcysteine prevents liverdamage?

Explanation

The answer is It replenishes glutathione –

The major mode of action of N-acetylcysteine is thought to be as a precursor forglutathione, which acts as a sulfhydryl donor to allow conjugation with N-acetyl-p-benzoquinone imine (NAPQI), the toxic metabolite of paracetamol. Patients with proteinmalnutrition who have fewer sulfhydryl groups available for conjugation or those onhepatic enzyme inducers such as alcohol are at higher risk from paracetamol toxicity.

It depletes cysteine (Option A) is incorrect. It increases intracellular cysteine.

It depletes glutathione (Option B) is incorrect. It increases intracellular stores.

It replenishes cysteine (Option C) is incorrect. Acetylcysteine enters hepatocytes and isconverted to cysteine, so that treatment replenishes cysteine concentrations and allowsrestoration of glutathione concentrations. Replenishment of glutathione is the preferredanswer because it is the glutathione depletion that permits liver injury and the therapeuticeffect is from glutathione replenishment.

It depletes cysteineA

It depletes glutathioneB

It replenishes cysteineC

It replenishes glutathioneD

It replenishes lysineE

Page 401: Back to Filters (/Secure/TestMe/Filter ... - 1 File Download

8/11/2016 MyPastest

https://mypastest.pastest.com/Secure/TestMe/Browser/429893#Top 2/2

46974

It replenishes lysine (Option E) is incorrect. There is no direct effect on lysine concentrations.

Next Question

Previous Question Tag Question Feedback End Review

Difficulty: Average

Peer Responses

Session Progress

0Responses Correct:

207Responses Incorrect:

207Responses Total:

0%Responses - % Correct:

Blog (https://www.pastest.com/blog) About Pastest (https://www.pastest.com/about-us)Contact Us (https://www.pastest.com/contact-us) Help (https://www.pastest.com/help)

© Pastest 2016

Page 402: Back to Filters (/Secure/TestMe/Filter ... - 1 File Download

8/11/2016 MyPastest

https://mypastest.pastest.com/Secure/TestMe/Browser/429893#Top 1/2

Back to Filters (/Secure/TestMe/Filter/429893/QA)

Question 22 of 207

You are treating a 56-year-old man with diabetes and hypertension and trying to optimise hismedication list.

Which of the following drugs is most likely to produce large clinical effects for a relativelysmall change in dose?

Explanation

The answer is Furosemide –

The dose–response curve for loop diuretics such as furosemide is progressive over abroad range of dosages, indicating that changes in dose are expected to alter clinicaldrug effects. This is in contrast to other medications where a flat dose–responserelationship exists, and modest changes in drug dose have negligible effects of drugresponse, eg thiazide diuretics, ACE inhibitors.

Bendrofluazide (Option A) is incorrect. The dose–response relationship for thiazides andblood pressure lowering reaches a plateau after the use of relatively low doses. In contrast,there is a linear relationship between thiazide dose and adverse effects, includinghypokalaemia. Therefore, increasing the dose of bendroflumethiazide beyond 2.5 mg dailyoffers little therapeutic benefit but will increase the risk of adverse effects.

Hydrochlorothiazide (Option C) is incorrect. The dose–response relationship for thiazides andblood pressure lowering reaches a plateau after the use of relatively low doses. In contrast,there is a linear relationship between thiazide dose and adverse effects, includinghypokalaemia. Therefore, increasing the drug dose offers little therapeutic benefit but willincrease the risk of adverse effects.

BendrofluazideA

FurosemideB

HydrochlorothiazideC

LosartanD

PioglitazoneE

Page 403: Back to Filters (/Secure/TestMe/Filter ... - 1 File Download

8/11/2016 MyPastest

https://mypastest.pastest.com/Secure/TestMe/Browser/429893#Top 2/2

46853

Losartan (Option D) is incorrect. ACE inhibitors and angiotensin receptor blockers are knownto observe a comparatively flat dose–response relationship, and doses higher than thetherapeutic range offer little additional blood pressure lowering effect but increase the riskof nephrotoxicity.

Pioglitazone (Option E) is incorrect. The dose–response curve reaches a plateau forpioglitazone at above 45 mg; there would be negligible additional blood glucose controlachieved by administered doses higher than the licensed dose range of 30–45 mg.

Next Question

Previous Question Tag Question Feedback End Review

Difficulty: Average

Peer Responses

Session Progress

0Responses Correct:

207Responses Incorrect:

207Responses Total:

0%Responses - % Correct:

Blog (https://www.pastest.com/blog) About Pastest (https://www.pastest.com/about-us)Contact Us (https://www.pastest.com/contact-us) Help (https://www.pastest.com/help)

© Pastest 2016

Page 404: Back to Filters (/Secure/TestMe/Filter ... - 1 File Download

8/11/2016 MyPastest

https://mypastest.pastest.com/Secure/TestMe/Browser/429893#Top 1/2

Back to Filters (/Secure/TestMe/Filter/429893/QA)

Question 23 of 207

46988

A 53-year-old man is admitted in an intoxicated state having drunk a large quantity ofmethanol. His investigations show that the osmolal gap is raised, and there is a metabolicacidosis. You decide to administer fomepizole antidote.

When treating a methanol overdose with fomepizole, which of the following describes thepharmacological action of fomepizole?

Explanation

The answer is Competitive inhibition –

Methanol is metabolised by alcohol dehydrogenase to formaldehyde and thence toformic acid, which leads to profound acidosis, renal failure and blindness. Fomepizole (4-methylpyrazole) is a competitive inhibitor of alcohol dehydrogenase and so is used totreat methanol and ethylene glycol poisoning. Ethanol also competes for metabolism byalcohol dehydrogenase and can lessen the extent of metabolism of methanol or ethyleneglycol; however, fomepizole is more effective and less toxic.

Competitive agonism (Option A) is incorrect. Fomepizole is a competitive inhibitor.

Non-competitive agonism (Option C) is incorrect. Fomepizole is a competitive inhibitor.

Non-competitive inhibition (Option D) is incorrect. Fomepizole is a competitive inhibitor.

Partial agonism (Option E) is incorrect. Fomepizole is a competitive inhibitor.

Competitive agonismA

Competitive inhibitionB

Non-competitive agonismC

Non-competitive inhibitionD

Partial agonismE

Next Question

Page 405: Back to Filters (/Secure/TestMe/Filter ... - 1 File Download

8/11/2016 MyPastest

https://mypastest.pastest.com/Secure/TestMe/Browser/429893#Top 2/2

Previous Question Tag Question Feedback End Review

Difficulty: Average

Peer Responses

Session Progress

0Responses Correct:

207Responses Incorrect:

207Responses Total:

0%Responses - % Correct:

Blog (https://www.pastest.com/blog) About Pastest (https://www.pastest.com/about-us)Contact Us (https://www.pastest.com/contact-us) Help (https://www.pastest.com/help)

© Pastest 2016

Page 406: Back to Filters (/Secure/TestMe/Filter ... - 1 File Download

8/11/2016 MyPastest

https://mypastest.pastest.com/Secure/TestMe/Browser/429893#Top 1/2

Back to Filters (/Secure/TestMe/Filter/429893/QA)

Question 24 of 207

46753

A child is born with an open spina bifida despite adequate folate intake by the mother duringpregnancy. The mother is a known epileptic and had elected to continue her routineantiepileptic medications during pregnancy, after the pros and cons had been discussed.

Which one of the following drugs would be most likely to cause this teratogenic effect?

Explanation

The answer is Sodium valproate –

Administration of sodium valproate during pregnancy may predispose to thedevelopment of neural tube defects in the fetus, including spina bifida.

Carbamazepine (Option A) is incorrect. Carbamazepine may also cause spina bifida as ateratogenic effect, although less commonly than sodium valproate, hence option E is thepreferred option.

Phenobarbital (Option B) is incorrect. Phenobarbital causes withdrawal symptoms in thenewborn; spina bifida is not a recognised adverse outcome.

Phenytoin (Option C) is incorrect. Phenytoin causes the fetal hydantoin syndrome with facialdysmorphism.

Primidone (Option D) is incorrect. Primidone causes withdrawal symptoms in the newborn;spina bifida is not a recognised adverse outcome.

CarbamazepineA

PhenobarbitalB

PhenytoinC

PrimidoneD

Sodium valproateE

Next Question

Page 407: Back to Filters (/Secure/TestMe/Filter ... - 1 File Download

8/11/2016 MyPastest

https://mypastest.pastest.com/Secure/TestMe/Browser/429893#Top 2/2

Previous Question Tag Question Feedback End Review

Difficulty: Average

Peer Responses

Session Progress

0Responses Correct:

207Responses Incorrect:

207Responses Total:

0%Responses - % Correct:

Blog (https://www.pastest.com/blog) About Pastest (https://www.pastest.com/about-us)Contact Us (https://www.pastest.com/contact-us) Help (https://www.pastest.com/help)

© Pastest 2016

Page 408: Back to Filters (/Secure/TestMe/Filter ... - 1 File Download

8/11/2016 MyPastest

https://mypastest.pastest.com/Secure/TestMe/Browser/429893#Top 1/2

Back to Filters (/Secure/TestMe/Filter/429893/QA)

Question 25 of 207

An 18-year-old boy is brought by ambulance to the Emergency Department. He has had a rowwith his father who found him in his bedroom some 2 h later in an unrousable state. It isknown that his father takes tablets for blood pressure. On examination the patient has a pulseof 42 bpm and a blood pressure of 74/40 mmHg.

Which of the following is the most appropriate treatment for this patient?

Explanation

The answer is Intravenous glucagon –

The prominent features in the scenario are bradycardia and hypotension, suggesting a β-blocker overdose. Where there is profound hypotension, as in this case, treatment is withintravenous glucagon, bolus 10 mg, followed by intravenous infusion. Glucagon exerts aninotropic effect independent of β-receptor activation and raises myocardial cAMP levels.Bradycardia may be corrected by glucagon; other treatments include atropine,isoprenaline infusion and insulin. Hypoglycaemia may also occur in association with β-blocker overdose – bolus doses of intravenous 50% dextrose followed by 10% dextroseinfusion is the management of choice.

External pacing (Option A) is incorrect. External pacing would only be considered if thepatient fails to respond to glucagon and other medical therapies.

Isoprenaline infusion (Option C) is incorrect. Intravenous isoprenaline infusion may beconsidered as a means of correcting bradycardia, normally only when patients have failed torespond to glucagon.

External pacingA

Intravenous glucagonB

Isoprenaline infusionC

Repeated small doses of adrenalineD

Temporary pacing wireE

Page 409: Back to Filters (/Secure/TestMe/Filter ... - 1 File Download

8/11/2016 MyPastest

https://mypastest.pastest.com/Secure/TestMe/Browser/429893#Top 2/2

46951

Repeated small doses of adrenaline (Option D) is incorrect. Small doses of adrenaline are lesslikely to be successful than glucagon, given that the β-receptors often become irreversiblyblocked in the setting of β-blocker overdose.

Temporary pacing wire (Option E) is incorrect. A temporary pacing wire may be neededwhen glucagon and other medical treatments fail.

Next Question

Previous Question Tag Question Feedback End Review

Difficulty: Average

Peer Responses

Session Progress

0Responses Correct:

207Responses Incorrect:

207Responses Total:

0%Responses - % Correct:

Blog (https://www.pastest.com/blog) About Pastest (https://www.pastest.com/about-us)Contact Us (https://www.pastest.com/contact-us) Help (https://www.pastest.com/help)

© Pastest 2016

Page 410: Back to Filters (/Secure/TestMe/Filter ... - 1 File Download

8/11/2016 MyPastest

https://mypastest.pastest.com/Secure/TestMe/Browser/429893#Top 1/2

Back to Filters (/Secure/TestMe/Filter/429893/QA)

Question 26 of 207

46907

A 60-year-old lady presents with a 5-day history of malaise, mild jaundice and abdominaldiscomfort. Transaminases are markedly raised and you diagnose a possible drug-inducedhepatitis.

Which of the following drugs would be most likely to be responsible?

Explanation

The answer is Amiodarone –

Drug-induced hepatitis may occur after high drug doses, eg azathioprine, isoniazid andpyrazinamide (note that it is the metabolite of isoniazid that causes hepatitis, and it ismore likely to occur in patients that have a slow acetylator genotype). In some cases theoccurrence of hepatitis is less clearly related to dose, eg amiodarone or nifedipine.Methyldopa may cause an autoimmune hepatitis.

Chlorpromazine (Option B) is incorrect. Chlorpromazine causes cholestatic jaundice ratherthan drug-induced hepatitis.

Ethinylestradiol (Option C) is incorrect. Oestrogens may interfere with bilirubin excretiongiving rise to jaundice without hepatitis.

Isoprenaline (Option D) is incorrect. Isoprenaline is not known to cause hepatitis.

Tetracyline (Option E) is incorrect. Tetracyclines may cause dose-dependent fatty change inthe liver.

AmiodaroneA

ChlorpromazineB

EthinylestradiolC

IsoprenalineD

TetracylineE

Page 411: Back to Filters (/Secure/TestMe/Filter ... - 1 File Download

8/11/2016 MyPastest

https://mypastest.pastest.com/Secure/TestMe/Browser/429893#Top 2/2

Next Question

Previous Question Tag Question Feedback End Review

Difficulty: Average

Peer Responses

Session Progress

0Responses Correct:

207Responses Incorrect:

207Responses Total:

0%Responses - % Correct:

Blog (https://www.pastest.com/blog) About Pastest (https://www.pastest.com/about-us)Contact Us (https://www.pastest.com/contact-us) Help (https://www.pastest.com/help)

© Pastest 2016

Page 412: Back to Filters (/Secure/TestMe/Filter ... - 1 File Download

8/11/2016 MyPastest

https://mypastest.pastest.com/Secure/TestMe/Browser/429893#Top 1/2

Back to Filters (/Secure/TestMe/Filter/429893/QA)

Question 27 of 207

46626

The blood test results of one of your clinic patients show serum potassium 3.4 mmol/l andserum bicarbonate 15 mmol/l.

Which one of the following diuretics is most likely to account for these biochemical findings?

Explanation

The answer is Acetazolamide –

The prominent finding is a metabolic acidosis, and very borderline hypokalaemia.Acetazolamide (carbonic anhydrase inhibitor) inhibits proximal tubule bicarbonatereabsorption in a similar fashion to type-2 renal tubular acidosis (RTA). In contrast, loopdiuretics and thiazide diuretics are associated with significant renal potassium andhydrogen ion loss, associated with a metabolic alkalosis.

Amiloride (Option B) is incorrect. Amiloride may also cause a metabolic acidosis byinhibiting the sodium channel in the collecting duct, which inhibits renal acid secretion orbicarbonate reabsorption; however, the metabolic acidosis is usually less prominent thanacetazolamide, which is why acetazolamide is the preferred answer.

Bumetanide (Option C) is incorrect. Loop diuretics promote sodium chloride loss and causevolume depletion, and are characteristically associated with metabolic alkalosis.

Furosemide (Option D) is incorrect. Loop diuretics promote sodium chloride loss and causevolume depletion, and are characteristically associated with metabolic alkalosis.

Metolazone (Option E) is incorrect. Metolazone produces significant hypokalaemia, and tendsto cause metabolic acidosis.

AcetazolamideA

AmilorideB

BumetanideC

FurosemideD

MetolazoneE

Page 413: Back to Filters (/Secure/TestMe/Filter ... - 1 File Download

8/11/2016 MyPastest

https://mypastest.pastest.com/Secure/TestMe/Browser/429893#Top 2/2

46626

Next Question

Previous Question Tag Question Feedback End Review

Difficulty: Average

Peer Responses

Session Progress

0Responses Correct:

207Responses Incorrect:

207Responses Total:

0%Responses - % Correct:

Blog (https://www.pastest.com/blog) About Pastest (https://www.pastest.com/about-us)Contact Us (https://www.pastest.com/contact-us) Help (https://www.pastest.com/help)

© Pastest 2016

Page 414: Back to Filters (/Secure/TestMe/Filter ... - 1 File Download

8/11/2016 MyPastest

https://mypastest.pastest.com/Secure/TestMe/Browser/429893#Top 1/2

Back to Filters (/Secure/TestMe/Filter/429893/QA)

Question 28 of 207

46970

You see a 53-year-old woman in the endocrinology clinic who is receiving treatment forGrave’s disease. She is concerned about the possible adverse effects of her medications.

Which one of the following is most likely to have been caused by carbimazole?

Explanation

The answer is Alopecia –

Carbimazole is used to treat hyperthyroidism. Its side-effects include nausea, rash,pruritus, arthralgia, alopecia, agranulocytosis and jaundice. Patients taking carbimazolemust be warned of the danger of neutropaenia and are advised to seek medical attentionquickly if they develop a sore throat or other symptoms of infection.

Acne (Option A) is incorrect. Acne is a recognised complication of phenytoin and certainhormone modulators.

Aplastic anaemia (Option C) is incorrect. Agranulocytosis may occur, but aplastic anaemia isnot a characteristic adverse effect.

Hypertrichosis (Option D) is incorrect. Hypertrichosis is a recognised adverse effect ofminoxidil and ciclosporin.

Tremor (Option E) is incorrect. Tremor is a characteristic adverse effect of excess thyroidhormone, β -adrenoceptor agonists (e.g. salbutamol), theophylline and lithium.

AcneA

AlopeciaB

Aplastic anaemiaC

HypertrichosisD

TremorE

2

Next Question

Page 415: Back to Filters (/Secure/TestMe/Filter ... - 1 File Download

8/11/2016 MyPastest

https://mypastest.pastest.com/Secure/TestMe/Browser/429893#Top 2/2

Previous Question Tag Question Feedback End Review

Difficulty: Average

Peer Responses

Session Progress

0Responses Correct:

207Responses Incorrect:

207Responses Total:

0%Responses - % Correct:

Blog (https://www.pastest.com/blog) About Pastest (https://www.pastest.com/about-us)Contact Us (https://www.pastest.com/contact-us) Help (https://www.pastest.com/help)

© Pastest 2016

Page 416: Back to Filters (/Secure/TestMe/Filter ... - 1 File Download

8/11/2016 MyPastest

https://mypastest.pastest.com/Secure/TestMe/Browser/429893#Top 1/2

Back to Filters (/Secure/TestMe/Filter/429893/QA)

Question 29 of 207

46956

When administering narcotic analgesics to patients with renal failure, which one of thefollowing is true?

Explanation

The answer is Prolonged elimination half-lives of both parent compound and metabolitescontribute to an increase in the risk of adverse effects –

Opiates and their metabolites have prolonged elimination half-lives and accumulate inrenal failure causing adverse effects. They should be used with caution, and the dosereduced and/or dosing interval increased.

Opiates are safe and no dosage adjustment is necessary (Option A) is incorrect. Adjustmentof dose and/or frequency is often needed.

Opiate metabolites are excreted in the same way as in a patient with normal renal function(Option B) is incorrect. Opioid metabolites, eg morphine-6-sulfate, may accumulate inpatients with renal impairment.

Therapeutic monitoring of morphine levels is commonly performed (Option D) is incorrect.Blood concentrations of drug and metabolites correlate poorly with clinical features ofopioid toxicity, so that therapeutic drug monitoring is inappropriate.

Use of opiates is absolutely contraindicated (Option E) is incorrect. Opioids may be used, butwith increased caution.

Opiates are safe and no dosage adjustment is necessaryA

Opiate metabolites are excreted in the same way as in a patient with normal renalfunction

B

Prolonged elimination half-lives of both parent compound and metabolitescontribute to an increase in the risk of adverse effects

C

Therapeutic monitoring of morphine levels is commonly performedD

Use of opiates is absolutely contraindicatedE

Page 417: Back to Filters (/Secure/TestMe/Filter ... - 1 File Download

8/11/2016 MyPastest

https://mypastest.pastest.com/Secure/TestMe/Browser/429893#Top 2/2

46956

Next Question

Previous Question Tag Question Feedback End Review

Difficulty: Average

Peer Responses

Session Progress

0Responses Correct:

207Responses Incorrect:

207Responses Total:

0%Responses - % Correct:

Blog (https://www.pastest.com/blog) About Pastest (https://www.pastest.com/about-us)Contact Us (https://www.pastest.com/contact-us) Help (https://www.pastest.com/help)

© Pastest 2016

Page 418: Back to Filters (/Secure/TestMe/Filter ... - 1 File Download

8/11/2016 MyPastest

https://mypastest.pastest.com/Secure/TestMe/Browser/429893#Top 1/2

Back to Filters (/Secure/TestMe/Filter/429893/QA)

Question 30 of 207

You are asked to review a 59-year-old man who has been referred to the hypertension clinicfor advice. He had attended the Emergency Department 2 weeks earlier due to a suddenattack of flushing, swelling of his tongue and airway, and hypotension. He has a history ofhypertension for which he takes amlodipine and enalapril.

What would be the best long-term course of action?

Explanation

The answer is Stop his enalapril tablets and substitute another antihypertensive agent -

Enalapril and other angiotensin converting enzyme (ACE) inhibitors, and angiotensin-II-receptor blockers are associated with the occurrence of angioedema, characterised bysweating, flushing, hypotension, facial oedema, airway oedema and respiratory distress.Treatment involves discontinuation of ACE inhibitor therapy.

Add regular treatment with an H1 blocker (Option A) is incorrect. The causative agent mustbe stopped, which takes priority over other treatments, including antihistamines for ongoingsymptoms.

Add regular treatment with an H2 blocker (Option B) is incorrect. The causative agent mustbe stopped, which takes priority over other treatments, including antihistamines for ongoingsymptoms.

Stop his amlodipine tablets and substitute another antihypertensive agent (Option C) isincorrect. Although angioedema may also be seen with a range of other drugs, including incase reports with calcium antagonists, the frequency is much higher with agents working viathe ACE pathway.

Add regular treatment with an H1 blockerA

Add regular treatment with an H2 blockerB

Stop his amlodipine tablets and substitute another antihypertensive agentC

Stop his enalapril tablets and substitute another antihypertensive agentD

Train him how to use an EpiPenE

Page 419: Back to Filters (/Secure/TestMe/Filter ... - 1 File Download

8/11/2016 MyPastest

https://mypastest.pastest.com/Secure/TestMe/Browser/429893#Top 2/2

46838

Train him how to use an EpiPen (Option E) is incorrect. ACE-inhibitor-induced angioedema isunlikely to recur provided that the patient avoids any further exposure to ACE inhibitors orangiotensin receptor blockers.

Next Question

Previous Question Tag Question Feedback End Review

Difficulty: Average

Peer Responses

Session Progress

0Responses Correct:

207Responses Incorrect:

207Responses Total:

0%Responses - % Correct:

Blog (https://www.pastest.com/blog) About Pastest (https://www.pastest.com/about-us)Contact Us (https://www.pastest.com/contact-us) Help (https://www.pastest.com/help)

© Pastest 2016

Page 420: Back to Filters (/Secure/TestMe/Filter ... - 1 File Download

8/11/2016 MyPastest

https://mypastest.pastest.com/Secure/TestMe/Browser/429893#Top 1/2

Back to Filters (/Secure/TestMe/Filter/429893/QA)

Question 31 of 207

You are asked to see a 78-year-old man, a nursing-home resident who has recently movedinto the care home due to progressive Alzheimer’s disease. He has had several subacuteconfusional episodes since his arrival, for which the duty GP has been called twice in the pastmonth, and he has been prescribed an antipsychotic to reduce his agitation. Past history ofnote includes previous alcoholism and an episode of biliary colic many years ago. He reportsno abdominal pain. On examination he is deeply jaundiced. On blood testing, his alkalinephosphatase activity and bilirubin concentration are markedly raised.

What diagnosis fits best with this clinical picture?

Explanation

The answer is Phenothiazine-related hepatotoxicity –

Adverse drug effects Phenothiazines such as chlorpromazine can producehepatocanalicular cholestasis (cholestatic jaundice) owing to a hypersensitivity reaction.It usually occurs within 1 month of starting the drug, in around 1% of patients. Otherdrugs that cause similar pathophysiology include erythromycin, cimetidine, ciclosporin,nifedipine, nitrofurantoin, imipramine, azathioprine and dextropropoxyphene. Hepaticadenomas, or even hepatocellular carcinoma, may occur in cases of prolonged use ofdanazol or high-hormone-content combined oral contraceptive medication.

Acute pancreatitis (Option A) is incorrect. The absence of abdominal pain or fever makes thisdiagnosis unlikely.

Cholecystitis (Option B) is incorrect. The absence of abdominal pain or fever makes thisdiagnosis unlikely.

Acute pancreatitisA

CholecystitisB

Pancreatic carcinomaC

Phenothiazine-related hepatotoxicityD

Renewed excess alcohol consumptionE

Page 421: Back to Filters (/Secure/TestMe/Filter ... - 1 File Download

8/11/2016 MyPastest

https://mypastest.pastest.com/Secure/TestMe/Browser/429893#Top 2/2

46878

Pancreatic carcinoma (Option C) is incorrect. Pancreatic cancer may cause obstructivejaundice, which is typically painless. Although the diagnosis is possible, it is uncommon andless likely than phenothiazine-related hepatotoxicity.

Renewed excess alcohol consumption (Option E) is incorrect. Care home residents maycontinue to drink alcohol, but alcoholic hepatitis would be less likely given the absence ofany other signs of liver disease.

Next Question

Previous Question Tag Question Feedback End Review

Difficulty: Average

Peer Responses

Session Progress

0Responses Correct:

207Responses Incorrect:

207Responses Total:

0%Responses - % Correct:

Blog (https://www.pastest.com/blog) About Pastest (https://www.pastest.com/about-us)Contact Us (https://www.pastest.com/contact-us) Help (https://www.pastest.com/help)

© Pastest 2016

Page 422: Back to Filters (/Secure/TestMe/Filter ... - 1 File Download

8/11/2016 MyPastest

https://mypastest.pastest.com/Secure/TestMe/Browser/429893#Top 1/2

Back to Filters (/Secure/TestMe/Filter/429893/QA)

Question 32 of 207

You review a 68-year-old man in the Outpatient Neurology Clinic who has been referred witha diagnosis of suspected Parkinson’s disease. After discussion with the patient and his family,you decide to initiate levodopa therapy.

Which one of the following statements best describes the pharmacological effects oflevodopa?

Explanation

The answer is Causes postural hypotension due to direct vascular effect -

Levodopa is a pro-drug that crosses the blood–brain barrier and is then converted todopamine. The direct effects of circulating levodopa and dopamine include nausea,arrhythmias and postural hypotension.

Can be given subcutaneously (Option A) is incorrect. Levodopa is available only in oralformulations. Apomorphine is administered subcutaneously in severe Parkinson’s disease.

Co-administration of benserazide increases the central nervous system (CNS) levels ofdopamine by inhibiting dopa-decarboxylase in the substantia nigra (Option C) is incorrect.Benserazide is a peripheral dopa-decarboxylase inhibitor that permits higher quantities oflevodopa to cross the blood–brain barrier; benserazide does not inhibit dopa-decarboxylaseactivity within the CNS.

It is an enzyme inducer (Option D) is incorrect. Levodopa does not have a direct effect onhepatic enzyme activity.

Can be given subcutaneouslyA

Causes postural hypotension due to direct vascular effectB

Co-administration of benserazide increases the central nervous system levels ofdopamine by inhibiting dopa-decarboxylase in the substantia nigra

C

It is an enzyme inducerD

Undergoes metabolism in the central but not the peripheral nervous systemE

Page 423: Back to Filters (/Secure/TestMe/Filter ... - 1 File Download

8/11/2016 MyPastest

https://mypastest.pastest.com/Secure/TestMe/Browser/429893#Top 2/2

46958

Undergoes metabolism in the central but not the peripheral nervous system (Option E) isincorrect. Levodopa is metabolised to dopa in the central and peripheral nervous system; theintended therapeutic effect requires metabolism in the central nervous system. Peripheralnervous system effects mediate adverse effects.

Next Question

Previous Question Tag Question Feedback End Review

Difficulty: Average

Peer Responses

Session Progress

0Responses Correct:

207Responses Incorrect:

207Responses Total:

0%Responses - % Correct:

Blog (https://www.pastest.com/blog) About Pastest (https://www.pastest.com/about-us)Contact Us (https://www.pastest.com/contact-us) Help (https://www.pastest.com/help)

© Pastest 2016

Page 424: Back to Filters (/Secure/TestMe/Filter ... - 1 File Download

8/11/2016 MyPastest

https://mypastest.pastest.com/Secure/TestMe/Browser/429893#Top 1/2

Back to Filters (/Secure/TestMe/Filter/429893/QA)

Question 33 of 207

40140

A 42-year-old patient who received a renal transplant some 4 months earlier comes to theEmergency Department complaining of nausea, anorexia and lethargy, which have increasedover the past 2 weeks since a new medication was started for control of palpitations andhypertension. She is taking a Ciclosporin based immunosuppressive regimen. On examinationher BP is 123/82 mmHg; pulse is 80/min and regular. Her BMI is 22. There is an abdominal scarcovering the recent transplant. Blood work reveals a 30% rise in creatinine over the past 2weeks, and Ciclosporin above the upper limit of the normal range.

Which of the following is the most likely cause?

Explanation

The answer is Verapamil -

It is important to be cautious with respect to introducing inhibitors of CYP 3A4 in conjunctionwith Ciclosporin. Inhibitors of 3A4 where significant caution is advised include Verapamil,Diltiazem, Imidazole anti-fungals, and macrolide antibiotics. Amlodipine is a substrate of 3A4,rather than being a potent inhibitor or inducer. Bisoprolol undergoes dual P450 metabolismvia 2D6 and 3A4. Ramipril is metabolised to an active metabolite, ramiliprat, by the liver, butdoes not lead to increased Ciclosporin levels. CYP 2C9 is the P450 enzyme most involved inthe metabolism of Valsartan.

target="_blank">http://www.medicines.org.uk/EMC/medicine/1307/SPC/Neoral+Soft+Gelatin+Capsules,+Neoral+Oral+Solution/(http://www.medicines.org.uk/EMC/medicine/1307/SPC/Neoral+Soft+Gelatin+Capsules,+Neoral+Oral+Solution/#INTERACTION

AmlodipineA

BisoprololB

RamiprilC

ValsartanD

VerapamilE

Next Question

Previous Question Tag Question Feedback End Review

Difficulty: Difficult

Peer Responses

Session Progress

Page 425: Back to Filters (/Secure/TestMe/Filter ... - 1 File Download

8/11/2016 MyPastest

https://mypastest.pastest.com/Secure/TestMe/Browser/429893#Top 2/2

0Responses Correct:

207Responses Incorrect:

207Responses Total:

0%Responses - % Correct:

Blog (https://www.pastest.com/blog) About Pastest (https://www.pastest.com/about-us)Contact Us (https://www.pastest.com/contact-us) Help (https://www.pastest.com/help)

© Pastest 2016

Page 426: Back to Filters (/Secure/TestMe/Filter ... - 1 File Download

8/11/2016 MyPastest

https://mypastest.pastest.com/Secure/TestMe/Browser/429893#Top 1/2

Back to Filters (/Secure/TestMe/Filter/429893/QA)

Question 34 of 207

46867

A patient with a history of angina is being investigated for dyspnoea. Blood tests confirmhaemolytic anaemia and a peripheral smear shows the presence of Heinz bodies andmethaemoglobinaemia.

Which of the following medications is most likely responsible for this complication?

Explanation

The answer is Isosorbine mononitrate –

Methaemoglobinaemia results from the oxidation of ferrous iron in the haemoglobin tothe ferric form; it is associated with precipitation of ferric iron as Heinz bodies, andeventually leads to haemolytic anaemia. Nitrates may cause this reaction, including amylnitrate.

Amlodipine (Option A) is incorrect. Amlodipine is not associated withmethaemoglobinaemia.

Aspirin (Option B) is incorrect. Aspirin is not associated with methaemoglobinaemia.

Metoprolol (Option D) is incorrect. Metoprolol is not associated with methaemoglobinaemia.

Verapamil (Option E) is incorrect. Verapamil is not associated with methaemoglobinaemia.

AmlodipineA

AspirinB

Isosorbide mononitrateC

MetoprololD

VerapamilE

Next Question

Page 427: Back to Filters (/Secure/TestMe/Filter ... - 1 File Download

8/11/2016 MyPastest

https://mypastest.pastest.com/Secure/TestMe/Browser/429893#Top 2/2

Previous Question Tag Question Feedback End Review

Difficulty: Average

Peer Responses

Session Progress

0Responses Correct:

207Responses Incorrect:

207Responses Total:

0%Responses - % Correct:

Blog (https://www.pastest.com/blog) About Pastest (https://www.pastest.com/about-us)Contact Us (https://www.pastest.com/contact-us) Help (https://www.pastest.com/help)

© Pastest 2016

Page 428: Back to Filters (/Secure/TestMe/Filter ... - 1 File Download

8/11/2016 MyPastest

https://mypastest.pastest.com/Secure/TestMe/Browser/429893#Top 1/2

Back to Filters (/Secure/TestMe/Filter/429893/QA)

Question 35 of 207

46741

An elderly, normotensive man with poor left ventricular function presents to the EmergencyDepartment with dizziness and palpitations. An ECG shows a broad-complex tachycardia.

Which of the following drugs would be the best choice of treatment?

Explanation

The answer is Amiodarone –

This patient, a normotensive man with poor left ventricular function and a broad-complex tachycardia, has ventricular tachycardia (VT). Amiodarone may be used tocardiovert VT to sinus rhythm and to prevent ventricular fibrillation. Amiodarone may beadministered by repeated IV bolus doses, or continuous IV infusion.

Flecainide (Option B) is incorrect. Flecainide may cause ventricular fibrillation in patientswith ventricular tachycardia, particularly in patients with underlying heart failure andischaemic heart disease.

Lidocaine (Option C) is incorrect. In the presence of poor left ventricular function, lidocaineand β-blockers should not be used.

Sotalol (Option D) is incorrect. In the presence of poor left ventricular function, lidocaine andβ-blockers should not be used.

Verapamil (Option E) is incorrect. Verapamil may precipitate a circulatory collapse in VT andis therefore contraindicated in VT.

AmiodaroneA

FlecainideB

LidocaineC

SotalolD

VerapamilE

Page 429: Back to Filters (/Secure/TestMe/Filter ... - 1 File Download

8/11/2016 MyPastest

https://mypastest.pastest.com/Secure/TestMe/Browser/429893#Top 2/2

Next Question

Previous Question Tag Question Feedback End Review

Difficulty: Average

Peer Responses

Session Progress

0Responses Correct:

207Responses Incorrect:

207Responses Total:

0%Responses - % Correct:

Blog (https://www.pastest.com/blog) About Pastest (https://www.pastest.com/about-us)Contact Us (https://www.pastest.com/contact-us) Help (https://www.pastest.com/help)

© Pastest 2016

Page 430: Back to Filters (/Secure/TestMe/Filter ... - 1 File Download

8/11/2016 MyPastest

https://mypastest.pastest.com/Secure/TestMe/Browser/429893#Top 1/2

Back to Filters (/Secure/TestMe/Filter/429893/QA)

Question 36 of 207

A 51-year-old woman with a history of type-2 diabetes and bipolar disorder is admitted forreview because of low sodium (118 mmol/l). On examination her blood pressure is 139/72mmHg, her pulse is 70 beats per minute, regular, and she is not in cardiac failure.

Investigations:

Hb 12.4 g/dl

White cell count 5.3 × 10 /l

Platelets 190 × 10 /l

Na 118 mmol/l

K 3.8 mmol/l

Creatinine 92 μmol/l

HbA1c 57.38 mmol/mol (7.4%)

Fasting glucose 6.4 mmol/l

Which one of the following drugs is most likely to be responsible?

Explanation

Drug-induced hyponatraemia

9

9

+

+

MoclobemideA

CarbamazepineB

LithiumC

GliclazideD

PioglitazoneE

Page 431: Back to Filters (/Secure/TestMe/Filter ... - 1 File Download

8/11/2016 MyPastest

https://mypastest.pastest.com/Secure/TestMe/Browser/429893#Top 2/2

20939

As well as being used for the management of epilepsy, carbamazepine is used in the

management of bipolar disorder

While moclobemide may rarely be associated with hyponatraemia, carbamazepine

possesses effects similar to those of antidiuretic hormone and hence it is commonly

associated with hyponatraemia

In this case other options for treatment of bipolar disorder could be considered

It is unlikely that chronic fluid restriction would resolve the degree of hyponatraemia

seen here

Next Question

Previous Question Tag Question Feedback End Review

Difficulty: Difficult

Peer Responses

Session Progress

0Responses Correct:

207Responses Incorrect:

207Responses Total:

0%Responses - % Correct:

Blog (https://www.pastest.com/blog) About Pastest (https://www.pastest.com/about-us)Contact Us (https://www.pastest.com/contact-us) Help (https://www.pastest.com/help)

© Pastest 2016

Page 432: Back to Filters (/Secure/TestMe/Filter ... - 1 File Download

8/11/2016 MyPastest

https://mypastest.pastest.com/Secure/TestMe/Browser/429893#Top 1/2

Back to Filters (/Secure/TestMe/Filter/429893/QA)

Question 37 of 207

46864

You are asked to arrange some blood tests to monitor a patient who is receivingamphotericin therapy, and you are considering which tests to arrange.

Which of the following metabolic disturbances is most likely to be associated withamphotericin treatment?

Explanation

The answer is Hypokalaemia –

Nephrotoxicity due to amphotericin is due to a tubular toxicity, and causes wasting ofpotassium, and to a lesser extent sodium and magnesium. Metabolic acidosis may occurdue to type 1 (distal) renal tubular acidosis, and polyuria may occur due to nephrogenicdiabetes insipidus. Renal adverse effects generally occur in patients that have receiveddaily doses of more than 3 g, and are usually reversible on decreasing the dosage orstopping treatment.

Metabolic alkalosis (Option A) is incorrect. Metabolic acidosis may occur (distal renal tubularacidosis).

Hypermagnesaemia (Option B) is incorrect. Hypomagnesaemia is a recognised complication.

Hypocalcaemia (Option C) is incorrect. Hypocalcaemia is not a recognised feature.

Hyponatraemia (Option E) is incorrect. Hyponatraemia may occur, but is usually lessprominent than hypokalaemia, and therefore represents a less suitable answer in this case.

Metabolic alkalosisA

HypermagnesaemiaB

HypocalcaemiaC

HypokalaemiaD

HyponatraemiaE

Page 433: Back to Filters (/Secure/TestMe/Filter ... - 1 File Download

8/11/2016 MyPastest

https://mypastest.pastest.com/Secure/TestMe/Browser/429893#Top 2/2

Next Question

Previous Question Tag Question Feedback End Review

Difficulty: Average

Peer Responses

Session Progress

0Responses Correct:

207Responses Incorrect:

207Responses Total:

0%Responses - % Correct:

Blog (https://www.pastest.com/blog) About Pastest (https://www.pastest.com/about-us)Contact Us (https://www.pastest.com/contact-us) Help (https://www.pastest.com/help)

© Pastest 2016

Page 434: Back to Filters (/Secure/TestMe/Filter ... - 1 File Download

8/11/2016 MyPastest

https://mypastest.pastest.com/Secure/TestMe/Browser/429893#Top 1/2

Back to Filters (/Secure/TestMe/Filter/429893/QA)

Question 38 of 207

37798

A 62-year-old man who takes multiple medications for cardiovascular disease and diabetescomes to the clinic for review. He has been complaining of a blue discolouration to his visionover the past 5 weeks since his last medication review. Examination does not reveal anysignificant change in acuity over the past 6 months and no abnormality is seen onfundoscopy.

Which of the following medications he is taking is the most likely cause?

Explanation

The answer is Sildenafil -

Sildenafil is a PDE-5 inhibitor, but at high doses it also inhibits PDE-6, which leads to bluediscoloration of vision. This can often be managed by reducing the dose of Sildenafil. It isseen across the class of PDE-5 inhibitors when they are used at high dose. Digoxin isrecognised at potentially toxic levels to lead to xanthopsia, or yellowing of vision. Reports ofhypoglycaemia and dizziness were made during the Sitagliptin clinical trial program, butexcess of visual system AEs or SAEs was reported. Diplopia is rarely reported in conjunctionwith Amlodipine therapy, and conjunctivitis with Ramipril therapy.

<ahref="https://www.medicines.org.uk/emc/medicine/1474/SPC/Viagra+25mg,+50mg,+100mg/"">https://www.medicines.org.uk/emc/medicine/1474/SPC/Viagra+25mg,+50mg,+100mg/

AmlodipineA

DigoxinB

RamiprilC

SildenafilD

SitagliptinE

Next Question

Page 435: Back to Filters (/Secure/TestMe/Filter ... - 1 File Download

8/11/2016 MyPastest

https://mypastest.pastest.com/Secure/TestMe/Browser/429893#Top 2/2

Previous QuestionTag Question Feedback End Review

Difficulty: Average

Peer Responses

Session Progress

0Responses Correct:

207Responses Incorrect:

207Responses Total:

0%Responses - % Correct:

Blog (https://www.pastest.com/blog) About Pastest (https://www.pastest.com/about-us)Contact Us (https://www.pastest.com/contact-us) Help (https://www.pastest.com/help)

© Pastest 2016

Page 436: Back to Filters (/Secure/TestMe/Filter ... - 1 File Download

8/11/2016 MyPastest

https://mypastest.pastest.com/Secure/TestMe/Browser/429893#Top 1/2

Back to Filters (/Secure/TestMe/Filter/429893/QA)

Question 39 of 207

46918

You review a 28-year-old man who has been admitted in a state of collapse from a night club.His friends admit that because of pressure at work he has been using increasing amounts ofcocaine recently.

Which of the following features is most commonly associated with cocaine?

Explanation

The answer is Metabolic acidosis –

Cocaine blocks the re-uptake of biogenic amines, and inhibition of dopamine re-uptake isthe cause of the psychomotor agitation that commonly accompanies cocaine use.Inhibtion of noradrenaline (norepinephrine) reuptake causes excess sympathomimeticactivity; the euphoric effect is caused by inhibition of serotonin reuptake. Adverse effectsinclude agitation, tachycardia, hypertension, sweating, hallucinations and seizures.Metabolic acidosis, hyperthermia, rhabdomyolysis, ventricular arrhythmias andmyocardial infarction are recognised. Regular use may be associated with prematurecoronary artery disease, dilated cardiomyopathy and increased risk of cerebralhaemorrhage.

Bradycardia (Option A) is incorrect. Tachycardia is a more typical finding.

Hypotension (Option B) is incorrect. High blood pressure rather than hypotension is typical.

Hypothermia (Option C) is incorrect. Hyperthermia is a recognised feature, not hypothermia.

Metabolic alkalosis (Option E) is incorrect. It is metabolic acidosis, rather than than alkalosis.

BradycardiaA

HypotensionB

HypothermiaC

Metabolic acidosisD

Metabolic alkalosisE

Page 437: Back to Filters (/Secure/TestMe/Filter ... - 1 File Download

8/11/2016 MyPastest

https://mypastest.pastest.com/Secure/TestMe/Browser/429893#Top 2/2

Next Question

Previous Question Tag Question Feedback End Review

Difficulty: Average

Peer Responses

Session Progress

0Responses Correct:

207Responses Incorrect:

207Responses Total:

0%Responses - % Correct:

Blog (https://www.pastest.com/blog) About Pastest (https://www.pastest.com/about-us)Contact Us (https://www.pastest.com/contact-us) Help (https://www.pastest.com/help)

© Pastest 2016

Page 438: Back to Filters (/Secure/TestMe/Filter ... - 1 File Download

8/11/2016 MyPastest

https://mypastest.pastest.com/Secure/TestMe/Browser/429893#Top 1/2

Back to Filters (/Secure/TestMe/Filter/429893/QA)

Question 40 of 207

46982

A 32-year-old patient was admitted with an acute overdose of lithium. She was immediatelystarted on an infusion of normal saline. On admission the lithium level was 2.4 mmol/l; after 10h the lithium level was found to be 1.8 mmol/l.

Based upon the lithium concentrations, what interval might be most likely before lithiumconcentrations are approaching an undetectable level?

Explanation

The answer is 72 h –

The half-life of lithium is around 20 h, although this time period may be prolonged in theelderly or in chronic lithium users. In this case, after 10 h the lithium level has fallen byaround 25%, consistent with a half-life of around 20 h. This is not, of course, a reliablemethod because the fall in lithium concentrations is due to elimination plus distributionfrom circulation to the tissues. Nonetheless, based on a crude estimate of half-life of 20h, then lithium concentrations would be 1.2 mmol/l after 20 h, 0.6 mmol/l after 40 h, 0.3mmol/l after 60 h and 0.15 mmol/l after 80 h.

4 h (Option A) is incorrect. This is too short, based upon a half-life estimate of 20 h.

8 h (Option B) is incorrect. This is too short, based upon a half-life estimate of 20 h.

12 h (Option C) is incorrect. This is too short, based upon a half-life estimate of 20 h.

24 h (Option D) is incorrect. This is too short, based upon a half-life estimate of 20 h.

4 hA

8 hB

12 hC

24 hD

72 hE

Page 439: Back to Filters (/Secure/TestMe/Filter ... - 1 File Download

8/11/2016 MyPastest

https://mypastest.pastest.com/Secure/TestMe/Browser/429893#Top 2/2

Next Question

Previous Question Tag Question Feedback End Review

Difficulty: Average

Peer Responses

Session Progress

0Responses Correct:

207Responses Incorrect:

207Responses Total:

0%Responses - % Correct:

Blog (https://www.pastest.com/blog) About Pastest (https://www.pastest.com/about-us)Contact Us (https://www.pastest.com/contact-us) Help (https://www.pastest.com/help)

© Pastest 2016

Page 440: Back to Filters (/Secure/TestMe/Filter ... - 1 File Download

8/11/2016 MyPastest

https://mypastest.pastest.com/Secure/TestMe/Browser/429893#Top 1/2

Back to Filters (/Secure/TestMe/Filter/429893/QA)

Question 41 of 207

You are involved in a meeting with your pharmacy and biochemistry department colleaguesto discuss the local hospital policy on therapeutic drug monitoring.

For which of the following drugs would therapeutic drug monitoring be most appropriate?

Explanation

The answer is Vancomycin –

Therapeutic drug monitoring is only useful for drugs that have a narrow therapeuticindex, and where there is a good correlation between serum concentration andtherapeutic effect. It should be remembered, however, that particularly withanticonvulsants and digoxin, it is important to rely on clinical judgement in interpretingthe drug concentrations, including response to treatment and occurrence of adverseeffects. Intravenous vancomycin has a narrow therapeutic window, and monitoring isneeded to minimise the risk of kidney injury.

Beta-interferon (Option A) is incorrect. There is no obvious relationship between interferondrug concentrations and clinical effectiveness or occurrence of adverse effects.

Cyclophosphamide (Option B) is incorrect. There is no obvious relationship betweencyclophosphamide drug concentrations and clinical effectiveness or occurrence of adverseeffects.

Propranolol (Option C) is incorrect. The dose of propranolol and other β-blockers may betitrated according to patient symptoms using heart rate as a guide to effectiveness.

Beta-interferonA

CyclophosphamideB

PropranololC

VancomycinD

VigabatrinE

Page 441: Back to Filters (/Secure/TestMe/Filter ... - 1 File Download

8/11/2016 MyPastest

https://mypastest.pastest.com/Secure/TestMe/Browser/429893#Top 2/2

46898

Vigabatrin (Option E) is incorrect. It is possible to measure vigabatrin concentrations toconfirm drug compliance, but there is a much poorer correlation between drugconcentrations and clinical response.

Next Question

Previous Question Tag Question Feedback End Review

Difficulty: Average

Peer Responses

Session Progress

0Responses Correct:

207Responses Incorrect:

207Responses Total:

0%Responses - % Correct:

Blog (https://www.pastest.com/blog) About Pastest (https://www.pastest.com/about-us)Contact Us (https://www.pastest.com/contact-us) Help (https://www.pastest.com/help)

© Pastest 2016

Page 442: Back to Filters (/Secure/TestMe/Filter ... - 1 File Download

8/11/2016 MyPastest

https://mypastest.pastest.com/Secure/TestMe/Browser/429893#Top 1/2

Back to Filters (/Secure/TestMe/Filter/429893/QA)

Question 42 of 207

46919

You review a 64-year-old man who is on warfarin therapy for recurrent atrial fibrillation. Hepresents to the Emergency Department with extensive unprovoked bruising and aninternational normalised ratio (INR) check reveals that his INR is raised at 6.5.

Which of the following drugs when co-administered with warfarin may result in increasedINR?

Explanation

The answer is Ciprofloxacin –

Chloramphenicol, ciprofloxacin, clarithromycin, erythromycin, metronidazole andomeprazole are all capable of inhibiting hepatic enzyme activity so that the effects ofwarfarin are that INR and bleeding risk increase.

Carbamazepine (Option A) is incorrect. Carbamazepine is a powerful inducer of hepaticenzyme activity such that over a period of time warfarin concentrations and INR decrease.

Phenobarbital (Option C) is incorrect. Phenobarbital is a powerful inducer of hepatic enzymeactivity such that over a period of time warfarin concentrations and INR decrease.

Primidone (Option D) is incorrect. Primodone is a prodrug metabolised to phenobarbital, apowerful inducer of hepatic enzyme activity such that over a period of time warfarinconcentrations and INR decrease.

Rifampicin (Option E) is incorrect. Rifampicin is an inducer of hepatic enzyme activity suchthat over a period of time warfarin concentrations and INR decrease.

CarbamazepineA

CiprofloxacinB

PhenobarbitalC

PrimidoneD

RifampicinE

Page 443: Back to Filters (/Secure/TestMe/Filter ... - 1 File Download

8/11/2016 MyPastest

https://mypastest.pastest.com/Secure/TestMe/Browser/429893#Top 2/2

Next Question

Previous Question Tag Question Feedback End Review

Difficulty: Average

Peer Responses

Session Progress

0Responses Correct:

207Responses Incorrect:

207Responses Total:

0%Responses - % Correct:

Blog (https://www.pastest.com/blog) About Pastest (https://www.pastest.com/about-us)Contact Us (https://www.pastest.com/contact-us) Help (https://www.pastest.com/help)

© Pastest 2016

Page 444: Back to Filters (/Secure/TestMe/Filter ... - 1 File Download

8/11/2016 MyPastest

https://mypastest.pastest.com/Secure/TestMe/Browser/429893#Top 1/2

Back to Filters (/Secure/TestMe/Filter/429893/QA)

Question 43 of 207

45803

You are reviewing a patient in the medical admissions unit who has been referred from theEmergency Department after a suspected overdose. The patient appears anxious and tearfulbut is refusing to disclose what he may have taken. On examination of his pupils, you notethat they are dilated.

Which of the following is most likely to account for the dilated pupils?

Explanation

The answer is Cocaine –

Dilated pupils (mydriasis) may be caused by drugs with sympathomimetic activity (egcocaine, amphetamines and pseudoephedrine) or anticholinergic drugs (egantihistamines, atropine and tricyclic antidepressants).

Chlorpromazine (Option A) is incorrect. Phenothiazines may cause miosis.

Fentanyl (Option C) is incorrect. Opioids cause miosis, which may be severe enough to cause‘pin-point’ pupils.

Mirtazapine (Option D) is incorrect. This may cause miosis. In mirtazapine overdose, theremay be ‘pin-point’ pupils that mimic the effects of opioids.

Sodium valproate (Option E) is incorrect. This may cause miosis.

ChlorpromazineA

CocaineB

FentanylC

MirtazapineD

Sodium valproateE

Next Question

Page 445: Back to Filters (/Secure/TestMe/Filter ... - 1 File Download

8/11/2016 MyPastest

https://mypastest.pastest.com/Secure/TestMe/Browser/429893#Top 2/2

Previous Question Tag Question Feedback End Review

Difficulty: Average

Peer Responses

Session Progress

0Responses Correct:

207Responses Incorrect:

207Responses Total:

0%Responses - % Correct:

Blog (https://www.pastest.com/blog) About Pastest (https://www.pastest.com/about-us)Contact Us (https://www.pastest.com/contact-us) Help (https://www.pastest.com/help)

© Pastest 2016

Page 446: Back to Filters (/Secure/TestMe/Filter ... - 1 File Download

8/11/2016 MyPastest

https://mypastest.pastest.com/Secure/TestMe/Browser/429893#Top 1/2

Back to Filters (/Secure/TestMe/Filter/429893/QA)

Question 44 of 207

A 56-year-old woman has been referred to the endocrinology clinic for review ofthyrotoxicosis and consideration of possible radioactive iodine treatment.

Which one of the following statements best describes the role of radioactive iodine (131I ) inthe treatment of thyrotoxicosis?

Explanation

The answer is May worsen Graves’ exophthalmos in the first 3 months –

I is administered as an oral solution or capsule of sodium 131I, which is rapidlyconcentrated in thyroid tissue. Beta-emissions result in ablation of the gland over thefollowing 6–18 weeks. Beta-emissions have very low tissue penetration capability. It isgenerally accepted that there is increased risk of worsening Graves’ exophthalmos duringradioactive iodine therapy

Associated with increased incidence of late leukaemia (Option A) is incorrect. Radioactiveiodine has been studied in very large numbers of patients, including a prospective study of36 000 patients that found no increased risk of cancer or leukaemia.

Hypoparathyroidism secondary to β-emissions and ablation of the parathyroid gland occursin 30% of cases (Option B) is incorrect. The parathyroid gland is not affected because theradiation is selectively taken up by thyroid tissue, and is capable of penetrating only 0.5 mmof tissue.

Intravenous administration is used to avoid gastrointestinal toxicity (Option C) is incorrect.Radioactive iodine is administered orally.

Associated with increased incidence of late leukaemiaA

Hypoparathyroidism secondary to β-emissions and ablation of the parathyroid glandoccurs in 30% of cases

B

Intravenous administration is used to avoid gastrointestinal toxicityC

May worsen Graves’ exophthalmos in the first 3 monthsD

Three doses 1 month apart is the optiomal regimenE

131

Page 447: Back to Filters (/Secure/TestMe/Filter ... - 1 File Download

8/11/2016 MyPastest

https://mypastest.pastest.com/Secure/TestMe/Browser/429893#Top 2/2

46627

Three doses 1 month apart is the optiomal regimen (Option E) is incorrect. Most patientsrequire only one treatment, although 10–15% experience treatment failure and require asecond or subsequent doses.

Next Question

Previous Question Tag Question Feedback End Review

Difficulty: Average

Peer Responses

Session Progress

0Responses Correct:

207Responses Incorrect:

207Responses Total:

0%Responses - % Correct:

Blog (https://www.pastest.com/blog) About Pastest (https://www.pastest.com/about-us)Contact Us (https://www.pastest.com/contact-us) Help (https://www.pastest.com/help)

© Pastest 2016

Page 448: Back to Filters (/Secure/TestMe/Filter ... - 1 File Download

8/11/2016 MyPastest

https://mypastest.pastest.com/Secure/TestMe/Browser/429893#Top 1/2

Back to Filters (/Secure/TestMe/Filter/429893/QA)

Question 45 of 207

A 45-year-old woman who smokes 30 cigarettes per day is admitted to the EmergencyDepartment resuscitation room after suffering an out-of-hospital cardiac arrest. Her husbandsays that she has recently suffered a chest infection treated by her GP with erythromycin. Shehas a past history of penicillin allergy, and has been taking some antifungal tablets for chronicfungal vaginal infection.

Given her medication history, which of the following causes of cardiac arrest is moreprobable?

Explanation

The answer is Torsades de pointes ventricular tachycardia –

This patient is likely to have suffered torsades de pointes ventricular tachycardia as aresult of an interaction between erythromycin (which is a cause of long QT) and herantifungal tablets, most likely ketoconazole, used for recurrent episodes of vaginalcandidiasis. Similar potential for drug-induced QT prolongation exists for a number ofdrugs, including the interaction between erythromycin and moxifloxacin. Cisapride, aprokinetic agent used for the treatment of gastro-oesophageal reflux disease, waswithdrawn due to the occurrence of QT prolongation, particularly when used incombination with erythromycin. When prescribing erythromycin, it is important toconsult the ‘Interactions’ section in the British National Formulary in order to rule outpossible interactions of this type.

Anaphylaxis to erythromycin (Option A) is incorrect. Although possible, it is uncommon forpatients with multidrug allergy to have allergy to macrolide antibiotics.

Anaphylaxis to erythromycinA

Myocardial infarctionB

Supraventricular tachycardiaC

Torsades de pointes ventricular tachycardiaD

Ventricular tachycardiaE

Page 449: Back to Filters (/Secure/TestMe/Filter ... - 1 File Download

8/11/2016 MyPastest

https://mypastest.pastest.com/Secure/TestMe/Browser/429893#Top 2/2

46883

Myocardial infarction (Option B) is incorrect. There are no obvious risk factors for myocardialinfarction, and no known association between her medications and atherosclerosis.

Supraventricular tachycardia (Option C) is incorrect. This is unrelated to any of the clinicaldetails provided.

Ventricular tachycardia (Option E) is incorrect. Although ventricular tachycardia is possiblycorrect, torsades de pointes offers a better explanation in view of her history of erythromycinand antifungal treatments.

Next Question

Previous Question Tag Question Feedback End Review

Difficulty: Average

Peer Responses

Session Progress

0Responses Correct:

207Responses Incorrect:

207Responses Total:

0%Responses - % Correct:

Blog (https://www.pastest.com/blog) About Pastest (https://www.pastest.com/about-us)Contact Us (https://www.pastest.com/contact-us) Help (https://www.pastest.com/help)

© Pastest 2016

Page 450: Back to Filters (/Secure/TestMe/Filter ... - 1 File Download

8/11/2016 MyPastest

https://mypastest.pastest.com/Secure/TestMe/Browser/429893#Top 1/2

Back to Filters (/Secure/TestMe/Filter/429893/QA)

Question 46 of 207

An elderly woman is taking furosemide and ramipril for heart failure. She visited her GPcomplaining of pain in her left knee and was prescribed rofecoxib for analgesia. Two weekslater, she was admitted to the A&E department complaining of breathlessness and pedaloedema.

What is the most likely cause of her having developed these symptoms?

Explanation

The answer is Fluid retention due to rofecoxib –

Rofecoxib is a cyclo-oxygenase-2 (Cox-2) specific inhibitor and has less of an adverseeffect on the gastrointestinal tract; thus gastric irritation and bleeding is much lesscommon than with other NSAIDs such as ibuprofen or mefenamic acid. However,rofecoxib can cause fluid retention and worsen pre-existing cardiac failure. One trialversus celecoxib demonstrated oedema rates of around 7.7% in patients treated withrofecoxib 25 mg, over a 6-week period.

Anaemia due to gastrointestinal bleeding (Option A) is incorrect. Coxibs can provokegastrointestinal irritation and bleeding, albeit much less commonly than after NSAIDs.

Drug interaction leading to decreased furosemide effect (Option B) is incorrect. Rofecoxib isnot known to impair response to furosemide.

Drug interaction leading to increased ramipril levels (Option C) is incorrect. Increasedramipril concentrations are unlikely to occur, and would not account for symptoms ofbreathlessness or oedema.

Anaemia due to gastrointestinal bleedingA

Drug interaction leading to decreased furosemide effectB

Drug interaction leading to increased ramipril levelsC

Fluid retention due to rofecoxibD

Rofecoxib induced deterioration in creatinineE

Page 451: Back to Filters (/Secure/TestMe/Filter ... - 1 File Download

8/11/2016 MyPastest

https://mypastest.pastest.com/Secure/TestMe/Browser/429893#Top 2/2

46743

Rofecoxib induced deterioration in creatinine (Option E) is incorrect. Coxibs may cause renalimpairment, but this is much less commonly encountered than after non-selective NSAIDs.

Next Question

Previous Question Tag Question Feedback End Review

Difficulty: Average

Peer Responses

Session Progress

0Responses Correct:

207Responses Incorrect:

207Responses Total:

0%Responses - % Correct:

Blog (https://www.pastest.com/blog) About Pastest (https://www.pastest.com/about-us)Contact Us (https://www.pastest.com/contact-us) Help (https://www.pastest.com/help)

© Pastest 2016

Page 452: Back to Filters (/Secure/TestMe/Filter ... - 1 File Download

8/11/2016 MyPastest

https://mypastest.pastest.com/Secure/TestMe/Browser/429893#Top 1/2

Back to Filters (/Secure/TestMe/Filter/429893/QA)

Question 47 of 207

An 18-year-old student presents to the Emergency Department with a short history of fever,vomiting and diarrhoea. She has recently returned to the UK after a holiday in Turkey duringwhich she had been commenced on an antibiotic for a suspected urinary tract infection. Onexamination she has a widespread macular rash associated with ring-like lesions, with someulceration in her oral cavity.

Which drug is most likely to account for these findings?

Explanation

The answer is Co-trimoxazole –

The widespread macular rash and target lesions are characteristic of erythemamultiforme, and the features are consistent with Stevens–Johnson syndrome. This is aparticularly serious and potentially fatal type of skin and mucous membrane eruption. Itcan occur after a number of antibiotics but is most strongly associated withsulfonamides including sulfamethoxazole (this is combined with trimethoprim in co-trimoxazole). Therefore, sulfonamides and co-trimoxazole are used less commonly, andreserved for severe infections.

Amoxicillin (Option A) is incorrect. Amoxicillin may cause widespread maculopapular rash,urticaria and fixed drug eruptions. It can be effective for empirical treatment of urinary tractinfections (UTIs) but in many countries and regions there is significant resistance amongvarious strains of Escherichia coli.

Ampicillin (Option B) is incorrect. Ampicillin may cause widespread maculopapular rash,urticaria and fixed drug eruptions. It can be effective for empirical treatment of urinary tract

AmoxicillinA

AmpicillinB

CiprofloxacinC

Co-trimoxazoleD

ErythromycinE

Page 453: Back to Filters (/Secure/TestMe/Filter ... - 1 File Download

8/11/2016 MyPastest

https://mypastest.pastest.com/Secure/TestMe/Browser/429893#Top 2/2

46781

infections (UTIs) but in many countries and regions there is significant resistance amongvarious strains of Escherichia coli.

Ciprofloxacin (Option C) is incorrect. Ciprofloxacin is effective in treating UTIs, even whenthey are caused by multidrug-resistant bacteria, eg Pseudomonas spp. They are normallyreserved for second-line treatment for UTIs to minimise emergence of resistant strains.Stevens–Johnson syndrome is not associated with ciprofloxacin.

Erythromycin (Option E) is incorrect. Erythromycin is generally regarded as much lesseffective versus gram-negative bacteria and unlikely to be effective treatment for UTI.

Next Question

Previous Question Tag Question Feedback End Review

Difficulty: Average

Peer Responses

Session Progress

0Responses Correct:

207Responses Incorrect:

207Responses Total:

0%Responses - % Correct:

Blog (https://www.pastest.com/blog) About Pastest (https://www.pastest.com/about-us)Contact Us (https://www.pastest.com/contact-us) Help (https://www.pastest.com/help)

© Pastest 2016

Page 454: Back to Filters (/Secure/TestMe/Filter ... - 1 File Download

8/11/2016 MyPastest

https://mypastest.pastest.com/Secure/TestMe/Browser/429893#Top 1/2

Back to Filters (/Secure/TestMe/Filter/429893/QA)

Question 48 of 207

You are asked to review a 63-year-old man as the medical registrar on duty. He is currently anin-patient on the psychiatric ward and receiving treatment for severe depression. Thepsychiatry team has considered a potential diagnosis of serotonin syndrome.

Which of the following features is most strongly suggestive of serotonin syndrome?

Explanation

The answer is Myoclonus at both ankles –

The serotonin syndrome occurs primarily because of interactions between drugs thatpromote serotonin release within the central nervous system. This includes monoamine-oxidase inhibitors (MAOIs), selective serotonin-reuptake inhibitors (SSRIs), tricyclicantidepressants, antipsychotics and tramadol. Typical features include hypertonia,hyperreflexia, hyperthermia, sweating, tremor, myoclonus and tachycardia. Treatmentconsists of stopping the causative drugs, and supportive measures to reduce agitationincluding benzodiazepines. Cyproheptadine possesses serotonin-antagonist activity andmay be helpful, and active cooling may be required in some patients.

Flaccid paresis of both legs (Option A) is incorrect. Tone and reflexes are normally increased.

Heart rate 92 bpm (Option B) is incorrect. Tachycardia is a recognised feature, but a heartrate of 92 is non-specific, and may be caused by a broad range of disorders.

Profuse sweating (Option D) is incorrect. Profuse sweating may be a feature, but is non-specific, and may be caused by a broad range of disorders.

Flaccid paresis of both legsA

Heart rate 92 bpmB

Myoclonus at both anklesC

Profuse sweatingD

Temperature 37.6 °CE

Page 455: Back to Filters (/Secure/TestMe/Filter ... - 1 File Download

8/11/2016 MyPastest

https://mypastest.pastest.com/Secure/TestMe/Browser/429893#Top 2/2

46490

Temperature 37.6 °C (Option E) is incorrect. Hyperthermia is a key feature, but a temperatureof 37.6 °C is non-specific, and may be caused by a broad range of disorders.

Next Question

Previous Question Tag Question Feedback End Review

Difficulty: Average

Peer Responses

Session Progress

0Responses Correct:

207Responses Incorrect:

207Responses Total:

0%Responses - % Correct:

Blog (https://www.pastest.com/blog) About Pastest (https://www.pastest.com/about-us)Contact Us (https://www.pastest.com/contact-us) Help (https://www.pastest.com/help)

© Pastest 2016

Page 456: Back to Filters (/Secure/TestMe/Filter ... - 1 File Download

8/11/2016 MyPastest

https://mypastest.pastest.com/Secure/TestMe/Browser/429893#Top 1/2

Back to Filters (/Secure/TestMe/Filter/429893/QA)

Question 49 of 207

A 59-year-old woman, who is being treated with trastuzumab for breast cancer, comes to theclinic complaining of increasing shortness of breath and ankle swelling. On examination herblood pressure is 115/80 mmHg and her pulse is 95 beats per minute and regular. She hasbiventricular failure with bilateral ankle swelling and inspiratory crackles in both lower zoneson auscultation.

Investigations:

Hb 10.9 g/dl

White cell count 9.1 × 10 /l

Platelets 165 × 10 /l

Na 141 mmol/l

K 4.2 mmol/l

Creatinine 129 μmol/l

ECHO suggestive of dilated cardiomyopathy

Which one of the following is the causative mechanism of cardiac failure in patients ontrastuzumab?

Explanation

Trastuzumab

9

9

+

+

Calcium channel blockadeA

Erb-b2 inhibitionB

Oestrogen receptor inhibitionC

Platelet-derived growth factor inhibitionD

Erb-b1 inhibitionE

Page 457: Back to Filters (/Secure/TestMe/Filter ... - 1 File Download

8/11/2016 MyPastest

https://mypastest.pastest.com/Secure/TestMe/Browser/429893#Top 2/2

22449

Studies have shown that activation of Erb-b2 (also known as HER-2), the receptor

blocked by trastuzumab (Herceptin), is important in preventing the development of

cardiomyopathy

A knockout mouse model with deletion of the erb-b2 receptor showed a predisposition

for the development of cardiomyopathy, which was actually worsened by anthracycline

therapy

Reference

Crone, S. A., Zhao, Y.-Y., Fan, L. et al. 2002. ErbBe2 is essential in the prevention of dilatedcardiomyopathy. Nature Medicine, 8, 459–465.

Next Question

Previous Question Tag Question Feedback End Review

Difficulty: Average

Peer Responses

Session Progress

0Responses Correct:

207Responses Incorrect:

207Responses Total:

0%Responses - % Correct:

Blog (https://www.pastest.com/blog) About Pastest (https://www.pastest.com/about-us)Contact Us (https://www.pastest.com/contact-us) Help (https://www.pastest.com/help)

© Pastest 2016

Page 458: Back to Filters (/Secure/TestMe/Filter ... - 1 File Download

8/11/2016 MyPastest

https://mypastest.pastest.com/Secure/TestMe/Browser/429893#Top 1/2

Back to Filters (/Secure/TestMe/Filter/429893/QA)

Question 50 of 207

A 35-year-old chronic alcoholic was admitted to hospital with severe tremor due tounplanned alcohol withdrawal, and has undergone a period of observed detoxification overthe past week. She is now mobilising independently and ready for discharge. She is keen toremain abstinent after discharge. She has already begun attending counselling meetings.Other past history of note includes irritable bowel syndrome (IBS).

Which of the following drugs might be most useful in maintaining abstinence from alcohol inthis patient?

Explanation

The answer is Acamprosate –

Acamprosate is a taurine derivative that increases γ-aminobutyric acid (GABA) activitywithin the brain, an inhibitory CNS neurotransmitter. It has relatively few side-effects butmay reduce alcohol craving and improves abstinence rates. It is contraindicated inpregnancy, and severe liver or kidney failure.

Chlordiazepoxide (Option B) is incorrect. Benzodiazepines such as chlordiazepoxide areuseful for managing the initial withdrawal phase by increasing seizure threshold and reducingacute withdrawal symptoms including delirium tremens.

Diazepam (Option C) is incorrect. Benzodiazepines such as diazepam are useful formanaging the initial withdrawal phase by increasing seizure threshold and reducing acutewithdrawal symptoms including delirium tremens.

Disulfiram (Option D) is incorrect. Disulfiram may provoke an unpleasant reaction if takenwith alcohol, and may promote alcohol avoidance. It is not used in acute management of

AcamprosateA

ChlordiazepoxideB

DiazepamC

DisulfiramD

NaltrexoneE

Page 459: Back to Filters (/Secure/TestMe/Filter ... - 1 File Download

8/11/2016 MyPastest

https://mypastest.pastest.com/Secure/TestMe/Browser/429893#Top 2/2

46762

alcohol withdrawal.

Naltrexone (Option E) is incorrect. Naltrexone reduces the pleasure that alcohol brings andcraving when it is withdrawn, and can reduce relapse rates by up to half. However, it isassociated with a number of adverse effects, including nausea, vomiting, anxiety,nervousness, insomnia, lethargy, arthralgia, increased sweating and lacrimation, diarrhoea orconstipation, increased thirst, and liver and kidney dysfunction. The adverse gastrointestinaleffects of naltrexone may discourage use in a patient with IBS.

Next Question

Previous Question Tag Question Feedback End Review

Difficulty: Average

Peer Responses

Session Progress

0Responses Correct:

207Responses Incorrect:

207Responses Total:

0%Responses - % Correct:

Blog (https://www.pastest.com/blog) About Pastest (https://www.pastest.com/about-us)Contact Us (https://www.pastest.com/contact-us) Help (https://www.pastest.com/help)

© Pastest 2016

Page 460: Back to Filters (/Secure/TestMe/Filter ... - 1 File Download

8/11/2016 MyPastest

https://mypastest.pastest.com/Secure/TestMe/Browser/429893#Top 1/2

Back to Filters (/Secure/TestMe/Filter/429893/QA)

Question 51 of 207

22480

A 58-year-old man presents to the clinic with easy bruising and bleeding from his gums. Hehas a history of type-2 diabetes and has recently been prescribed a medication forneuropathy. In addition he has an infection but cannot remember which antibiotic his doctorhas prescribed for it. Chronic medication of note includes warfarin, which is prescribed foratrial fibrillation after a failed cardioversion. His INR is normally stable at 2.1; it has recentlyincreased to 5.0.

Which one of the following medications is most likely to be responsible for his recentdisturbance in coagulation control?

Explanation

Drug interactions

Rifampicin and carbamazepine are inducers of CYP-450 and hence are associated with

decreased INR

Lansoprazole and other proton pump inhibitors, while they affect stomach pH, are not

usually associated with significant disturbance in INR

While a caution is advised for use of macrolides with warfarin, azithromycin appears to

be least likely to lead to an interaction

This leaves ciprofloxacin, for which a strong caution is advised because of potentiation

of warfarin’s effect

CiprofloxacinA

AzithromycinB

LansoprazoleC

RifampicinD

CarbamazepineE

Next Question

Page 461: Back to Filters (/Secure/TestMe/Filter ... - 1 File Download

8/11/2016 MyPastest

https://mypastest.pastest.com/Secure/TestMe/Browser/429893#Top 2/2

Previous Question Tag Question Feedback End Review

Difficulty: Average

Peer Responses

Session Progress

0Responses Correct:

207Responses Incorrect:

207Responses Total:

0%Responses - % Correct:

Blog (https://www.pastest.com/blog) About Pastest (https://www.pastest.com/about-us)Contact Us (https://www.pastest.com/contact-us) Help (https://www.pastest.com/help)

© Pastest 2016

Page 462: Back to Filters (/Secure/TestMe/Filter ... - 1 File Download

8/11/2016 MyPastest

https://mypastest.pastest.com/Secure/TestMe/Browser/429893#Top 1/2

Back to Filters (/Secure/TestMe/Filter/429893/QA)

Question 52 of 207

46909

A 60-year-old lady, who has been an inpatient in the medical unit for several weeks, is notedto have a haemoglobin concentration of 9.9 g/dl (13–16). The blood film shows red cellfragments and you suspect haemolysis. The haematology specialist registrar has asked for alist of all her medications because she suspects a drug cause.

Which of the following drugs is most likely to cause haemolysis and anaemia?

Explanation

The answer is Penicillin –

Drug-induced haemolysis may be related to underlying genetic causes, for exampleglucose-6-phophate dehydrogenase (G6PD) deficiency predisposes to haemolysis whenpatients receive dapsone, primaquine, aspirin or quinolones. Acquired autoimmunehaemolysis is a recognised complication of methyldopa, penicillin, quinine and quinidine.Direct (non-immune) red cell toxicity may occur after lead exposure.

Atenolol (Option A) is incorrect. Atenolol may cause bradyarrhythmia and bronchospasm butanaemia is not a recognised complication.

Captopril (Option B) is incorrect. Captopril may cause renal failure.

Erythromycin (Option C) is incorrect. Erythromycin is a powerful liver enzyme inhibitor;certain preparations may cause cholestasis.

Verapamil (Option E) is incorrect. Verapamil may cause bradycardia and heart block, andgastrointestinal upset, but it is not a recognised cause of haemolysis.

AtenololA

CaptoprilB

ErythromycinC

PenicillinD

VerapamilE

Page 463: Back to Filters (/Secure/TestMe/Filter ... - 1 File Download

8/11/2016 MyPastest

https://mypastest.pastest.com/Secure/TestMe/Browser/429893#Top 2/2

Next Question

Previous Question Tag Question Feedback End Review

Difficulty: Average

Peer Responses

Session Progress

0Responses Correct:

207Responses Incorrect:

207Responses Total:

0%Responses - % Correct:

Blog (https://www.pastest.com/blog) About Pastest (https://www.pastest.com/about-us)Contact Us (https://www.pastest.com/contact-us) Help (https://www.pastest.com/help)

© Pastest 2016

Page 464: Back to Filters (/Secure/TestMe/Filter ... - 1 File Download

8/11/2016 MyPastest

https://mypastest.pastest.com/Secure/TestMe/Browser/429893#Top 1/2

Back to Filters (/Secure/TestMe/Filter/429893/QA)

Question 53 of 207

46813

A patient with metastatic carcinoma of the breast is admitted for chemotherapy. Her historyreveals that she has had several deep vein thromboses in the past, as well as a pulmonaryembolism when she was younger.

Which of the following chemotherapeutic agents should be particularly avoided in her case?

Explanation

The answer is Tamoxifen –

Tamoxifen is a partial oestrogen agonist and can increase the risk of thromboembolism,particularly during and immediately after major surgery or periods of immobility.

Anastrozole (Option A) is incorrect. Anastrazole reduces sex hormone concentrations anddoes not increase the risk of venous thromboembolism.

Buserelin (Option B) is incorrect. Buserelin reduces sex hormone concentrations and doesnot increase the risk of venous thromboembolism.

Goserelin (Option C) is incorrect. Goserelin reduces sex hormone concentrations and doesnot increase the risk of venous thromboembolism.

Letrozole (Option D) is incorrect. Letrozole reduces oestrogen concentrations and does notincrease the risk of venous thromboembolism.

AnastrozoleA

BuserelinB

GoserelinC

LetrozoleD

TamoxifenE

Next Question

Page 465: Back to Filters (/Secure/TestMe/Filter ... - 1 File Download

8/11/2016 MyPastest

https://mypastest.pastest.com/Secure/TestMe/Browser/429893#Top 2/2

Previous Question Tag Question Feedback End Review

Difficulty: Average

Peer Responses

Session Progress

0Responses Correct:

207Responses Incorrect:

207Responses Total:

0%Responses - % Correct:

Blog (https://www.pastest.com/blog) About Pastest (https://www.pastest.com/about-us)Contact Us (https://www.pastest.com/contact-us) Help (https://www.pastest.com/help)

© Pastest 2016

Page 466: Back to Filters (/Secure/TestMe/Filter ... - 1 File Download

8/11/2016 MyPastest

https://mypastest.pastest.com/Secure/TestMe/Browser/429893#Top 1/2

Back to Filters (/Secure/TestMe/Filter/429893/QA)

Question 54 of 207

Which of the following is the main reaction involved in the normal metabolism ofparacetamol?

Explanation

The answer is Conjugation to glucoronic acid –

Paracetamol is a commonly used painkiller, and when taken in overdose may causesevere and fatal liver damage. The key pathways responsible for metabolism areglucuronidation and sulfation. In acute overdose or when the maximum daily dose isexceeded over a prolonged period, these pathways become saturated so that anincreasing quantity is metabolised via non-specific P450 oxidative metabolism. Excessparacetamol is then oxidatively metabolised in the liver via the mixed function oxidaseP450 system to a toxic metabolite, N-acetyl-p-benzoquinone imine (NAPQI). NAPQI hasan extremely short half-life and is rapidly conjugated wtih glutathione, a sulfhydryl donor,and is renally excreted. Under conditions of excessive NAPQI formation or reducedglutathione stores, NAPQI covalently binds to vital proteins and the lipid bilayer ofhepatocyte membranes. This results in hepatocellular death and centrilobular livernecrosis.

Acetylation (Option A) is incorrect. Acetylation does not contribute to paracetamolmetabolism.

Conjugation to glutathione (Option C) is incorrect. Glutathione forms conjugates with theNAPQI metabolite rather than paracetamol itself.

AcetylationA

Conjugation to glucuronic acidB

Conjugation to glutathioneC

Cytochrome P450-dependent oxidationD

HydrolysisE

Page 467: Back to Filters (/Secure/TestMe/Filter ... - 1 File Download

8/11/2016 MyPastest

https://mypastest.pastest.com/Secure/TestMe/Browser/429893#Top 2/2

46936

Cytochrome P450-dependent oxidation (Option D) is incorrect. Oxidative metabolismaccounts for only a small proportion of paracetamol metabolism after therapeutic doses.

Hydrolysis (Option E) is incorrect. This is not relevant to paracetamol metabolism.

Next Question

Previous Question Tag Question Feedback End Review

Difficulty: Average

Peer Responses

Session Progress

0Responses Correct:

207Responses Incorrect:

207Responses Total:

0%Responses - % Correct:

Blog (https://www.pastest.com/blog) About Pastest (https://www.pastest.com/about-us)Contact Us (https://www.pastest.com/contact-us) Help (https://www.pastest.com/help)

© Pastest 2016

Page 468: Back to Filters (/Secure/TestMe/Filter ... - 1 File Download

8/11/2016 MyPastest

https://mypastest.pastest.com/Secure/TestMe/Browser/429893#Top 1/2

Back to Filters (/Secure/TestMe/Filter/429893/QA)

Question 55 of 207

A 72-year-old woman is admitted with ureteric colic. There is a past history of epilepsy butshe has been fit-free for nearly 15 years. This patient has suffered multiple renal stones andhas chronic renal impairment with a serum creatinine of 210 μmol/l. You prescribe pethidinefor pain relief and she initially settles. However, you are asked to see her urgently on call thefollowing morning as she has suffered a generalised seizure.

What is the likely cause of the generalised seizure?

Explanation

The answer is Accumulation of norpethidine leading to toxicity –

Pethidine is metabolised to norpethidine, but in patients with renal impairmentnorpethidine accumulates rather than being excreted through the kidneys. Norpethidineis toxic and is associated with a risk of seizures.

Accumulation of morphine 6-glucuronide after pethidine administration (Option A) isincorrect. Morphine 6-glucuronide accumulates in renal failure when patients are treated withmorphine, increasing the likelihood of opiod toxicity, but this is irrelevant to administration ofpethidine.

Accumulation of pethidine leading to toxicity (Option C) is incorrect. It is the metabolite ofpethidine rather than pethidine itself that causes seizures.

An epileptic fit in keeping with her history of previous epilepsy (Option D) is incorrect.Seizures happen in patients with epilepsy, but this is a less satisfactory explanation in thissituation, as norpethedine will have provoked seizure.

Accumulation of morphine 6-glucuronide after pethidine administrationA

Accumulation of norpethidine leading to toxicityB

Accumulation of pethidine leading to toxicityC

An epileptic fit in keeping with her history of previous epilepsyD

PseudoseizureE

Page 469: Back to Filters (/Secure/TestMe/Filter ... - 1 File Download

8/11/2016 MyPastest

https://mypastest.pastest.com/Secure/TestMe/Browser/429893#Top 2/2

46833

Pseudoseizure (Option E) is incorrect. There is no association between pethidine andpseudoseizures.

Next Question

Previous Question Tag Question Feedback End Review

Difficulty: Average

Peer Responses

Session Progress

0Responses Correct:

207Responses Incorrect:

207Responses Total:

0%Responses - % Correct:

Blog (https://www.pastest.com/blog) About Pastest (https://www.pastest.com/about-us)Contact Us (https://www.pastest.com/contact-us) Help (https://www.pastest.com/help)

© Pastest 2016

Page 470: Back to Filters (/Secure/TestMe/Filter ... - 1 File Download

8/11/2016 MyPastest

https://mypastest.pastest.com/Secure/TestMe/Browser/429893#Top 1/2

Back to Filters (/Secure/TestMe/Filter/429893/QA)

Question 56 of 207

46902

A 39-year-old lady is noted to have a low serum sodium concentration of 127 mmol/l (137–144), as well as a high urine osmolality. A diagnosis of syndrome of inappropriate antidiuretichormone secretion (SIADH) is made, and a drug-related cause is suspected.

Which of the following drugs is most likely to be responsible?

Explanation

The answer is Carbamazepine –

Antidiuretic hormone (ADH) secretion from the posterior pituitary can be stimulated bysome drugs, notably carbamazepine. The responsiveness of the collecting ducts to ADHcan be sensitised by rifampicin and sulfonylureas, especially chlorpropamide.

Chlorpropramide (Option B) is incorrect. Chlorpropamide may cause SIADH by increasingthe sensitivity of collecting duct ADH receptors, but is a less noticeable adverse effect thanthat of carbamazepine.

Demeclocycline (Option C) is incorrect. Demeclocycline interferes with renal responsivenessto ADH and is used as a treatment of SIADH, although it may cause diabetes insipidus.

Lithium (Option D) is incorrect. Lithium typically causes nephrogenic diabetes insipidus,especially after long-term use. SIADH is a very rare but recognised complication of lithiumdue to increased secretion of ADH by the posterior pituitary.

Rifampicin (Option E) is incorrect. Rifampicin is a recognised cause of SIADH, but occursmuch less commonly than after carbamazepine, hence option A is the preferred answer.

CarbamazepineA

ChlorpropamideB

DemeclocyclineC

LithiumD

RifampicinE

Page 471: Back to Filters (/Secure/TestMe/Filter ... - 1 File Download

8/11/2016 MyPastest

https://mypastest.pastest.com/Secure/TestMe/Browser/429893#Top 2/2

Next Question

Previous Question Tag Question Feedback End Review

Difficulty: Average

Peer Responses

Session Progress

0Responses Correct:

207Responses Incorrect:

207Responses Total:

0%Responses - % Correct:

Blog (https://www.pastest.com/blog) About Pastest (https://www.pastest.com/about-us)Contact Us (https://www.pastest.com/contact-us) Help (https://www.pastest.com/help)

© Pastest 2016

Page 472: Back to Filters (/Secure/TestMe/Filter ... - 1 File Download

8/11/2016 MyPastest

https://mypastest.pastest.com/Secure/TestMe/Browser/429893#Top 1/2

Back to Filters (/Secure/TestMe/Filter/429893/QA)

Question 57 of 207

A 60-year-old man, who has been taking warfarin for 3 years for atrial fibrillation, with apreviously stable INR, presents for review. He noticed some bruising on his arms fromworking in the garden. INR is now markedly elevated at 7.0

Which of the following interactions with warfarin is the most likely cause of this clinicalpicture?

Explanation

The answer is Cranberry juice –

A warning was given by the Committee on Safety of Medicines in 2003 about thepossible interaction between cranberry juice and warfarin levels. The cause is thought tobe bioflavonoids contained in the cranberry juice, which block cytochrome-P450-relatedwarfarin metabolism (CYP2C9). Of note, grapefruit juice interferes with CYP3A4metabolism, which affects levels of a number of drugs, including ciclosporin andsimvastatin.

Carrot juice (Option A) is incorrect. There is no significant interaction between warfarin andcarrot juice.

Orange juice (Option C) is incorrect. There is no significant interaction between warfarin andorange juice.

St John’s wort (Option D) is incorrect. St John’s Wort is an enzyme inducer and regulartreatment will lead to reduced levels of warfarin and INR.

Carrot juiceA

Cranberry juiceB

Orange juiceC

St John’s wortD

Tomato juiceE

Page 473: Back to Filters (/Secure/TestMe/Filter ... - 1 File Download

8/11/2016 MyPastest

https://mypastest.pastest.com/Secure/TestMe/Browser/429893#Top 2/2

46971

Tomato juice (Option E) is incorrect. There is no significant interaction between warfarin andtomato juice.

Next Question

Previous Question Tag Question Feedback End Review

Difficulty: Average

Peer Responses

Session Progress

0Responses Correct:

207Responses Incorrect:

207Responses Total:

0%Responses - % Correct:

Blog (https://www.pastest.com/blog) About Pastest (https://www.pastest.com/about-us)Contact Us (https://www.pastest.com/contact-us) Help (https://www.pastest.com/help)

© Pastest 2016

Page 474: Back to Filters (/Secure/TestMe/Filter ... - 1 File Download

8/11/2016 MyPastest

https://mypastest.pastest.com/Secure/TestMe/Browser/429893#Top 1/2

Back to Filters (/Secure/TestMe/Filter/429893/QA)

Question 58 of 207

You are asked by the hospital formulary to review a pharmaceutical company’s application fora new drug to be added to the list of approved medications. The drug is a combinationproduct made up of two long-standing drugs and the firm claims bioequivalence.

What is the best definition of bioequivalence in this context?

Explanation

The answer is The two drugs compared have the same pharmacokinetic andpharmacodynamic effects –

Bioequivalence is all about demonstrating similar biological effect, including bothpharmacodynamic and pharmacokinetic properties. Considering only pharmacokinetic orpharmacodynamic characteristics in isolation provides less comparative data. There maybe differences between products in excipients or delivery vehicle, which means that thebiological effects are different when the products are compared. Combination productsbring different issues.

The two drugs compared contain the same active components (Option A) is incorrect.Containing the same active chemicals is not sufficient to ensure bioequivalence owing todifferences in excipients or formulation that might affect pharmacokinetic orpharmacodynamic properties.

The two drugs compared contain the same active componentsA

The two drugs compared contain the same ingredients and have the samepharmacodynamics

B

The two drugs compared contain the same ingredients and have the samepharmacokinetics

C

The two drugs compared have similar bioavailabilityD

The two drugs compared have the same pharmacokinetic and pharmacodynamiceffects

E

Page 475: Back to Filters (/Secure/TestMe/Filter ... - 1 File Download

8/11/2016 MyPastest

https://mypastest.pastest.com/Secure/TestMe/Browser/429893#Top 2/2

46843

The two drugs compared contain the same ingredients and have the samepharmacodynamics (Option B) is incorrect. Pharmacodynamic equivalence alone isinsufficient to ensure bioequivalence.

The two drugs compared contain the same ingredients and have the same pharmacokinetics(Option C) is incorrect. Pharmacokinetic equivalence alone is insufficient to ensurebioequivalence.

The two drugs compared have similar bioavailability (Option D) is incorrect. Bioavailability,one aspect of pharmacokinetic characteristics, does not ensure bioequivalence.

Next Question

Previous Question Tag Question Feedback End Review

Difficulty: Average

Peer Responses

Session Progress

0Responses Correct:

207Responses Incorrect:

207Responses Total:

0%Responses - % Correct:

Blog (https://www.pastest.com/blog) About Pastest (https://www.pastest.com/about-us)Contact Us (https://www.pastest.com/contact-us) Help (https://www.pastest.com/help)

© Pastest 2016

Page 476: Back to Filters (/Secure/TestMe/Filter ... - 1 File Download

8/11/2016 MyPastest

https://mypastest.pastest.com/Secure/TestMe/Browser/429893#Top 1/2

Back to Filters (/Secure/TestMe/Filter/429893/QA)

Question 59 of 207

46769

You are the medical registrar on-call and asked to review a 46-year-old patient who isreceiving medications for a severe respiratory tract infection. The nurse has noted that hisurine has become discoloured orange–red and is worried that this might be related to histreatment.

Which of the following drugs would be most likely to cause this?

Explanation

The answer is Rifampicin –

Patients on rifampicin should be warned that their urine, tears and other secretions willdevelop a bright orange–red colour. This is dose-dependent and fully reverses oncetreatment ceases.

B complex vitamins (Option A) is incorrect. B-complex vitamins may darken the urine, but donot produce widespread changes in other body secretions.

Erythromycin (Option B) is incorrect. Erythromycin does not cause a change in urine colour.

Nelfinavir (Option C) is incorrect. Nelfinavir, a protease inhibitor, may be associated withhaematuria (dark or bright red discoloration) and crystalluria owing to renal stone formation.There is no effect on other bodily secretions. The occurrence of discoloration is much lesscommon than with rifampicin, hence option E is the preferred answer.

Phenolphthalein (Option D) is incorrect. Phenolphthalein stains alkaline urine pink.

B complex vitaminsA

ErythromycinB

NelfinavirC

PhenolphthaleinD

RifampicinE

Page 477: Back to Filters (/Secure/TestMe/Filter ... - 1 File Download

8/11/2016 MyPastest

https://mypastest.pastest.com/Secure/TestMe/Browser/429893#Top 2/2

Next Question

Previous Question Tag Question Feedback End Review

Difficulty: Average

Peer Responses

Session Progress

0Responses Correct:

207Responses Incorrect:

207Responses Total:

0%Responses - % Correct:

Blog (https://www.pastest.com/blog) About Pastest (https://www.pastest.com/about-us)Contact Us (https://www.pastest.com/contact-us) Help (https://www.pastest.com/help)

© Pastest 2016

Page 478: Back to Filters (/Secure/TestMe/Filter ... - 1 File Download

8/11/2016 MyPastest

https://mypastest.pastest.com/Secure/TestMe/Browser/429893#Top 1/2

Back to Filters (/Secure/TestMe/Filter/429893/QA)

Question 60 of 207

46858

A 35-year-old Asian man is admitted to hospital with fever and rigors. Blood films arereported as showing the presence of Plasmodium vivax and you diagnose acute malaria.

Which of the following antimalarial treatments is most likely to be a slow-actingschizonticide?

Explanation

The answer is Pyrimethamine –

Antimalarials that act on the erythrocytic phase of schizogony are called ‘schizonticides’.Antimalarials may be considered as fast-acting, high-efficacy blood schizonticides thatmay be effective as monotherapy, or slow-acting, low-efficacy schizonticides thatnormally need to be administered in combination. Pyrimethamine is used in thetreatment of uncomplicated malaria, particularly for chloroquine-resistant P. falciparum.It acts on both the erythrocytic and hepatic phases of infection. It inhibits dihydrofolatereductase in the parasite thus preventing the biosynthesis of purines and pyrimidines,and thereby halting the processes of DNA replication, cell division and reproduction. It isnormally used alongside a sulfonamide. Doxycycline is also a very slow-actingantimalarial.

Artemesinin (Option A) is incorrect. Artemisinin is a fast-acting schizonticides.

Mefloquine (Option B) is incorrect. Mefloquine is an intermediate-acting schizonticide.

Mepacrine (Option C) is incorrect. Mepacrine is a fast-acting schizonticide.

Quinine (Option E) is incorrect. Quinine is an intermediate-acting schizonticide.

ArtemesininA

MefloquineB

MepacrineC

PyrimethamineD

QuinineE

Page 479: Back to Filters (/Secure/TestMe/Filter ... - 1 File Download

8/11/2016 MyPastest

https://mypastest.pastest.com/Secure/TestMe/Browser/429893#Top 2/2

46858

Next Question

Previous Question Tag Question Feedback End Review

Difficulty: Average

Peer Responses

Session Progress

0Responses Correct:

207Responses Incorrect:

207Responses Total:

0%Responses - % Correct:

Blog (https://www.pastest.com/blog) About Pastest (https://www.pastest.com/about-us)Contact Us (https://www.pastest.com/contact-us) Help (https://www.pastest.com/help)

© Pastest 2016

Page 480: Back to Filters (/Secure/TestMe/Filter ... - 1 File Download

8/11/2016 MyPastest

https://mypastest.pastest.com/Secure/TestMe/Browser/429893#Top 1/2

Back to Filters (/Secure/TestMe/Filter/429893/QA)

Question 61 of 207

A 25-year-old woman is diagnosed as having Graves’ disease with hyperthyroidism. She isprescribed carbimazole treatment, and she has a number of questions concerning thepossible adverse effects.

Which of the following serious adverse effects are most likely to occur after carbimazole?

Explanation

The answer is Reversible agranulocytosis –

The active metabolite of carbimazole, methimazole, reduces the synthesis of new thyroidhormones by inhibiting the iodination of tyrosine and blocks the coupling ofiodotyrosine. Methimazole also has a minor immunosuppressive action, leading to areduction in serum thyroid-stimulating-hormone-receptor antibody (TRAb)concentrations. Symptoms improve within 10–14 days, and euthyroid levels achieved after3–4 weeks. Adverse effects are usually seen within 7–28 days of starting treatment.Agranulocytosis cannot be predicted by routine white cell counts, but is fortunatelyreversible when the drug is discontinued; patients are warned to stop the drugimmediately and contact their doctor should they develop a sore throat or fever.

Cholestatic jaundice (Option A) is incorrect. Cholestatic jaundice is a recognised feature butthis is very uncommon, and much less clinically significant than agranulocytosis.

Hepatitis (Option B) is incorrect. Hepatitis is a very rare complication of carbimazoletreatment.

Hypoprothrombinaemia (Option C) is incorrect. Hypoprothombinaemia may occur as thyroidfunction falls to normal, but the clinical impact is very small, hence reversible

Cholestatic jaundiceA

HepatitisB

HypoprothrombinaemiaC

ImmunosuppressionD

Reversible agranulocytosisE

Page 481: Back to Filters (/Secure/TestMe/Filter ... - 1 File Download

8/11/2016 MyPastest

https://mypastest.pastest.com/Secure/TestMe/Browser/429893#Top 2/2

46739

agranulocystosis is the preferred answer.

Immunosuppression (Option D) is incorrect. Although an immunosuppressive effect is notedconcerning TSH receptor antibodies, the clinical significance is uncertain. Agranulocytosis isa better characterised adverse effect and hence is the preferred answer.

Next Question

Previous Question Tag Question Feedback End Review

Difficulty: Average

Peer Responses

Session Progress

0Responses Correct:

207Responses Incorrect:

207Responses Total:

0%Responses - % Correct:

Blog (https://www.pastest.com/blog) About Pastest (https://www.pastest.com/about-us)Contact Us (https://www.pastest.com/contact-us) Help (https://www.pastest.com/help)

© Pastest 2016

Page 482: Back to Filters (/Secure/TestMe/Filter ... - 1 File Download

8/11/2016 MyPastest

https://mypastest.pastest.com/Secure/TestMe/Browser/429893#Top 1/2

Back to Filters (/Secure/TestMe/Filter/429893/QA)

Question 62 of 207

A 51-year-old man with a history of alcoholism comes to the Cardiology Clinic for review. Hehas failed a cardioversion for atrial fibrillation, and despite trying Bisoprolol and Flecainide,remains in an irregular rhythm. On examination in the clinic his BP is 108/72 mmHg, pulse is70/min (irregular). His chest is clear, there is minor bilateral pitting oedema affecting bothankles. He has some spider naevi on examination of the torso.

Investigations:

Hb 10.9 g/dl

WCC 9.2 x10 /l

PLT 211 x10 /l

Na 137 mmol/l

K 4.3 mmol/l

Creatinine 88 micromol/l

CXR reveals cardiomegaly

You are considering anti-coagulating him.

Which of the following is the strongest predictor of bleeding in this situation?

Explanation

The answer is AST/ALT ratio -

9

9

+

+

AST/ALT ratioA

History of depressionB

Serum albumin < 35 g/lC

Signs of fluid overloadD

Units alcohol per weekE

Page 483: Back to Filters (/Secure/TestMe/Filter ... - 1 File Download

8/11/2016 MyPastest

https://mypastest.pastest.com/Secure/TestMe/Browser/429893#Top 2/2

40174

One study, the Veterans AffaiRs Study to Improve Anticoagulation (VARIA) database of103,897 patients receiving warfarin across 100 sites, provides us with useful data in thisrespect. Among alcohol abusers, elevated AST: ALT >2.0 corresponded to more than threetimes the number of haemorrhages (HR 3.02, p?<?0.001 compared to nonusers). It is alsolikely that low albumin (i.e.<30), an indicator of impaired synthetic function is also associatedwith risk of bleeding, although AST/ALT is the strongest predictor.

http://www.ncbi.nlm.nih.gov/pubmed/23620189(http://www.ncbi.nlm.nih.gov/pubmed/23620189)

Next Question

Previous Question Tag Question Feedback End Review

Difficulty: Difficult

Peer Responses

Session Progress

0Responses Correct:

207Responses Incorrect:

207Responses Total:

0%Responses - % Correct:

Blog (https://www.pastest.com/blog) About Pastest (https://www.pastest.com/about-us)Contact Us (https://www.pastest.com/contact-us) Help (https://www.pastest.com/help)

© Pastest 2016

Page 484: Back to Filters (/Secure/TestMe/Filter ... - 1 File Download

8/11/2016 MyPastest

https://mypastest.pastest.com/Secure/TestMe/Browser/429893#Top 1/2

Back to Filters (/Secure/TestMe/Filter/429893/QA)

Question 63 of 207

46744

An 18-year-old young woman presents with a 4-day history of cough, headache, fever andjoint pains. Blood tests show the presence of raised antibody titres and the presence of coldagglutinins. A diagnosis of Mycoplasma pneumoniae infection is made.

Which drug would be the most appropriate first-line treatment for this patient?

Explanation

The answer is Clarithromycin –

Clarithromycin or tetracyclines are preferred for Mycoplasma infections; tetracyclines arealso used to treat a Coxiella burnetii infection and psittacosis.

Cefuroxime (Option A) is incorrect. Cephalosporins are inactive against Mycoplasmainfection.

Co-trimoxazole (Option C) is incorrect. Co-trimoxazole is given in Pneumocystis jirovecii(previously known as Pneumocystis carinii) infection.

Penicillin (Option D) is incorrect. Penicillin is used commonly in pneumococcal infection butis ineffective against Mycoplasma pneumoniae.

Rifampicin (Option E) is incorrect. Rifampicin therapy may be effective in severe cases ofMycoplasma pneumoniae but would rarely be considered as first line owing to a greater rateof occurrence of adverse effects.

CefuroximeA

ClarithromycinB

Co-trimoxazoleC

PenicillinD

RifampicinE

Next Question

Page 485: Back to Filters (/Secure/TestMe/Filter ... - 1 File Download

8/11/2016 MyPastest

https://mypastest.pastest.com/Secure/TestMe/Browser/429893#Top 2/2

Previous Question Tag Question Feedback End Review

Difficulty: Average

Peer Responses

Session Progress

0Responses Correct:

207Responses Incorrect:

207Responses Total:

0%Responses - % Correct:

Blog (https://www.pastest.com/blog) About Pastest (https://www.pastest.com/about-us)Contact Us (https://www.pastest.com/contact-us) Help (https://www.pastest.com/help)

© Pastest 2016

Page 486: Back to Filters (/Secure/TestMe/Filter ... - 1 File Download

8/11/2016 MyPastest

https://mypastest.pastest.com/Secure/TestMe/Browser/429893#Top 1/2

Back to Filters (/Secure/TestMe/Filter/429893/QA)

Question 64 of 207

A 60-year-old patient has been under regular blood pressure monitoring by the practicenurse, and a decision is made to commence antihypertensive therapy. Two days later hepresents to the Emergency Department with a severe dizziness on standing, fast palpitationsand lightheadedness.

Which one of the following drugs is most likely to account for these symptoms?

Explanation

The answer is Doxazosin –

Alpha-selective antagonists in particular can cause significant vasodilatation, orthostatichypotension and postural hypotension. Patients require careful dose titration and shouldbe warned of the possibility of postural dizziness. The same side-effects may also beobserved with ACE inhibitors, although these tend to lessen with continued therapy (firstdose hypotension).

Amlodipine (Option A) is incorrect. Dihydropyridine-type (eg nifedipine) calcium-channelblockers may cause vasodilatation and postural dizziness, but to a lesser extent thanrecognised after α-blockers or ACE inhibitors.

Atenolol (Option B) is incorrect. Atenolol lowers blood pressure by significantly reducingcardiac output and heart rate; it tends to increase peripheral vascular resistance, which iscounterproductive as monotherapy but makes the addition of a vasodilating drug a logicalchoice.

Hydrochlorothiazide (Option D) is incorrect. Hydrochlorothiazide is expected to cause somevolume depletion, but its blood pressure lowering effects are of gradual onset over several

AmlodipineA

AtenololB

DoxazosinC

HydrochlorothiazideD

VerapamilE

Page 487: Back to Filters (/Secure/TestMe/Filter ... - 1 File Download

8/11/2016 MyPastest

https://mypastest.pastest.com/Secure/TestMe/Browser/429893#Top 2/2

46785

weeks, and less likely to cause a dramatic blood pressure drop over several days.

Verapamil (Option E) is incorrect. Verapamil predominantly blocks cardiac calcium channelsin therapeutic doses, and is unlikely to cause significant vasodilatation and would suppressheart rate.

Next Question

Previous Question Tag Question Feedback End Review

Difficulty: Average

Peer Responses

Session Progress

0Responses Correct:

207Responses Incorrect:

207Responses Total:

0%Responses - % Correct:

Blog (https://www.pastest.com/blog) About Pastest (https://www.pastest.com/about-us)Contact Us (https://www.pastest.com/contact-us) Help (https://www.pastest.com/help)

© Pastest 2016

Page 488: Back to Filters (/Secure/TestMe/Filter ... - 1 File Download

8/11/2016 MyPastest

https://mypastest.pastest.com/Secure/TestMe/Browser/429893#Top 1/2

Back to Filters (/Secure/TestMe/Filter/429893/QA)

Question 65 of 207

You review a 58-year-old man with type-2 diabetes treated with metformin, and hypertensionthat is being treated with nifedipine. His weight has increased and his glycaemic control ispoor, so you wish to escalate his medications. You consider adding a glitazone to his therapy.

Which metabolic pathway is mainly responsible for the metabolism of pioglitazone?

Explanation

The answer is CYP2C8 –

The main cytochrome P450 enzyme pathway responsible for pioglitazone metabolism isCYP2C8. Gemfibrozil and trimethoprim inhibit activity of the CYP2C8 isoenzyme and,therefore, increase pioglitazone concentrations and increase the risk of adverse effects.Pioglitazone lowers HbA1c by around 1–1.3% and raises HDL cholesterol. Its adverseeffects include weight gain, fluid retention, heart failure and reduced bone mineraldensity.

CYP2C9 (Option B) is incorrect. CYP2C9 is responsible for metabolism of many non-steroidalanti-inflammatory drugs (NSAIDs) and warfarin; it may be inhibited by fluconazole.

CYP2D6 (Option C) is incorrect. CYP2D6 is responsible for metabolism of a number ofantipsychotics, and converts codeine to morphine; it may be inhibited by fluoxetine andparoxetine.

CYP3A2 (Option D) is incorrect. CYP3A2 catalyses conversion of testosterone to 6-β-hydroxytestosterone; it may be inhibited by macrolide antibiotics.

CYP2C8A

CYP2C9B

CYP2D6C

CYP3A2D

CYP3A4E

Page 489: Back to Filters (/Secure/TestMe/Filter ... - 1 File Download

8/11/2016 MyPastest

https://mypastest.pastest.com/Secure/TestMe/Browser/429893#Top 2/2

46881

CYP3A4 (Option E) is incorrect. CYP3A4 is responsible for metabolism of a very largenumber of different drugs, including simvastatin; CYP3A4 may be inhibited by proteaseinhibitors and macrolide antibiotics.

Next Question

Previous Question Tag Question Feedback End Review

Difficulty: Average

Peer Responses

Session Progress

0Responses Correct:

207Responses Incorrect:

207Responses Total:

0%Responses - % Correct:

Blog (https://www.pastest.com/blog) About Pastest (https://www.pastest.com/about-us)Contact Us (https://www.pastest.com/contact-us) Help (https://www.pastest.com/help)

© Pastest 2016

Page 490: Back to Filters (/Secure/TestMe/Filter ... - 1 File Download

8/11/2016 MyPastest

https://mypastest.pastest.com/Secure/TestMe/Browser/429893#Top 1/2

Back to Filters (/Secure/TestMe/Filter/429893/QA)

Question 66 of 207

46998

A 42-year-old man with long-standing epilepsy presents to the clinic for review. He complainsof increasing ataxia over the past few months and is particularly distressed by bilateralDupuytren’s contractures that he is developing. On examination he has bilateral poor co-ordination, nystagmus and ataxia on walking; there is axillary and inguinal lymphadenopathy.Sensory testing reveals decreased sensation in both feet.

Which one of the following medications is most likely to be responsible for these findings?

Explanation

The answer is Phenytoin –

Phenytoin has a range of adverse effects, which include peripheral neuropathy,Dupuytren’s contractures, benign lymphadenopathy, acne and gum hyperplasia.

Carbamazepine (Option A) is incorrect. Although carbamazepine may cause ataxia, it ismuch less likely to cause peripheral neuropathy than phenytoin.

Lamotrigine (Option B) is incorrect. Lamotrigine is rarely associated with peripheralneuropathy, and there is no association with Dupuytren’s contracture.

Sodium valproate (Option D) is incorrect. Sodium valproate may be a suitable alternativemedication to substitute in this patient.

Topiramate (Option E) is incorrect. Neuropathy is rarely associated with topiramate.

CarbamazepineA

LamotrigineB

PhenytoinC

Sodium valproateD

TopiramateE

Next Question

Page 491: Back to Filters (/Secure/TestMe/Filter ... - 1 File Download

8/11/2016 MyPastest

https://mypastest.pastest.com/Secure/TestMe/Browser/429893#Top 2/2

Previous Question Tag Question Feedback End Review

Difficulty: Average

Peer Responses

Session Progress

0Responses Correct:

207Responses Incorrect:

207Responses Total:

0%Responses - % Correct:

Blog (https://www.pastest.com/blog) About Pastest (https://www.pastest.com/about-us)Contact Us (https://www.pastest.com/contact-us) Help (https://www.pastest.com/help)

© Pastest 2016

Page 492: Back to Filters (/Secure/TestMe/Filter ... - 1 File Download

8/11/2016 MyPastest

https://mypastest.pastest.com/Secure/TestMe/Browser/429893#Top 1/2

Back to Filters (/Secure/TestMe/Filter/429893/QA)

Question 67 of 207

You review a 72-year-old woman who is complaining of severe nausea and lethargy. She haschronic atrial fibrillation for which she takes digoxin 125 μg/day. Her GP has recently added athiazide diuretic to her antihypertensive regime. Serum potassium level is 3.0 mmol/l (3.5–4.9). Her pulse is 42 bpm, with a blood pressure of 122/70 mmHg.

What is the best course of action in this case?

Explanation

The answer is Stop her thiazide diuretic, temporarily stop the digoxin, and substituteanother antihypertensive agent –

It is likely that, in this patient, the recent addition of the thiazide has precipitated a fall inserum potassium concentration. The symptoms of digoxin toxicity include anorexia andnausea, often with altered vision. Arrhythmias may occur, including ventricular prematurebeats, bigeminy, ventricular tachycardia and atrioventricular (AV) block. Digoxin toxicityis potentially severe when concentrations exceed 2.5 nmol/l. Management includescautious restoration of serum potassium levels, and symptomatic management ofarrhythmias. In severe cases of toxicity, digoxin may be permanently stopped andanother antiarrhythmic substituted if needed.

Administer FAB fragment antidigoxin antibodies (Option A) is incorrect. In this case there isno circulatory compromise, and so a temporary reduction/cessation in digoxin therapy withcorrection of serum potassium is the best course of action.

Administer FAB fragment antidigoxin antibodiesA

Introduce a small dose of spironolactoneB

Permanently stop her digoxin therapyC

Start potassium supplements but continue diuretic therapyD

Stop her thiazide diuretic, temporarily stop the digoxin, and substitute anotherantihypertensive agent

E

Page 493: Back to Filters (/Secure/TestMe/Filter ... - 1 File Download

8/11/2016 MyPastest

https://mypastest.pastest.com/Secure/TestMe/Browser/429893#Top 2/2

46875

Introduce a small dose of spironolactone (Option B) is incorrect. Spironolactone may be asuitable alternative to bendroflumethiazide, but digoxin must be stopped, at leasttemporarily, which is why option B is a less good answer.

Permanently stop her digoxin therapy (Option C) is incorrect. Digoxin can normally bereintroduced, perhaps at a lower dose or after cessation of diuretic treatment.

Start potassium supplements but continue diuretic therapy (Option D) is incorrect. Short-term potassium supplements may be considered, but it would be illogical to continue diuretictreatment at this stage.

Next Question

Previous Question Tag Question Feedback End Review

Difficulty: Average

Peer Responses

Session Progress

0Responses Correct:

207Responses Incorrect:

207Responses Total:

0%Responses - % Correct:

Blog (https://www.pastest.com/blog) About Pastest (https://www.pastest.com/about-us)Contact Us (https://www.pastest.com/contact-us) Help (https://www.pastest.com/help)

© Pastest 2016

Page 494: Back to Filters (/Secure/TestMe/Filter ... - 1 File Download

8/11/2016 MyPastest

https://mypastest.pastest.com/Secure/TestMe/Browser/429893#Top 1/2

Back to Filters (/Secure/TestMe/Filter/429893/QA)

Question 68 of 207

46984

A total of 630 mg of a new investigational intravenous anaesthetic agent is injected into a 28-year-old man as part of a phase 2 study. The half-life of the agent is 30 min.

How much time will it take before the total body drug content falls below 20 mg?

Explanation

The answer is 150 min –

Half-life is the time taken for the drug concentration to fall by 50%. 20 mg isapproximately 1/32nd of a 630-mg dose given IV originally; that is to say 1/32nd = 1/2 ×1/2 × 1/2 × 1/2 × 1/2, or five sequential half-lives; 5 × 30 min = 150 min.

90 min (Option A) is incorrect. It will take five half-lives, namely 150 min.

2 h (Option B) is incorrect. It will take five half-lives, namely 150 min.

4 h (Option D) is incorrect. It will take five half-lives, namely 150 min.

8 h (Option E) is incorrect. It will take five half-lives, namely 150 min.

90 minA

2 hB

150 minC

4 hD

8 hE

Next Question

Previous Question Tag Question Feedback End Review

Page 495: Back to Filters (/Secure/TestMe/Filter ... - 1 File Download

8/11/2016 MyPastest

https://mypastest.pastest.com/Secure/TestMe/Browser/429893#Top 2/2

Difficulty: Average

Peer Responses

Session Progress

0Responses Correct:

207Responses Incorrect:

207Responses Total:

0%Responses - % Correct:

Blog (https://www.pastest.com/blog) About Pastest (https://www.pastest.com/about-us)Contact Us (https://www.pastest.com/contact-us) Help (https://www.pastest.com/help)

© Pastest 2016

Page 496: Back to Filters (/Secure/TestMe/Filter ... - 1 File Download

8/11/2016 MyPastest

https://mypastest.pastest.com/Secure/TestMe/Browser/429893#Top 1/2

Back to Filters (/Secure/TestMe/Filter/429893/QA)

Question 69 of 207

A 35-year-old woman, who is receiving fortnightly intramuscular injections for rheumatoidarthritis, presents to her GP with sore throat, cough, and tingling and numbness in her handsand feet. Full blood count shows haemoglobin 78 g/l, white cell count 1.8 × 10 /l and platelets34 × 10 /l.

Which of the following medications is most likely to account for these findings?

Explanation

The answer is Gold –

Gold (sodium aurothiomalate) is given by deep intramuscular injection, initially weekly,then fortnightly, and maintained monthly thereafter. A number of serious adverse effectsare recognised, including peripheral neuropathy, skin reactions, nephrotic syndrome,pulmonary fibrosis and aplastic pancytopenia.

Hydroxychloroquine (Option B) is incorrect. Hydroxychloroquine may cause bone marrowsuppression and pancytopenia. Gold is the preferred answer because hydroxychloroquinewould be less likely to cause neuropathy and would not be given by monthly injection (oraladministration).

Indometacin (Option C) is incorrect. Indometacin and other NSAIDs may promotegastrointestinal blood loss and iron deficiency anaemia; these are normally administeredorally.

Methotrexate (Option D) is incorrect. Methotrexate may cause hepatic and pulmonaryfibrosis; it is normally given weekly by oral, subcutaneous or intramuscular routes.

9

9

GoldA

HydroxychloroquineB

IndometacinC

MethotrexateD

SulfasalazineE

Page 497: Back to Filters (/Secure/TestMe/Filter ... - 1 File Download

8/11/2016 MyPastest

https://mypastest.pastest.com/Secure/TestMe/Browser/429893#Top 2/2

46615

Sulfasalazine (Option E) is incorrect. Sulfasalazine may cause bone marrow suppression andpancytopenia. Gold is the preferred answer because sulfasalazine would be less likely tocause neuropathy and would not be given by monthly injection (oral administration).

Next Question

Previous Question Tag Question Feedback End Review

Difficulty: Average

Peer Responses

Session Progress

0Responses Correct:

207Responses Incorrect:

207Responses Total:

0%Responses - % Correct:

Blog (https://www.pastest.com/blog) About Pastest (https://www.pastest.com/about-us)Contact Us (https://www.pastest.com/contact-us) Help (https://www.pastest.com/help)

© Pastest 2016

Page 498: Back to Filters (/Secure/TestMe/Filter ... - 1 File Download

8/11/2016 MyPastest

https://mypastest.pastest.com/Secure/TestMe/Browser/429893#Top 1/2

Back to Filters (/Secure/TestMe/Filter/429893/QA)

Question 70 of 207

A 40-year-old man has undergone bone marrow transplant treatment for leukaemia.Unfortunately, his recovery has been complicated by the occurrence of systemic fungalinfection, and he is prescribed amphotericin.

Which of the following statements best describes the pharmacological characteristic of thisdrug?

Explanation

The answer is Lipid-bound preparations cause fewer adverse effects –

Amphotericin is a polyene antifungal agent that binds to ergosterol and thereby formsmicropores. These increase fungal cell wall permeability and cause lysis. Amphotericinmay exert either fungicidal or fungistatic activity, depending on its concentration at thesite of infection and sensitivity of the organism. It is administered intravenously andadverse effects are common, the most important being dose-related nephrotoxicity andhypokalaemia. Acute infusion-related adverse effects include fever and rigors,hypotension, nausea and vomiting, headache, and thrombophlebitis. Adverse effectsassociated with long-term therapy include anaemia, and renal tubular loss of potassium,sodium and magnesium. Lipisomal preparations are associated with less toxicity.

Adverse effects are idiosyncratic and unpredictable (Option A) is incorrect. Adverse effectsare dose and concentration dependent, and therefore predictable.

Adverse effects are uncommon after intravenous use (Option B) is incorrect. Adverse effectsoccur commonly.

Adverse effects are idiosyncratic and unpredictableA

Adverse effects are uncommon after intravenous useB

It has a low affinity for ergosterol present in the fungal cell wallC

It may be administered intramuscularlyD

Lipid-bound preparations cause fewer adverse effectsE

Page 499: Back to Filters (/Secure/TestMe/Filter ... - 1 File Download

8/11/2016 MyPastest

https://mypastest.pastest.com/Secure/TestMe/Browser/429893#Top 2/2

46863

It has a low affinity for ergosterol present in the fungal cell wall (Option C) is incorrect.Ergosterol is a major therapeutic target for amphotericin.

It may be administered intramuscularly (Option D) is incorrect. Amphotericin may only beadministered by intravenous route.

Next Question

Previous Question Tag Question Feedback End Review

Difficulty: Average

Peer Responses

Session Progress

0Responses Correct:

207Responses Incorrect:

207Responses Total:

0%Responses - % Correct:

Blog (https://www.pastest.com/blog) About Pastest (https://www.pastest.com/about-us)Contact Us (https://www.pastest.com/contact-us) Help (https://www.pastest.com/help)

© Pastest 2016

Page 500: Back to Filters (/Secure/TestMe/Filter ... - 1 File Download

8/11/2016 MyPastest

https://mypastest.pastest.com/Secure/TestMe/Browser/429893#Top 1/2

Back to Filters (/Secure/TestMe/Filter/429893/QA)

Question 71 of 207

46601

A 49-year-old man has returned to the hypertension clinic for review. He is complaining of anirritating dry cough that has developed after he commenced treatment with lisinopril.

Which of the following mechanisms is most likely to account for development of cough?

Explanation

The answer is Excess bradykinin within lung parenchyma –

ACE inhibitors prevent the breakdown of bradykinin within the lungs, and coughing isthought to be triggered by excess bradykinin that accumulates. Cough due to ACEinhibitors is reported in 10% of men and 20% of women, although the reason for a genderdifference is unclear. Other adverse effecs of ACE inhibitors include dysgeusia (anunpleasant metallic taste), postural hypotension, first dose hypotension and renal failure.

Bronchoconstriction (Option A) is incorrect. This is not a recognised feature.

Hypersalivation (Option C) is incorrect. This is not a recognised feature.

Increased bronchial mucous secretion (Option D) is incorrect. This is not a recognisedfeature.

Vasodilation causing pulmonary congestion (Option E) is incorrect. ACE inhibitors are knownto cause systemic vasodilation and possibly also within the pulmonary vasculature, but this isnot thought to be relevant to the development of cough.

BronchoconstrictionA

Excess bradykinin within lung parenchymaB

HypersalivationC

Increased bronchial mucous secretionD

Vasodilation causing pulmonary congestionE

Next Question

Page 501: Back to Filters (/Secure/TestMe/Filter ... - 1 File Download

8/11/2016 MyPastest

https://mypastest.pastest.com/Secure/TestMe/Browser/429893#Top 2/2

Previous Question Tag Question Feedback End Review

Difficulty: Average

Peer Responses

Session Progress

0Responses Correct:

207Responses Incorrect:

207Responses Total:

0%Responses - % Correct:

Blog (https://www.pastest.com/blog) About Pastest (https://www.pastest.com/about-us)Contact Us (https://www.pastest.com/contact-us) Help (https://www.pastest.com/help)

© Pastest 2016

Page 502: Back to Filters (/Secure/TestMe/Filter ... - 1 File Download

8/11/2016 MyPastest

https://mypastest.pastest.com/Secure/TestMe/Browser/429893#Top 1/2

Back to Filters (/Secure/TestMe/Filter/429893/QA)

Question 72 of 207

A 17-year-old boy presents to the endocrinology outpatient clinic for investigation ofanosmia. His parents mention that he is childlike and has not developed secondary sexualcharacteristics like his peers. On examination he is overweight but euthyroid, and you notethe absence of secondary hair growth.

What treatment would you consider most appropriate in this condition?

Explanation

The answer is Human chorionic gonadotrophin –

This 17-year-old boy, presenting with obesity, anosmia and failure to develop secondarysexual characteristics, most probably has Kallmann syndrome. This syndrome is acombination of anosmia, obesity and hypogonadotrophic hypogonadism, and is an X-linked recessive disorder that causes an isolated deficiency of gonadotrophin-releasinghormone (GnRH). Long-term treatment of males is with human chorionic gonadotrophinor testosterone, which allows restoration of pubertal development and secondary sexcharacteristics.

Buserelin (Option A) is incorrect. Buserelin is a GnRH analogue that initially stimulates andlater inhibits release of FSH and LH, and is used to suppress the growth of some hormone-sensitive tumours, eg prostate cancer. Pulsed GnRH analogue therapy may, however, be usedin an attempt to maintain secondary sexual characteristics and fertility in males who requireit.

Cyclic oestrogen and progestogen (Option B) is incorrect. Cyclic oestrogen and progestogenare indicated only for female patients.

BuserelinA

Cyclic oestrogen and progestogenB

Human chorionic gonadotrophinC

Nafarelin acetateD

OctreotideE

Page 503: Back to Filters (/Secure/TestMe/Filter ... - 1 File Download

8/11/2016 MyPastest

https://mypastest.pastest.com/Secure/TestMe/Browser/429893#Top 2/2

46750

Nafarelin acetate (Option D) is incorrect. Nafarelin acetate is a GnRH analogue that inhibitspituitary release of FSH and LH, and is used to suppress hormone-sensitive disordersincluding endometriosis and uterine fibroids.

Octreotide (Option E) is incorrect. Octreotide is a long-acting analogue of somatostatin andis used in the treatment of acromegaly. It also suppresses the luteinising hormone responseto GnRH.

Next Question

Previous Question Tag Question Feedback End Review

Difficulty: Average

Peer Responses

Session Progress

0Responses Correct:

207Responses Incorrect:

207Responses Total:

0%Responses - % Correct:

Blog (https://www.pastest.com/blog) About Pastest (https://www.pastest.com/about-us)Contact Us (https://www.pastest.com/contact-us) Help (https://www.pastest.com/help)

© Pastest 2016

Page 504: Back to Filters (/Secure/TestMe/Filter ... - 1 File Download

8/11/2016 MyPastest

https://mypastest.pastest.com/Secure/TestMe/Browser/429893#Top 1/2

Back to Filters (/Secure/TestMe/Filter/429893/QA)

Question 73 of 207

You review a 45-year-old woman in the respiratory clinic for review of her asthma symptoms.After discussion with the consultant, you decide to prescribe aminophylline. The patient asksyou about the possible side effects of the medication.

Which of the following adverse effects is most likely to occur?

Explanation

The answer is Agitation and insomnia –

Theophylline is the pharmacologically active molecule but is sparingly soluble in water;aminophylline is a combination of theophylline and ethylenediamine that is water solubleand increases theophylline absorption around 20-fold. Aminophylline is normallyreserved for patients with severe asthma because there is a high rate of occurrence ofadverse effects, including effets on the central nervous system: headache, insomnia,agitation, seizures, severe nausea and vomiting. Gastrointestinal irritation may occurcausing vomiting and peptic ulceration. Metabolic disturbances include hypokalaemiaand metabolic acidosis. In the setting of theophylline toxicity or overdose, hypotension,profound hypokalaemia, and ventricular and supraventricular arrhythmias may occur,especially when drug concentrations are > 25 μg/ml.

Arrhythmias (Option B) is incorrect. Arrhythmias are anticipated only in the setting oftheophylline toxicity, and therapeutic drug monitoring is required to minimise the risk oftoxicity.

Diarrhoea (Option C) is incorrect. Diarrhoea is a recognised feature but is much less commonthan nausea, vomiting, headache, agitation and insomnia.

Agitation and insomniaA

ArrhythmiasB

DiarrhoeaC

HyperkalaemiaD

HypotensionE

Page 505: Back to Filters (/Secure/TestMe/Filter ... - 1 File Download

8/11/2016 MyPastest

https://mypastest.pastest.com/Secure/TestMe/Browser/429893#Top 2/2

46600

Hyperkalaemia (Option D) is incorrect. Theophylline is associated with hypokalaemia,particularly in the context of theophylline toxicity.

Hypotension (Option E) is incorrect. Hypotension is a recognised feature of theophyllinetoxicity, but does not normally occur after therapeutic doses.

Next Question

Previous Question Tag Question Feedback End Review

Difficulty: Average

Peer Responses

Session Progress

0Responses Correct:

207Responses Incorrect:

207Responses Total:

0%Responses - % Correct:

Blog (https://www.pastest.com/blog) About Pastest (https://www.pastest.com/about-us)Contact Us (https://www.pastest.com/contact-us) Help (https://www.pastest.com/help)

© Pastest 2016

Page 506: Back to Filters (/Secure/TestMe/Filter ... - 1 File Download

8/11/2016 MyPastest

https://mypastest.pastest.com/Secure/TestMe/Browser/429893#Top 1/2

Back to Filters (/Secure/TestMe/Filter/429893/QA)

Question 74 of 207

46945

You are asked to review a patient attending the endocrine clinic for investigation ofgalactorrhoea.

Which of the following drugs is most likely to cause this adverse effect?

Explanation

The answer is Metoclopramide –

Metoclopramide causes extrapyramidal effects (especially in children and young adults),hyperprolactinaemia and occasionally tardive dyskinesia on prolonged administration.Hyperprolactinaemia may give rise to galactorrhoea. Other adverse effects ofmetoclopramide include drowsiness, restlessness, diarrhoea, depression, neurolepticmalignant syndrome, rashes, pruritus and oedema. Rare adverse effects include cardiacconduction abnormalities, including methaemoglobinaemia (more severe in glucose-6-phosphate dehydrogenase deficiency). Other drug causes include chlorpromazine,haloperidol, metoclopramide, α-methyldopa, opioids, fluoxetine and ranitidine.Recreational agents including cocaine and marijuana are also recognised causes.

Atenolol (Option A) is incorrect. Atenolol is not a recognised cause.

Furosemide (Option B) is incorrect. Furosemide is not a recognised cause.

Rifampicin (Option D) is incorrect. Rifampicin is not a recognised cause.

Verapamil (Option E) is incorrect. Verapamil is a recognised cause, but is less commonlyimplicated than metoclopramide, which is why metoclopramide is the preferred answer.

AtenololA

FurosemideB

MetoclopramideC

RifampicinD

VerapamilE

Page 507: Back to Filters (/Secure/TestMe/Filter ... - 1 File Download

8/11/2016 MyPastest

https://mypastest.pastest.com/Secure/TestMe/Browser/429893#Top 2/2

Next Question

Previous Question Tag Question Feedback End Review

Difficulty: Average

Peer Responses

Session Progress

0Responses Correct:

207Responses Incorrect:

207Responses Total:

0%Responses - % Correct:

Blog (https://www.pastest.com/blog) About Pastest (https://www.pastest.com/about-us)Contact Us (https://www.pastest.com/contact-us) Help (https://www.pastest.com/help)

© Pastest 2016

Page 508: Back to Filters (/Secure/TestMe/Filter ... - 1 File Download

8/11/2016 MyPastest

https://mypastest.pastest.com/Secure/TestMe/Browser/429893#Top 1/2

Back to Filters (/Secure/TestMe/Filter/429893/QA)

Question 75 of 207

A 39-year-old woman who is planning to travel overseas to a malaria-endemic area later thismonth presents to the Emergency Department after an intentional overdose of herchloroquine medications.

Which of the following features is most likely attributable to chloroquine toxicity?

Explanation

The answer is QRS prolongation on the ECG –

Symptoms of chloroquine toxicity includes nausea, headache, visual disturbance, cardiacarrhythmia, convulsions and coma. Ingestion of more than 2 g in an adult may be fatal.Hypoglycaemia is a recognised adverse effect. Treatment includes oral activatedcharcoal, which should be given to patients who present within 1 hour. Chloroquineinterferes with cardiac sodium channel conductance, which results in prolongation of theQRS duration.

Acute blindness (Option A) is incorrect. Quinine toxicity is similar to chloroquine, and inaddition quinine may cause blindness, normally 16–20 hours after ingestion.

Hyperglycaemia (Option B) is incorrect. Hypoglycaemia is a recognised adverse effect but isa less specific indicator of severe toxicity than QRS prolongation.

Hyperkalaemia (Option C) is incorrect. Hypokalaemia is a recognised feature, although thisappears to confer some cardioprotective effects, and potassium replacement should becautious, ideally avoided within the first 8 hours after ingestion, if possible.

Acute blindnessA

HyperglycaemiaB

HyperkalaemiaC

Increased PR interval on the ECGD

QRS prolongation on the ECGE

Page 509: Back to Filters (/Secure/TestMe/Filter ... - 1 File Download

8/11/2016 MyPastest

https://mypastest.pastest.com/Secure/TestMe/Browser/429893#Top 2/2

45800

Increased PR interval on the ECG (Option D) is incorrect. Chloroquine may interfere withsodium channel conductance, giving rise to QRS prolongation.

Next Question

Previous Question Tag Question Feedback End Review

Difficulty: Average

Peer Responses

Session Progress

0Responses Correct:

207Responses Incorrect:

207Responses Total:

0%Responses - % Correct:

Blog (https://www.pastest.com/blog) About Pastest (https://www.pastest.com/about-us)Contact Us (https://www.pastest.com/contact-us) Help (https://www.pastest.com/help)

© Pastest 2016

Page 510: Back to Filters (/Secure/TestMe/Filter ... - 1 File Download

8/11/2016 MyPastest

https://mypastest.pastest.com/Secure/TestMe/Browser/429893#Top 1/2

Back to Filters (/Secure/TestMe/Filter/429893/QA)

Question 76 of 207

A 42-year-old woman has recently undergone a total abdominal hysterectomy and bilateralsalpingo-oophorectomy for pelvic inflammatory disease. She asks you about the risks andbenefits of oestrogen replacement therapy.

Which of the following statements is most accurate regarding long-term oestrogen therapy?

Explanation

The answer is Oestrogen therapy reduces the occurrence of osteoporitic fractures in oldage –

Osteoporosis occurs in all races. Reduction in total-body bone mass begins in women intheir late 20s and progresses through adult life. The rate of bone loss is accelerated atthe time of menopause. Oestrogen minimises the rate of trabecular bone loss invertebrae and cortical bone loss at the radius.

A combination of oestrogen and progesterone is more effective than oestrogen alone(Option A) is incorrect. As the uterus has been removed in this patient, there is no need foradditional progesterone therapy.

Loss of trabecular and not cortical bone is prevented by oestrogen therapy (Option B) isincorrect. Oestrogen replacement is capable of preventing trabecular and cortical bone loss.

Oestrogen therapy may fully prevent bone loss (Option C) is incorrect. Signficant bone losswill have already occurred by age 42, and oestrogen replacement will slow the rate ofongoing bone loss but not prevent it fully.

A combination of oestrogen and progesterone is more effective than oestrogenalone

A

Loss of trabecular and not cortical bone is prevented by oestrogen therapyB

Oestrogen therapy may fully prevent bone lossC

Oestrogen therapy reduces the occurrence of osteoporotic fractures in old ageD

The benefits of oestrogen are greatest when treatment is maintained more than 10years

E

Page 511: Back to Filters (/Secure/TestMe/Filter ... - 1 File Download

8/11/2016 MyPastest

https://mypastest.pastest.com/Secure/TestMe/Browser/429893#Top 2/2

46594

The benefits of oestrogen are greatest when treatment is maintained more than 10 years(Option E) is incorrect. The benefits of oestrogen therapy for preventing bone loss do notextend beyond 10 years.

Next Question

Previous Question Tag Question Feedback End Review

Difficulty: Average

Peer Responses

Session Progress

0Responses Correct:

207Responses Incorrect:

207Responses Total:

0%Responses - % Correct:

Blog (https://www.pastest.com/blog) About Pastest (https://www.pastest.com/about-us)Contact Us (https://www.pastest.com/contact-us) Help (https://www.pastest.com/help)

© Pastest 2016

Page 512: Back to Filters (/Secure/TestMe/Filter ... - 1 File Download

8/11/2016 MyPastest

https://mypastest.pastest.com/Secure/TestMe/Browser/429893#Top 1/2

Back to Filters (/Secure/TestMe/Filter/429893/QA)

Question 77 of 207

A 40-year-old bank clerk has been receiving phenelzine for severe depression under the careof the psychiatry team. She is brought to the Emergency Department in an agitated statewith high fever, tremor and agitation. Her partner says that 1 week earlier, an out-of-hours GPhad prescribed an additional medication to address her worsening low mood.

Which of the following is most likely to have provoked the adverse reaction?

Explanation

The answer is Fluoxetine –

A dangerous pharmacodynamic interaction can occur when fluoxetine or one of thenewer selective serotonin-reuptake inhibitors (SSRIs) is used in the presence of amonoamine oxidase inhibitor (MAOI). The combination of increased stores of themonoamine plus inhibition of reuptake after release is thought to result in markedincreases of serotonin in the synapses, leading to a serotonin syndrome. This rare butserious disorder is characterised by agitation, hyperthermia, dysarthria, tremor, musclerigidity, myoclonus and autonomic instability. A similar interaction may occur with otherdrugs capable of increasing serotonergic activity, including tricyclic antidepressants,antipsychotics and tramadol.

Isocarboxazid (Option B) is incorrect. Isocarboxazid is an MAOI that may also provokeserotonergic syndrome in combination with other agents, but because it shares the samebasic mechanism as phenylzine it is less likely to provoke serotonergic syndrome than thecombination of MAOI and SSRI.

FluoxetineA

IsocarboxazidB

LithiumC

St John’s WortD

TranylcypromideE

Page 513: Back to Filters (/Secure/TestMe/Filter ... - 1 File Download

8/11/2016 MyPastest

https://mypastest.pastest.com/Secure/TestMe/Browser/429893#Top 2/2

46782

Lithium (Option C) is incorrect. Lithium is not a significant risk factor for serotonergicsyndrome.

St John’s Wort (Option D) is incorrect. St John’s Wort may be used to treat mild depression,but it is not known to significantly increase serotonin release within the central nervoussystem.

Tranylcypromide (Option E) is incorrect. Tranylcypromine is an MAOI that may also provokeserotonergic syndrome in combination with other agents, but because it shares the samebasic mechanism as phenylzine it is less likely to provoke serotonergic syndrome than thecombination of MAOI and SSRI.

Next Question

Previous Question Tag Question Feedback End Review

Difficulty: Average

Peer Responses

Session Progress

0Responses Correct:

207Responses Incorrect:

207Responses Total:

0%Responses - % Correct:

Blog (https://www.pastest.com/blog) About Pastest (https://www.pastest.com/about-us)Contact Us (https://www.pastest.com/contact-us) Help (https://www.pastest.com/help)

© Pastest 2016

Page 514: Back to Filters (/Secure/TestMe/Filter ... - 1 File Download

8/11/2016 MyPastest

https://mypastest.pastest.com/Secure/TestMe/Browser/429893#Top 1/2

Back to Filters (/Secure/TestMe/Filter/429893/QA)

Question 78 of 207

46941

A 56-year-old patient is currently being treated for rheumatoid arthritis, depression andepilepsy. He presents with bilateral central visual field defects.

Which of the following drugs is most likely to be responsible for this adverse effect?

Explanation

The answer is Vigabatrin –

Vigabatrin is associated with visual field defects. The onset of symptoms varies from 1month to several years after starting therapy. In most cases, visual field defects persistdespite discontinuation of the drug. Visual field assessment should be carried out beforetreatment and at 6-month intervals, and patients should be warned to report any newvisual symptoms that develop so they can be referred for an urgent ophthalmologicalopinion; gradual withdrawal of vigabatrin should be considered.

Amitriptyline (Option A) is incorrect. Amitriptyline may cause mydriasis owing to itsanticholinergic properties.

Carbamazepine (Option B) is incorrect. Carbamazepine may cause nystagmus, especially inhigh doses.

Hydroxychloroquine (Option C) is incorrect. Hydroxychloroquine may cause retinopathy, butthis is a very rare adverse effect, which is why vigabatrin is the preferred answer.

Prednisolone (Option D) is incorrect. Prednisolone may predispose to development ofcataracts.

AmitriptylineA

CarbamazepineB

HydroxychloroquineC

PrednisoloneD

VigabatrinE

Page 515: Back to Filters (/Secure/TestMe/Filter ... - 1 File Download

8/11/2016 MyPastest

https://mypastest.pastest.com/Secure/TestMe/Browser/429893#Top 2/2

Next Question

Previous Question Tag Question Feedback End Review

Difficulty: Average

Peer Responses

Session Progress

0Responses Correct:

207Responses Incorrect:

207Responses Total:

0%Responses - % Correct:

Blog (https://www.pastest.com/blog) About Pastest (https://www.pastest.com/about-us)Contact Us (https://www.pastest.com/contact-us) Help (https://www.pastest.com/help)

© Pastest 2016

Page 516: Back to Filters (/Secure/TestMe/Filter ... - 1 File Download

8/11/2016 MyPastest

https://mypastest.pastest.com/Secure/TestMe/Browser/429893#Top 1/2

Back to Filters (/Secure/TestMe/Filter/429893/QA)

Question 79 of 207

46903

A 60-year-old man presents with polyuria and is noted to have a serum sodium level of 159mmol/l (137–144). A diagnosis of nephrogenic diabetes insipidus is considered.

Which of the following medications would be most likely to have caused this?

Explanation

The answer is Demeclocycline –

Nephrogenic diabetes insipidus is caused by acquired resistance of the renal tubules tovasopressin (antidiuretic hormone, ADH). Drugs capable of causing nephrogenicdiabetes insipidus include: lithium, demeclocycline, amphotericin and glibenclamide. Thisproperty of demeclocycline is occasionally exploited in the treatment of syndrome ofinappropriate ADH secretion.

Aspirin (Option A) is incorrect. Low-dose aspirin interferes with renal tubular secretion ofurate, so that in time total body urate accumulates and increases the risk of precipitatingacute gout.

Methotrexate (Option C) is incorrect. Methotrexate may cause retroperitoneal fibrosis andobstructive uropathy.

Propranolol (Option D) is incorrect. Beta-blockers and ACE inhibitors are recognised causesof type-4 renal tubular acidosis.

Thiazide diuretics (Option E) is incorrect. Thiazide diuretics cause sodium and potassiumdepletion.

AspirinA

DemeclocyclineB

MethotrexateC

PropranololD

Thiazide diureticsE

Page 517: Back to Filters (/Secure/TestMe/Filter ... - 1 File Download

8/11/2016 MyPastest

https://mypastest.pastest.com/Secure/TestMe/Browser/429893#Top 2/2

Next Question

Previous Question Tag Question Feedback End Review

Difficulty: Average

Peer Responses

Session Progress

0Responses Correct:

207Responses Incorrect:

207Responses Total:

0%Responses - % Correct:

Blog (https://www.pastest.com/blog) About Pastest (https://www.pastest.com/about-us)Contact Us (https://www.pastest.com/contact-us) Help (https://www.pastest.com/help)

© Pastest 2016

Page 518: Back to Filters (/Secure/TestMe/Filter ... - 1 File Download

8/11/2016 MyPastest

https://mypastest.pastest.com/Secure/TestMe/Browser/429893#Top 1/2

Back to Filters (/Secure/TestMe/Filter/429893/QA)

Question 80 of 207

A 72-year-old man presents with acute-onset lumbar spine pain. This occurred while he wasdigging the vegetable patch at home; he tried paracetamol and ibuprofen at home with littleeffect. His daughter brought him to the Emergency Department as he was unable to mobiliseto the toilet at home because of pain. There is no significant neurology on examination. Hehas a history of chronic obstructive pulmonary disease (COPD) and rheumatoid arthritis. Hetakes high-dose seretide for his COPD and low-dose prednisolone (5 mg) for his rheumatoidarthritis. X-ray reveals an osteoporotic fracture of L4.

Which of the following would be the most appropriate short-term pain relief in this case?

Explanation

The answer is Tramadol 50–100 mg qds and paracetamol 1 g qds –

This man (a 72-year-old with a history of COPD) has an acute osteoporotic fracture ofthe lumbar spine, likely caused by chronic use of corticosteroids. Any single class ofanalgesic is unlikely to provide adequate pain relief and hence the combination oftramadol and paracetamol may be most effective. Suitable longer-term treatments forhis osteoporosis would include either the use of bisphosphonate therapy or calcium andvitamin D supplementation.

Naproxen 1.5 g bd (Option A) is incorrect. A non-steroidal drug may be an effectiveanalgesic, but this high dose of naproxen would be hazardous in view of the risk of pepticulceration, particularly in the context of corticosteroid use. Moderate non-steroidal anti-inflammatory drug (NSAID) doses could be considered, and gastroprotection with a proteinpump inhibitor may be needed.

Naproxen 1.5 g bdA

Paracetamol 1 g po qdsB

Prednisolone 40 mg odC

Tramadol 50–100 mg qdsD

Tramadol 50–100 mg qds and paracetamol 1 g qdsE

Page 519: Back to Filters (/Secure/TestMe/Filter ... - 1 File Download

8/11/2016 MyPastest

https://mypastest.pastest.com/Secure/TestMe/Browser/429893#Top 2/2

46949

Paracetamol 1 g po qds (Option B) is incorrect. Although regular paracetamol may be animportant component, it is unlikely to offer sufficient analgesia by itself.

Prednisolone 40 mg od (Option C) is incorrect. Increased prednisolone doses may offer anti-inflammatory effects, but would not provide effective analgesia in the setting of vertebralfracture.

Tramadol 50–100 mg qds (Option D) is incorrect. Tramadol may offer effective analgesia butoption E is preferred since the combination of tramadol with paracetamol is likely to be moreeffective, and avoid the need for very large doses of tramadol as monotherapy.

Next Question

Previous Question Tag Question Feedback End Review

Difficulty: Average

Peer Responses

Session Progress

0Responses Correct:

207Responses Incorrect:

207Responses Total:

0%Responses - % Correct:

Blog (https://www.pastest.com/blog) About Pastest (https://www.pastest.com/about-us)Contact Us (https://www.pastest.com/contact-us) Help (https://www.pastest.com/help)

© Pastest 2016

Page 520: Back to Filters (/Secure/TestMe/Filter ... - 1 File Download

8/11/2016 MyPastest

https://mypastest.pastest.com/Secure/TestMe/Browser/429893#Top 1/2

Back to Filters (/Secure/TestMe/Filter/429893/QA)

Question 81 of 207

You are responsible for treating a 56-year-old woman with an acute attack of gout affectingher left knee. She had previously been receiving allopurinol treatment.

Which of the following statements most accurately depicts the effects of allopurinol in thesetting of acute gout?

Explanation

The answer is Exacerbation and prolongation of the attack –

Non-steroidal anti-inflammatory drugs (NSAIDs) in high doses are the drugs of choice inacute gout; colchicine is a suitable alternative but may cause diarrhoea. Allopurinol isused when the attacks are frequent and severe, or there is associated renal impairment.Initiation of allopurinol treatment during an attack can exacerbate and prolong theepisode, hence it is prudent to wait until the attack settles. Allopurinol starting dose isnormally 100 mg/day building up to 300 mg/day or more if needed to prevent acutegout episodes.

Complete remission of symptoms within a few days of treatment (Option A) is incorrect.Allopurinol will not be expected to allow prompt resolution of acute gout. High doses ofNSAIDs may allow remission of acute gout, typically within 5–10 days.

May cause renal impairment with doses > 300 mg/day (Option C) is incorrect. Renalimpairment impairs urate clearance, leading to accumulation of urate within the body andincreased risk of gout.

Reduces swelling and erythema of the affected joint (Option D) is incorrect. NSAIDs andcolchicine will reduce acute inflammation of the affected joint, but allopurinol would not be

Complete remission of symptoms within a few days of treatmentA

Exacerbation and prolongation of the attackB

May cause renal impairment with doses > 300 mg/dayC

Reduces swelling and erythema of the affected jointD

Reduction of uric acid levels to normal within a week of commencing treatmentE

Page 521: Back to Filters (/Secure/TestMe/Filter ... - 1 File Download

8/11/2016 MyPastest

https://mypastest.pastest.com/Secure/TestMe/Browser/429893#Top 2/2

46604

expected to have any significant effect.

Reduction of uric acid levels to normal within a week of commencing treatment (Option E) isincorrect. Allopurinol may lower serum urate by around 20–30%; with prolonged therapy, theaim of using allopurinol is to lower total body urate content and reduce the risk of acute goutepisodes.

Next Question

Previous Question Tag Question Feedback End Review

Difficulty: Average

Peer Responses

Session Progress

0Responses Correct:

207Responses Incorrect:

207Responses Total:

0%Responses - % Correct:

Blog (https://www.pastest.com/blog) About Pastest (https://www.pastest.com/about-us)Contact Us (https://www.pastest.com/contact-us) Help (https://www.pastest.com/help)

© Pastest 2016

Page 522: Back to Filters (/Secure/TestMe/Filter ... - 1 File Download

8/11/2016 MyPastest

https://mypastest.pastest.com/Secure/TestMe/Browser/429893#Top 1/2

Back to Filters (/Secure/TestMe/Filter/429893/QA)

Question 82 of 207

46870

A patient with AIDS has been prescribed a non-nucleoside reverse transcriptase inhibitor.

Which of the following drugs acts predominantly by this mechanism?

Explanation

The answer is Nevirapine –

Nevirapine is the only non-nucleoside reverse transcriptase inhibitor in this list.

Abacavir (Option A) is incorrect. Abacavir is a nucleoside analogue.

Lopinavir R (Option B) is incorrect. Lopinavir R is a protease inhibitor.

Nelfinavir (Option C) is incorrect. Nelfinavir is a protease inhibitor.

Stavudine (Option E) is incorrect. Stavudine is a nucleoside analogue.

AbacavirA

Lopinavir RB

NelfinavirC

NevirapineD

StavudineE

Next Question

Previous Question Tag Question Feedback End Review

Difficulty: Average

Peer Responses

Page 523: Back to Filters (/Secure/TestMe/Filter ... - 1 File Download

8/11/2016 MyPastest

https://mypastest.pastest.com/Secure/TestMe/Browser/429893#Top 2/2

Session Progress

0Responses Correct:

207Responses Incorrect:

207Responses Total:

0%Responses - % Correct:

Blog (https://www.pastest.com/blog) About Pastest (https://www.pastest.com/about-us)Contact Us (https://www.pastest.com/contact-us) Help (https://www.pastest.com/help)

© Pastest 2016

Page 524: Back to Filters (/Secure/TestMe/Filter ... - 1 File Download

8/11/2016 MyPastest

https://mypastest.pastest.com/Secure/TestMe/Browser/429893#Top 1/2

Back to Filters (/Secure/TestMe/Filter/429893/QA)

Question 83 of 207

You review a 69-year-old man in the cardiology clinic who is attending for routine follow-upof his angina. He is receiving clopidogrel, simvastatin, bisoprolol, amlodipine and ramipril. Hetells you that he has been experiencing occasional angina on exertion, and you consider thata trial of nicorandil therapy is appropriate.

Which of the following statements best describes the pharmacological effects of nicorandil?

Explanation

The answer is Oral ulceration is a recognised adverse effect –

Nicorandil is an activator of ATP-dependent potassium channels. Its pharmacologicalactions are relaxation of smooth muscle in veins and increased venous capacitance,which leads to reduced ventricular filling pressures and dilatation of the coronaryarterioles. Headache is the most common unwanted effect (~35% of patients), whichappears to be dose-dependent and lessens with continued treatment. Other adverseeffects include oral ulceration, flushing and gastrointestinal disturbance.

Headache occurs in 2–4% of patients (Option A) is incorrect. Headache occurs in up to 35%of patients initiated on nicorandil.

It increases ventricular filling pressures (Option B) is incorrect. It reduces venous return tothe heart and filling pressures.

It inhibits ATP-dependent potassium channels (Option C) is incorrect. Nicorandil activatesATP-dependent potassium channels.

Headache occurs in 2–4% of patientsA

It increases ventricular filling pressuresB

It inhibits ATP-dependent potassium channelsC

It reverses the hypotensive effect of sildenafilD

Oral ulceration is a recognised adverse effectE

Page 525: Back to Filters (/Secure/TestMe/Filter ... - 1 File Download

8/11/2016 MyPastest

https://mypastest.pastest.com/Secure/TestMe/Browser/429893#Top 2/2

46378

It reverses the hypotensive effect of sildenafil (Option D) is incorrect. Nicorandil canpotentiate the hypotensive effect of phosphodiesterase inhibitors such as sildenafil, and co-prescribing is contraindicated.

Next Question

Previous Question Tag Question Feedback End Review

Difficulty: Average

Peer Responses

Session Progress

0Responses Correct:

207Responses Incorrect:

207Responses Total:

0%Responses - % Correct:

Blog (https://www.pastest.com/blog) About Pastest (https://www.pastest.com/about-us)Contact Us (https://www.pastest.com/contact-us) Help (https://www.pastest.com/help)

© Pastest 2016

Page 526: Back to Filters (/Secure/TestMe/Filter ... - 1 File Download

8/11/2016 MyPastest

https://mypastest.pastest.com/Secure/TestMe/Browser/429893#Top 1/2

Back to Filters (/Secure/TestMe/Filter/429893/QA)

Question 84 of 207

46934

What is the main reason for the difference between the dose of sublingual glyceryl trinitrate(GTN) and oral isosorbide mononitrate needed to exert the same therapeutic effect?

Explanation

The answer is First-pass metabolism –

Among the various routes of drug delivery, the oral route is preferred for convenience.However, a key disadvantage for some drugs is first-pass metabolism and enzymaticdegradation within the gastrointestinal tract. This renders oral administration of certainclasses of drugs infeasible, especially peptides and proteins, eg insulin. Where there issignificant first-pass metabolism, other transmucosal routes of drug delivery (ie themucosal linings of the nasal, rectal, vaginal, ocular and oral cavities) offer distinctadvantages over peroral administration for systemic drug delivery.

Absorption (Option A) is incorrect. Drug absorption is normally effective; it is metabolism ofthe absorbed drug that substantially lowers drug bioavailability.

Lipid solubility (Option C) is incorrect. Both drugs have similar lipid solubility.

More rapid renal clearance (Option D) is incorrect. Neither drug is significantly dependentupon renal elimination for clearance.

Phase II metabolism (Option E) is incorrect. Phase II metabolism refers to conjugation. It isphase I, oxidative metabolism of nitrates, that diminishes oral bioavailabilty.

AbsorptionA

First-pass metabolismB

Lipid solubilityC

More rapid renal clearanceD

Phase II metabolismE

Page 527: Back to Filters (/Secure/TestMe/Filter ... - 1 File Download

8/11/2016 MyPastest

https://mypastest.pastest.com/Secure/TestMe/Browser/429893#Top 2/2

Next Question

Previous Question Tag Question Feedback End Review

Difficulty: Average

Peer Responses

Session Progress

0Responses Correct:

207Responses Incorrect:

207Responses Total:

0%Responses - % Correct:

Blog (https://www.pastest.com/blog) About Pastest (https://www.pastest.com/about-us)Contact Us (https://www.pastest.com/contact-us) Help (https://www.pastest.com/help)

© Pastest 2016

Page 528: Back to Filters (/Secure/TestMe/Filter ... - 1 File Download

8/11/2016 MyPastest

https://mypastest.pastest.com/Secure/TestMe/Browser/429893#Top 1/2

Back to Filters (/Secure/TestMe/Filter/429893/QA)

Question 85 of 207

A 45-year-old, petrol-station attendant complains of tingling and numbness in his hands andfeet, breathlessness, lethargy, weight gain and fatigue. He has a past medical history ofparoxysmal atrial fibrillation and depression and is receiving a number of differentmedications. He attends the dermatology outpatient clinic because he has developed agreyish-blue discoloration affecting his face.

Which of the following factors is most likely to account for his symptoms?

Explanation

The answer is Amiodarone –

Amiodarone may cause skin deposits, photodermatitis, a greyish-blue discoloration.Other adverse effects include tingling and numbness in the hands and feet,breathlessness, lethargy, weight gain, fatigue and slowing of peripheral reflexes.Pulmonary toxicity is one of the most important adverse effects and requires immediatedrug cessation. Other effects are inhibition of peripheral conversion of thyroxine (T ) totriiodothyronine (T ); it also contains large amounts of inorganic iodine; treatment maytherefore cause hypo- or hyperthyroidism.

Lithium (Option B) is incorrect. Lithium blocks the endocytosis of monoiodotyrosine (MIT)and diiodotyrosine (DIT) by the follicular cells of the thyroid and thereby interferes with Tand T synthesis. The interaction between lithium and thyroid function is complex because italso impairs TSH receptor sensitivity. Even at therapeutic doses, lithium may causehypothyroidism.

AmiodaroneA

LithiumB

Occupational lead exposureC

Occupational mercury exposureD

PrednisoloneE

4

3

3

4

Page 529: Back to Filters (/Secure/TestMe/Filter ... - 1 File Download

8/11/2016 MyPastest

https://mypastest.pastest.com/Secure/TestMe/Browser/429893#Top 2/2

46780

Occupational lead exposure (Option C) is incorrect. Chronic lead poisoning may occur due toexposure to leaded petrol, but this is very uncommon now. Lead poisoning does not causeskin discoloration but may give rise to blue lead lines in the gingival margins.

Occupational mercury exposure (Option D) is incorrect. Mercury poisoning is extremely rare.Chronic mercury poisoning by mercury vapour results in a classic triad of tremor,neuropsychiatric disturbance and gingivostomatitis. Elemental mercury is less toxic thanmercury salts that may be encountered in the manufacture of electrical equipment and otherindustrial uses. Methylmercury intoxication mainly affects the central nervous system andresults in paraesthesias, ataxia, deafness, dysarthria and progressive constriction of visualfields.

Prednisolone (Option E) is incorrect. Prednisolone is associated with salt and water retention,weight gain and a Cushingoid appearance when used at high dose for a prolonged period.

Next Question

Previous Question Tag Question Feedback End Review

Difficulty: Average

Peer Responses

Session Progress

0Responses Correct:

207Responses Incorrect:

207Responses Total:

0%Responses - % Correct:

Blog (https://www.pastest.com/blog) About Pastest (https://www.pastest.com/about-us)Contact Us (https://www.pastest.com/contact-us) Help (https://www.pastest.com/help)

© Pastest 2016

Page 530: Back to Filters (/Secure/TestMe/Filter ... - 1 File Download

8/11/2016 MyPastest

https://mypastest.pastest.com/Secure/TestMe/Browser/429893#Top 1/2

Back to Filters (/Secure/TestMe/Filter/429893/QA)

Question 86 of 207

A patient in a general surgical ward has received aminoglycosides and cephalosporinsintravenously for 10 days. Shortly after completion of this treatment he developed waterydiarrhoea that was associated with abdominal pain. You are asked to review him because hehas been complaining of generalised abdominal pain, fever and frequent bloody diarrhoea.

Which one of the following oral therapies would be most appropriate?

Explanation

The answer is Vancomycin –

The clinical features are consistent with pseudomembranous colitis, an acute exudativeinfection of the colon caused by Clostridium difficile. The name ‘pseudomembranouscolitis’ derives from plaques of necrotic membrane that adhere to the mucosal surface inthe clinically most severe form of the disease. The single most pertinent detail of thisman’s medical history is his previous extensive antimicrobial treatment.Pseudomembranous colitis has been reported to follow the use of every broad-spectrumantimicrobial in common medical practice, including ciprofloxacin, clindamycin,ampicillin, amoxicillin and cephalosporins. Treatment involves stopping the causativeantimicrobial if possible. Oral vancomycin has minimal gastrointestinal absorption so thatthe effect of oral administration is confined mainly to the gastrointestinal tract. Oral orintravenous metronidazole is an effective alternative treatment.

Amphotericin B (Option A) is incorrect. Amphotercin is an antifungal that is ineffectiveagainst C. difficile.

Erythromycin (Option B) is incorrect. Erythromycin is ineffective against C. difficile.

Amphotericin BA

ErythromycinB

Ispaghula huskC

PrednisoloneD

VancomycinE

Page 531: Back to Filters (/Secure/TestMe/Filter ... - 1 File Download

8/11/2016 MyPastest

https://mypastest.pastest.com/Secure/TestMe/Browser/429893#Top 2/2

46787

Ispaghula husk (Option C) is incorrect. A high-fibre diet is unlikely to confer any benefit.Probiotic yoghurts may be of some value in treatment of C. difficile.

Prednisolone (Option D) is incorrect. Prednisolone may be effective in the setting of non-infective inflammatory bowel disorders, but the clinical context makes C. difficile infectionmuch more likely here.

Next Question

Previous Question Tag Question Feedback End Review

Difficulty: Average

Peer Responses

Session Progress

0Responses Correct:

207Responses Incorrect:

207Responses Total:

0%Responses - % Correct:

Blog (https://www.pastest.com/blog) About Pastest (https://www.pastest.com/about-us)Contact Us (https://www.pastest.com/contact-us) Help (https://www.pastest.com/help)

© Pastest 2016

Page 532: Back to Filters (/Secure/TestMe/Filter ... - 1 File Download

8/11/2016 MyPastest

https://mypastest.pastest.com/Secure/TestMe/Browser/429893#Top 1/2

Back to Filters (/Secure/TestMe/Filter/429893/QA)

Question 87 of 207

You have seen a patient with bronchiectasis and wish to prescribe a theophylline preparationas a bronchodilator.

Which of the following best describes a main feature of this drug’s activity?

Explanation

The answer is It blocks the adenosine receptor –

Adenosine acts as a local mediator for many organ systems. Blockade of the receptorsby theophylline causes relaxation of bronchial smooth muscle, constriction of cerebralblood vessels, stimulation of the cardiac pacemaker and stimulation of gastric secretions.In addition to blocking adenosine, theophylline inhibits phosphodiesterase, resulting incalcium ion release from the sarcoplasmic reticulum in skeletal and cardiac muscle, thusenhancing their contractility. Theophylline is extensively metabolised by the liver; it has anarrow therapeutic window, so drugs that inhibit liver metabolism may causetheophylline toxicity.

50% of the drug is plasma-protein bound (Option A) is incorrect. Theophylline is highlyprotein bound.

It enhances diaphragm contractility (Option C) is incorrect. Theophylline may enhancediaphragm contractility, which may assist respiration but this is not the majorpharmacological mechanism of action.

Most of the drug is excreted unchanged in urine (Option D) is incorrect. Only 10% of the drugis excreted unchanged in the urine as most of it is extensively metabolised in the liver.

50% of the drug is plasma-protein boundA

It blocks the adenosine receptorB

It enhances diaphragm contractilityC

Most of the drug is excreted unchanged in urineD

Side-effects increase in a linear way versus doseE

Page 533: Back to Filters (/Secure/TestMe/Filter ... - 1 File Download

8/11/2016 MyPastest

https://mypastest.pastest.com/Secure/TestMe/Browser/429893#Top 2/2

46869

Side-effects increase in a linear way versus dose (Option D) is incorrect. Theophylline issubject to saturable liver metabolism pathways, and a small increase in dose may provoketoxicity.

Next Question

Previous Question Tag Question Feedback End Review

Difficulty: Average

Peer Responses

Session Progress

0Responses Correct:

207Responses Incorrect:

207Responses Total:

0%Responses - % Correct:

Blog (https://www.pastest.com/blog) About Pastest (https://www.pastest.com/about-us)Contact Us (https://www.pastest.com/contact-us) Help (https://www.pastest.com/help)

© Pastest 2016

Page 534: Back to Filters (/Secure/TestMe/Filter ... - 1 File Download

8/11/2016 MyPastest

https://mypastest.pastest.com/Secure/TestMe/Browser/429893#Top 1/2

Back to Filters (/Secure/TestMe/Filter/429893/QA)

Question 88 of 207

A 43-year-old woman is admitted to the Intensive Care Unit after an intentional overdoseinvolving a mixture of different medications. You notice that the patient has beencommenced on haemodialysis.

Which one of the following drug toxicities is LEAST likely to be improved by haemodialysis?

Explanation

The answer is Ecstasy –

There is no antidote for ecstasy overdose, and haemodialysis has not been shown tooffer any benefit in treatment of acute toxicity. Haemodialysis, peritoneal dialysis,charcoal or resin haemoperfusion, haemofiltration, plasmapheresis and exchangetransfusion are used to enhance clearance of drug from the body; greatest clearance isseen with drugs that have high water solubility, low plasma protein binding, smallvolumes of distribution and a long half-life.

Alcohol (Option A) is incorrect. Ethanol, methanol and ethylene glycol clearance can besignificantly enhanced by dialysis.

Barbiturates (Option B) is incorrect. Although now an uncommon means of overdose,haemodialysis is particularly effective in removal of barbiturates.

Lithium (Option D) is incorrect. Lithium toxicity may be severe and result in cognitive andneurological impairment. Haemodialysis is highly effective in removing lithium from thecirculating compartment, but prolonged dialysis (18 h) is required to achieve an effectiveremoval of total body lithium.

AlcoholA

BarbituratesB

EcstasyC

LithiumD

SalicylateE

Page 535: Back to Filters (/Secure/TestMe/Filter ... - 1 File Download

8/11/2016 MyPastest

https://mypastest.pastest.com/Secure/TestMe/Browser/429893#Top 2/2

46968

Salicylate (Option E) is incorrect. Severe salicylate poisoning may cause metabolic acidosisthat is difficult to correct by bicarbonate administration alone; haemodialysis is highlyeffective in removing salicylates and restoring acid–base balance.

Next Question

Previous Question Tag Question Feedback End Review

Difficulty: Average

Peer Responses

Session Progress

0Responses Correct:

207Responses Incorrect:

207Responses Total:

0%Responses - % Correct:

Blog (https://www.pastest.com/blog) About Pastest (https://www.pastest.com/about-us)Contact Us (https://www.pastest.com/contact-us) Help (https://www.pastest.com/help)

© Pastest 2016

Page 536: Back to Filters (/Secure/TestMe/Filter ... - 1 File Download

8/11/2016 MyPastest

https://mypastest.pastest.com/Secure/TestMe/Browser/429893#Top 1/2

Back to Filters (/Secure/TestMe/Filter/429893/QA)

Question 89 of 207

46894

A 57-year-old woman with cardiac failure is being managed in the High Dependency Unit. Herheart rate and systolic blood pressure are low, and after discussion with the cardiology team,you decide that she should receive inotropic support.

Of the following drugs, which is most likely to cause significant tachycardia?

Explanation

The answer is Adrenaline –

Adrenaline exerts agonist effects on both α- and β-adrenoceptors and less intensivevasoconstriction than norepinephrine. Adrenaline stimulates both β - and β -receptorswith approximately equal potency; β effects may cause tachycardia, particularly athigher doses, whereas β effects include vasodilatation.

Dobutamine (Option B) is incorrect. Dobutamine is relatively selective for peripheral β -receptors and may cause tachycardia, with less effect on peripheral vasculature.

Dopamine (Option C) is incorrect. Dopamine causes peripheral vasoconstriction, and hasequivocal effects on heart rate (in some cases there may be a baroreflex mediatedbradycardia).

Noradrenaline (Option D) is incorrect. Norepinephrine stimulates mainly α-adrenoceptors inthe periphery, resulting in intense vasoconstriction; the increase in afterload may worsenheart failure.

Phenylephrine (Option E) is incorrect. Phenylephrine primarily results in a rise in BP, it mayresult in reflex bradycardia when given IV because of carotid body stimulation.

AdrenalineA

DobutamineB

DopamineC

NoradrenalineD

PhenylephrineE

1 2

1

2

1

Page 537: Back to Filters (/Secure/TestMe/Filter ... - 1 File Download

8/11/2016 MyPastest

https://mypastest.pastest.com/Secure/TestMe/Browser/429893#Top 2/2

46894

Next Question

Previous Question Tag Question Feedback End Review

Difficulty: Average

Peer Responses

Session Progress

0Responses Correct:

207Responses Incorrect:

207Responses Total:

0%Responses - % Correct:

Blog (https://www.pastest.com/blog) About Pastest (https://www.pastest.com/about-us)Contact Us (https://www.pastest.com/contact-us) Help (https://www.pastest.com/help)

© Pastest 2016

Page 538: Back to Filters (/Secure/TestMe/Filter ... - 1 File Download

8/11/2016 MyPastest

https://mypastest.pastest.com/Secure/TestMe/Browser/429893#Top 1/2

Back to Filters (/Secure/TestMe/Filter/429893/QA)

Question 90 of 207

A 55-year-old man diagnosed with hypertension who was not responding to recommendedlifestyle changes was commenced on drug treatment 1 month ago. There is a past history ofbenign prostatic hypertropy and the GP has tailored therapy to try and treat both conditions.He complains of dizziness, and severe postural hypotension is found.

What is the most likely aetiological agent?

Explanation

The answer is Doxazosin –

Doxazosin, like other α-adrenergic blocking agents, can cause marked hypotension,especially postural hypotension, with syncope and other postural symptoms such asdizziness. In this case, doxazosin may be used to control the symptoms of benignprostatic hypertropy (BPH) as an additional potential benefit. Orthostatic hypotension ismost common after the first dose, but can also occur when there is a dosage increase, orif the dose is interrupted for more than a few days and restarted. To decrease thelikelihood of excessive hypotension and syncope, it is essential that treatment is initiatedwith the low dose and then gradually increased.

Bendrofluazide (Option A) is incorrect. Thiazides exert a modest diuretic effect soon afterinitiation; however, the blood pressure lowering effects are more gradual in onset overseveral days to weeks.

Bisoprolol (Option B) is incorrect. Beta-blockers are less likely to provoke posturalhypotension than α-blockers, ACE inhibitors, or angiotensin receptor blockers; this may be

BendrofluazideA

BisoprololB

DoxazosinC

LisinoprilD

LosartanE

Page 539: Back to Filters (/Secure/TestMe/Filter ... - 1 File Download

8/11/2016 MyPastest

https://mypastest.pastest.com/Secure/TestMe/Browser/429893#Top 2/2

46939

due to their propensity to cause vasoconstriction owing to the unopposed effect ofcatecholamines causing vasocontriction mediated by α-adrenoceptors.

Lisinopril (Option D) is incorrect. ACE inhibitors commonly cause first-dose hypotension andpostural hypotension, but the co-existent BPH makes doxazosin a better option.

Losartan (Option E) is incorrect. Angiotension receptor blockers commonly cause first-dosehypotension and postural hypotension, but the co-existent BPH makes doxazosin a betteroption.

Next Question

Previous Question Tag Question Feedback End Review

Difficulty: Average

Peer Responses

Session Progress

0Responses Correct:

207Responses Incorrect:

207Responses Total:

0%Responses - % Correct:

Blog (https://www.pastest.com/blog) About Pastest (https://www.pastest.com/about-us)Contact Us (https://www.pastest.com/contact-us) Help (https://www.pastest.com/help)

© Pastest 2016

Page 540: Back to Filters (/Secure/TestMe/Filter ... - 1 File Download

8/11/2016 MyPastest

https://mypastest.pastest.com/Secure/TestMe/Browser/429893#Top 1/2

Back to Filters (/Secure/TestMe/Filter/429893/QA)

Question 91 of 207

46978

A 17-year-old girl is admitted from her birthday party. She does not normally drink alcohol,but you understand she was given some by a friend, along with a pill to ‘loosen up and havefun’. On examination she has GCS 14, blood pressure is 155/95 mmHg and pulse is 125/min.Her respiratory rate is 24/min. Her pupils are dilated.

Which one of the following offers the best explanation for her clinical features?

Explanation

The answer is Ecstasy –

Ecstasy or MDMA is a common drug of abuse, closely related to methamphetamine, andcauses release of both serotonin and catecholamines, leading to feelings of euphoria. Itcauses hyperthermia and is often associated with rhabdomyolysis and hyponatraemia.Pupils are dilated.

Diclofenac (Option A) is incorrect. Diclofenac ingestion may cause gastrointestinal irritationbut would not explain the reduced conscious level and tachycardia.

Morphine (Option C) is incorrect. Morphine would suppress conscious level but would beexpected to suppress respiration and cause small pupils.

Thyroxine (Option D) is incorrect. Thyroxine is unlikely to alter conscious level or respiratoryrate; the clinical effects, eg increased heart rate, may take several hours to days to manifest.

Verapamil (Option E) is incorrect. Verapamil would be expected to cause bradycardia.

DiclofenacA

EcstasyB

MorphineC

ThyroxineD

VerapamilE

Page 541: Back to Filters (/Secure/TestMe/Filter ... - 1 File Download

8/11/2016 MyPastest

https://mypastest.pastest.com/Secure/TestMe/Browser/429893#Top 2/2

Next Question

Previous Question Tag Question Feedback End Review

Difficulty: Average

Peer Responses

Session Progress

0Responses Correct:

207Responses Incorrect:

207Responses Total:

0%Responses - % Correct:

Blog (https://www.pastest.com/blog) About Pastest (https://www.pastest.com/about-us)Contact Us (https://www.pastest.com/contact-us) Help (https://www.pastest.com/help)

© Pastest 2016

Page 542: Back to Filters (/Secure/TestMe/Filter ... - 1 File Download

8/11/2016 MyPastest

https://mypastest.pastest.com/Secure/TestMe/Browser/429893#Top 1/2

Back to Filters (/Secure/TestMe/Filter/429893/QA)

Question 92 of 207

You see a 76-year-old man in the Coronary Care Unit 3 days after sustaining a non-STelevation myocardial infarction (NSTEMI). He has been started on a number of newcardioactive medications.

Which one of the following antiplatelet agents increases cellular concentration of cyclicadenosine monophosphate (cAMP) by inhibiting phosphodiesterase?

Explanation

The answer is Dipyridamole –

Dipyridamole is a weak antiplatelet agent that acts by increasing the cellularconcentration of cyclic adenosine monophosphate (cAMP). It inhibits thephosphodiesterase enzyme, which is responsible for convertin cAMP to inactive 5' AMP,thereby increasing cAMP concentrations. Raised cAMP and cyclic guanosinemonophosphate (cGMP) inhibit platelet activation and aggregation.

Abciximab (Option A) is incorrect. Abciximab is a humanised monoclonal antibody thatselectively blocks the GPIIb–IIIa receptor on the platelet surface.

Aspirin (Option B) is incorrect. Aspirin is a potent inhibitor of platelet cyclo-oxygenase, anenzyme that converts arachidonic acid to thromboxane A (TxA ), a strong promoter ofplatelet aggregation. The platelet has no protein synthetic apparatus; therefore, the effectsof aspirin are irreversible and last for the life of the platelet (8–10 days), even though aspirinhas only a short plasma half-life.

Clopidogrel (Option C) is incorrect. Clopidogrel, like ticlopidine, antagonises platelet ADPreceptor, resulting in inhibition of platelet activation. Antagonism is non-competitive,

AbciximabA

AspirinB

ClopidogrelC

DipyridamoleD

TiclopidineE

2 2

Page 543: Back to Filters (/Secure/TestMe/Filter ... - 1 File Download

8/11/2016 MyPastest

https://mypastest.pastest.com/Secure/TestMe/Browser/429893#Top 2/2

46628

irreversible and results in 50–70% inhibition of fibrinogen binding.

Ticlopidine (Option E) is incorrect. Ticlopidine is an adenosine diphosphate (ADP) receptorantagonist.

Next Question

Previous Question Tag Question Feedback End Review

Difficulty: Average

Peer Responses

Session Progress

0Responses Correct:

207Responses Incorrect:

207Responses Total:

0%Responses - % Correct:

Blog (https://www.pastest.com/blog) About Pastest (https://www.pastest.com/about-us)Contact Us (https://www.pastest.com/contact-us) Help (https://www.pastest.com/help)

© Pastest 2016

Page 544: Back to Filters (/Secure/TestMe/Filter ... - 1 File Download

8/11/2016 MyPastest

https://mypastest.pastest.com/Secure/TestMe/Browser/429893#Top 1/2

Back to Filters (/Secure/TestMe/Filter/429893/QA)

Question 93 of 207

46865

A cancer patient is admitted for investigation of severe abdominal pain, diarrhoea and feverafter receiving radiation therapy treatment for underlying cancer. On questioning, it is foundthat the patient had been receiving chemotherapy.

Which one of the following chemotherapy agents is most likely to have increased the risk ofradiation toxicity?

Explanation

The answer is Dactinomycin –

Dactinomycin is a radiosensitiser agent that increases cellular response to radiationtreatment. A number of other chemotherapy agents are capable of promoting the effectsof radiation, including: metronidazole, 5-fluorouracil, gemcitabine, cisplatin, hydroxyurea,paclitaxel, mitomycin C and topotecan.

6-Mercaptopurine (Option A) is incorrect. 6-Mercaptopurine is not thought to act as aradiosensitiser.

Amifostine (Option B) is incorrect. In contrast to the effects of radiosensitisers, amifostineserves to protect against radiation toxicity.

Cyclophosphamide (Option C) is incorrect. Cyclophosphamide is not thought to act as aradiosensitiser.

Vincristine (Option E) is incorrect. Vincristine is not thought to act as a radiosensitiser.

6-MercaptopurineA

AmifostineB

CyclophosphamideC

DactinomycinD

VincristineE

Page 545: Back to Filters (/Secure/TestMe/Filter ... - 1 File Download

8/11/2016 MyPastest

https://mypastest.pastest.com/Secure/TestMe/Browser/429893#Top 2/2

Next Question

Previous Question Tag Question Feedback End Review

Difficulty: Average

Peer Responses

Session Progress

0Responses Correct:

207Responses Incorrect:

207Responses Total:

0%Responses - % Correct:

Blog (https://www.pastest.com/blog) About Pastest (https://www.pastest.com/about-us)Contact Us (https://www.pastest.com/contact-us) Help (https://www.pastest.com/help)

© Pastest 2016

Page 546: Back to Filters (/Secure/TestMe/Filter ... - 1 File Download

8/11/2016 MyPastest

https://mypastest.pastest.com/Secure/TestMe/Browser/429893#Top 1/2

Back to Filters (/Secure/TestMe/Filter/429893/QA)

Question 94 of 207

46812

A 58-year-old obese patient presents with swelling of the first metatarsophalangeal joint. Onexamination it is acutely inflamed, tender, warm to the touch, and the overlying skin is red,shiny and itchy.

What is the most appropriate therapy?

Explanation

The answer is Indometacin –

The clinical scenario is typical of acute gout. Full doses of any of the non-steroidal anti-inflammatory drugs (NSAIDs) are effective in terminating attacks of acute gout. Oralcorticosteroids may be considered, but are less effective in reducing pain and swelling.

Allopurinol (Option A) is incorrect. Allopurinol is a xanthine oxidase inhibitor that causes amodest reduction in serum urate concentration and, over a sufficient period of time, willreduce total body burden of urate thereby making gout episodes less frequent. It mayprovoke acute gout, and it is normally introduced only after the acute episode has resolved.

Aspirin (Option B) is incorrect. Aspirin inhibits tubular secretion of urate so that serum urateconcentrations increase. High-dose aspirin may be as effective as NSAIDs in the acutesetting, but is less commonly used.

Paracetamol (Option D) is incorrect. Paracetamol is less effective than NSAIDs for reducingacute inflammatory joint disease.

Prednisolone (Option E) is incorrect. Corticosteroids are a second-line therapy that may beconsidered when NSAIDs are not tolerated or contraindicated.

AllopurinolA

AspirinB

IndometacinC

ParacetamolD

PrednisoloneE

Page 547: Back to Filters (/Secure/TestMe/Filter ... - 1 File Download

8/11/2016 MyPastest

https://mypastest.pastest.com/Secure/TestMe/Browser/429893#Top 2/2

46812

Next Question

Previous Question Tag Question Feedback End Review

Difficulty: Average

Peer Responses

Session Progress

0Responses Correct:

207Responses Incorrect:

207Responses Total:

0%Responses - % Correct:

Blog (https://www.pastest.com/blog) About Pastest (https://www.pastest.com/about-us)Contact Us (https://www.pastest.com/contact-us) Help (https://www.pastest.com/help)

© Pastest 2016

Page 548: Back to Filters (/Secure/TestMe/Filter ... - 1 File Download

8/11/2016 MyPastest

https://mypastest.pastest.com/Secure/TestMe/Browser/429893#Top 1/2

Back to Filters (/Secure/TestMe/Filter/429893/QA)

Question 95 of 207

A 32-year-old man presents to the Emergency Department following an intentional overdoseof an unknown quantity of ferrous sulfate. He is complaining of abdominal pain and nausea.

Which of the following features is most likely to indicate severe iron poisoning?

Explanation

The answer is Blood iron concentration of 120 μmol/l at 6 hours post-ingestion –

Patients with serum iron concentrations higher than 90 μmol/l may have moderate tosevere toxicity, and normally require chelation therapy with intravenous desferrioxamine.Undissolved iron tablets are radiopaque, but the presence on a plain X-ray does notcorrelate well with toxicity. Early features of iron overdose are due to the direct irritanteffects of iron and include vomiting, diarrhoea and abdominal pain. Severe symptomsmay occur after a latent period of up to 20 hours, including metabolic acidosis, livertoxicity and widespread organ failure. Initial hyperglycaemia can occur followingsignificant ingestion of iron, but hypoglycaemia can be seen later in cases of severepoisoning with associated hepatic failure.

Heart rate 110 per minute (Option B) is incorrect. Tachycardia may occur, but is a secondaryfeature, and not specific to iron poisoning.

Hypocalcaemia on the initial blood tests (Option C) is incorrect. Iron poisoning is not usuallyassociated with any significant effect on serum calcium concentration.

Presence of radio-opaque tablets on plain abdominal X-ray (Option D) is incorrect. Althoughundissolved iron tablets are radio-opaque, their presence on a plain X-ray does not correlatewell with toxicity.

Blood iron concentration of 120 μmol/l at 6 hours post-ingestionA

Heart rate 110 per minuteB

Hypocalcaemia on the initial blood testsC

Presence of radio-opaque tablets on plain abdominal X-rayD

Unable to administer oral activated charcoal due to vomitingE

Page 549: Back to Filters (/Secure/TestMe/Filter ... - 1 File Download

8/11/2016 MyPastest

https://mypastest.pastest.com/Secure/TestMe/Browser/429893#Top 2/2

45792

Unable to administer oral activated charcoal due to vomiting (Option E) is incorrect. Severevomiting is a recognised feature of iron poisoning but is non-specific. Iron is not adsorbed byactivated charcoal (neither is lithium) and this treatment is not expected to be helpful.

Next Question

Previous Question Tag Question Feedback End Review

Difficulty: Average

Peer Responses

Session Progress

0Responses Correct:

207Responses Incorrect:

207Responses Total:

0%Responses - % Correct:

Blog (https://www.pastest.com/blog) About Pastest (https://www.pastest.com/about-us)Contact Us (https://www.pastest.com/contact-us) Help (https://www.pastest.com/help)

© Pastest 2016

Page 550: Back to Filters (/Secure/TestMe/Filter ... - 1 File Download

8/11/2016 MyPastest

https://mypastest.pastest.com/Secure/TestMe/Browser/429893#Top 1/2

Back to Filters (/Secure/TestMe/Filter/429893/QA)

Question 96 of 207

A 50-year-old, mildly hypertensive businessman, on low-dose aspirin, is a regular at partieswhere he scouts for more business. He has type-2 diabetes and is taking metformin andgliclazide. Recently, he was prescribed erythromycin and paracetamol for an ear infection. Hepresents to the Emergency Department with abdominal pain and nausea, and investigationsshow serum bicarbonate 14 mmol/l and lactic 4.7 mmol/l.

Which of the following drugs is most likely to have interacted with metformin to cause thiscomplication?

Explanation

The answer is Alcohol –

Excess alcohol intake may significantly increase the risk of lactic acidosis in patientstaking metformin therapy. Metformin is not subject to metabolism and is normallysubject to renal excretion; it may accumulate and cause lactic acidosis in the setting ofacute kidney injury. Drugs capable of provoking kidney injury may increase the risk ofmetformin toxicity, including contrast media, cyclosporin and aminoglycosides.Metformin should therefore be stopped before, and for 48 h after, contrast radiography.Metformin is excreted by the renal tubules and this process can be inhibited bycimetidine, but not the other H2 receptor antagonists.

Aspirin (Option B) is incorrect. Aspirin may cause metabolic acidosis, particularly in thecontext of an aspirin overdose; however, aspirin does not cause a lactic acidosis.

Erythromycin (Option C) is incorrect. Metformin is not metabolised, and therefore notsubject to the effects of erythromycin on hepatic metabolism.

AlcoholA

AspirinB

ErythromycinC

GliclazideD

ParacetamolE

Page 551: Back to Filters (/Secure/TestMe/Filter ... - 1 File Download

8/11/2016 MyPastest

https://mypastest.pastest.com/Secure/TestMe/Browser/429893#Top 2/2

46767

Gliclazide (Option D) is incorrect. Gliclazide may be administered safely alongsidemetformin.

Paracetamol (Option E) is incorrect. Paracetamol does not interact with metformin.

Next Question

Previous Question Tag Question Feedback End Review

Difficulty: Average

Peer Responses

Session Progress

0Responses Correct:

207Responses Incorrect:

207Responses Total:

0%Responses - % Correct:

Blog (https://www.pastest.com/blog) About Pastest (https://www.pastest.com/about-us)Contact Us (https://www.pastest.com/contact-us) Help (https://www.pastest.com/help)

© Pastest 2016

Page 552: Back to Filters (/Secure/TestMe/Filter ... - 1 File Download

8/11/2016 MyPastest

https://mypastest.pastest.com/Secure/TestMe/Browser/429893#Top 1/2

Back to Filters (/Secure/TestMe/Filter/429893/QA)

Question 97 of 207

You are involved in developing a study protocol to test a new drug for heart failure, and wishto conduct a 6-month-long clinical trial. During this time you are hoping to detect adverseevents that might be expected to occur with an incidence of 1 in 2000 patients or morefrequently.

How many patients would need to be recruited for treatment with the new drug to have a95% chance of detecting the occurrence of one adverse event?

Explanation

The answer is 6000 –

Roughly speaking, for a study to have 95% power to detect a certain adverse event in aclinical trial you would need to enrol around three times as many patients as theexpected event frequency, so in this case 6000 patients would be needed. If thefrequency expected was 1 in 10 000, then you would need to recruit around 30 000patients and so on. The practicalities and costs of conducting very large-scale trialsmean that most new drugs are registered after trials in only a few thousand patients atmost; therefore, adverse events occurring with a frequency of 1 in 10 000 or less areoften only detected after the launch of a drug. For this reason, new drugs are afforded‘Black triangle’ status in the UK to signify the need for intensive monitoring and reportingof all adverse effects to the regulatory authorities via the ‘Yellow-card’ system.

1000 (Option A) is incorrect. This would reliably allow detection of adverse effects that occurin 1 in 300 patients or more frequently.

1000A

2000B

6000C

12 000D

24 000E

Page 553: Back to Filters (/Secure/TestMe/Filter ... - 1 File Download

8/11/2016 MyPastest

https://mypastest.pastest.com/Secure/TestMe/Browser/429893#Top 2/2

46837

2000 (Option B) is incorrect. This would reliably allow detection of adverse effects thatoccur in 1 in 650 patients or more frequently.

12 000 (Option D) is incorrect. This would reliably allow detection of adverse effects thatoccur in 1 in 4000 patients or more frequently.

24 000 (Option E) is incorrect. Not this many patients would require recruitment; this wouldreliably allow detection of adverse effects that occur as rarely as 1 in 8000 patients.

Next Question

Previous Question Tag Question Feedback End Review

Difficulty: Average

Peer Responses

Session Progress

0Responses Correct:

207Responses Incorrect:

207Responses Total:

0%Responses - % Correct:

Blog (https://www.pastest.com/blog) About Pastest (https://www.pastest.com/about-us)Contact Us (https://www.pastest.com/contact-us) Help (https://www.pastest.com/help)

© Pastest 2016

Page 554: Back to Filters (/Secure/TestMe/Filter ... - 1 File Download

8/11/2016 MyPastest

https://mypastest.pastest.com/Secure/TestMe/Browser/429893#Top 1/2

Back to Filters (/Secure/TestMe/Filter/429893/QA)

Question 98 of 207

46597

You are reviewing a 72-year-old woman in the cardiology outpatient clinic. She has a 3-yearhistory of congestive cardiac failure and is receiving a number of different medications.

Which of the following drugs is most likely to prolong survival in patients with heart failure?

Explanation

The answer is Enalapril –

Treatment with ACE inhibitors and angiotensin receptor antagonists has been shown toprolong survival rates in patients with congestive heart failure. These agents exert anumber of beneficial actions on the cardiovascular system, including vasodilatation andminimisation of cardiac dilatation.

Aspirin (Option A) is incorrect. Aspirin prolongs survival in patients with coronary occlusionor myocardial infarction.

Digoxin (Option B) is incorrect. Digoxin reduces heart failure deaths but has no effect onoverall cardiovascular mortality.

Furosemide (Option D) is incorrect. Furosemide is effective in reducing heart failuresymptoms and may reduce hospitalisations, but does not alter mortality.

Isosorbide mononitrate (Option E) is incorrect. Nitrates improve cardiovascularhaemodynamics and may reduce symptoms. Nitrates combined with hydralazine (an arterialvasodilator) reduce heart failure mortality, but there is no evidence that nitrates alone dothis.

AspirinA

DigoxinB

EnalaprilC

FurosemideD

Isosorbide mononitrateE

Page 555: Back to Filters (/Secure/TestMe/Filter ... - 1 File Download

8/11/2016 MyPastest

https://mypastest.pastest.com/Secure/TestMe/Browser/429893#Top 2/2

Next Question

Previous Question Tag Question Feedback End Review

Difficulty: Average

Peer Responses

Session Progress

0Responses Correct:

207Responses Incorrect:

207Responses Total:

0%Responses - % Correct:

Blog (https://www.pastest.com/blog) About Pastest (https://www.pastest.com/about-us)Contact Us (https://www.pastest.com/contact-us) Help (https://www.pastest.com/help)

© Pastest 2016

Page 556: Back to Filters (/Secure/TestMe/Filter ... - 1 File Download

8/11/2016 MyPastest

https://mypastest.pastest.com/Secure/TestMe/Browser/429893#Top 1/2

Back to Filters (/Secure/TestMe/Filter/429893/QA)

Question 99 of 207

46900

You are asked to review a 50-year-old man in the Intensive Care Unit who has apparently hada single epileptic seizure, several days after admission. In the absence of other obviouscauses, you consider the possibility that this might have been an adverse drug effect.

Which of the following drugs might be most likely to have caused this?

Explanation

The answer is Intravenous benzylpenicillin –

High doses of intravenous penicillin can provoke seizures; other antibiotics are also arecognised cause of seizures, namely quinolones such as ciprofloxacin. Overdoseinvolving a number of different drugs may provoke seizures, including tricyclicantidepressants, mefenamic acid, anticonvulsants and class I antiarrhythmics.

Amiodarone (Option A) is incorrect. Amiodarone has rarely been reported as a possiblecause of seizures.

Chlordiazepoxide (Option B) is incorrect. Chlordiazepoxide raises seizure threshold and maybe used to treat seizures. Sudden cessation may provoke seizures.

Clomethiazole (Option C) is incorrect. Clomethiazole (chlormethiazole) raises seizurethreshold and may be used to prevent or treat seizures.

Diazepam (Option D) is incorrect. Diazepam raises seizure threshold and may be used totreat seizures. Sudden cessation may provoke seizures.

AmiodaroneA

ChlordiazepoxideB

ClomethiazoleC

DiazepamD

Intravenous benzylpenicillinE

Page 557: Back to Filters (/Secure/TestMe/Filter ... - 1 File Download

8/11/2016 MyPastest

https://mypastest.pastest.com/Secure/TestMe/Browser/429893#Top 2/2

Next Question

Previous Question Tag Question Feedback End Review

Difficulty: Average

Peer Responses

Session Progress

0Responses Correct:

207Responses Incorrect:

207Responses Total:

0%Responses - % Correct:

Blog (https://www.pastest.com/blog) About Pastest (https://www.pastest.com/about-us)Contact Us (https://www.pastest.com/contact-us) Help (https://www.pastest.com/help)

© Pastest 2016

Page 558: Back to Filters (/Secure/TestMe/Filter ... - 1 File Download

8/11/2016 MyPastest

https://mypastest.pastest.com/Secure/TestMe/Browser/429893#Top 1/2

Back to Filters (/Secure/TestMe/Filter/429893/QA)

Question 100 of 207

46938

A 57-year-old woman attends the medical clinic for review. She was commenced on a newmedicine 6 months earlier, and has noticed that her weight has progressively increased.

Which of the following drugs is most likely to cause weight gain?

Explanation

The answer is Sodium valproate –

Sodium valproate causes the following side-effects: gastric irritation, nausea, ataxia andtremor, hyperammonaemia, increased appetite and weight gain.

Carbamazepine (Option A) is incorrect. Carbamazepine may cause minor weight gain, and isa less preferred answer than sodium valproate in this instance.

Diazepam (Option B) is incorrect. Diazepam may cause drowsiness, but weight gain is not asignificant adverse effect.

Phenytoin (Option C) is incorrect. Phenytoin may cause weight gain, but this is normallyminor, hence sodium valproate is the preferred answer here.

Pregabalin (Option D) is incorrect. Pregabalin may cause drowsiness, but weight gain is not asignificant adverse effect.

CarbamazepineA

DiazepamB

PhenytoinC

PregabalinD

Sodium valproateE

Next Question

Page 559: Back to Filters (/Secure/TestMe/Filter ... - 1 File Download

8/11/2016 MyPastest

https://mypastest.pastest.com/Secure/TestMe/Browser/429893#Top 2/2

Previous Question Tag Question Feedback End Review

Difficulty: Average

Peer Responses

Session Progress

0Responses Correct:

207Responses Incorrect:

207Responses Total:

0%Responses - % Correct:

Blog (https://www.pastest.com/blog) About Pastest (https://www.pastest.com/about-us)Contact Us (https://www.pastest.com/contact-us) Help (https://www.pastest.com/help)

© Pastest 2016

Page 560: Back to Filters (/Secure/TestMe/Filter ... - 1 File Download

8/11/2016 MyPastest

https://mypastest.pastest.com/Secure/TestMe/Browser/429893#Top 1/2

Back to Filters (/Secure/TestMe/Filter/429893/QA)

Question 101 of 207

A 74-year-old woman with a history of ischaemic heart disease and chronic atrial fibrillation istreated with Digoxin, Ramipril, Furosemide, Bisoprolol and Clopidogrel. She has suffered fromdiarrhoea and vomiting over the past few days and you are concerned that she may have pre-renal failure. Examination reveals a BP of 95/60 mmHg, her pulse is 42/min, AF. Her chest isclear and her abdomen is generally tender as a result of the gastroenteritis, although it is soft.

Investigations;

Hb 12.8 g/dl

WCC 7.9 x10 /l

PLT 221 x10 /l

Na 137 mmol/l

K 5.6 mmol/l

Creatinine255 micromol/l (up from 135 micromol/l at last clinicvisit)

Urea 21.2 mmol/l

Which of the following features, related to her medication, may be seen in this situation?

Explanation

The answer is Yellowing of vision -

9

9

+

+

Blue discoloration of visionA

Blurred visionB

Loss of colour visionC

Loss of night visionD

Yellowing of visionE

Page 561: Back to Filters (/Secure/TestMe/Filter ... - 1 File Download

8/11/2016 MyPastest

https://mypastest.pastest.com/Secure/TestMe/Browser/429893#Top 2/2

38084

Digoxin toxicity is known to be associated with yellow discoloration of vision, such that whenpatients are proactively questioned about their eyesight they often mention that ”everythinglooks jaundiced”. Blue discoloration of vision is seen in patients who use high dose PDE-5inhibitors for the treatment of erectile dysfunction because of crossover inhibition of thePDE-6 enzyme. Loss of night vision is seen with congenital conditions such as retinitispigmentosa, while loss of contrast and colour vision is seen in optic neuritis.

Next Question

Previous Question Tag Question Feedback End Review

Difficulty: Easy

Peer Responses

Session Progress

0Responses Correct:

207Responses Incorrect:

207Responses Total:

0%Responses - % Correct:

Blog (https://www.pastest.com/blog) About Pastest (https://www.pastest.com/about-us)Contact Us (https://www.pastest.com/contact-us) Help (https://www.pastest.com/help)

© Pastest 2016

Page 562: Back to Filters (/Secure/TestMe/Filter ... - 1 File Download

8/11/2016 MyPastest

https://mypastest.pastest.com/Secure/TestMe/Browser/429893#Top 1/2

Back to Filters (/Secure/TestMe/Filter/429893/QA)

Question 102 of 207

A patient with type-2 diabetes is found to have consistently high blood pressure in clinic, withhis latest recording 156/90 mmHg. Resting ECG shows normal sinus rhythm, and urinalysisshows the presence of microalbuminuria.

Which of the following would be the most appropriate class of antihypertensive drugs toconsider for this patient?

Explanation

The answer is ACE inhibitor –

Antihypertensive treatment retards the progression of diabetic nephropathy from itsearliest stages, slowing the progression of microalbuminuria to overt proteinuria and thedecline of glomerular filtration rate in established diabetic nephropathy.

Strongest evidence of benefit is from clinical trials involving angiotensin convertingenzyme (ACE) inhibitors and angiotensin receptor blockers (ARBs), suggesting thatthere may be specific mechanisms related to the renin–angiotensin–aldosterone systemin addition to blood pressure lowering. The blood pressure target is the lowest that canbe achieved without adverse effects, ideally less than 130/80 mmHg, or less than 125/75mmHg when there is proteinuria.

Alpha-blocker (Option B) is incorrect. Alpha-blocker therapy is associated with a greater riskof developing heart failure, and is used only where blood pressure cannot be controlledadequately by other antihypertensive agents.

Beta-blocker (Option C) is incorrect. Beta-blockers show less effective prevention of strokeand new-onset atrial fibrillation compared to agents that block the renin–angiotensin–

ACE inhibitorA

Alpha-blockerB

Beta-blockerC

Calcium-channel blockerD

DiureticE

Page 563: Back to Filters (/Secure/TestMe/Filter ... - 1 File Download

8/11/2016 MyPastest

https://mypastest.pastest.com/Secure/TestMe/Browser/429893#Top 2/2

46795

aldosterone pathway, even when capable of achieving the same blood pressure control.Therefore, they are normally reserved for second-line therapy.

Calcium-channel blocker (Option D) is incorrect. Calcium-channel blockers are useful whenadded to an ACE inhibitor in order to achieve target blood pressure, but there is lesssupportive data for calcium-channel blockers in delaying progression of diabeticnephropathy.

Diuretic (Option E) is incorrect. Thiazide diuretics are effective in lowering blood pressure,but may aggravate hyperglycaemia and dyslipidaemia.

Next Question

Previous Question Tag Question Feedback End Review

Difficulty: Average

Peer Responses

Session Progress

0Responses Correct:

207Responses Incorrect:

207Responses Total:

0%Responses - % Correct:

Blog (https://www.pastest.com/blog) About Pastest (https://www.pastest.com/about-us)Contact Us (https://www.pastest.com/contact-us) Help (https://www.pastest.com/help)

© Pastest 2016

Page 564: Back to Filters (/Secure/TestMe/Filter ... - 1 File Download

8/11/2016 MyPastest

https://mypastest.pastest.com/Secure/TestMe/Browser/429893#Top 1/2

Back to Filters (/Secure/TestMe/Filter/429893/QA)

Question 103 of 207

A 32-year-old man presents with epigastric tenderness and fever. He has a past history oftreated epilepsy. On examination he has a blood pressure of 100/60 mmHg, pulse of 110 bpmand regular and severe pain on palpation of the epigastrium. Blood tests revealhypocalcaemia, metabolic acidosis and a markedly elevated serum amylase. He cannotremember what he takes for his epilepsy.

Which of the following antiepileptic agents is most likely to have caused this acute illness?

Explanation

The answer is Valproate –

The clinical scenario is strongly suggestive of acute pancreatitis. The commonest factorscausing this are biliary disease and alcohol, but in up to 10% of cases a drug cause maybe identified:

Thiazides, furosemide diuretics

Corticosteroids

Tetracyclines, erythromycin, nitrofurantoin, metronidazole

Oestrogens

Valproate

Azathioprine

Methyldopa

Procainamide

Angiotensin converting enzyme (ACE) inhibitors

CarbamazepineA

LamotrigineB

PhenytoinC

TopiramateD

ValproateE

Page 565: Back to Filters (/Secure/TestMe/Filter ... - 1 File Download

8/11/2016 MyPastest

https://mypastest.pastest.com/Secure/TestMe/Browser/429893#Top 2/2

46952

Ranitidine

Carbamazepine (Option A) is incorrect. Acute pancreatitis is not a characteristic adverseeffect of carbamazepine; acute hepatitis may occur.

Lamotrigine (Option B) is incorrect. Acute pancreatitis is not a characteristic adverse effectof lamotrigine.

Phenytoin (Option C) is incorrect. Acute pancreatitis is not a characteristic adverse effect ofphenytoin; acute hepatitis may occur.

Topiramate (Option D) is incorrect. Acute pancreatitis is not a characteristic adverse effect oftopiramate.

Next Question

Previous Question Tag Question Feedback End Review

Difficulty: Average

Peer Responses

Session Progress

0Responses Correct:

207Responses Incorrect:

207Responses Total:

0%Responses - % Correct:

Blog (https://www.pastest.com/blog) About Pastest (https://www.pastest.com/about-us)Contact Us (https://www.pastest.com/contact-us) Help (https://www.pastest.com/help)

© Pastest 2016

Page 566: Back to Filters (/Secure/TestMe/Filter ... - 1 File Download

8/11/2016 MyPastest

https://mypastest.pastest.com/Secure/TestMe/Browser/429893#Top 1/2

Back to Filters (/Secure/TestMe/Filter/429893/QA)

Question 104 of 207

46854

A 40-year-old woman with type-2 diabetes is receiving regular insulin therapy.

Which of the following enzymes is most likely to be inhibited by insulin?

Explanation

The answer is Pyruvate carboxylase –

Insulin inhibits enzymes involved in gluconeogenesis, including pyruvate carboxylase.Additional effects are to stimulate glycogenesis by stimulating glycogen synthetase inliver and muscle. Other effects include activation of the hexose monophosphate (HMP)shunt by inducing the synthesis of glucose 6-phosphate dehydrogenase, and activationof acetyl-CoA carboxylase and ATP citrate lyase to increase the synthesis of fatty acids.

Acetyl-CoA carboxylase (Option A) is incorrect. Insulin stimulates acetyl-CoA carboxylaseand ATP citrate lyase, which are central elements of the Krebs cycle and result in formationof free fatty acids.

ATP citrate lyase (Option B) is incorrect. Insulin stimulates acetyl-CoA carboxylase and ATPcitrate lyase, which are central elements of the Krebs cycle and result in formation of freefatty acids.

Glucose 6-phosphate dehydrogenase (Option C) is incorrect. Insulin stimulates activity ofglucose 6-phosphate dehydrogenase activity.

Glycogen synthetase (Option D) is incorrect. Insulin stimulates glycogen storage, in part byactivating glycogen synthetase.

Acetyl-CoA carboxylaseA

ATP citrate lyaseB

Glucose 6-phosphate dehydrogenaseC

Glycogen synthetaseD

Pyruvate carboxylaseE

Page 567: Back to Filters (/Secure/TestMe/Filter ... - 1 File Download

8/11/2016 MyPastest

https://mypastest.pastest.com/Secure/TestMe/Browser/429893#Top 2/2

Next Question

Previous Question Tag Question Feedback End Review

Difficulty: Average

Peer Responses

Session Progress

0Responses Correct:

207Responses Incorrect:

207Responses Total:

0%Responses - % Correct:

Blog (https://www.pastest.com/blog) About Pastest (https://www.pastest.com/about-us)Contact Us (https://www.pastest.com/contact-us) Help (https://www.pastest.com/help)

© Pastest 2016

Page 568: Back to Filters (/Secure/TestMe/Filter ... - 1 File Download

8/11/2016 MyPastest

https://mypastest.pastest.com/Secure/TestMe/Browser/429893#Top 1/2

Back to Filters (/Secure/TestMe/Filter/429893/QA)

Question 105 of 207

A 64-year-old woman with long-standing osteoarthritis presents to hospital with a 3-dayhistory of melaena. She has been receiving naproxen 500 mg twice daily for the past 4months.

Which of the following statements best pertains to gastrointestinal bleeding due to NSAIDtherapy?

Explanation

The answer is It is due to depletion of mucosal prostaglandin E (PGE) levels –

Patients who have pre-existing gastric or duodenal ulcers are more prone togastrointestinal (GI) bleeds with non-steroidal anti-inflammatory drug (NSAID)treatment. However, GI bleeds are also common in those with no such previous historyand may occur in patients without any significant history of previous dyspepsia. The riskof causing GI bleeds differs with different agents; ibuprofen confers a relatively low riskwhereas piroxicam and azapropazone confer the highest risk. Endoscopic evidence ofpeptic ulceration is found in 20% of NSAID users even in the absence of symptoms.NSAIDs cause direct damage to the gastric mucosal barrier by depleting mucosal PGElevels; NSAIDs decrease the gastroduodenal defence mechanisms and cytoprotectiveeffect of PGE, resulting in mucosal injury, erosions and ulceration.

It is due to decreased platelet adhesiveness (Option A) is incorrect. Disruption of themucosal integrity causes bleeding, not decreased platelet adhesiveness.

Occurs when the patient complains of severe dyspepsia (Option C) is incorrect. There is apoor correlation between symptoms and severity of ulceration and bleeding risk.

It is due to decreased platelet adhesivenessA

It is due to depletion of mucosal prostaglandin E levelsB

Occurs when the patient complains of severe dyspepsiaC

Only occurs in patients who have pre-existing gastric and/or duodenal ulcersD

Only occurs with high-risk NSAIDs such as piroxicamE

Page 569: Back to Filters (/Secure/TestMe/Filter ... - 1 File Download

8/11/2016 MyPastest

https://mypastest.pastest.com/Secure/TestMe/Browser/429893#Top 2/2

46612

Only occurs in patients who have pre-existing gastric and/or duodenal ulcers (Option D) isincorrect. NSAIDs may provoke peptic ulceration even in the absence of prior ulceration.

Only occurs with high-risk NSAIDs such as piroxicam (Option E) is incorrect. Ulceration mayoccur after exposure to any NSAID.

Next Question

Previous Question Tag Question Feedback End Review

Difficulty: Average

Peer Responses

Session Progress

0Responses Correct:

207Responses Incorrect:

207Responses Total:

0%Responses - % Correct:

Blog (https://www.pastest.com/blog) About Pastest (https://www.pastest.com/about-us)Contact Us (https://www.pastest.com/contact-us) Help (https://www.pastest.com/help)

© Pastest 2016

Page 570: Back to Filters (/Secure/TestMe/Filter ... - 1 File Download

8/11/2016 MyPastest

https://mypastest.pastest.com/Secure/TestMe/Browser/429893#Top 1/2

Back to Filters (/Secure/TestMe/Filter/429893/QA)

Question 106 of 207

A 24-year-old woman is undergoing chemotherapy for acute leukaemia. She is admitted tothe Acute Medical Unit due to severe vomiting and you decide to administer intravenoushydration and antiemetic treatment.

Which of the following would be the most appropriate antiemetic choice?

Explanation

The answer is Ondansetron IV –

Ondansetron is a potent 5-hydroxytryptamine-3 (5-HT ) receptor antagonist that is welltolerated. The main adverse effect is headache. Given that vomiting is likely to continue,intravenous would be preferred rather than oral route of administration. Studies haveshown that 5-HT antagonists are generally more effective in treating post-chemotherapy nausea and vomiting than other antiemetic agents.

Metoclopramide IM (Option A) is incorrect. Metoclopramide is less likely to be effective thanondansetron. Antidopaminergic drugs, such as metoclopramide, should be avoided owing toincreased risk of side-effects in young women in particular.

Oral domperidone (Option C) is incorrect. The oral route would be inappropriate;domperidone would be less likely to be effective than ondansetron, but could be consideredas a second-line agent if ondansetron is ineffective or not tolerated.

Oral ondansetron (Option D) is incorrect. The oral route would be inappropriate in the settingof active vomiting.

Metoclopramide IMA

Ondansetron IVB

Oral domperidoneC

Oral ondansetronD

Prochlorperazine IME

3

3

Page 571: Back to Filters (/Secure/TestMe/Filter ... - 1 File Download

8/11/2016 MyPastest

https://mypastest.pastest.com/Secure/TestMe/Browser/429893#Top 2/2

46954

Prochlorperazine IM (Option E) is incorrect. Prochlorperazine is less likely to be effectivethan ondansetron, and the onset of action would be less predictable after intramuscularrather than intravenous administration.

Next Question

Previous Question Tag Question Feedback End Review

Difficulty: Average

Peer Responses

Session Progress

0Responses Correct:

207Responses Incorrect:

207Responses Total:

0%Responses - % Correct:

Blog (https://www.pastest.com/blog) About Pastest (https://www.pastest.com/about-us)Contact Us (https://www.pastest.com/contact-us) Help (https://www.pastest.com/help)

© Pastest 2016

Page 572: Back to Filters (/Secure/TestMe/Filter ... - 1 File Download

8/11/2016 MyPastest

https://mypastest.pastest.com/Secure/TestMe/Browser/429893#Top 1/2

Back to Filters (/Secure/TestMe/Filter/429893/QA)

Question 107 of 207

You are called by a local GP who asks for advice about the best antibiotic to use to treat anacute skin infection in a woman who is breast-feeding her 2-week-old baby.

Which of the following antibiotics would be the best option in this patient?

Explanation

The answer is Flucloxacillin –

The penicillins are considered to be relatively safe in breast-feeding.

Ciprofloxacin (Option A) is incorrect. Ciprofloxacin is subject to widespread tissuedistribution, including secretion in significant quantities in breast milk. Quinolones arethought to increase the risk of tendon abnormalities.

Clarithromycin (Option B) is incorrect. Clarithromycin is not recommended by themanufacturer unless the benefits of treatment outweigh the risk, although macrolides arerelatively well used in breast-feeding women with no apparent adverse events.

Fluconazole (Option D) is incorrect. Fluconazole is also not recommended for use by themanufacturer in patients who are breast-feeding. It is a lipophilic agent and levelsaccumulated in the infant via breast-feeding are greater than those recommended for use inchildren. It is important to note that significant changes in cytochrome P450 metabolismtake place during the first few months of life, so that safety in a parent or adult population isno guarantee of safety in the neonate.

Ofloxacin (Option E) is incorrect. Ofloxacin is subject to widespread tissue distribution,including secretion in significant quantities in breast milk. Quinolones are thought to increase

CiprofloxacinA

ClarithromycinB

FlucloxacillinC

FluconazoleD

OfloxacinE

Page 573: Back to Filters (/Secure/TestMe/Filter ... - 1 File Download

8/11/2016 MyPastest

https://mypastest.pastest.com/Secure/TestMe/Browser/429893#Top 2/2

46841

the risk of tendon abnormalities.

Next Question

Previous Question Tag Question Feedback End Review

Difficulty: Average

Peer Responses

Session Progress

0Responses Correct:

207Responses Incorrect:

207Responses Total:

0%Responses - % Correct:

Blog (https://www.pastest.com/blog) About Pastest (https://www.pastest.com/about-us)Contact Us (https://www.pastest.com/contact-us) Help (https://www.pastest.com/help)

© Pastest 2016

Page 574: Back to Filters (/Secure/TestMe/Filter ... - 1 File Download

8/11/2016 MyPastest

https://mypastest.pastest.com/Secure/TestMe/Browser/429893#Top 1/2

Back to Filters (/Secure/TestMe/Filter/429893/QA)

Question 108 of 207

Probenecid may be used to reduce the renal clearance of certain drugs, thereby prolongingtheir half-life and duration of action.

For which one of the following drugs is the co-administration of probenecid most likely toprovide improved clinical effectiveness?

Explanation

The answer is Penicillin –

Drugs may be secreted into the proximal convoluted tubule of the nephron by cationtransporters (eg quinine, pethidine, morphine) or anion transporters (eg penicillins,bendroflumethiazide, furosemide, cephalosporins). Probenecid interferes with aniontransport and therby reduces the renal elimination of these drugs, and prolongs theirduration of action. Penicillin has a short duration of action and requires multiple dailydose administration for efficacy; co-administration of probenecid allows maintenance ofpenicillin blood concentrations higher than the minimum inhibitory concentrations withless frequent dose administration.

Aspirin (Option A) is incorrect. Aspirin has a short duration of action of 1–2 hours, but this issufficient to inhibit platelet function for much longer because platelets are unable toregenerate cyclo-oxygenase. Probenecid may prolong the duration of aspirin action, but thiswould not alter its pharmacodynamic actions.

Bendroflumethiazide (Option B) is incorrect. Probenecid is capable of interfering withbendroflumethiazide elimination, but this will not produce any meaningful clinical benefitbecause the blood-pressure-lowering effects of bendroflumethiazide persist for several

AspirinA

BendroflumethiazideB

FurosemideC

PenicillinD

QuinineE

Page 575: Back to Filters (/Secure/TestMe/Filter ... - 1 File Download

8/11/2016 MyPastest

https://mypastest.pastest.com/Secure/TestMe/Browser/429893#Top 2/2

45806

weeks.

Furosemide (Option C) is incorrect. Probenecid may enhance the actions of furosemide, butthis is unlikely to offer any meaningful clinical benefit.

Quinine (Option E) is incorrect. Probenecid does not interfere with quinine elimination.

Next Question

Previous Question Tag Question Feedback End Review

Difficulty: Average

Peer Responses

Session Progress

0Responses Correct:

207Responses Incorrect:

207Responses Total:

0%Responses - % Correct:

Blog (https://www.pastest.com/blog) About Pastest (https://www.pastest.com/about-us)Contact Us (https://www.pastest.com/contact-us) Help (https://www.pastest.com/help)

© Pastest 2016

Page 576: Back to Filters (/Secure/TestMe/Filter ... - 1 File Download

8/11/2016 MyPastest

https://mypastest.pastest.com/Secure/TestMe/Browser/429893#Top 1/2

Back to Filters (/Secure/TestMe/Filter/429893/QA)

Question 109 of 207

46821

A 25-year-old patient, who has a past history of cluster headaches, presents to the AcuteMedical Unit with an acute attack of cluster headache.

Which one of the following drugs is most likely to be an effective treatment?

Explanation

The answer is Sumatriptan –

Sumatriptan is effective, and offers rapid relief of symptoms. Alternatively, administrationof 100% oxygen at a rate of 7–12 l/min is useful in aborting an attack. Prophylaxis isconsidered if cluster headaches are frequent or last more than 3 weeks, normally withverapamil or lithium. Ergotamine may be used intermittently, but should be used withcaution in view of adverse effects related to vasospasm.

Aspirin (Option A) is incorrect. Aspirin is not a recognised treatment for cluster headaches.

Atenolol (Option B) is incorrect. Atenolol is ineffective. Non-cardioselective β-blockersincluding propranolol may be effective in preventing migraine.

Levocabastine (Option C) is incorrect. Levocabastine is an anti-histamine used for thetreatment of allergic conjunctivitis.

Paracetamol (Option D) is incorrect. Paracetamol is often ineffective for cluster headache.

AspirinA

AtenololB

LevocabastineC

ParacetamolD

SumatriptanE

Next Question

Page 577: Back to Filters (/Secure/TestMe/Filter ... - 1 File Download

8/11/2016 MyPastest

https://mypastest.pastest.com/Secure/TestMe/Browser/429893#Top 2/2

Previous Question Tag Question Feedback End Review

Difficulty: Average

Peer Responses

Session Progress

0Responses Correct:

207Responses Incorrect:

207Responses Total:

0%Responses - % Correct:

Blog (https://www.pastest.com/blog) About Pastest (https://www.pastest.com/about-us)Contact Us (https://www.pastest.com/contact-us) Help (https://www.pastest.com/help)

© Pastest 2016

Page 578: Back to Filters (/Secure/TestMe/Filter ... - 1 File Download

8/11/2016 MyPastest

https://mypastest.pastest.com/Secure/TestMe/Browser/429893#Top 1/2

Back to Filters (/Secure/TestMe/Filter/429893/QA)

Question 110 of 207

46993

A patient is admitted to the ward for observation after sustaining a head injury while runningaway from the police. After 36 h on the ward he is suffering from hallucinations,hypersalivation, irritability and diarrhoea. You suspect he is withdrawing from the effects ofprevious recreational drug abuse.

Withdrawal of which of the following would be most likely to account for these features?

Explanation

The answer is Heroin –

The following are all signs of heroin withdrawal: sweating, diarrhoea, nausea andvomiting, irritability and restlessness, and muscle ache. Management of long-term heroinwithdrawal involves methadone or buprenorphine maintenance programmes to avoid thehazards of heroin injection.

Alcohol (Option A) is incorrect. Ethanol withdrawal may cause similar symptoms, but is lesslikely to cause excess salivation or diarrhoea.

Amphetamine (Option B) is incorrect. Amphetamines are predominantly short-actingstimulants, and withdrawal is uncommon.

Cocaine (Option C) is incorrect. Cocaine is a short-acting potent stimulant.

Ecstasy (Option D) is incorrect. Ecstasy is an intermediate-acting stimulant that enhancesserotonin activity within the central nervous system. Withdrawal may cause dysphoria.

AlcoholA

AmphetamineB

CocaineC

EcstasyD

HeroinE

Page 579: Back to Filters (/Secure/TestMe/Filter ... - 1 File Download

8/11/2016 MyPastest

https://mypastest.pastest.com/Secure/TestMe/Browser/429893#Top 2/2

Next Question

Previous Question Tag Question Feedback End Review

Difficulty: Average

Peer Responses

Session Progress

0Responses Correct:

207Responses Incorrect:

207Responses Total:

0%Responses - % Correct:

Blog (https://www.pastest.com/blog) About Pastest (https://www.pastest.com/about-us)Contact Us (https://www.pastest.com/contact-us) Help (https://www.pastest.com/help)

© Pastest 2016

Page 580: Back to Filters (/Secure/TestMe/Filter ... - 1 File Download

8/11/2016 MyPastest

https://mypastest.pastest.com/Secure/TestMe/Browser/429893#Top 1/2

Back to Filters (/Secure/TestMe/Filter/429893/QA)

Question 111 of 207

A 74-year-old woman with a history of depression presents to the Emergency Departmentwith a history of agitation, slurred speech, tremor and reduced conscious level. Serum lithiumconcentration is reported as 3.3 mmol/l.

Which one of the following treatments would be most effective in treating lithium toxicity?

Explanation

The answer is Haemodialysis –

Toxicity is usually associated with levels > 1.5 mmol/l, and clinical features include tremor,thirst, polyuria, diarrhoea, hypertonia, seizures and impaired consciousness.Haemodialysis is the treatment of choice for severe lithium toxicity. In milder cases,symptomatic treatment is usually all that is needed. All patients with lithium poisoningshould have measurement of lithium levels and plasma urea, electrolytes and osmolality.

Activated charcoal (Option A) is incorrect. Activated charcoal adsorbs drugs in the gut, andincreases removal of drugs from the body by interfering with entero-hepatic recirculation.

Forced diuresis with sodium chloride (Option B) is incorrect. Forced diuresis is not effectivein minimising toxicity and may be hazardous; sufficient IV fluids should be administered tomaintain hydration, with correction of electrolytes.

Methionine (Option D) is incorrect. Methionine is used as an oral antidote for paracetamolpoisoning in those who cannot tolerate acetylcysteine or where acetylcysteine is notavailable.

Activated charcoalA

Forced diuresis with sodium chlorideB

HaemodialysisC

MethionineD

MethylprednisoloneE

Page 581: Back to Filters (/Secure/TestMe/Filter ... - 1 File Download

8/11/2016 MyPastest

https://mypastest.pastest.com/Secure/TestMe/Browser/429893#Top 2/2

46733

Methylprednisolone (Option E) is incorrect. Methylprednisolone may enhance renal clearanceto a very limited extent, but would be too slow and ineffective to have a role in lithiumtoxicity.

Next Question

Previous Question Tag Question Feedback End Review

Difficulty: Average

Peer Responses

Session Progress

0Responses Correct:

207Responses Incorrect:

207Responses Total:

0%Responses - % Correct:

Blog (https://www.pastest.com/blog) About Pastest (https://www.pastest.com/about-us)Contact Us (https://www.pastest.com/contact-us) Help (https://www.pastest.com/help)

© Pastest 2016

Page 582: Back to Filters (/Secure/TestMe/Filter ... - 1 File Download

8/11/2016 MyPastest

https://mypastest.pastest.com/Secure/TestMe/Browser/429893#Top 1/2

Back to Filters (/Secure/TestMe/Filter/429893/QA)

Question 112 of 207

38677

A 72-year-old man comes to the Elderly Care clinic with his wife for the results of tests todetermine the underlying cause of dementia, diagnosed some 2 months earlier because ofprogressively increasing confusion and memory loss over the past year. Whilst he is still ableto wash and dress himself, his wife is finding it increasingly difficult to cope with himwandering and trying to get out of the house at night. He takes no regular medication. Onexamination his BP is 132/72 mmHg; pulse is 79/min and regular. He looks slightly unkempt.His BMI is 22 kg/m2. MMSE is 12/30.

They are given a diagnosis of Alzheimer’s disease.

Which of the following is the most appropriate initial therapy?

Explanation

The answer is Donepezil -

Acetylcholinesterase inhibitor treatment (Donepezil, Galantamine or Rivastigmine) should beconsidered in patients with mild or moderate Alzheimer's disease although should ideally beinitiated by a specialist. In patients where these first line therapies are not tolerated, or arecontra-indicated, NICE recommends use of Memantine. Tricyclic antidepressants such asAmitriptyline are not recommended because they may worsen underlying confusion. Anti-psychotics should be avoided if possible, although if needed for significant delusions andaggressive or confused behaviour, Risperidone would be considered first line. Lorazepam is apotential option, particularly for IM use in acute confusion and aggression.

AmitriptylineA

DonepezilB

LorazepamC

MemantineD

RisperidoneE

Next Question

Page 583: Back to Filters (/Secure/TestMe/Filter ... - 1 File Download

8/11/2016 MyPastest

https://mypastest.pastest.com/Secure/TestMe/Browser/429893#Top 2/2

Previous Question Tag Question Feedback End Review

Difficulty: Average

Peer Responses

Session Progress

0Responses Correct:

207Responses Incorrect:

207Responses Total:

0%Responses - % Correct:

Blog (https://www.pastest.com/blog) About Pastest (https://www.pastest.com/about-us)Contact Us (https://www.pastest.com/contact-us) Help (https://www.pastest.com/help)

© Pastest 2016

Page 584: Back to Filters (/Secure/TestMe/Filter ... - 1 File Download

8/11/2016 MyPastest

https://mypastest.pastest.com/Secure/TestMe/Browser/429893#Top 1/2

Back to Filters (/Secure/TestMe/Filter/429893/QA)

Question 113 of 207

A 22-year-old female presents to the Emergency Department complaining of rapid, regularpalpitations, which started suddenly while she was studying for exams. On examination she istachycardic at 155 bpm, normotensive and the rest of the examination is unremarkable.Electrocardiogram (ECG) shows a narrow, complex tachycardia, and this is unaltered by vagalmanoeuvres.

Which of the following subsequent measures would be the most appropriate next step?

Explanation

The answer is Intravenous adenosine -

The narrow, complex tachycardia indicates a supraventricular arrhythmia, possibly due toatrial flutter, nodal tachycardia or atrial fibrillation. Such arrhythmias may be precipitatedby ingestion of large quantities of caffeine or other stimulant drugs. Adenosine may beeffective in terminating supraventricular re-entrant tachycardias, and can help todistinguish underlying atrial rhythm.

24-h urine collection for catecholamines (Option A) is incorrect. This may be considered at alater stage, depending on the clinical history.

Direct current (DC) cardioversion (Option B) is incorrect. DC shock would be inappropriateas a first measure before contemplating adenosine, but may be considered if the rhythmdisturbance persists.

Intravenous amiodarone (Option D) is incorrect. Intravenous amiodarone is not normallyindicated in the first instance because the rhythm is often benign and may self-terminate orterminate promptly after adenosine.

24-h urine collection for catecholaminesA

Direct current (DC) cardioversionB

Intravenous adenosineC

Intravenous amiodaroneD

Intravenous digoxinE

Page 585: Back to Filters (/Secure/TestMe/Filter ... - 1 File Download

8/11/2016 MyPastest

https://mypastest.pastest.com/Secure/TestMe/Browser/429893#Top 2/2

46916

Intravenous digoxin (Option E) is incorrect. Intravenous digoxin could render subsequent DCshock (if needed) more hazardous in provoking a more sinister arrhythmia.

Next Question

Previous Question Tag Question Feedback End Review

Difficulty: Average

Peer Responses

Session Progress

0Responses Correct:

207Responses Incorrect:

207Responses Total:

0%Responses - % Correct:

Blog (https://www.pastest.com/blog) About Pastest (https://www.pastest.com/about-us)Contact Us (https://www.pastest.com/contact-us) Help (https://www.pastest.com/help)

© Pastest 2016

Page 586: Back to Filters (/Secure/TestMe/Filter ... - 1 File Download

8/11/2016 MyPastest

https://mypastest.pastest.com/Secure/TestMe/Browser/429893#Top 1/2

Back to Filters (/Secure/TestMe/Filter/429893/QA)

Question 114 of 207

46381

You review a 54-year-old woman in clinic who has recently started on ezetimibe for treatmentof hypercholesterolaemia.

Which of the following statements best describes the pharmacological actions of ezetimibe?

Explanation

The answer is Reduces intestinal cholesterol absorption –

Ezetimibe prevents cholesterol absorption from the gut by directly inhibiting cholesterolreceptors on enterocytes. In contrast to anion-exchange resins (e.g. cholestyramine),ezetimibe does not significantly alter absorption of fat-soluble drugs (e.g. digoxin,thyroxine) or fat-soluble vitamins (A, D, E and K).

It inhibits the cytochrome P450 enzyme system (Option A) is incorrect. Ezetimibe has noeffect on the cytochrome P450 enzyme system.

Its mechanism of action is to reduce cholesterol synthesis (Option B) is incorrect. This is themechanism of statins, which inhibit HMG-CoA reductase, the rate-limiting enzyme incholesterol synthesis.

May lower serum digoxin concentrations (Option C) is incorrect. Despite digoxin being a fat-soluble drug, ezetimibe has no significant effect on its absorption (this is unlike the effect ofcholestyramine that significantly reduces digoxin absorption).

Significantly reduces absorption of fat-soluble vitamins (Option E) is incorrect. Ezetimibedoes not significantly alter absorption of fat-soluble vitamins.

It inhibits the cytochrome P450 enzyme systemA

Its mechanism of action is to reduce cholesterol synthesisB

May lower serum digoxin concentrationsC

Reduces intestinal cholesterol absorptionD

Significantly reduces absorption of fat-soluble vitaminsE

Page 587: Back to Filters (/Secure/TestMe/Filter ... - 1 File Download

8/11/2016 MyPastest

https://mypastest.pastest.com/Secure/TestMe/Browser/429893#Top 2/2

Next Question

Previous Question Tag Question Feedback End Review

Difficulty: Average

Peer Responses

Session Progress

0Responses Correct:

207Responses Incorrect:

207Responses Total:

0%Responses - % Correct:

Blog (https://www.pastest.com/blog) About Pastest (https://www.pastest.com/about-us)Contact Us (https://www.pastest.com/contact-us) Help (https://www.pastest.com/help)

© Pastest 2016

Page 588: Back to Filters (/Secure/TestMe/Filter ... - 1 File Download

8/11/2016 MyPastest

https://mypastest.pastest.com/Secure/TestMe/Browser/429893#Top 1/2

Back to Filters (/Secure/TestMe/Filter/429893/QA)

Question 115 of 207

A 56-year-old man presents to the Emergency Department with jaundice three weeks afterdischarge following failed cardioversion for atrial fibrillation. He is a smoker of 10 cigarettesper day and was started on a number of new medications during his hospital stay.Examination reveals a BP of 125/72 mmHg, pulse of 80/min, atrial fibrillation. He hasjaundiced sclerae and mild tenderness in the right upper quadrant of the abdomen onpalpation. He does not appear to be in cardiac failure.

Investigations;

Hb 13.1 g/dl

WCC 8.7 x10 /l

PLT 197 x10 /l

Na 138 mmol/l

K 4.3 mmol/l

Creatinine 100 micromol/l

Bilirubin 82 micromol/l

AST 1420 U/l

ALP 395 U/l

PT 38.1 s

EchocardiogramStructurally normal heart with no evidence ofvalve disease

Which of the following is the most likely cause?

9

9

+

+

AmiodaroneA

AmlodipineB

FurosemideC

RamiprilD

Page 589: Back to Filters (/Secure/TestMe/Filter ... - 1 File Download

8/11/2016 MyPastest

https://mypastest.pastest.com/Secure/TestMe/Browser/429893#Top 2/2

38083

Explanation

The answer is Amiodarone -

Amiodarone leads to abnormal liver function in 15-50% of patients for whom it is prescribed.This can range from an isolated modest range in transaminases, to fulminant hepatitis. Whilstonset is rare within the first month of therapy, it is well described. In this situation it shouldbe discontinued as soon as possible. One potential alternative in this situation given thestructurally normal heart on echocardiography is flecanide. Hepatitis is very rarely describedin conjunction with Amlodipine therapy. Cholestatic jaundice, rather than a hepatitic picture,is more characteristically described with Ramipril therapy.

WarfarinE

Next Question

Previous Question Tag Question Feedback End Review

Difficulty: Easy

Peer Responses

Session Progress

0Responses Correct:

207Responses Incorrect:

207Responses Total:

0%Responses - % Correct:

Blog (https://www.pastest.com/blog) About Pastest (https://www.pastest.com/about-us)Contact Us (https://www.pastest.com/contact-us) Help (https://www.pastest.com/help)

© Pastest 2016

Page 590: Back to Filters (/Secure/TestMe/Filter ... - 1 File Download

8/11/2016 MyPastest

https://mypastest.pastest.com/Secure/TestMe/Browser/429893#Top 1/2

Back to Filters (/Secure/TestMe/Filter/429893/QA)

Question 116 of 207

A 17-year-old woman is brought into the Emergency Department by friends after she hadtaken an overdose of an unknown quantity of a non-steroidal anti-inflammatory drug(NSAID), thought to be mefenamic acid.

Which of the following statements is true concerning NSAID overdoses?

Explanation

The answer is Mefenamic acid is the NSAID most likely to cause convulsions –

Following overdoses of most NSAIDs, the main effects are mild gastrointestinal upsetwith epigastric tenderness, nausea, vomiting and diarrhea. These effects are mainly dueto the inhibition of cyclo-oxygenase. Up to 10% of patients will have seizures after NSAIDoverdose, particularly after mefenamic acid. Large ingestions may cause acidosis, renalimpairment, gastrointestinal haemorrhage and CNS effects (drowsiness, coma, cerebellarsigns). Treatment is with activated charcoal in patients presenting within 1 h, andsupportive care. Proton-pump inhibitors may reduce the symptoms of gastrointestinaltoxicity.

Convulsions occur in 50% of patients following NSAID overdose (Option A) is incorrect.Seizures may occur, but the rate of occurrence is around 10%.

Gastrointestinal symptoms are due to the stimulation of cyclo-oxygenase (Option B) isincorrect. It is inhibition rather than stimulation of cyclo-oxygenase that occurs.

Metabolic alkalosis usually occurs following large ingestions of NSAIDs (Option D) isincorrect. NSAID ingestion may cause metabolic acidosis.

Convulsions occur in 50% of patients following NSAID overdoseA

Gastrointestinal symptoms are due to the stimulation of cyclo-oxygenaseB

Mefenamic acid is the NSAID most likely to cause convulsionsC

Metabolic alkalosis usually occurs following large ingestions of NSAIDsD

Multi-dose activated charcoal is normally used for NSAID overdosesE

Page 591: Back to Filters (/Secure/TestMe/Filter ... - 1 File Download

8/11/2016 MyPastest

https://mypastest.pastest.com/Secure/TestMe/Browser/429893#Top 2/2

46493

Multi-dose activated charcoal is normally used for NSAID overdoses (Option E) is incorrect.This is normally indicated after aspirin overdose, to enhance drug clearance, but is notnormally used for NSAIDs.

Next Question

Previous Question Tag Question Feedback End Review

Difficulty: Average

Peer Responses

Session Progress

0Responses Correct:

207Responses Incorrect:

207Responses Total:

0%Responses - % Correct:

Blog (https://www.pastest.com/blog) About Pastest (https://www.pastest.com/about-us)Contact Us (https://www.pastest.com/contact-us) Help (https://www.pastest.com/help)

© Pastest 2016

Page 592: Back to Filters (/Secure/TestMe/Filter ... - 1 File Download

8/11/2016 MyPastest

https://mypastest.pastest.com/Secure/TestMe/Browser/429893#Top 1/2

Back to Filters (/Secure/TestMe/Filter/429893/QA)

Question 117 of 207

46969

A 54-year-old woman with a long history of rheumatoid arthritis presents with a dry coughand increasing dyspnoea. Investigation shows SaO2 85% on air. On examination she displaysbilateral crackles. An infection screen is negative and you suspect drug-induced fibrosis.

Which one of the following is most likely to have been the causative drug?

Explanation

The answer is Methotrexate –

Methotrexate can also cause pneumonitis, hepatic fibrosis and folate deficiency.

Azathioprine (Option A) is incorrect. Azathioprine causes dose-related myelosuppression,hair loss, hypotension, cholestatic jaundice, arrhythmia and hypersensitivity reactions.

Ciclosporin (Option B) is incorrect. Ciclosporin is nephrotoxic and can cause cholestasis,hypertrichosis, hypertension, gingival hyperplasia, tremor, nausea and vomiting.

Prednisolone (Option D) is incorrect. Prednisolone has numerous side-effects affecting manyorgan systems but does not cause pulmonary fibrosis; it may be used as a treatment forfibrosing alveolitis.

Sulfasalazine (Option E) is incorrect. Both sulfasalazine and methotrexate can causepulmonary fibrosis, but it is more commonly encountered with methotrexate therapy.Sulfasalazine can cause myelosuppression, decreased sperm count, hepatitis, oral ulcers, rashand haemolytic anaemia.

AzathioprineA

CiclosporinB

MethotrexateC

PrednisoloneD

SulfasalazineE

Page 593: Back to Filters (/Secure/TestMe/Filter ... - 1 File Download

8/11/2016 MyPastest

https://mypastest.pastest.com/Secure/TestMe/Browser/429893#Top 2/2

Next Question

Previous Question Tag Question Feedback End Review

Difficulty: Average

Peer Responses

Session Progress

0Responses Correct:

207Responses Incorrect:

207Responses Total:

0%Responses - % Correct:

Blog (https://www.pastest.com/blog) About Pastest (https://www.pastest.com/about-us)Contact Us (https://www.pastest.com/contact-us) Help (https://www.pastest.com/help)

© Pastest 2016

Page 594: Back to Filters (/Secure/TestMe/Filter ... - 1 File Download

8/11/2016 MyPastest

https://mypastest.pastest.com/Secure/TestMe/Browser/429893#Top 1/2

Back to Filters (/Secure/TestMe/Filter/429893/QA)

Question 118 of 207

46929

A 23-year-old man was brought in to the Emergency Department from a nightclub after hewas found unconscious. His heart rate is 134 bpm, blood pressure was measured at 165/90mmHg on admission. He had the following investigation results: K+ 2.5 mmol/l, Na+ 138mmol/l, urea 5.5 mmol/l and creatinine 85 μmol/l.

Which of the following is the most likely cause?

Explanation

The answer is Amphetamine –

Amphetamine use may be associated with mydriasis, hypertension, tachycardia, skinpallor, hyperexcitability and, in the initial stages, agitation and increased talkativenessand euphoria. Hypokalaemia is seen as a result of sympathetic stimulation. Severetoxicity may be associated with hyperpyrexia, rhabdomyolysis, acute renal failure andacute liver failure. High blood pressure may require β-blockade, and agitated patientsmay be sedated with benzodiazepines.

Atenolol (Option B) is incorrect. Atenolol may cause significant bradycardia and heart block.

Cannabis (Option C) is incorrect. Cannabis rarely causes such significant increased heart rateand blood pressure.

Ethylene glycol (Option D) is incorrect. Ethylene glycol toxicity may cause profoundhypocalcaemia and renal failure; hypokalaemia is uncommon.

Heroin (Option E) is incorrect. Opioids are associated with hypotension and bradycardia.

AmphetamineA

AtenololB

CannabisC

Ethylene glycolD

HeroinE

Page 595: Back to Filters (/Secure/TestMe/Filter ... - 1 File Download

8/11/2016 MyPastest

https://mypastest.pastest.com/Secure/TestMe/Browser/429893#Top 2/2

Next Question

Previous Question Tag Question Feedback End Review

Difficulty: Average

Peer Responses

Session Progress

0Responses Correct:

207Responses Incorrect:

207Responses Total:

0%Responses - % Correct:

Blog (https://www.pastest.com/blog) About Pastest (https://www.pastest.com/about-us)Contact Us (https://www.pastest.com/contact-us) Help (https://www.pastest.com/help)

© Pastest 2016

Page 596: Back to Filters (/Secure/TestMe/Filter ... - 1 File Download

8/11/2016 MyPastest

https://mypastest.pastest.com/Secure/TestMe/Browser/429893#Top 1/2

Back to Filters (/Secure/TestMe/Filter/429893/QA)

Question 119 of 207

A 54-year-old gardener with long-standing depression presents to the EmergencyDepartment after taking a deliberate overdose of pesticide product. The package label statesthat the produce contains Paraquat.

What is the main mechanism of Paraquat toxicity?

Explanation

The correct answer is Cell injury and death is mediated by oxygen free radical toxicity –

Paraquat is highly toxic, and as little as 2 g in an adult may be fatal (10 ml of aconcentrated 20% solution). Paraquat is rapidly absorbed and is sequestered in thelungs. Its mechanism of toxicity is reaction with oxygen to produce hydrogen peroxideand superoxide, which cause oxidative tissue injury, inflammation and an acute alveolitis.Death tends to occur due to respiratory failure within hours to days in patients thatingest more than 6 g of Paraquat. Pulmonary fibrosis is a later complication that candevelop up to 6 weeks after ingestion. Early features may be caused by corrosive effectson the gastrointestinal tract and oropharynx.

Early death may be caused by pulmonary fibrosis (Option B) is incorrect. Pulmonary fibrosisis a later complication that can develop 6 weeks or more after ingestion.

Liver injury (Option C) is incorrect. This is a less important mechanism than pulmonary andgastrointestinal toxicity.

Oesophageal irritation and obstruction (Option D) is incorrect. Although these arerecognised features, the main mechanism of death is pulmonary toxicity.

Cell injury and death mediated by oxygen free radical toxicityA

Early death may be caused by pulmonary fibrosisB

Liver injuryC

Oesophageal irritation and obstructionD

Tissue hypoxiaE

Page 597: Back to Filters (/Secure/TestMe/Filter ... - 1 File Download

8/11/2016 MyPastest

https://mypastest.pastest.com/Secure/TestMe/Browser/429893#Top 2/2

45796

Tissue hypoxia (Option E) is incorrect. Although severe tissue hypoxia is undesirable, minorhypoxia can be tolerated. Oxygen supplementation can increase free radical formation andworsen pulmonary toxicity; supplementary administration should be avoided if possible.

Next Question

Previous Question Tag Question Feedback End Review

Difficulty: Average

Peer Responses

Session Progress

0Responses Correct:

207Responses Incorrect:

207Responses Total:

0%Responses - % Correct:

Blog (https://www.pastest.com/blog) About Pastest (https://www.pastest.com/about-us)Contact Us (https://www.pastest.com/contact-us) Help (https://www.pastest.com/help)

© Pastest 2016

Page 598: Back to Filters (/Secure/TestMe/Filter ... - 1 File Download

8/11/2016 MyPastest

https://mypastest.pastest.com/Secure/TestMe/Browser/429893#Top 1/2

Back to Filters (/Secure/TestMe/Filter/429893/QA)

Question 120 of 207

46736

The husband of a woman admitted to hospital with meningococcal meningitis asks you aboutthe risks that he too may develop meningitis, and asks you about the possible role ofprophylactic antibiotic treatment.

What is the best choice of antibiotic prophylaxis for close contacts of patients withmeningococcal meningitis?

Explanation

The answer is Ciprofloxacin –

Household and other close contacts of patients with pyogenic meningitis should begiven oral ciprofloxacin.

Ceftriaxone (Option A) is incorrect. Ceftriaxone is the drug of choice for treatment ofmeningitis without a blanching rash, aged 18–50 years.

Co-trimoxazole (Option C) is incorrect. Co-trimoxazole is useful for adults over 50 years ofage with bacterial meningitis, where Listeria monocytogenes infection is suspected.

Rifampicin (Option D) is incorrect. Rifampicin is effective against meningococcal meningitisbut a recent Cochrane review has supported ciprofloxacin ahead of it as prophylaxis ofchoice owing to greater tolerability.

Vancomycin (Option E) is incorrect. Vancomycin is given in penicillin-resistant pneumococcalinfection and to those with a history of anaphylaxis to ß-lactam antibiotics.

CeftriaxoneA

CiprofloxacinB

Co-trimoxazoleC

RifampicinD

VancomycinE

Page 599: Back to Filters (/Secure/TestMe/Filter ... - 1 File Download

8/11/2016 MyPastest

https://mypastest.pastest.com/Secure/TestMe/Browser/429893#Top 2/2

Next Question

Previous Question Tag Question Feedback End Review

Difficulty: Average

Peer Responses

Session Progress

0Responses Correct:

207Responses Incorrect:

207Responses Total:

0%Responses - % Correct:

Blog (https://www.pastest.com/blog) About Pastest (https://www.pastest.com/about-us)Contact Us (https://www.pastest.com/contact-us) Help (https://www.pastest.com/help)

© Pastest 2016

Page 600: Back to Filters (/Secure/TestMe/Filter ... - 1 File Download

8/11/2016 MyPastest

https://mypastest.pastest.com/Secure/TestMe/Browser/429893#Top 1/2

Back to Filters (/Secure/TestMe/Filter/429893/QA)

Question 121 of 207

46836

You are reviewing a 73-year-old man with multiple cardiac problems. He is currently takingramipril, metoprolol, furosemide, amiodarone and aspirin. Over the past few months he hasnoted problems with night glare and his optician has diagnosed corneal microdeposits.

Which of the following drugs in his regime is most likely to be responsible?

Explanation

The answer is Amiodarone –

Amiodarone has a number of long-term side-effects, one of which is the development ofcorneal microdeposits; this may manifest in night-time visual glare. Amiodarone containsiodine and may result in the development of hypo- or hyperthyroidism, both of whichmay be complex to manage. Rare but recognised respiratory adverse effects includepneumonitis and pulmonary fibrosis, and should be considered in patients that presentwith increasing shortness of breath and cough. Hepatotoxicity may occur, and patientson amiodarone should be monitored for liver function test abnormalities.

Aspirin (Option B) is incorrect. Aspirin is not expected to cause visual disturbance.

Furosemide (Option C) is incorrect. Furosemide is not expected to cause visual disturbance.

Metoprolol (Option D) is incorrect. Beta-blockers are not expected to cause visualimpairment; they may be used to treat glaucoma.

Ramipril (Option E) is incorrect. ACE inhibitors are not expected to cause visual impairment.

AmiodaroneA

AspirinB

FurosemideC

MetoprololD

RamiprilE

Page 601: Back to Filters (/Secure/TestMe/Filter ... - 1 File Download

8/11/2016 MyPastest

https://mypastest.pastest.com/Secure/TestMe/Browser/429893#Top 2/2

Next Question

Previous Question Tag Question Feedback End Review

Difficulty: Average

Peer Responses

Session Progress

0Responses Correct:

207Responses Incorrect:

207Responses Total:

0%Responses - % Correct:

Blog (https://www.pastest.com/blog) About Pastest (https://www.pastest.com/about-us)Contact Us (https://www.pastest.com/contact-us) Help (https://www.pastest.com/help)

© Pastest 2016

Page 602: Back to Filters (/Secure/TestMe/Filter ... - 1 File Download

8/11/2016 MyPastest

https://mypastest.pastest.com/Secure/TestMe/Browser/429893#Top 1/2

Back to Filters (/Secure/TestMe/Filter/429893/QA)

Question 122 of 207

46775

A 25-year-old woman is admitted to hospital with severe diarrhoea following treatment withamoxicillin for a bad throat infection. Stool culture yields Clostridium difficile. She becomesdehydrated requiring IV fluids and is unable to tolerate oral medication owing to her sorethroat.

What is the best IV treatment?

Explanation

The answer is Metronidazole –

If parenteral treatment is required, IV metronidazole is effective, as excretion ofmetronidazole into bile occurs, and exudation from the inflamed colon results inbactericidal levels in faeces.

Erythromycin (Option A) is incorrect. Erythromycin may itself cause an overgrowth of C.difficile and is a prokinetic agent that may worsen diarrhoea symptoms.

Neomycin (Option C) is incorrect. Drugs of the neomycin group are not significantlyabsorbed from the gut and oral formulations are effective against bacterial gastroenteritis.

Tobramycin (Option D) is incorrect. Tobramycin is an aminoglycoside active against Gram-positive and Gram-negative bacteria; it is not useful in the treatment of C. difficile infection.

Vancomycin (Option E) is incorrect. Both vancomycin and metronidazole are effective afteroral administration. Intravenous vancomycin is less effective than metronidazole as it haslower availability within the gut after intravenous administration.

ErythromycinA

MetronidazoleB

NeomycinC

TobramycinD

VancomycinE

Page 603: Back to Filters (/Secure/TestMe/Filter ... - 1 File Download

8/11/2016 MyPastest

https://mypastest.pastest.com/Secure/TestMe/Browser/429893#Top 2/2

Next Question

Previous Question Tag Question Feedback End Review

Difficulty: Average

Peer Responses

Session Progress

0Responses Correct:

207Responses Incorrect:

207Responses Total:

0%Responses - % Correct:

Blog (https://www.pastest.com/blog) About Pastest (https://www.pastest.com/about-us)Contact Us (https://www.pastest.com/contact-us) Help (https://www.pastest.com/help)

© Pastest 2016

Page 604: Back to Filters (/Secure/TestMe/Filter ... - 1 File Download

8/11/2016 MyPastest

https://mypastest.pastest.com/Secure/TestMe/Browser/429893#Top 1/2

Back to Filters (/Secure/TestMe/Filter/429893/QA)

Question 123 of 207

Therapeutic drug monitoring involving direct measurement of circulating drug concentrationsmay be of help in some clinical situations.

Which of the following drugs is most likely to benefit from therapeutic drug monitoring?

Explanation

The answer is Theophylline -

Therapeutic drug monitoring may be helpful where (1) there is a close relationshipbetween drug concentrations and drug effect or toxicity, and (2) there is a narrowtherapeutic index, ie small difference between drug concentrations required for efficacyversus those that might cause toxicity. Theophylline has a narrow therapeutic windowand needs close monitoring of its serum level to avoid toxicity; many drugs that interferewith P450 enzyme activity may influence theophylline concentrations.

Carbimazole (Option A) is incorrect. The dose of carbimazole required to treathyperthyroidism is best determined by correlation with clinical features and measuringthyroxine and thyroid-stimulating hormone levels.

Cyclophosphamide (Option B) is incorrect. Bone marrow and bladder toxicity related tocyclophosphamide may be monitored by regular full blood count measurement andurinalysis.

Rifampicin (Option C) is incorrect. Rifampicin has a wide therapeutic window, thereforemonitoring serum concentrations is not indicated.

CarbimazoleA

CyclophosphamideB

RifampicinC

TheophyllineD

WarfarinE

Page 605: Back to Filters (/Secure/TestMe/Filter ... - 1 File Download

8/11/2016 MyPastest

https://mypastest.pastest.com/Secure/TestMe/Browser/429893#Top 2/2

46607

Warfarin (Option E) is incorrect. Measurements of warfarin concentrations are of limitedclinical value. The INR (International Normalised Ratio) is used to monitor the effect ofwarfarin, which has greater practical significance in terms of efficacy and bleeding risk.

Next Question

Previous Question Tag Question Feedback End Review

Difficulty: Average

Peer Responses

Session Progress

0Responses Correct:

207Responses Incorrect:

207Responses Total:

0%Responses - % Correct:

Blog (https://www.pastest.com/blog) About Pastest (https://www.pastest.com/about-us)Contact Us (https://www.pastest.com/contact-us) Help (https://www.pastest.com/help)

© Pastest 2016

Page 606: Back to Filters (/Secure/TestMe/Filter ... - 1 File Download

8/11/2016 MyPastest

https://mypastest.pastest.com/Secure/TestMe/Browser/429893#Top 1/2

Back to Filters (/Secure/TestMe/Filter/429893/QA)

Question 124 of 207

46888

You are asked to review a 55-year-old woman who is currently taking aciclovir for an episodeof severe shingles. She is also taking digoxin for atrial fibrillation and diazepam for anxiety,fluoxetine for depression and temazepam at night.

Which of these drugs would be expected to have greatest water solubility?

Explanation

The answer is Digoxin –

Digoxin is highly soluble at physiological pH and is readily reconstituted for intravenousadministration. Drugs with high water solubility may be more readily absorbed in thefasted state, whereas drugs with high lipid solubility may be more readily absorbed in thepresence of dietary fat intake.

Aciclovir (Option A) is incorrect. Aciclovir is water soluble at 25°C, but has poor solubility atphysiological pH and body temperature.

Diazepam (Option B) is incorrect. Diazepam is highly lipid soluble, and readily crosses theblood–brain barrier. Intravenous formulations are prepared as a soya emulsion to overcomethe lack of water solubility.

Fluoxetine (Option D) is incorrect. Fluoxetine is lipid soluble, which may be beneficial forcrossing the blood–brain barrier.

Temazepam (Option E) is incorrect. Temazpam is highly lipid soluble, and readily crosses theblood brain–barrier.

AciclovirA

DiazepamB

DigoxinC

FluoxetineD

TemazepamE

Page 607: Back to Filters (/Secure/TestMe/Filter ... - 1 File Download

8/11/2016 MyPastest

https://mypastest.pastest.com/Secure/TestMe/Browser/429893#Top 2/2

Next Question

Previous Question Tag Question Feedback End Review

Difficulty: Average

Peer Responses

Session Progress

0Responses Correct:

207Responses Incorrect:

207Responses Total:

0%Responses - % Correct:

Blog (https://www.pastest.com/blog) About Pastest (https://www.pastest.com/about-us)Contact Us (https://www.pastest.com/contact-us) Help (https://www.pastest.com/help)

© Pastest 2016

Page 608: Back to Filters (/Secure/TestMe/Filter ... - 1 File Download

8/11/2016 MyPastest

https://mypastest.pastest.com/Secure/TestMe/Browser/429893#Top 1/2

Back to Filters (/Secure/TestMe/Filter/429893/QA)

Question 125 of 207

20944

You are at the scene of a cardiac arrest within the hospital, when one of the nursesaccidentally injects the distal part of her thumb with adrenaline from an auto injector. Thefinger has become white and intensely painful.

Which one of the following is the most appropriate way to manage the problem?

Explanation

Accidental administration of adrenaline

Where there is significant digital ischaemia, emergency medicine protocols (Velissariou

et al., 2004) suggest that local infiltration of phentolamine (an α-blocker) is the most

effective treatment

An alternative possibility is locally applied GTN paste

Oral or IV options are much less effective because when phentolamine is infiltrated

locally, much higher doses can be achieved

Reference

Velissariou, I., Cottrell, S., Berry, K., Wilson, B. 2004. Management of adrenaline (epinephrine)induced digital ischaemia in children after accidental injection from an EpiPen.

Emergency Medicine Journal 21, 387–388.

Administer 60 mg of nifedipine orallyA

Administer 50 mg of oral atenololB

Give sublingual GTNC

Give local phentolamineD

Administer IV labetalolE

Next Question

Page 609: Back to Filters (/Secure/TestMe/Filter ... - 1 File Download

8/11/2016 MyPastest

https://mypastest.pastest.com/Secure/TestMe/Browser/429893#Top 2/2

Previous Question Tag Question Feedback End Review

Difficulty: Average

Peer Responses

Session Progress

0Responses Correct:

207Responses Incorrect:

207Responses Total:

0%Responses - % Correct:

Blog (https://www.pastest.com/blog) About Pastest (https://www.pastest.com/about-us)Contact Us (https://www.pastest.com/contact-us) Help (https://www.pastest.com/help)

© Pastest 2016

Page 610: Back to Filters (/Secure/TestMe/Filter ... - 1 File Download

8/11/2016 MyPastest

https://mypastest.pastest.com/Secure/TestMe/Browser/429893#Top 1/2

Back to Filters (/Secure/TestMe/Filter/429893/QA)

Question 126 of 207

46959

You see a 67-year-old man in the medical outpatient department for investigation of low backpain. He has a past medical history of prostate cancer and is currently receiving buserelin.

Which of the following best explains the pharmacological mechanism of action of buserelin?

Explanation

The answer is Decreased androgen production –

Buserelin is a gonadotrophin releasing hormone agonist that exerts its actions at thelevel of the pituitary gland. Initially treatment causes increased gonadotrophin release;however, after a few weeks of continued therapy, gonadotrophin production is inhibited,and testosterone levels fall. The initial increase in testosterone levels may beaccompanied by a ‘flare’ in disease symptoms in some patients.

Blockade of androgen receptors (Option A) is incorrect. Flutamide, nilutamide, bicalutamideand enzalutamide are androgen receptor blockers.

Increased androgen metabolism (Option C) is incorrect. This mechanism is not relevant tobuserelin.

Inhibition of 5α-reductase activity (Option D) is incorrect. 5α-reductase inhibitors (e.g.finasteride, dutasteride and alfatradiol) prevent conversion of testosterone to its more potentmetabolite dihydrotestosterone (DHT).

Prevention of the progression of bone lesions (Option E) is incorrect. Bisphosphonates maybe helpful.

Blockade of androgen receptorsA

Decreased androgen productionB

Increased androgen metabolismC

Inhibition of 5α-reductase activityD

Prevention of the progression of bone lesionsE

Page 611: Back to Filters (/Secure/TestMe/Filter ... - 1 File Download

8/11/2016 MyPastest

https://mypastest.pastest.com/Secure/TestMe/Browser/429893#Top 2/2

Next Question

Previous Question Tag Question Feedback End Review

Difficulty: Average

Peer Responses

Session Progress

0Responses Correct:

207Responses Incorrect:

207Responses Total:

0%Responses - % Correct:

Blog (https://www.pastest.com/blog) About Pastest (https://www.pastest.com/about-us)Contact Us (https://www.pastest.com/contact-us) Help (https://www.pastest.com/help)

© Pastest 2016

Page 612: Back to Filters (/Secure/TestMe/Filter ... - 1 File Download

8/11/2016 MyPastest

https://mypastest.pastest.com/Secure/TestMe/Browser/429893#Top 1/2

Back to Filters (/Secure/TestMe/Filter/429893/QA)

Question 127 of 207

A 66-year-old man with a history of chronic obstructive airways disease asks you about thebenefits of acetylcysteine therapy.

Which of the following best describes the mode of action of N-acetylcysteine?

Explanation

The answer is Reduction of circulating oxidative stress –

N-acetylcysteine (NAC) exhibits direct and indirect antioxidant properties. NAC’s freethiol group is capable of directly interacting with electrophilic groups. This leads tointermediate formation of NAC thiol, with NAC disulfide as a major end-product. NACexerts an indirect antioxidant effect related to its role as a glutathione (GSH) precursor;GSH is a tripeptide made up of glutamic acid, cysteine and glycine, and is a major factorthat protects against intracellular oxidative stress. It is the sulfhydryl group ofacetylcysteine and GSH that confers the antioxidant effects. GSH synthesis takes placemainly in the liver (which acts as a reservoir) and the lungs.

Dopamine agonism (Option A) is incorrect. Acetylcysteine has no significant effect ondopaminergic pathways.

Histamine antagonism (Option B) is incorrect. Acetylcysteine provokes histamine release, amajor mechanism that underlies the occurrence of anaphylactoid reactions.

Phase I induction (Option C) is incorrect. Acetylcysteine has no significant effects on hepaticenzyme activity.

Dopamine agonismA

Histamine antagonismB

Phase I inductionC

Reduction of circulating oxidative stressD

Reduction of the formation of nitric oxideE

Page 613: Back to Filters (/Secure/TestMe/Filter ... - 1 File Download

8/11/2016 MyPastest

https://mypastest.pastest.com/Secure/TestMe/Browser/429893#Top 2/2

46944

Reduction of the formation of nitric oxide (Option E) is incorrect. Acetylcysteine promotesformation of nitric oxide, one of the factors that promotes flushing and vasodilatation.

Next Question

Previous Question Tag Question Feedback End Review

Difficulty: Average

Peer Responses

Session Progress

0Responses Correct:

207Responses Incorrect:

207Responses Total:

0%Responses - % Correct:

Blog (https://www.pastest.com/blog) About Pastest (https://www.pastest.com/about-us)Contact Us (https://www.pastest.com/contact-us) Help (https://www.pastest.com/help)

© Pastest 2016

Page 614: Back to Filters (/Secure/TestMe/Filter ... - 1 File Download

8/11/2016 MyPastest

https://mypastest.pastest.com/Secure/TestMe/Browser/429893#Top 1/2

Back to Filters (/Secure/TestMe/Filter/429893/QA)

Question 128 of 207

46624

You are asked to review a 46-year-old man who is being treated for suspected occupationalpoisoning by heavy metals, and is receiving treatment with d-penicillamine.

For which of the following heavy metals is d-penicillamine most likely to be effective?

Explanation

The answer is Copper –

d-Penicillamine may be used to reduce the body burden of copper after occupationalexposure or in patients with Wilson’s disease. It acts as a chelating agent, and is alsoeffective as a means of reducing total body lead burden.

Arsenic (Option A) is incorrect. The mainstay of treatment is supportive care, but in severecases then DMPS or DMSA may be considered as chelating agents to reduce arsenic toxicity.

Cadmium (Option B) is incorrect. Cadmium may be chelated by sodium calcium edetate insevere cases.

Mercury (Option D) is incorrect. Mercury metal is inert, but various mercurial salts areassociated with toxic effects. Historically, dimercaprol has been used to chelate mercury, butit is associated with adverse effects; DMPS or DMSA are preferred chelating agents in severemercury poisoning.

Thallium (Option E) is incorrect. Thallium metal is poorly absorbed and of limited toxicity;thallium salts are more readily absorbed and toxic. The most appropriate antidote in thalliumpoisoning is Prussian Blue.

ArsenicA

CadmiumB

CopperC

MercuryD

ThalliumE

Page 615: Back to Filters (/Secure/TestMe/Filter ... - 1 File Download

8/11/2016 MyPastest

https://mypastest.pastest.com/Secure/TestMe/Browser/429893#Top 2/2

Next Question

Previous Question Tag Question Feedback End Review

Difficulty: Average

Peer Responses

Session Progress

0Responses Correct:

207Responses Incorrect:

207Responses Total:

0%Responses - % Correct:

Blog (https://www.pastest.com/blog) About Pastest (https://www.pastest.com/about-us)Contact Us (https://www.pastest.com/contact-us) Help (https://www.pastest.com/help)

© Pastest 2016

Page 616: Back to Filters (/Secure/TestMe/Filter ... - 1 File Download

8/11/2016 MyPastest

https://mypastest.pastest.com/Secure/TestMe/Browser/429893#Top 1/2

Back to Filters (/Secure/TestMe/Filter/429893/QA)

Question 129 of 207

46773

A 39-year-old woman, on oral medication for severe rheumatoid arthritis, attends the clinicwith a history of nausea, lethargy and drowsiness. Investigations reveal a pancytopaenia, urea30 mmol/l and creatinine 600 μmol/l.

Which one of the following drugs is most likely to cause these adverse effects?

Explanation

The answer is d-penicillamine –

d-Penicillamine is associated with development of pancytopaenia and acutetubulointerstitial nephritis.

Gold (Option B) is incorrect. Gold may present with marrow suppression and nephropathy aswell; however, gold is administered by deep intramuscular injection.

Hydroxychloroquine (Option C) is incorrect. Hydroxychloroquine is usually prescribed formild to moderate rheumatoid arthritis and not in severe cases, as it is a relatively weakantirheumatic drug; its main side-effects include corneal deposits, decrease in peripheralvision and retinopathy.

Infliximab (Option D) is incorrect. Although major side-effects with infliximab are rare,suppression of the immune response and reactivation of latent tuberculosis or thedevelopment of immunosuppression-related malignancy may occur.

Methotrexate (Option E) is incorrect. Methotrexate may rarely cause myelosuppression;hepatotoxicity and hepatic fibrosis are much more common adverse reactions.

d-PenicillamineA

GoldB

HydroxychloroquineC

InfliximabD

MethotrexateE

Page 617: Back to Filters (/Secure/TestMe/Filter ... - 1 File Download

8/11/2016 MyPastest

https://mypastest.pastest.com/Secure/TestMe/Browser/429893#Top 2/2

Next Question

Previous Question Tag Question Feedback End Review

Difficulty: Average

Peer Responses

Session Progress

0Responses Correct:

207Responses Incorrect:

207Responses Total:

0%Responses - % Correct:

Blog (https://www.pastest.com/blog) About Pastest (https://www.pastest.com/about-us)Contact Us (https://www.pastest.com/contact-us) Help (https://www.pastest.com/help)

© Pastest 2016

Page 618: Back to Filters (/Secure/TestMe/Filter ... - 1 File Download

8/11/2016 MyPastest

https://mypastest.pastest.com/Secure/TestMe/Browser/429893#Top 1/2

Back to Filters (/Secure/TestMe/Filter/429893/QA)

Question 130 of 207

You review a 36-year-old man with a family history of premature cardiovascular disease.Fasting cholesterol is 8.2 mmol/l and high-density lipoprotein (HDL) is 1.4 mmol/l. You electto commence him on atorvastatin 20 mg daily.

Which of the following is best recognised as a mechanism of action of statins?

Explanation

The answer is They lower very-low-density lipoprotein (VLDL) synthesis –

Statins inhibit 3-hydroxy-3-methylglutaryl coenzyme A (HMG-CoA) reductase, whichcatalyses the rate-limiting step in cholesterol synthesis. Statins lead to increased hepaticlow-density lipoprotein (LDL) receptors and reduced hepatic very-low-densitylipoprotein (VLDL) synthesis, coupled with increased VLDL clearance. Statins may causemyositis and abnormal liver function, although the risk of these adverse effects isnormally outweighed by the potential benefits of reduction of cardiovascular disease inhigh-risk patients. Incidence of myositis is increased with concomitant use of fibric acidderivatives, and statins are known to interact with both ciclosporin and nicotinic acid.Macrolide antibiotics and digoxin also interact with some statins.

They increase lipoprotein lipase (Option A) is incorrect. Some statins are recognised toincrease lipoprotein lipase activity, thereby reducing triglyceride concentrations; the effect ismodest, and data concerning atorvastatin are controversial.

They stimulate fatty acid synthetase (Option C) is incorrect. Statins have a modest effect toreduce the synthesis of triglycerides.

They increase lipoprotein lipaseA

They lower very-low-density lipoprotein (VLDL) synthesisB

They stimulate fatty acid synthetaseC

They stimulate 3-hydroxy-3-methylglutaryl coenzyme A (HMG-CoA) reductaseD

They stimulate sodium–potassium ATPase activityE

Page 619: Back to Filters (/Secure/TestMe/Filter ... - 1 File Download

8/11/2016 MyPastest

https://mypastest.pastest.com/Secure/TestMe/Browser/429893#Top 2/2

46948

They stimulate 3-hydroxy-3-methylglutaryl coenzyme A (HMG-CoA) reductase (Option D) isincorrect. Statins inhibit the activity of this enzyme.

They stimulate sodium–potassium ATPase activity (Option E) is incorrect. Statins have nosignificant effect on sodium–potassium ATPase activity.

Next Question

Previous Question Tag Question Feedback End Review

Difficulty: Average

Peer Responses

Session Progress

0Responses Correct:

207Responses Incorrect:

207Responses Total:

0%Responses - % Correct:

Blog (https://www.pastest.com/blog) About Pastest (https://www.pastest.com/about-us)Contact Us (https://www.pastest.com/contact-us) Help (https://www.pastest.com/help)

© Pastest 2016

Page 620: Back to Filters (/Secure/TestMe/Filter ... - 1 File Download

8/11/2016 MyPastest

https://mypastest.pastest.com/Secure/TestMe/Browser/429893#Top 1/2

Back to Filters (/Secure/TestMe/Filter/429893/QA)

Question 131 of 207

A 56-year-old recently bereaved man is admitted with bradycardia and profoundhypotension. He has a history of treatment for hypertension, and you are concerned that hiscardiovascular findings are caused by atenolol toxicity.

Which of the following pharmaceutical agents is most suitable in initial treatment ofsuspected β-blocker overdose?

Explanation

The answer is Glucagon –

Overdose The initial treatment of choice for a β-blocker overdose is atropine. However,the preferred treatment for patients with profound bradycardia and hypotension is anintravenous bolus of a glucagon followed by an intravenous glucagon infusion. Careshould be taken to protect the airway because vomiting is a characteristic adverse effectof glucagon administration.

Adrenaline (Option A) is incorrect. Adrenaline and isoprenaline can be given as an alternativeif glucagon is unavailable. In severe β-blocker poisoning there is irreversible binding of β-blocker to the β-adrenoceptor, so that isoprenaline is of limited efficacy. In cases of severebradycardia unresponsive to medical therapy, cardiac pacing may be necessary, but this mayfail to ‘capture’ and cardiopulmonary bypass may be required until the β-blocker is clearedfrom the body.

Insulin (Option C) is incorrect. Insulin therapy plays an important role as an antidote incalcium-channel blocker poisoining. Insulin administration may be indicated in β-blockerpoisoning that fails to respond to inotropes and glucagon administration.

AdrenalineA

GlucagonB

InsulinC

NeostigmineD

PyridostigmineE

Page 621: Back to Filters (/Secure/TestMe/Filter ... - 1 File Download

8/11/2016 MyPastest

https://mypastest.pastest.com/Secure/TestMe/Browser/429893#Top 2/2

46830

Neostigmine (Option D) is incorrect. Neostigmine and pyridostigmine are used for thereversal of muscle relaxants.

Pyridostigmine (Option E) is incorrect. Neostigmine and pyridostigmine are used for thereversal of muscle relaxants.

Next Question

Previous Question Tag Question Feedback End Review

Difficulty: Average

Peer Responses

Session Progress

0Responses Correct:

207Responses Incorrect:

207Responses Total:

0%Responses - % Correct:

Blog (https://www.pastest.com/blog) About Pastest (https://www.pastest.com/about-us)Contact Us (https://www.pastest.com/contact-us) Help (https://www.pastest.com/help)

© Pastest 2016

Page 622: Back to Filters (/Secure/TestMe/Filter ... - 1 File Download

8/11/2016 MyPastest

https://mypastest.pastest.com/Secure/TestMe/Browser/429893#Top 1/2

Back to Filters (/Secure/TestMe/Filter/429893/QA)

Question 132 of 207

46990

A 20-year-old woman who suffers from intermittent torticollis has severe vomiting anddehydration after a food-poisoning incident. You decide to give her an antiemetic as she isunable to tolerate oral intake.

Which medication would be most likely to cause adverse effects and should be avoided?

Explanation

The answer is Prochlorperazine –

Torticollis or acute dystonia may occur in between 0.5% and 1% of patients given eitherprochlorperazine or metoclopramide, and the rate of occurrence is higher in youngwomen. Benzatropine and procyclidine may be used to manage acute drug-induceddystonia.

Cetirizine (Option A) is incorrect. Cetirizine may worsen dry mouth sensation, but this posesless of a hazard than prochlorperazine.

Domperidone (Option B) is incorrect. Domperidone would be effective in this situation.

Ondansetron (Option C) is incorrect. Ondansetron would be effective in this situation.

Promethazine (Option E) is incorrect. Promethazine may cause drowsiness.

CetirizineA

DomperidoneB

OndansetronC

ProchlorperazineD

PromethazineE

Next Question

Page 623: Back to Filters (/Secure/TestMe/Filter ... - 1 File Download

8/11/2016 MyPastest

https://mypastest.pastest.com/Secure/TestMe/Browser/429893#Top 2/2

Previous Question Tag Question Feedback End Review

Difficulty: Average

Peer Responses

Session Progress

0Responses Correct:

207Responses Incorrect:

207Responses Total:

0%Responses - % Correct:

Blog (https://www.pastest.com/blog) About Pastest (https://www.pastest.com/about-us)Contact Us (https://www.pastest.com/contact-us) Help (https://www.pastest.com/help)

© Pastest 2016

Page 624: Back to Filters (/Secure/TestMe/Filter ... - 1 File Download

8/11/2016 MyPastest

https://mypastest.pastest.com/Secure/TestMe/Browser/429893#Top 1/2

Back to Filters (/Secure/TestMe/Filter/429893/QA)

Question 133 of 207

46779

A 22-year-old man was admitted to the Acute Medical Unit after partial seizures, and hasbeen discharged home with a supply of new medications. You are reviewing him 3 weekslater in the outpatient department and he complains of double vision and unsteady gait,which have progressively worsened over the past 2 weeks.

Which one of the following drugs is most likely to have caused these symptoms?

Explanation

The answer is Carbamazepine –

The most common dose-related adverse effects of carbamazepine are related tocerebellar toxicity, including diplopia and ataxia. The effects are dose-dependent, andare fully reversible on stopping therapy. Phenytoin may also cause dose-dependentcerebellar toxicity.

Gabapentin (Option B) is incorrect. Gabapentin causes drowsiness, headache and tremor.

Tiagabine (Option C) is incorrect. Tiagabine may cause dizziness, confusion, somnolence andataxia.

Topiramate (Option D) is incorrect. Topirimate typically causes drowsiness, altered mood,acute myopia, glaucoma and urolithiasis.

Vigabatrin (Option E) is incorrect. Vigabatrin can cause drowsiness, dizziness and weightgain. Long-term therapy has been associated with visual field defects in up to one-third ofpatients, which may be irreversible.

CarbamazepineA

GabapentinB

TiagabineC

TopiramateD

VigabatrinE

Page 625: Back to Filters (/Secure/TestMe/Filter ... - 1 File Download

8/11/2016 MyPastest

https://mypastest.pastest.com/Secure/TestMe/Browser/429893#Top 2/2

Next Question

Previous Question Tag Question Feedback End Review

Difficulty: Average

Peer Responses

Session Progress

0Responses Correct:

207Responses Incorrect:

207Responses Total:

0%Responses - % Correct:

Blog (https://www.pastest.com/blog) About Pastest (https://www.pastest.com/about-us)Contact Us (https://www.pastest.com/contact-us) Help (https://www.pastest.com/help)

© Pastest 2016

Page 626: Back to Filters (/Secure/TestMe/Filter ... - 1 File Download

8/11/2016 MyPastest

https://mypastest.pastest.com/Secure/TestMe/Browser/429893#Top 1/2

Back to Filters (/Secure/TestMe/Filter/429893/QA)

Question 134 of 207

A patient on regular treatment for anxiety symptoms stops his medication abruptly on theadvice of one of his friends. Several days later, he develops acute anxiety, insomnia, irritability,rage, feelings of unreality and depersonalisation, diplopia, paraesthesias, palpitations, flushingand hyperventilation.

Cessation of which drug is most likely to have caused these withdrawal symptoms?

Explanation

The answer is Lorazepam –

Benzodiazepines (lorazepam, diazepam, chlordiazepoxide) withdrawal symptoms may beeven worse than the condition for which the drug was originally prescribed. Symptomsappear sooner with the rapidly eliminated benzodiazepines such as lorazepam.

Amitriptyline (Option A) is incorrect. Withdrawal symptoms due to tricyclic antidepressants(eg amitriptyline) are rare and include cholinergic effects such as abdominal cramps,diarrhoea, vomiting and dehydration, extrapyramidal symptoms, anxiety, psychosis, deliriumand mania.

Buspirone (Option B) is incorrect. Buspirone is claimed not to have withdrawal effects; nordoes it have sedative or muscle relaxant properties. It is an azapirone, and is thereforedistinct from other anxiolytics such as benzodiazepines.

Paroxetine (Option D) is incorrect. Withdrawal of paroxetine or other SSRI antidepressantscan lead to deterioration in mood and cognition and orofacial dystonias.

AmitriptylineA

BuspironeB

LorazepamC

ParoxetineD

PhenelzineE

Page 627: Back to Filters (/Secure/TestMe/Filter ... - 1 File Download

8/11/2016 MyPastest

https://mypastest.pastest.com/Secure/TestMe/Browser/429893#Top 2/2

46740

Phenelzine (Option E) is incorrect. Abrupt withdrawal of phenelzine leads to panic, shaking,sweats and nausea.

Next Question

Previous Question Tag Question Feedback End Review

Difficulty: Average

Peer Responses

Session Progress

0Responses Correct:

207Responses Incorrect:

207Responses Total:

0%Responses - % Correct:

Blog (https://www.pastest.com/blog) About Pastest (https://www.pastest.com/about-us)Contact Us (https://www.pastest.com/contact-us) Help (https://www.pastest.com/help)

© Pastest 2016

Page 628: Back to Filters (/Secure/TestMe/Filter ... - 1 File Download

8/11/2016 MyPastest

https://mypastest.pastest.com/Secure/TestMe/Browser/429893#Top 1/2

Back to Filters (/Secure/TestMe/Filter/429893/QA)

Question 135 of 207

22481

You are drawing up guidelines for the use of insulin glargine in basal bolus regimes ahead ofisophane insulin.

When considering your guidelines, in which of the following situations does insulin glarginehave the clearest advantage over isophane?

Explanation

Insulin

The greatest benefit with respect to reduction in the absolute number of nocturnal

hypoglycaemic events for glargine versus NPH is in Type 1 diabetes patients

Day time events are not significantly impacted

HbA1c results are similar between treated groups

In paediatric patients no significant improvement/impact of glargine on hypos has been

seen in patients with Type 1 diabetes

NICE only recommends use of insulin glargine in patients with type-2 diabetes who

have significant hypoglycaemia on isophane insulin

In patients with type-1 diabetes who have significant daytime hypoglycaemia onisophane

A

In patients with type-2 diabetes who are taking more than 15 U of isophane per dayB

In patients with type-2 diabetes who have episodes of daytime hypoglycaemia onisophane

C

In patients with type-1 diabetes who have significant nocturnal hypoglycaemia onisophane

D

In patients with type-2 diabetes who have weight gain on isophaneE

Next Question

Page 629: Back to Filters (/Secure/TestMe/Filter ... - 1 File Download

8/11/2016 MyPastest

https://mypastest.pastest.com/Secure/TestMe/Browser/429893#Top 2/2

Previous Question Tag Question Feedback End Review

Difficulty: Average

Peer Responses

Session Progress

0Responses Correct:

207Responses Incorrect:

207Responses Total:

0%Responses - % Correct:

Blog (https://www.pastest.com/blog) About Pastest (https://www.pastest.com/about-us)Contact Us (https://www.pastest.com/contact-us) Help (https://www.pastest.com/help)

© Pastest 2016

Page 630: Back to Filters (/Secure/TestMe/Filter ... - 1 File Download

8/11/2016 MyPastest

https://mypastest.pastest.com/Secure/TestMe/Browser/429893#Top 1/2

Back to Filters (/Secure/TestMe/Filter/429893/QA)

Question 136 of 207

46950

A 72-year-old man is reviewed in the Emergency Department. He has been feeling tired andunwell for several weeks. There is a past history of glaucoma, chronic obstructive pulmonarydisease, congestive heart failure and type-2 diabetes. His GP has recently been investigatingfor anaemia. Blood tests show a metabolic acidosis with normal anion gap.

Which of the following drugs is most likely to be responsible for the acid–base disturbance?

Explanation

The answer is Acetazolamide –

Normal anion gap acidosis may be caused by gastrointestinal (GI) bicarbonate loss, egdiarrhoea, pancreatic fistula, renal bicarbonate loss, renal failure and hypoaldosteronism.Acetazolamide is a carbonic anhydrase inhibitor that causes renal sodium andbicarbonate loss.

Aspirin (Option B) is incorrect. Aspirin provides a source of unmeasured anion so thatmetabolic acidosis is associated with a raised anion gap.

Iron sulfate (Option C) is incorrect. Iron overdose may be associated with the development oflactic acidosis.

Metformin (Option D) is incorrect. Metformin is a lactate dehydrogenase inhibitor that causesmetabolic acidosis due to accumulation of lactate; lactic acidosis causes a high anion gap.

Ramipril (Option E) is incorrect. Ramipril may uncommonly cause type-4 renal tubularacidosis.

AcetazolamideA

AspirinB

Iron sulfateC

MetforminD

RamiprilE

Page 631: Back to Filters (/Secure/TestMe/Filter ... - 1 File Download

8/11/2016 MyPastest

https://mypastest.pastest.com/Secure/TestMe/Browser/429893#Top 2/2

Next Question

Previous Question Tag Question Feedback End Review

Difficulty: Average

Peer Responses

Session Progress

0Responses Correct:

207Responses Incorrect:

207Responses Total:

0%Responses - % Correct:

Blog (https://www.pastest.com/blog) About Pastest (https://www.pastest.com/about-us)Contact Us (https://www.pastest.com/contact-us) Help (https://www.pastest.com/help)

© Pastest 2016

Page 632: Back to Filters (/Secure/TestMe/Filter ... - 1 File Download

8/11/2016 MyPastest

https://mypastest.pastest.com/Secure/TestMe/Browser/429893#Top 1/2

Back to Filters (/Secure/TestMe/Filter/429893/QA)

Question 137 of 207

A 27-year-old farmer presents with fever, cough and dyspnoea. On auscultation, expiratorycrackles are heard at both apices. A chest radiograph shows a round lesion in the right apexwith an air halo above it. A full blood count reveals a high eosinophil count.

Which of the following drugs would be most effective in this case?

Explanation

The answer is Amphotericin –

This patient, a farmer presenting with fever, cough and dyspnoea, most probably has anaspergilloma. If asymptomatic, patients may simply require follow-up, and 10% resolvespontaneously. Where patients are symptomatic, particularly if there is haemoptysis, thenaspergilloma is often managed surgically by lobectomy. Alternative treatments includebronchial arterial embolisation and local instillation of amphotericin. Pre- and post-operative systemic antifungal treatment is routinely used to prevent recurrence.Treatment options include amphotericin with flucytosine, itraconazole and voraconazole,but the benefit of long-term antifungal treatment is uncertain.

Fluconazole (Option B) is incorrect. Fluconazole is useful in candidiasis and central nervoussystem infections with Cryptococcus neoformans, and is usually commenced after initialtreatment with amphotericin and flucytosine.

Itraconazole (Option C) is incorrect. Itraconazole is the agent of choice for non-life-threatening blastomycosis and histoplasmosis. It is moderately effective against invasiveaspergillosis but there is less evidence to support its use than amphotericin, which is why it isa less good answer.

AmphotericinA

FluconazoleB

ItraconazoleC

NystatinD

TerbinafineE

Page 633: Back to Filters (/Secure/TestMe/Filter ... - 1 File Download

8/11/2016 MyPastest

https://mypastest.pastest.com/Secure/TestMe/Browser/429893#Top 2/2

46862

Nystatin (Option D) is incorrect. Local instillation of nystatin may be effective, but is oftengiven with amphotericin. Nystatin is poorly absorbed through mucous membranes and isthus useful in oral, vaginal and enteric candidiasis.

Terbinafine (Option E) is incorrect. Terbinafine is used to treat superficial mycoses such asdermatophyte infections.

Next Question

Previous Question Tag Question Feedback End Review

Difficulty: Average

Peer Responses

Session Progress

0Responses Correct:

207Responses Incorrect:

207Responses Total:

0%Responses - % Correct:

Blog (https://www.pastest.com/blog) About Pastest (https://www.pastest.com/about-us)Contact Us (https://www.pastest.com/contact-us) Help (https://www.pastest.com/help)

© Pastest 2016

Page 634: Back to Filters (/Secure/TestMe/Filter ... - 1 File Download

8/11/2016 MyPastest

https://mypastest.pastest.com/Secure/TestMe/Browser/429893#Top 1/2

Back to Filters (/Secure/TestMe/Filter/429893/QA)

Question 138 of 207

46953

You review a 42-year-old woman with type-1 diabetes who has undergone a renal transplant.She is taking azathioprine and tacrolimus for long-term immunosuppression.

Which of the following most accurately characterises the pharmacological effects of post-transplant immunosuppressant agents?

Explanation

The answer is Tacrolimus is a calcineurin inhibitor –

This is the primary mechanism by which tacrolimus prevents cytokine formation within T-lymphocytes, including reduced expression of interleukin-2.

Azathioprine has a half-life of 2.5 days (Option A) is incorrect. Azathioprine has a half-life ofaround 5 h, but the duration of effect may be significantly longer.

Azathioprine inhibits pyrimidine synthesis (Option B) is incorrect. Azathioprine inhibitspurine synthesis, an essential step in the proliferation of leukocytes.

Sirolimus is a calcineurin inhibitor (Option C) is incorrect. Sirolimus is not a calcineurininhibitor, despite having a similar name to tacrolimus. Sirolimus inhibits the cellular responseto interleukin-2.

Tacrolimus has a half-life of around 1 h (Option D) is incorrect. Tacrolimus has a half-life ofaround 11 h (similar to sirolimus).

Azathioprine has a half-life of 2.5 daysA

Azathioprine inhibits pyrimidine synthesisB

Sirolimus is a calcineurin inhibitorC

Tacrolimus has a half-life of around 1 hD

Tacrolimus is a calcineurin inhibitorE

Next Question

Page 635: Back to Filters (/Secure/TestMe/Filter ... - 1 File Download

8/11/2016 MyPastest

https://mypastest.pastest.com/Secure/TestMe/Browser/429893#Top 2/2

Previous Question Tag Question Feedback End Review

Difficulty: Average

Peer Responses

Session Progress

0Responses Correct:

207Responses Incorrect:

207Responses Total:

0%Responses - % Correct:

Blog (https://www.pastest.com/blog) About Pastest (https://www.pastest.com/about-us)Contact Us (https://www.pastest.com/contact-us) Help (https://www.pastest.com/help)

© Pastest 2016

Page 636: Back to Filters (/Secure/TestMe/Filter ... - 1 File Download

8/11/2016 MyPastest

https://mypastest.pastest.com/Secure/TestMe/Browser/429893#Top 1/2

Back to Filters (/Secure/TestMe/Filter/429893/QA)

Question 139 of 207

A 72-year-old woman presents for review in the fracture clinic. There is evidence ofosteoporosis and you decide to start bisphosphonate therapy. Nausea is listed as a verycommon side-effect of the drug, and the patient asks for clarification of this.

What is the expected rate of occurrence for side-effects listed as ‘very common’?

Explanation

The answer is More than 10% of patients –

Oesophagitis is also recognised after regular bisphosphonate use, but much lesscommon than simple nausea.

Very common side-effects are said to occur with a frequency of greater than 1 in 10patients (10%).

Very common side-effects More than 1 in 10 (>10%)

Common side-effects 1 in 100 to 1 in 10 (1 - 10%)

Uncommon side-effects 1 in 1000 to 1 in 100 (0.1 - 1%)

Rare side-effects 1 in 10 000 to 1 in 1000 (0.01 - 0.1%)

Very rare side-effects Less than 1 in 10 000 (<0.01%)

1–10% of patientsA

25–50% of patientsB

50–75% of patientsC

More than 10% of patientsD

More than 75% of patientsE

Page 637: Back to Filters (/Secure/TestMe/Filter ... - 1 File Download

8/11/2016 MyPastest

https://mypastest.pastest.com/Secure/TestMe/Browser/429893#Top 2/2

46885

1–10% of patients (Option A) is incorrect. 1–10% would be ‘common’ rather than ‘verycommon’.

25–50% of patients (Option B) is incorrect. This would be extremely common. Very commonis defined as anything that occurs with a frequency greater than 10%.

50–75% of patients (Option C) is incorrect. This would be extremely common. Very commonis defined as anything that occurs with a frequency greater than 10%.

More than 75% of patients (Option E) is incorrect. This would be extremely common! Verycommon is defined as anything that occurs with a frequency greater than 10%.

Next Question

Previous Question Tag Question Feedback End Review

Difficulty: Average

Peer Responses

Session Progress

0Responses Correct:

207Responses Incorrect:

207Responses Total:

0%Responses - % Correct:

Blog (https://www.pastest.com/blog) About Pastest (https://www.pastest.com/about-us)Contact Us (https://www.pastest.com/contact-us) Help (https://www.pastest.com/help)

© Pastest 2016

Page 638: Back to Filters (/Secure/TestMe/Filter ... - 1 File Download

8/11/2016 MyPastest

https://mypastest.pastest.com/Secure/TestMe/Browser/429893#Top 1/2

Back to Filters (/Secure/TestMe/Filter/429893/QA)

Question 140 of 207

You are called to see a 16-year-old girl who told her parents that she partied all night andtook five Ecstasy tablets. The most recent tablets were ingested 6 h ago. On examination sheis restless, dehydrated, her blood pressure is 100/60 mmHg and her pulse is 100/min.

What is the next step in her management?

Explanation

The answer is Intravenous NaCl infusion –

Ecstasy (3,4-methylenedioxymethamfetamine, MDMA) stimulates serotonergic activitywithin the central nervous system. Gastric lavage may be considered if a substantialoverdose has been ingested in the preceding 1 h. Treatment involves supportive care, IVfluids to maintain hydration status, correction of biochemical disturbance, diazepam toreduce agitation, and active cooling if needed. Nitrates and calcium antagonists may beneeded to control blood pressure.

Gastric lavage (Option A) is incorrect. Gastric lavage is unlikely to be of benefit at 6 h post-ingestion.

Intubation (Option C) is incorrect. Intubation is not indicated at present, but this may beindicated if hyperthermia fails to settle after adequate sedation.

Provocation of vomiting (Option D) is incorrect. Induced vomiting is rarely, if ever, effective inreducing drug toxicity, and may be complicated by oesophageal rupture and gastrointestinalbleeding.

Gastric lavageA

Intravenous NaCl infusionB

IntubationC

Provocation of vomitingD

SedationE

Page 639: Back to Filters (/Secure/TestMe/Filter ... - 1 File Download

8/11/2016 MyPastest

https://mypastest.pastest.com/Secure/TestMe/Browser/429893#Top 2/2

46817

Sedation (Option E) is incorrect. Sedation is an important aspect of treatment but wouldnormally follow measures to address fluid and electrolyte balance.

Next Question

Previous Question Tag Question Feedback End Review

Difficulty: Average

Peer Responses

Session Progress

0Responses Correct:

207Responses Incorrect:

207Responses Total:

0%Responses - % Correct:

Blog (https://www.pastest.com/blog) About Pastest (https://www.pastest.com/about-us)Contact Us (https://www.pastest.com/contact-us) Help (https://www.pastest.com/help)

© Pastest 2016

Page 640: Back to Filters (/Secure/TestMe/Filter ... - 1 File Download

8/11/2016 MyPastest

https://mypastest.pastest.com/Secure/TestMe/Browser/429893#Top 1/2

Back to Filters (/Secure/TestMe/Filter/429893/QA)

Question 141 of 207

A 19-year-old man attends the early medical review clinic 2 days after presenting to theEmergency Department because of witnessed tonic–clonic seizures. A CT head scan isnormal, and neurological examination findings are normal. He suffered a similar episode 18months ago, and you decide to commence antiepileptic medications.

Which of the following medications would be the most appropriate first-line therapy for thispatient?

Explanation

The answer is Sodium valproate –

The most common drugs used to control generalised seizures in young adults are sodiumvalproate and carbamazepine. Carbamazepine is a powerful enzyme inducer that mayinteract with other medications; toxicity is associated with ataxia. Sodium valproate isrecognised to cause weight gain and associated with a polycystic ovary (PCOS)-likesyndrome in young women.

Lamotrigine (Option A) is incorrect. Lamotrigine is commonly used as an adjunctive drug, oras an alternative to valproate; it has a relatively benign side-effect profile versus the otheroptions.

Phenobarbital (Option B) is incorrect. Phenobarbital can cause folate deficiency,osteomalacia, neuropathy and agitation in children.

Phenytoin (Option C) is incorrect. Phenytoin may also be effective but is known to causeserious adverse effects, including gingival hypertrophy, hirsutism, cerebellar signs, ricketsand lymphadenopathy.

LamotrigineA

PhenobarbitalB

PhenytoinC

Sodium valproateD

TopiramateE

Page 641: Back to Filters (/Secure/TestMe/Filter ... - 1 File Download

8/11/2016 MyPastest

https://mypastest.pastest.com/Secure/TestMe/Browser/429893#Top 2/2

46613

Topiramate (Option E) is incorrect. Topiramate may be used for intractable seizures or addedwhen seizures are difficult to control with other drugs.

Next Question

Previous Question Tag Question Feedback End Review

Difficulty: Average

Peer Responses

Session Progress

0Responses Correct:

207Responses Incorrect:

207Responses Total:

0%Responses - % Correct:

Blog (https://www.pastest.com/blog) About Pastest (https://www.pastest.com/about-us)Contact Us (https://www.pastest.com/contact-us) Help (https://www.pastest.com/help)

© Pastest 2016

Page 642: Back to Filters (/Secure/TestMe/Filter ... - 1 File Download

8/11/2016 MyPastest

https://mypastest.pastest.com/Secure/TestMe/Browser/429893#Top 1/2

Back to Filters (/Secure/TestMe/Filter/429893/QA)

Question 142 of 207

46859

A 25-year-old woman is prescribed an antimicrobial agent to treat a tooth infection. Shepresents to the Acute Medical Unit with severe headache, blurred vision and a convergentsquint. On examination, there is diplopia, maximal on looking to the left side. The patient isunable to abduct her left eye beyond the midline.

Which of the following drugs is most likely to be responsible for her present condition?

Explanation

The answer is Tetracycline –

This patient has clinical features indicating a left VIth cranial nerve palsy, a finding inintracranial hypertension. A number of drugs are associated with intracranialhypertension, including tetracyclines, amiodarone, oral contraceptive pills and withdrawalof corticosteroid therapy. The underlying mechanism is unclear.

Amoxicillin (Option A) is incorrect. Amoxicillin is not known to cause raised intracranialpressure.

Ceftriaxone (Option B) is incorrect. Ceftriaxone is not known to cause raised intracranialpressure.

Erythromycin (Option C) is incorrect. Erythromycin is less likely to cause raised intracranialpressure than tetracyclines.

Gentamicin (Option D) is incorrect. Gentamicin is not known to cause raised intracranialpressure.

AmoxicillinA

CeftriaxoneB

ErythromycinC

GentamicinD

TetracyclineE

Page 643: Back to Filters (/Secure/TestMe/Filter ... - 1 File Download

8/11/2016 MyPastest

https://mypastest.pastest.com/Secure/TestMe/Browser/429893#Top 2/2

Next Question

Previous Question Tag Question Feedback End Review

Difficulty: Average

Peer Responses

Session Progress

0Responses Correct:

207Responses Incorrect:

207Responses Total:

0%Responses - % Correct:

Blog (https://www.pastest.com/blog) About Pastest (https://www.pastest.com/about-us)Contact Us (https://www.pastest.com/contact-us) Help (https://www.pastest.com/help)

© Pastest 2016

Page 644: Back to Filters (/Secure/TestMe/Filter ... - 1 File Download

8/11/2016 MyPastest

https://mypastest.pastest.com/Secure/TestMe/Browser/429893#Top 1/2

Back to Filters (/Secure/TestMe/Filter/429893/QA)

Question 143 of 207

46868

A patient in status asthmaticus is not responding to nebulised salbutamol and terbutaline.The consultant respiratory physician has advised an intravenous infusion of aminophylline.

Which of the following enzymes is this drug most likely to inhibit in order to relieve thesymptoms?

Explanation

The answer is Phosphodiesterase –

Phosphodiesterase catalyses the conversion of cAMP to 5'-adenosine monophosphate(5'-AMP). This reaction is blocked by aminophylline or theophylline, which causes theaccumulation of cAMP intracellularly and this results in bronchodilatation, cardiacstimulation and vasodilatation.

Adenyl cyclase (Option A) is incorrect. Adenyl cyclase catalyses the conversion of ATP tocyclic adenosine monophosphate (cAMP).

Alcohol dehydrogenase (Option B) is incorrect. Alcohol dehydrogenase is a key enzymeinvolved in metabolism of ethanol, methanol and ethylene glycol; fomepizole is a competitiveinhibitor of alcohol dehydrogenase.

Guanyl cyclase (Option C) is incorrect. Guanyl cyclase catalyses the formation of guanosinemonophosphate, an important intracellular messenger.

Monoamine oxidase (Option D) is incorrect. Monamine oxidase inhibition is reserved fordepression that fails to respond to other treatments. Peripheral effects are mediated bypotentiation of catecholamines and include tachycardia and hypertension.

Adenyl cyclaseA

Alcohol dehydrogenaseB

Guanyl cyclaseC

Monoamine oxidaseD

PhosphodiesteraseE

Page 645: Back to Filters (/Secure/TestMe/Filter ... - 1 File Download

8/11/2016 MyPastest

https://mypastest.pastest.com/Secure/TestMe/Browser/429893#Top 2/2

46868

Next Question

Previous Question Tag Question Feedback End Review

Difficulty: Average

Peer Responses

Session Progress

0Responses Correct:

207Responses Incorrect:

207Responses Total:

0%Responses - % Correct:

Blog (https://www.pastest.com/blog) About Pastest (https://www.pastest.com/about-us)Contact Us (https://www.pastest.com/contact-us) Help (https://www.pastest.com/help)

© Pastest 2016

Page 646: Back to Filters (/Secure/TestMe/Filter ... - 1 File Download

8/11/2016 MyPastest

https://mypastest.pastest.com/Secure/TestMe/Browser/429893#Top 1/2

Back to Filters (/Secure/TestMe/Filter/429893/QA)

Question 144 of 207

46935

A 34-year-old patient has been treated for rosacea for the last 3 months. He develops blue-grey skin pigmentation.

Which of the following drugs is most likely to be responsible for this?

Explanation

The answer is Minocycline –

Cutaneous pigmentation due to minocycline causes discoloration of the skin, which canbe slate grey in appearance and is reversible after drug cessation. Other adverse effectsinclude anorexia, dizziness, tinnitus and vertigo (more common in women), acute renalfailure, discoloration of conjunctiva, tears and sweat, as well as systemic lupuserythematosus.

Amiodarone (Option A) is incorrect. Amiodarone may cause slate grey discoloration of theskin, but this would fit less well with the clinical scenario, hence option E is the preferredanswer.

Ciprofloxacin (Option B) is incorrect. Ciprofloxacin does not cause altered skin pigmentationor discoloration.

Doxycycline (Option C) is incorrect. Doxycycline does not cause altered skin pigmentation ordiscoloration.

Erythromycin (Option D) is incorrect. Erythromycin does not cause altered skin pigmentationor discoloration.

AmiodaroneA

CiprofloxacinB

DoxycyclineC

ErythromycinD

MinocyclineE

Page 647: Back to Filters (/Secure/TestMe/Filter ... - 1 File Download

8/11/2016 MyPastest

https://mypastest.pastest.com/Secure/TestMe/Browser/429893#Top 2/2

Next Question

Previous Question Tag Question Feedback End Review

Difficulty: Average

Peer Responses

Session Progress

0Responses Correct:

207Responses Incorrect:

207Responses Total:

0%Responses - % Correct:

Blog (https://www.pastest.com/blog) About Pastest (https://www.pastest.com/about-us)Contact Us (https://www.pastest.com/contact-us) Help (https://www.pastest.com/help)

© Pastest 2016

Page 648: Back to Filters (/Secure/TestMe/Filter ... - 1 File Download

8/11/2016 MyPastest

https://mypastest.pastest.com/Secure/TestMe/Browser/429893#Top 1/2

Back to Filters (/Secure/TestMe/Filter/429893/QA)

Question 145 of 207

46871

A patient who has mild benign prostatic hyperplasia has been advised to take finasteride.

Production of which of the following androgens is most likely to be inhibited as a result ofintake of this drug?

Explanation

The answer is Dihydrotestosterone –

Finasteride inhibits 5α-reductase, which converts testosterone to dihydrotestosterone(DHT). DHT is much more active than testosterone and binds more avidly to cytoplasmicreceptors to stimulate prostate growth. DTH may promote development of benignprostatic hyperplasia in the elderly.

Androstenedione (Option A) is incorrect. Androstenedione concentrations are not diminishedsignificantly.

Androsterone (Option B) is incorrect. Androsterone is a very weak androgen that ismetabolised to DHT by hydroxytestosterone dehydrogenase; this reaction is not blocked byfinasteride.

Dehydroepiandrosterone (Option C) is incorrect. Dihydroepiandrosterone is produced by theadrenals, gonads and other tissues, and is a precursor in synthesis of androgen.

Testosterone (Option E) is incorrect. Testosterone concentrations are not diminishedsignificantly.

AndrostenedioneA

AndrosteroneB

DehydroepiandrosteroneC

DihydrotestosteroneD

TestosteroneE

Page 649: Back to Filters (/Secure/TestMe/Filter ... - 1 File Download

8/11/2016 MyPastest

https://mypastest.pastest.com/Secure/TestMe/Browser/429893#Top 2/2

Next Question

Previous Question Tag Question Feedback End Review

Difficulty: Average

Peer Responses

Session Progress

0Responses Correct:

207Responses Incorrect:

207Responses Total:

0%Responses - % Correct:

Blog (https://www.pastest.com/blog) About Pastest (https://www.pastest.com/about-us)Contact Us (https://www.pastest.com/contact-us) Help (https://www.pastest.com/help)

© Pastest 2016

Page 650: Back to Filters (/Secure/TestMe/Filter ... - 1 File Download

8/11/2016 MyPastest

https://mypastest.pastest.com/Secure/TestMe/Browser/429893#Top 1/2

Back to Filters (/Secure/TestMe/Filter/429893/QA)

Question 146 of 207

46814

A 50-year-old patient takes 3.5 g aspirin daily together with omeprazole for her rheumatoidarthritis. One morning her pain is worse and she takes double the dose of aspirin beforebreakfast (7 g). Shortly after, she vomits and complains of tinnitus, sweating, dizziness andhyperventilation.

What is the most likely immediate effect of excess aspirin on acid–base status?

Explanation

The answer is Respiratory alkalosis –

When ingested in overdose, salicylates directly stimulate the respiratory centre toproduce an increase in both the depth and rate of breathing, thereby causing arespiratory alkalosis. This occurs even before metabolic acidosis, and is not solely acompensatory response.

Metabolic acidosis (Option A) is incorrect. Metabolic acidosis is not an immediate effect ofaspirin ingestion, but is a characteristic feature as toxicity progresses and may be verysevere, possibly requiring administration of intravenous sodium bicarbonate or haemodialysisto restore normal acid–base status.

Metabolic alkalosis (Option B) is incorrect. Metabolic alkalosis is not a recognised feature.

Normal pH (Option C) is incorrect. pH may be normal after minor aspirin overdose, but inaspirin overdose with severe symptoms this is unlikely to remain normal.

Respiratory acidosis (Option D) is incorrect. Respiratory acidosis is most likely to occur afteroverdose involving sedative agents, eg opioids or benzodiazepines.

Metabolic acidosisA

Metabolic alkalosisB

Normal pHC

Respiratory acidosisD

Respiratory alkalosisE

Page 651: Back to Filters (/Secure/TestMe/Filter ... - 1 File Download

8/11/2016 MyPastest

https://mypastest.pastest.com/Secure/TestMe/Browser/429893#Top 2/2

46814

Next Question

Previous Question Tag Question Feedback End Review

Difficulty: Average

Peer Responses

Session Progress

0Responses Correct:

207Responses Incorrect:

207Responses Total:

0%Responses - % Correct:

Blog (https://www.pastest.com/blog) About Pastest (https://www.pastest.com/about-us)Contact Us (https://www.pastest.com/contact-us) Help (https://www.pastest.com/help)

© Pastest 2016

Page 652: Back to Filters (/Secure/TestMe/Filter ... - 1 File Download

8/11/2016 MyPastest

https://mypastest.pastest.com/Secure/TestMe/Browser/429893#Top 1/2

Back to Filters (/Secure/TestMe/Filter/429893/QA)

Question 147 of 207

46960

A 49-year-old woman suffers with migraine, and smokes 30 cigarettes per day. She hasnoticed that taking paracetamol 1 g as advised by her GP has no effect on pain relief.

Which one of the following factors is the most likely to account for the lack of response toparacetamol?

Explanation

The answer is Hepatic enzyme induction –

The contents of smoke inhaled from cigarettes can induce hepatic microsomal enzymesof the cytochrome P450 system, one of the pathways involved in metabolism ofparacetamol. Common drugs that induce hepatic enzymes are phenytoin,carbamazepine, barbiturates, rifampicin, chronic alcohol excess and sulfonylureas.

Altered volume of distribution (Option A) is incorrect. The volume of distribution isunaltered.

Delayed gastric emptying in migraine (Option B) is incorrect. There may be delayed gastricemptying in migraine, but this has a limited effect on paracetamol absorption.

First-pass metabolism (Option C) is incorrect. Although chronic smokers will have higherlevels of hepatic enzyme activity, the effect of this is greater on paracetamol metabolism andclearance than on initial drug metabolism after absorption.

Reduced gut blood flow (Option E) is incorrect. Blood flow is unlikely to be alteredsignificantly.

Altered volume of distributionA

Delayed gastric emptying in migraineB

First-pass metabolismC

Hepatic enzyme inductionD

Reduced gut blood flowE

Page 653: Back to Filters (/Secure/TestMe/Filter ... - 1 File Download

8/11/2016 MyPastest

https://mypastest.pastest.com/Secure/TestMe/Browser/429893#Top 2/2

Next Question

Previous Question Tag Question Feedback End Review

Difficulty: Average

Peer Responses

Session Progress

0Responses Correct:

207Responses Incorrect:

207Responses Total:

0%Responses - % Correct:

Blog (https://www.pastest.com/blog) About Pastest (https://www.pastest.com/about-us)Contact Us (https://www.pastest.com/contact-us) Help (https://www.pastest.com/help)

© Pastest 2016

Page 654: Back to Filters (/Secure/TestMe/Filter ... - 1 File Download

8/11/2016 MyPastest

https://mypastest.pastest.com/Secure/TestMe/Browser/429893#Top 1/2

Back to Filters (/Secure/TestMe/Filter/429893/QA)

Question 148 of 207

A 29-year-old woman is undergoing treatment for secondary infertility after resection of apituitary adenoma. She is reviewed in the endocrinology clinic and tells you that she has beensuffering from headache, dizziness on standing, and symptoms suggestive of Raynaud’sphenomenon since starting a new medication prescribed at her previous clinic appointment.

Which one of the following drugs is most likely to explain these adverse effects?

Explanation

The answer is Bromocriptine –

Bromocriptine is a dopamine agonist used to treat Parkinson’s disease. It inhibitsprolactin release from the anterior pituitary and is used in treatment ofhyperprolactinaemia and acromegaly. Adverse effects include nausea, headache andlight-headedness due to orthostatic hypotension. High doses are associated with cold-induced peripheral digital vasospasm. It should be used with caution in patients with ahistory of coronary artery disease and Raynaud’s disease.

Buserelin (Option B) is incorrect. Buserelin is a gonadotrophin-releasing hormone (GnRH)agonist and may cause hot flushes, decreased libido and vaginal dryness.

Clomifene citrate (Option C) is incorrect. Clomifene blocks oestrogen receptors in thehypothalamus and stimulates pituitary release of gonadotrophin-releasing hormone (GnRH),which in turn stimulates ovarian function in women suffering from anovulation oroligovulation. Adverse effects characteristically include ovarian hyperstimulation (grossovarian enlargement, ascites, hydrothorax, hypovolaemia and shock), hot flushes, headache,weight gain and depression.

BromocriptineA

BuserelinB

Clomifene citrateC

Human chorionic gonadotrophin (hCG)D

Human menopausal gonadotrophin (hMG)E

Page 655: Back to Filters (/Secure/TestMe/Filter ... - 1 File Download

8/11/2016 MyPastest

https://mypastest.pastest.com/Secure/TestMe/Browser/429893#Top 2/2

46778

Human chorionic gonadotrophin (hCG) (Option D) is incorrect. Human chorionicgonadotrophin treatment may cause headache, depression and peripheral oedema.

Human menopausal gonadotrophin (hMG) (Option E) is incorrect. Human menopausalgonadotrophin can cause ovarian hyperstimulation, and reversible ovarian enlargement in20% of cases.

Next Question

Previous Question Tag Question Feedback End Review

Difficulty: Average

Peer Responses

Session Progress

0Responses Correct:

207Responses Incorrect:

207Responses Total:

0%Responses - % Correct:

Blog (https://www.pastest.com/blog) About Pastest (https://www.pastest.com/about-us)Contact Us (https://www.pastest.com/contact-us) Help (https://www.pastest.com/help)

© Pastest 2016

Page 656: Back to Filters (/Secure/TestMe/Filter ... - 1 File Download

8/11/2016 MyPastest

https://mypastest.pastest.com/Secure/TestMe/Browser/429893#Top 1/2

Back to Filters (/Secure/TestMe/Filter/429893/QA)

Question 149 of 207

46747

A 22-year-old man is diagnosed with pulmonary tuberculosis and is started on treatment. Amonth later, he presents with anorexia, malaise and fever. Laboratory investigations reveal: Hb12.8 g/dl; WCC 12.0 × 10 /l; urea 27 mmol/l; creatinine 480 μmol/l. Urinalysis shows thepresence of numerous pus cells but no growth on culture.

What is the most likely cause for his symptoms and investigations?

Explanation

The answer is Acute interstitial nephritis caused by rifampicin –

Renal failure in this patient, who presents with anorexia, malaise, fever and pus cells onurinalysis 1 month after starting treatment for primary tuberculosis, is likely to be due toacute interstitial nephritis caused by rifampicin toxicity.

Adverse effects of ethambutol (Option B) is incorrect. Ethambutol is associated withretrobulbar neuritis and arthralgia. Uncommonly it may be associated with hyperuricaemia,and interstitial nephritis (less common than anticipated after rifampicin).

Isoniazid toxicity (Option C) is incorrect. Isoniazid therapy may give rise to peripheralneuropathy, hepatitis and rash.

Renal tuberculosis (Option D) is incorrect. As the patient is already on treatment, spread oftuberculosis to the kidneys is unlikely to develop.

Superimposed urinary tract infection (Option E) is incorrect. A superimposed urinary tractinfection, by itself, would not be expected to cause renal failure.

9

Acute interstitial nephritis caused by rifampicinA

Adverse effects of ethambutolB

Isoniazid toxicityC

Renal tuberculosisD

Superimposed urinary tract infectionE

Page 657: Back to Filters (/Secure/TestMe/Filter ... - 1 File Download

8/11/2016 MyPastest

https://mypastest.pastest.com/Secure/TestMe/Browser/429893#Top 2/2

Next Question

Previous Question Tag Question Feedback End Review

Difficulty: Average

Peer Responses

Session Progress

0Responses Correct:

207Responses Incorrect:

207Responses Total:

0%Responses - % Correct:

Blog (https://www.pastest.com/blog) About Pastest (https://www.pastest.com/about-us)Contact Us (https://www.pastest.com/contact-us) Help (https://www.pastest.com/help)

© Pastest 2016

Page 658: Back to Filters (/Secure/TestMe/Filter ... - 1 File Download

8/11/2016 MyPastest

https://mypastest.pastest.com/Secure/TestMe/Browser/429893#Top 1/2

Back to Filters (/Secure/TestMe/Filter/429893/QA)

Question 150 of 207

You are considering the use of a new antihypertensive medication in an 81-year-old woman,but have some concerns about age-related differences in metabolism in the elderly.

Which of the following factors is most likely to account for differences in drug metabolismbetween the elderly and younger age groups?

Explanation

The answer is Reduced cardiac output in the elderly –

All of the following may account for differences in drug metabolism in the elderly:

Diminished renal function

Altered proportions of body fat and water

Reduced cardiac output

Some degree of altered hepatic metabolism

Disease

Concomitant medication use

For these reasons, clinical studies in drug development are rarely able to detectproblems that may occur with the use of new drugs in older adults. Many problemsassociated with the use of new drugs in older adults may only be discovered throughadverse event reporting during the post-marketing period.

Improved hepatic metabolism in the elderlyA

Increased cardiac output in the elderlyB

Increased GFR in the elderlyC

Less likelihood of concomitant medication in the elderlyD

Reduced cardiac output in the elderlyE

Page 659: Back to Filters (/Secure/TestMe/Filter ... - 1 File Download

8/11/2016 MyPastest

https://mypastest.pastest.com/Secure/TestMe/Browser/429893#Top 2/2

46852

Improved hepatic metabolism in the elderly (Option A) is incorrect. Hepatic metabolism isoften slightly lower in older adults, so that a dose reduction should be considered for drugsextensively cleared by liver metabolism.

Increased cardiac output in the elderly (Option B) is incorrect. Cardiac output often declinesin adulthood, although the impact on drug handling is often minimal.

Increased GFR in the elderly (Option C) is incorrect. eGFR is often decreased in olderpatients so that drugs cleared by renal elimination may accumulate.

Less likelihood of concomitant medication in the elderly (Option D) is incorrect. On thecontrary, older adults are susceptible to polypharmacy as a result of multiple long-termhealth disorders accumulating.

Next Question

Previous Question Tag Question Feedback End Review

Difficulty: Average

Peer Responses

Session Progress

0Responses Correct:

207Responses Incorrect:

207Responses Total:

0%Responses - % Correct:

Blog (https://www.pastest.com/blog) About Pastest (https://www.pastest.com/about-us)Contact Us (https://www.pastest.com/contact-us) Help (https://www.pastest.com/help)

© Pastest 2016

Page 660: Back to Filters (/Secure/TestMe/Filter ... - 1 File Download

8/11/2016 MyPastest

https://mypastest.pastest.com/Secure/TestMe/Browser/429893#Top 1/2

Back to Filters (/Secure/TestMe/Filter/429893/QA)

Question 151 of 207

A 63-year-old patient has a history of chronic renal failure and atrial fibrillation for which hereceives warfarin. He presented with an acutely painful right big toe. Investigation: uric acid390 micromol/l, creatinine 200.

What is the most appropriate treatment?

Explanation

The answer is Colchicine –

Colchicine may be given to patients with acute gout, even if there is renal failure. Mostcommon side-effects of colchicine include nausea, diarrhoea and abdominal pain. Up to6 mg may be used in divided doses to gain control of symptoms after an acute attack,and maintenance dose is around 1.5 mg given, once again, in divided doses.

Allopurinol (Option A) is incorrect. Allopurinol has a long half-life and may accumulate inpatients with chronic renal failure and paradoxically worsen gout symptoms when used in theinitial stages without the protection of non-steroidal anti-inflammatory drugs (NSAIDs) orcolchicine.

Ibuprofen (Option C) is incorrect. NSAIDs such as ibuprofen and indometacin are effective inalleviating the pain and inflammation of acute gout, but they interact with warfarin therapyto increase the risk of bleeding and may worsen renal impairment.

Indometacin (Option D) is incorrect. NSAIDs such as ibuprofen and indometacin are effectivein alleviating the pain and inflammation of acute gout, but they interact with warfarin therapyto increase the risk of bleeding and may worsen renal impairment.

AllopurinolA

ColchicineB

IbuprofenC

IndometacinD

ParacetamolE

Page 661: Back to Filters (/Secure/TestMe/Filter ... - 1 File Download

8/11/2016 MyPastest

https://mypastest.pastest.com/Secure/TestMe/Browser/429893#Top 2/2

46932

Paracetamol (Option E) is incorrect. Paracetamol is effective for mild to moderate pain butpossesses little anti-inflammatory action, so it has limited therapeutic benefit in acute gout.

Next Question

Previous Question Tag Question Feedback End Review

Difficulty: Average

Peer Responses

Session Progress

0Responses Correct:

207Responses Incorrect:

207Responses Total:

0%Responses - % Correct:

Blog (https://www.pastest.com/blog) About Pastest (https://www.pastest.com/about-us)Contact Us (https://www.pastest.com/contact-us) Help (https://www.pastest.com/help)

© Pastest 2016

Page 662: Back to Filters (/Secure/TestMe/Filter ... - 1 File Download

8/11/2016 MyPastest

https://mypastest.pastest.com/Secure/TestMe/Browser/429893#Top 1/2

Back to Filters (/Secure/TestMe/Filter/429893/QA)

Question 152 of 207

46923

A 58-year-old man has a history of asthma, obesity, gastro-oesophageal reflux disease, lowback pain and ischaemic heart disease (IHD). He presents with large, itchy wheals over thetrunk and limbs and a sensation of tightness in his throat.

Which one of following drugs is likely to have triggered this skin eruption?

Explanation

The answer is Aspirin –

This is the typical picture of aspirin sensitivity with flushing, bronchospasm and urticariabeing common features. Other drugs that may cause this type of picture on firstexposure include intravenous morphine and N-acetylcysteine used to treat paracetamoloverdose. An alternative to aspirin use in this patient is clopidogrel, an alternativeplatelet inhibitor. Aspirin desensitisation may be used to overcome this problem.

Glyceryl trinitrate (GTN) spray (Option B) is incorrect. Hypersensitivity or allergic reactionsto GTN are very uncommon. Common adverse effects include flushing, hypotension, andheadache.

Omeprazole (Option C) is incorrect. Omeprazole may cause acute interstitial nephritis.

Paracetamol (Option D) is incorrect. Paracetamol is not associated with hypersensitivity orallergic reactions.

Simvastatin (Option E) is incorrect. Simvastatin may be associated with allergic reactions, butmuch less commonly than aspirin, hence aspirin is the preferred answer. Other adverseeffects include hepatitis and myopathy.

AspirinA

Glyceryl trinitrate sprayB

OmeprazoleC

ParacetamolD

SimvastatinE

Page 663: Back to Filters (/Secure/TestMe/Filter ... - 1 File Download

8/11/2016 MyPastest

https://mypastest.pastest.com/Secure/TestMe/Browser/429893#Top 2/2

46923

Next Question

Previous Question Tag Question Feedback End Review

Difficulty: Average

Peer Responses

Session Progress

0Responses Correct:

207Responses Incorrect:

207Responses Total:

0%Responses - % Correct:

Blog (https://www.pastest.com/blog) About Pastest (https://www.pastest.com/about-us)Contact Us (https://www.pastest.com/contact-us) Help (https://www.pastest.com/help)

© Pastest 2016

Page 664: Back to Filters (/Secure/TestMe/Filter ... - 1 File Download

8/11/2016 MyPastest

https://mypastest.pastest.com/Secure/TestMe/Browser/429893#Top 1/2

Back to Filters (/Secure/TestMe/Filter/429893/QA)

Question 153 of 207

You see a 57-year-old business man in the hypertension clinic for review. He has been recentlystarted on losartan by his GP and he is referred for further advice and investigation.

Which mechanism of action best accounts for the blood pressure lowering action oflosartan?

Explanation

The answer is Angiotensin-II receptor blockade –

Angiotensin II is a powerful vasoconstrictor. Angiotensin-II subtype-1 receptorantagonists, also known as angiotensin receptor blockers (ARBs), include losartan,candesartan and valsartan. The major actions of angiotensin receptor antagonists reflectinhibition of the classic effects of the renin–angiotensin system mediated by the AT1receptor on the blood vessels, heart, adrenal gland, kidneys, brain and sympatheticnervous system. Like ACE inhibitors, ARBs may cause hyperkalaemia and renal failure,especially in sodium-depleted patients and in those with a critical reduction of renalblood flow. They are less likely than ACE inhibitors to cause cough (1% versus 15%).

Calcium-channel blockade (Option B) is incorrect. Calcium-channel blockers may berelatively selective for cardiac channels (diltiazem, verapamil) or peripheral vascular channels(eg nifedipine, amlodipine). This relative selectivity is lost in overdose when effects areexerted at both cardiac and peripheral channels.

Fall in plasma renin concentrations (Option C) is incorrect. Neither ARBs nor ACE inhibitorshave a significant effect on renin concentrations; both act at downstream parts of the renin–angiotensin–aldosterone system.

Angiotension-II receptor blockadeA

Calcium-channel blockadeB

Fall in plasma renin concentrationsC

Increased plasma aldosterone concentrationsD

Inhibition of angiotensin converting enzymeE

Page 665: Back to Filters (/Secure/TestMe/Filter ... - 1 File Download

8/11/2016 MyPastest

https://mypastest.pastest.com/Secure/TestMe/Browser/429893#Top 2/2

46788

Increased plasma aldosterone concentrations (Option D) is incorrect. Both ARBs and ACEinhibitors are capable of lowering aldosterone concentrations.

Inhibition of angiotensin converting enzyme (Option E) is incorrect. ACE inhibitors (egramipril, lisinopril) block a different step in the renin–angiotensin system, the enzyme thatconverts angiotensin I to angiotensin II (the same enzyme breaks down bradykinin to kinins,so that ACE inhibitors cause accumulation of bradykinin in the lungs and cough).

Next Question

Previous Question Tag Question Feedback End Review

Difficulty: Average

Peer Responses

Session Progress

0Responses Correct:

207Responses Incorrect:

207Responses Total:

0%Responses - % Correct:

Blog (https://www.pastest.com/blog) About Pastest (https://www.pastest.com/about-us)Contact Us (https://www.pastest.com/contact-us) Help (https://www.pastest.com/help)

© Pastest 2016

Page 666: Back to Filters (/Secure/TestMe/Filter ... - 1 File Download

8/11/2016 MyPastest

https://mypastest.pastest.com/Secure/TestMe/Browser/429893#Top 1/2

Back to Filters (/Secure/TestMe/Filter/429893/QA)

Question 154 of 207

40147

A 34-year-old man is brought to the Emergency Department by the police having been foundin the street in an agitated state. He was apparently shouting and aggressively beggingoutside a local burger bar when he was noted to collapse, suffering a short (1 minute) tonicclonic seizure. Previous attendances at the Emergency Department with opiate,benzodiazepine and alcohol abuse are noted. Most recently he was enrolled in an addictiontreatment program and had been prescribed an SSRI. He tells you he can feel insects crawlingall over his skin and hears voices telling him that he is worthless. On examination his BP is145/85 mmHg; pulse is 92/min and regular. He is covered in sweat.

Which of the following is he most likely to be withdrawing from?

Explanation

The answer is Alcohol -

The withdrawal seizure, signs of autonomic hyperactivity and tactile hallucinations fit wellwith alcohol withdrawal. Sedative withdrawal is associated with hyperactivity and potentiallyseizures, although tactile hallucinations are usually not a feature. Generally opiate withdrawalis not associated with seizures, but is associated with a flu-like illness and features such asrhinorrhoea, sneezing, yawning, lacrimation, abdominal cramping, leg cramping, piloerection,nausea and vomiting. GHB withdrawal symptoms are similar to those of benzodiazepinewithdrawal. Sertraline withdrawal is associated with anxiety.

AlcoholA

DiazepamB

DiamorphineC

GHBD

SertralineE

Next Question

Previous Question Tag Question Feedback End Review

Page 667: Back to Filters (/Secure/TestMe/Filter ... - 1 File Download

8/11/2016 MyPastest

https://mypastest.pastest.com/Secure/TestMe/Browser/429893#Top 2/2

Difficulty: Average

Peer Responses

Session Progress

0Responses Correct:

207Responses Incorrect:

207Responses Total:

0%Responses - % Correct:

Blog (https://www.pastest.com/blog) About Pastest (https://www.pastest.com/about-us)Contact Us (https://www.pastest.com/contact-us) Help (https://www.pastest.com/help)

© Pastest 2016

Page 668: Back to Filters (/Secure/TestMe/Filter ... - 1 File Download

8/11/2016 MyPastest

https://mypastest.pastest.com/Secure/TestMe/Browser/429893#Top 1/2

Back to Filters (/Secure/TestMe/Filter/429893/QA)

Question 155 of 207

The use of performance-enhancing anabolic steroids by professional athletes is generallydecreasing owing to random drug testing. However, the use among members of the publicattending gyms may be increasing. A 19-year-old man asks you some questions about thepossible effects.

Which of the following statements best describes the possible cardiovascular and metaboliceffects of anabolic steroid use?

Explanation

The answer is Haematocrit increases during prolonged use –

Anabolic steroids can be taken orally (eg stanozolol) but are often injected owing toextensive first-pass metabolism (eg testosterone enantate). Among their many unwantedeffects, they increase the risk of cardiovascular disease, particularly with prolonged use.These effects include blood pressure elevation, increased LDL-cholesterol and decreasedHDL-cholesterol. With prolonged use haematocrit increases, which may cause aprothrombotic tendency; plasma fibrinogen concentrations decrease with prolonged use.

Blood concentrations of HDL-cholesterol are increased (Option A) is incorrect. HDL-cholesterol concentrations tend to decrease.

Blood concentrations of LDL-cholesterol are decreased (Option B) is incorrect. LDL-cholesterol concentrations may increase.

Blood pressure decreases during prolonged use (Option C) is incorrect. Blood pressure tendsto increase, increasing stroke risk.

Blood concentrations of HDL-cholesterol are increasedA

Blood concentrations of LDL-cholesterol are decreasedB

Blood pressure decreases during prolonged useC

Erythrocyte sedimentation rate increasesD

Haematocrit increases during prolonged useE

Page 669: Back to Filters (/Secure/TestMe/Filter ... - 1 File Download

8/11/2016 MyPastest

https://mypastest.pastest.com/Secure/TestMe/Browser/429893#Top 2/2

46379

Erythrocyte sedimentation rate (ESR) increases (Option D) is incorrect. There is nosignificant effect on ESR.

Next Question

Previous Question Tag Question Feedback End Review

Difficulty: Average

Peer Responses

Session Progress

0Responses Correct:

207Responses Incorrect:

207Responses Total:

0%Responses - % Correct:

Blog (https://www.pastest.com/blog) About Pastest (https://www.pastest.com/about-us)Contact Us (https://www.pastest.com/contact-us) Help (https://www.pastest.com/help)

© Pastest 2016

Page 670: Back to Filters (/Secure/TestMe/Filter ... - 1 File Download

8/11/2016 MyPastest

https://mypastest.pastest.com/Secure/TestMe/Browser/429893#Top 1/2

Back to Filters (/Secure/TestMe/Filter/429893/QA)

Question 156 of 207

A 45-year-old woman has been diagnosed with polycystic ovarian syndrome. She has beenreceiving anticonvulsant treatment for a number of years.

Which one of the following anticonvulsants is most likely to be associated with thedevelopment of polycystic ovarian syndrome (PCOS)?

Explanation

The answer is Sodium valproate –

Sodium valproate has been associated with the development of polycystic ovariansyndrome (PCOS), a condition associated with central obesity, hirsutism, irregularperiods or secondary amenorrhoea, infertility and insulin resistance. The underlyingpathogenesis of PCOS is not yet established, but there appears to be defective ovarianproduction of oestradiol, with overproduction of precursor molecules, which areconverted into androgens in extra-glandular tissue. Characteristic features of PCOSinclude:

A low follicle-stimulating hormone level

High luteinising hormone level

High oestradiol level

High androgen levels

Occasionally virilism may occur

PhenobarbitoneA

PhenytoinB

Sodium valproateC

TopiramateD

VigabatrinE

Page 671: Back to Filters (/Secure/TestMe/Filter ... - 1 File Download

8/11/2016 MyPastest

https://mypastest.pastest.com/Secure/TestMe/Browser/429893#Top 2/2

46729

Phenobarbitone (Option A) is incorrect. PCOS is not a recognised adverse effect ofphenobarbitone.

Phenytoin (Option B) is incorrect. PCOS is not a recognised adverse effect of phenytoin.

Topiramate (Option D) is incorrect. PCOS is not a recognised adverse effect of Topiramate.

Vigabatrin (Option E) is incorrect. PCOS is not a recognised adverse effect of vigabatrin.

Next Question

Previous Question Tag Question Feedback End Review

Difficulty: Average

Peer Responses

Session Progress

0Responses Correct:

207Responses Incorrect:

207Responses Total:

0%Responses - % Correct:

Blog (https://www.pastest.com/blog) About Pastest (https://www.pastest.com/about-us)Contact Us (https://www.pastest.com/contact-us) Help (https://www.pastest.com/help)

© Pastest 2016

Page 672: Back to Filters (/Secure/TestMe/Filter ... - 1 File Download

8/11/2016 MyPastest

https://mypastest.pastest.com/Secure/TestMe/Browser/429893#Top 1/2

Back to Filters (/Secure/TestMe/Filter/429893/QA)

Question 157 of 207

45812

You are discussing with an obstetric FY2 some of the potential risks of prescribing to apatient in early pregnancy.

Which of the drugs listed below would be considered safe in the first trimester of pregnancy?

Explanation

The answer is Acetylcysteine –

A number of drugs are known to have teratogenic effects, and should normally beavoided. These are listed in British National Formulary, including androgens causingcardiac deformities, ethanol causing foetal alcohol syndrome, diethylstilbestrol causingvaginal carcinoma, phenobarbital causing cleft palate, and thalidomide causingphocomelia. For many drugs, insufficient information exists to inform outcomes after usein pregnancy. Data collected by the UK Teratology Information Service indicate thatacetylcysteine, as used in the setting of paracetamol overdose, does not appear toincrease risks.

Carbamazepine (Option B) is incorrect. This is a recognised cause of microcephaly.

Lithium (Option C) is incorrect. This is a recognised cause of neonatal goitre andhypothyroidism.

Sodium valproate (Option D) is incorrect. This is a recognised cause of neural tube defects.

Warfarin (Option E) is incorrect. This is a recognised cause of chondrodysplasia punctate.

AcetylcysteineA

CarbamazepineB

LithiumC

Sodium valproateD

WarfarinE

Page 673: Back to Filters (/Secure/TestMe/Filter ... - 1 File Download

8/11/2016 MyPastest

https://mypastest.pastest.com/Secure/TestMe/Browser/429893#Top 2/2

Next Question

Previous Question Tag Question Feedback End Review

Difficulty: Average

Peer Responses

Session Progress

0Responses Correct:

207Responses Incorrect:

207Responses Total:

0%Responses - % Correct:

Blog (https://www.pastest.com/blog) About Pastest (https://www.pastest.com/about-us)Contact Us (https://www.pastest.com/contact-us) Help (https://www.pastest.com/help)

© Pastest 2016

Page 674: Back to Filters (/Secure/TestMe/Filter ... - 1 File Download

8/11/2016 MyPastest

https://mypastest.pastest.com/Secure/TestMe/Browser/429893#Top 1/2

Back to Filters (/Secure/TestMe/Filter/429893/QA)

Question 158 of 207

21360

A 59-year-old man is admitted with unstable angina. He has a history of type-2 diabetes anda previous inferior myocardial infarction. His ECG shows anterior ST depression and he hasongoing chest pain despite nitrates. He goes for angioplasty and is treated with abciximab.

Which one of the following correctly describes the mode of action of abciximab?

Explanation

Abciximab

Abciximab is an inhibitor of the glycoprotein 2b 3a receptor on the platelet membrane

The receptor mediates platelet aggregation

Inhibition by abciximab, a chimeric/human monoclonal antibody, leads to decreased

thrombus formation

Abciximab is indicated in both the management of unstable angina and the prevention

of ischaemic complications in patients who have undergone percutaneous coronary

intervention

Other notes

Aspirin inhibits cyclooxygenase

Clopidogrel inhibits the platelet ADP receptor

Cyclooxygenase inhibitorA

Prostaglandin E inhibitorB

Glycoprotein 2b 3a inhibitorC

Phosphodiesterase inhibitorD

Thromboxane A2 inhibitorE

Next Question

Page 675: Back to Filters (/Secure/TestMe/Filter ... - 1 File Download

8/11/2016 MyPastest

https://mypastest.pastest.com/Secure/TestMe/Browser/429893#Top 2/2

Previous Question Tag Question Feedback End Review

Difficulty: Easy

Peer Responses

Session Progress

0Responses Correct:

207Responses Incorrect:

207Responses Total:

0%Responses - % Correct:

Blog (https://www.pastest.com/blog) About Pastest (https://www.pastest.com/about-us)Contact Us (https://www.pastest.com/contact-us) Help (https://www.pastest.com/help)

© Pastest 2016

Page 676: Back to Filters (/Secure/TestMe/Filter ... - 1 File Download

8/11/2016 MyPastest

https://mypastest.pastest.com/Secure/TestMe/Browser/429893#Top 1/2

Back to Filters (/Secure/TestMe/Filter/429893/QA)

Question 159 of 207

A 22-year-old woman is brought to the Emergency Department having ingested at least 20tablets of paracetamol 8 h earlier in front of her boyfriend. She had initially refused to let himcall an ambulance but then relented. She weighs 61 kg.

What is the most appropriate immediate management?

Explanation

The answer is Intravenous N-acetylcysteine –

Activated charcoal is useful if given within 1 h of the paracetamol overdose, and maysufficiently reduce paracetamol absorption so that acetylcysteine antidote is notrequired. At 8 h post-ingestion, charcoal would be ineffective. It would be helpful toestablish baseline liver function tests, electrolytes, prothrombin time and paracetamolconcentrations, but acetylcysteine treatment should not be delayed while these resultsare awaited. Acetylcysteine is normally advised in cases where the paracetamol doseexceeds 75 mg/kg body weight, and where paracetamol concentrations are above thenomogram if patients have taken an overdose at a single timepoint. If patients presentsufficiently early after overdose, then it may be reasonable to await paracetamolconcentrations before treating. However, treatment should not be delayed for more than8 h after ingestion; therefore acetylcysteine should be commenced while theparacetamol concentrations are awaited.

Inform the local liver unit for management of acute liver failure (Option A) is incorrect. Itwould be too early for fulminant liver failure to have occurred.

Inform the local liver unit for management of acute liver failureA

Intravenous N-acetylcysteineB

Liver function tests, prothrombin time and INR estimations then decide treatmentC

Oral activated charcoalD

Plasma paracetamol concentration estimation then decide treatmentE

Page 677: Back to Filters (/Secure/TestMe/Filter ... - 1 File Download

8/11/2016 MyPastest

https://mypastest.pastest.com/Secure/TestMe/Browser/429893#Top 2/2

46602

Liver function tests, prothrombin time and INR estimations then decide treatment (Option C)is incorrect. Although it will be important to check these blood tests, treatment should notbe delayed while the results are awaited.

Oral activated charcoal (Option D) is incorrect. At 8 h after ingestion, oral activated charcoalwill be ineffective in reducing paracetamol exposure.

Plasma paracetamol concentration estimation then decide treatment (Option E) is incorrect.A paracetamol concentration should be sent, but antidote treatment should not be delayedwhile the results are awaited.

Next Question

Previous Question Tag Question Feedback End Review

Difficulty: Average

Peer Responses

Session Progress

0Responses Correct:

207Responses Incorrect:

207Responses Total:

0%Responses - % Correct:

Blog (https://www.pastest.com/blog) About Pastest (https://www.pastest.com/about-us)Contact Us (https://www.pastest.com/contact-us) Help (https://www.pastest.com/help)

© Pastest 2016

Page 678: Back to Filters (/Secure/TestMe/Filter ... - 1 File Download

8/11/2016 MyPastest

https://mypastest.pastest.com/Secure/TestMe/Browser/429893#Top 1/2

Back to Filters (/Secure/TestMe/Filter/429893/QA)

Question 160 of 207

46820

A patient assessed in the Dementia Clinic is found to meet the diagnostic criteria formoderately severe Alzheimer’s disease with a MMSE of 12/30.

Which of the following drugs is most likely to slow progression of the disease?

Explanation

The answer is Donepezil –

Pharmacological treatments can be divided into ‘symptom-oriented’ treatments(depression, agitation and psychotic symptoms) and ‘disease-orientated’ treatments thatinclude cholinesterase inhibitors (eg donepezil, rivastigmine) and memantine. Therapycan achieve modest improvements in cognition in 25–50% of patients.

Atropine (Option A) is incorrect. Atropine confers anticholinergic effects but has no value intreatment of dementia.

Diphenhydramine (Option B) is incorrect. Diphenhydramine possesses anticholinergic andantihistamine effects but would have no effect on the progression of dementia.

Levodopa (Option D) is incorrect. Levodopa is effective in Parkinson’s disease anddegenerative disorders with Parkinsonian features.

Promethazine (Option E) is incorrect. Promethazine may be effective in reducing nausea buthas no impact on the progression of dementia.

AtropineA

DiphenhydramineB

DonepezilC

LevodopaD

PromethazineE

Next Question

Page 679: Back to Filters (/Secure/TestMe/Filter ... - 1 File Download

8/11/2016 MyPastest

https://mypastest.pastest.com/Secure/TestMe/Browser/429893#Top 2/2

Previous Question Tag Question Feedback End Review

Difficulty: Average

Peer Responses

Session Progress

0Responses Correct:

207Responses Incorrect:

207Responses Total:

0%Responses - % Correct:

Blog (https://www.pastest.com/blog) About Pastest (https://www.pastest.com/about-us)Contact Us (https://www.pastest.com/contact-us) Help (https://www.pastest.com/help)

© Pastest 2016

Page 680: Back to Filters (/Secure/TestMe/Filter ... - 1 File Download

8/11/2016 MyPastest

https://mypastest.pastest.com/Secure/TestMe/Browser/429893#Top 1/2

Back to Filters (/Secure/TestMe/Filter/429893/QA)

Question 161 of 207

A 90-year-old man with chronic leukaemia presents with gout, which his general practitionertreats with allopurinol.

What is the primary pharmacological action of allopurinol?

Explanation

The answer is By inhibiting purine synthesis and urate production –

Leukaemia is associated with gout because of the increased turnover of blood cells,which leads to increased uric acid production. Other conditions associated with goutinclude: myeloproliferative disorders, such as polycythaemia vera, glucose-6-phospatedehydrogenase (G6PD) deficiency, chronic renal disease, drug therapy (thiazidediuretics), lead toxicity, primary hyperparathyroidism, hypothyroidism, increasedproduction of lactic acid from alcohol, exercise, or starvation. Allopurinol blocks theenzyme xanthine oxidase, which is responsible for the conversion of xanthine into urate,which is the final step in the pathway of purine breakdown.

By competing for the uric acid transporter in the kidney (Option A) is incorrect. Probenecidenhances excretion of uric acid.

By enhancing its solubility (Option B) is incorrect. Sodium bicarbonate creates an alkalineenvironment and solubilises urate so that its renal clearance is enhanced.

By inhibiting a step in urate breakdown (Option C) is incorrect. There is limited uratebreakdown to allantoin by oxidative metabolism; other mammals that express urate oxidaseextensively metabolise urate to allantoin.

By competing for the uric acid transporter in the kidneyA

By enhancing its solubilityB

By inhibiting a step in urate breakdownC

By inhibiting purine synthesis and urate productionD

By inhibiting the inflammatory response associated with leukaemiaE

Page 681: Back to Filters (/Secure/TestMe/Filter ... - 1 File Download

8/11/2016 MyPastest

https://mypastest.pastest.com/Secure/TestMe/Browser/429893#Top 2/2

46925

By inhibiting the inflammatory response associated with leukaemia (Option E) is incorrect.Treatment of classic acute gout involves the use of non-steroidal anti-inflammatory drugs(NSAIDs) during the initial period.

Next Question

Previous Question Tag Question Feedback End Review

Difficulty: Average

Peer Responses

Session Progress

0Responses Correct:

207Responses Incorrect:

207Responses Total:

0%Responses - % Correct:

Blog (https://www.pastest.com/blog) About Pastest (https://www.pastest.com/about-us)Contact Us (https://www.pastest.com/contact-us) Help (https://www.pastest.com/help)

© Pastest 2016

Page 682: Back to Filters (/Secure/TestMe/Filter ... - 1 File Download

8/11/2016 MyPastest

https://mypastest.pastest.com/Secure/TestMe/Browser/429893#Top 1/2

Back to Filters (/Secure/TestMe/Filter/429893/QA)

Question 162 of 207

46758

A 16-year-old boy has been receiving antiepileptic medications for several years. He attendsthe outpatient clinic for review and reports suffering from nausea and diplopia. Onexamination, there are marked visual-field defects noted affecting central and peripheralvisual fields.

Which of the following drugs is most likely to be responsible?

Explanation

The answer is Vigabatrin –

Vigabatrin is the drug of choice for infantile spasms in childhood. It is, however,associated with a number of other adverse effects and as such is not widely used apartfrom this indication. Adverse effects include aggression, alopecia, retinal atrophy andreduced peripheral vision. Treatment should be stopped immediately.

Ethosuximide (Option A) is incorrect. Ethosuximide only rarely causes visual disturbance.

Primidone (Option B) is incorrect. Primidone is a prodrug of phenobarbital, which is capableof causing diplopia, ataxia and slurred speech. It is not associated with visual-field defects.

Sodium valproate (Option C) is incorrect. Sodium valproate may cause liver toxicity,hyperammonaemia, pancreatitis and weight gain.

Tiagabine (Option D) is incorrect. Tiagabine may be useful in partial seizures or secondarygeneralised tonic–clonic seizures, but is not prescribed for infantile spasms; it may alsoimpair peripheral vision (visual disturbance is less well characterised than for vigabatrin).

EthosuximideA

PrimidoneB

Sodium valproateC

TiagabineD

VigabatrinE

Page 683: Back to Filters (/Secure/TestMe/Filter ... - 1 File Download

8/11/2016 MyPastest

https://mypastest.pastest.com/Secure/TestMe/Browser/429893#Top 2/2

Next Question

Previous Question Tag Question Feedback End Review

Difficulty: Average

Peer Responses

Session Progress

0Responses Correct:

207Responses Incorrect:

207Responses Total:

0%Responses - % Correct:

Blog (https://www.pastest.com/blog) About Pastest (https://www.pastest.com/about-us)Contact Us (https://www.pastest.com/contact-us) Help (https://www.pastest.com/help)

© Pastest 2016

Page 684: Back to Filters (/Secure/TestMe/Filter ... - 1 File Download

8/11/2016 MyPastest

https://mypastest.pastest.com/Secure/TestMe/Browser/429893#Top 1/2

Back to Filters (/Secure/TestMe/Filter/429893/QA)

Question 163 of 207

22423

You are trialling a new drug for type-2 diabetes for a major drug company. One of your jobs isto determine whether it can be given as a once-per-day agent. To do this you need to have anidea of the half-life and plasma concentration at various time points. The peak initial plasmaconcentration is 2 mg/dl, achieved 20 min after oral administration. The half-life is measuredat 4 h.

What is the plasma concentration likely to be 12 h after peak concentration is reached?

Explanation

Pharmacokinetics

The half-life is the time taken for the concentration of a drug to reduce by 50%

We are given the half-life here, which is 4 h, and are required to calculate the plasma

concentration 12 h after administration of a quantity of drug that gives a peak initial

plasma concentration of 2 mg/dl

12 h is equal to three half-lives

Therefore, the plasma concentration at 12 h will be 2/(2 × 2 ×2) = 0.25

The t (time taken to reach peak concentration) has nothing at all to do with the

calculation

0.1 mg/dlA

1.8 mg/dlB

0.25 mg/dlC

0.66 mg/dlD

0.5 mg/dlE

max

Next Question

Page 685: Back to Filters (/Secure/TestMe/Filter ... - 1 File Download

8/11/2016 MyPastest

https://mypastest.pastest.com/Secure/TestMe/Browser/429893#Top 2/2

Previous Question Tag Question Feedback End Review

Difficulty: Easy

Peer Responses

Session Progress

0Responses Correct:

207Responses Incorrect:

207Responses Total:

0%Responses - % Correct:

Blog (https://www.pastest.com/blog) About Pastest (https://www.pastest.com/about-us)Contact Us (https://www.pastest.com/contact-us) Help (https://www.pastest.com/help)

© Pastest 2016

Page 686: Back to Filters (/Secure/TestMe/Filter ... - 1 File Download

8/11/2016 MyPastest

https://mypastest.pastest.com/Secure/TestMe/Browser/429893#Top 1/2

Back to Filters (/Secure/TestMe/Filter/429893/QA)

Question 164 of 207

46962

An 18-year-old woman attends the Emergency Department with clinical features that makeyou suspicious of recreational drug use. A bedside urine test gives a positive reading foramphetamine.

Which of the following agents would be most strongly implicated?

Explanation

The answer is Ecstasy –

Ecstasy (3, 4-methylenedioxy methamphetamine) is a semi-synthetic amphetaminederivative that is used as an illicit recreational substance and may cross-react withamphetamine screening tests. The main adverse effects are nausea, muscle pains,confusion, ataxia, hyperkalaemia, metabolic acidosis, hyperthermia, cardiovascularcollapse and acute respiratory distress. There is no specific antidote, and treatmentinvolves supportive care.

Cannabis (Option A) is incorrect. Cannabis and its metabolites may be detectable in urine forseveral days to weeks after drug exposure.

Cocaine (Option B) is incorrect. Cocaine metabolites may be detectable in urine withinseveral hours after exposure.

Cyanide (Option C) is incorrect. Cyanide is converted to thiocyanates, which may bedetectable in blood or urine.

Morphine (Option E) is incorrect. Morphine and its metabolites may be readily detectable inurine within several hours up to around 1 day after exposure.

CannabisA

CocaineB

CyanideC

EcstasyD

MorphineE

Page 687: Back to Filters (/Secure/TestMe/Filter ... - 1 File Download

8/11/2016 MyPastest

https://mypastest.pastest.com/Secure/TestMe/Browser/429893#Top 2/2

46962

Next Question

Previous Question Tag Question Feedback End Review

Difficulty: Average

Peer Responses

Session Progress

0Responses Correct:

207Responses Incorrect:

207Responses Total:

0%Responses - % Correct:

Blog (https://www.pastest.com/blog) About Pastest (https://www.pastest.com/about-us)Contact Us (https://www.pastest.com/contact-us) Help (https://www.pastest.com/help)

© Pastest 2016

Page 688: Back to Filters (/Secure/TestMe/Filter ... - 1 File Download

8/11/2016 MyPastest

https://mypastest.pastest.com/Secure/TestMe/Browser/429893#Top 1/2

Back to Filters (/Secure/TestMe/Filter/429893/QA)

Question 165 of 207

46485

An obstetric FY2 stops you in the corridor to ask for some advice about a 44-year-oldwoman that presented at 24-weeks gestation. An obstetric ultrasound has identifiedsignificant intrauterine growth retardation. The patient had inadvertently been exposed to anumber of different drugs during early pregnancy.

Which of the following drugs would be most likely to cause intrauterine growth retardation?

Explanation

The answer is Bisoprolol –

Many drugs are capable of causing teratogenic risks and complications duringpregnancy. Beta-blockers may cause hypoglycaemia, intrauterine growth retardation andfetal bradycardia.

Aspirin (Option A) is incorrect. Aspirin is a recognised cause of kernicterus.

Bendroflumethiazide (Option B) is incorrect. Thiazide diuretics may cause neonatalthrombocytopaenia.

Ramipril (Option D) is incorrect. Angiotensin-converting enzyme (ACE) inhibitors may causeoligohydramnios and impaired renal function.

Simvastatin (Option E) is incorrect. Statins are not typically associated with intrauterinegrowth retardation.

AspirinA

BendroflumethiazideB

BisoprololC

RamiprilD

SimvastatinE

Next Question

Page 689: Back to Filters (/Secure/TestMe/Filter ... - 1 File Download

8/11/2016 MyPastest

https://mypastest.pastest.com/Secure/TestMe/Browser/429893#Top 2/2

Previous Question Tag Question Feedback End Review

Difficulty: Average

Peer Responses

Session Progress

0Responses Correct:

207Responses Incorrect:

207Responses Total:

0%Responses - % Correct:

Blog (https://www.pastest.com/blog) About Pastest (https://www.pastest.com/about-us)Contact Us (https://www.pastest.com/contact-us) Help (https://www.pastest.com/help)

© Pastest 2016

Page 690: Back to Filters (/Secure/TestMe/Filter ... - 1 File Download

8/11/2016 MyPastest

https://mypastest.pastest.com/Secure/TestMe/Browser/429893#Top 1/2

Back to Filters (/Secure/TestMe/Filter/429893/QA)

Question 166 of 207

You are asked to review a 46-year-old lady who has been referred by her GP with a creatinineof 342 micromol/l. Renal function was normal before commencing chemotherapy recently.

Which one of the following chemotherapeutic agents is most likely to result innephrotoxicity?

Explanation

The answer is Ciclosporin –

Ciclosporin is used as an immunosuppressive agent after renal transplantation. It has anarrow therapeutic window and can cause direct nephrotoxicity; as renal functionworsens plasma concentrations increase and renal function progressively deteriorates.

Cyclophosphamide (Option B) is incorrect. Cyclophosphamide is an alkylating agent thattypically causes bone marrow suppression.

Methotrexate (Option C) is incorrect. Methotrexate is a dihydrofolate reductase inhibitor thatis used to treat acute lymphoblastic leukaemia, non-Hodgkin’s lymphoma, some solidtumours and psoriasis. Folinic acid is concomitantly administered to reduce the risk ofmucositis or myelosuppression.

Sulfasalazine (Option D) is incorrect. Sulfasalazine is an amino salicylate that is used in thetreatment of ulcerative colitis, Crohn’s disease and rheumatoid arthritis. Adverse effectsinclude diarrhea, nausea, rash, Stevens–Johnson syndrome and hepatitis. Interstitial nephritismay occur, but this is rare and so ciclosporin is the preferred answer.

CiclosporinA

CyclophosphamideB

MethotrexateC

SulfasalazineD

VincristineE

Page 691: Back to Filters (/Secure/TestMe/Filter ... - 1 File Download

8/11/2016 MyPastest

https://mypastest.pastest.com/Secure/TestMe/Browser/429893#Top 2/2

46961

Vincristine (Option E) is incorrect. Vincristine is a vinca alkaloid that is used to treat acuteleukaemia and some solid tumours. Its characteristic adverse effect profile is neurotoxicity; itdoes not cause prominent bone marrow suppression or renal impairment.

Next Question

Previous Question Tag Question Feedback End Review

Difficulty: Average

Peer Responses

Session Progress

0Responses Correct:

207Responses Incorrect:

207Responses Total:

0%Responses - % Correct:

Blog (https://www.pastest.com/blog) About Pastest (https://www.pastest.com/about-us)Contact Us (https://www.pastest.com/contact-us) Help (https://www.pastest.com/help)

© Pastest 2016

Page 692: Back to Filters (/Secure/TestMe/Filter ... - 1 File Download

8/11/2016 MyPastest

https://mypastest.pastest.com/Secure/TestMe/Browser/429893#Top 1/2

Back to Filters (/Secure/TestMe/Filter/429893/QA)

Question 167 of 207

A 70-year-old man is found by his home help lying on the floor of his lounge. An emptybottle which contained 5 mg diazepam tablets and half a bottle of whisky are on the table.He has a past medical history of severe rheumatoid arthritis and you understand his pain anddisability had worsened significantly over the past few months. On examination in theEmergency Room his temperature is 34°C. His blood pressure is 105/65 mmHg, with a pulseof 60 beats per minute.

Investigations:

Hb 12.1 g/dl

White cell count 12.1 × 10 /l

Platelets 230 × 10 /l

Na 141 mmol/l

K 6.0 mmol/l

Creatinine 240 μmol/l

ALT 550 U/l

Urine blood ++

Which one of the following is the most likely diagnosis?

Explanation

9

9

+

+

Additional paracetamol overdoseA

RhabdomyolysisB

Urinary sepsisC

HepatitisD

Myocardial infarctionE

Page 693: Back to Filters (/Secure/TestMe/Filter ... - 1 File Download

8/11/2016 MyPastest

https://mypastest.pastest.com/Secure/TestMe/Browser/429893#Top 2/2

20937

Multidrug overdose

This man has taken an overdose of both diazepam and whisky and it is likely he

remained on the floor for a significant period after losing consciousness

This period on the floor has led to significant muscle necrosis, resulting in

myoglobinuria (urine dipstick positive to blood), acute renal impairment and raised

alanine aminotransferase

Whilst we are not given the result for creatine kinase, it will be raised in this case

Key in the early management of this patient is treatment of his hyperkalaemia, with

insulin and dextrose infusion the first-line option along with gentle rewarming

Next Question

Previous Question Tag Question Feedback End Review

Difficulty: Easy

Peer Responses

Session Progress

0Responses Correct:

207Responses Incorrect:

207Responses Total:

0%Responses - % Correct:

Blog (https://www.pastest.com/blog) About Pastest (https://www.pastest.com/about-us)Contact Us (https://www.pastest.com/contact-us) Help (https://www.pastest.com/help)

© Pastest 2016

Page 694: Back to Filters (/Secure/TestMe/Filter ... - 1 File Download

8/11/2016 MyPastest

https://mypastest.pastest.com/Secure/TestMe/Browser/429893#Top 1/2

Back to Filters (/Secure/TestMe/Filter/429893/QA)

Question 168 of 207

46802

You are asked by one of the obstetrics staff to see a 24-year-old woman who is 13 weeks’pregnant and has been found to have had high blood pressure recordings during the pastthree visits. Today her blood pressure is 170/110 mmHg. Her antenatal records indicate healthyblood pressure before pregnancy.

What would be the most appropriate antihypertensive drug to use in this patient?

Explanation

The answer is Methyldopa –

One of the earliest clinical trials on the treatment of hypertension during pregnancyinvolved a comparison of methyldopa with no treatment, which found no adverse effectson pregnancy outcomes or subsequent development in childhood.

Doxazosin (Option A) is incorrect. Doxazosin may be safe, there is no evidence ofteratogenicity, but has less robust outcome data when compared to methyldopa.

Enalapril (Option B) is incorrect. ACE inhibitors may cause oligohydramnios.

Hydrochlorothiazide (Option C) is incorrect. Thiazides may cause neonatalthrombocytopaenia, although this is a comparatively rare outcome. Insufficient data areavailable to consider thiazides safe for use.

Losartan (Option D) is incorrect. Angiotensin receptor blockers may cause oligohydramniosand neonatal hyperkalaemia.

DoxazosinA

EnalaprilB

HydrochlorothiazideC

LosartanD

MethyldopaE

Page 695: Back to Filters (/Secure/TestMe/Filter ... - 1 File Download

8/11/2016 MyPastest

https://mypastest.pastest.com/Secure/TestMe/Browser/429893#Top 2/2

Next Question

Previous Question Tag Question Feedback End Review

Difficulty: Average

Peer Responses

Session Progress

0Responses Correct:

207Responses Incorrect:

207Responses Total:

0%Responses - % Correct:

Blog (https://www.pastest.com/blog) About Pastest (https://www.pastest.com/about-us)Contact Us (https://www.pastest.com/contact-us) Help (https://www.pastest.com/help)

© Pastest 2016

Page 696: Back to Filters (/Secure/TestMe/Filter ... - 1 File Download

8/11/2016 MyPastest

https://mypastest.pastest.com/Secure/TestMe/Browser/429893#Top 1/2

Back to Filters (/Secure/TestMe/Filter/429893/QA)

Question 169 of 207

46489

A 67-year-old man has recently started taking dothiepin for the management of depression.His GP has contacted you for advice regarding the optimal dose, and for advice on how tomonitor for development of anticholinergice adverse effects.

Which of the following clinical features would be most strongly indicative of anticholinergicdrug effects?

Explanation

The answer is Hot facial sensation –

Common features include hot, dry skin, hypertension, tachycardia, dilated pupils, urinaryretention, constipation and delirium. Drugs with anticholinergic properties includetricyclic antidepressants, antipsychotics and antihistamines. In most cases, withdrawal ofthe offending drug is all that is required, along with supportive care.

Bradycardia (Option A) is incorrect. Tachycardia is a common feature.

Constricted pupils (Option B) is incorrect. Mydriasis is a recognised feature.

Hypotension (Option D) is incorrect. Slight blood pressure elevation may occur.

Urinary incontinence (Option E) is incorrect. Acute urinary retention is common.

BradycardiaA

Constricted pupilsB

Hot facial sensationC

HypotensionD

Urinary incontinenceE

Next Question

Page 697: Back to Filters (/Secure/TestMe/Filter ... - 1 File Download

8/11/2016 MyPastest

https://mypastest.pastest.com/Secure/TestMe/Browser/429893#Top 2/2

Previous Question Tag Question Feedback End Review

Difficulty: Average

Peer Responses

Session Progress

0Responses Correct:

207Responses Incorrect:

207Responses Total:

0%Responses - % Correct:

Blog (https://www.pastest.com/blog) About Pastest (https://www.pastest.com/about-us)Contact Us (https://www.pastest.com/contact-us) Help (https://www.pastest.com/help)

© Pastest 2016

Page 698: Back to Filters (/Secure/TestMe/Filter ... - 1 File Download

8/11/2016 MyPastest

https://mypastest.pastest.com/Secure/TestMe/Browser/429893#Top 1/2

Back to Filters (/Secure/TestMe/Filter/429893/QA)

Question 170 of 207

A 29-year-old man is found collapsed and unresponsive in the entrance to the hospital withan empty bottle of medicine. His partner thinks he was supplied with a new medication whenhe attended the psychiatry outpatient clinic on the day before. On examination he is drowsy,heart rate is 140 bpm, and an ECG shows wide complexes.

Which one of the following is the best initial treatment?

Explanation

The answer is Sodium bicarbonate –

The history is suggestive of an overdose of tricyclic antidepressants leading toventricular tachycardia. Symptoms and signs of tricyclic overdose include dry mouth,hypotension, hypothermia, urinary retention, hyperreflexia, up-going planters, coma,seizures and cardiac arrhythmias. Metabolic acidosis is a common feature, whichenhances cardiotoxicity. Sodium bicarbonate diminishes the sodium-channel-blockingeffect of tricyclics, thereby lessening the risk of seizures and arrhythmia.

Adenosine (Option A) is incorrect. Adenosine would not be expected to have any significanteffect.

Digoxin (Option B) is incorrect. Digoxin may increase the risk of cardiotoxicity in thissituation, and may worsen hyperkalaemia.

Lidocaine (Option C) is incorrect. Lidocaine is a class I antiarrhythmic agent that may worsenthe tricyclic-induced sodium channel blockade.

AdenosineA

DigoxinB

LidocaineC

PropranololD

Sodium bicarbonateE

Page 699: Back to Filters (/Secure/TestMe/Filter ... - 1 File Download

8/11/2016 MyPastest

https://mypastest.pastest.com/Secure/TestMe/Browser/429893#Top 2/2

46963

Propranolol (Option D) is incorrect. Propranolol and other β-blockers may be helpful incontrolling heart rate but are less effective in preventing arrhythmia and seizure, hencesodium bicarbonate is the preferred answer.

Next Question

Previous Question Tag Question Feedback End Review

Difficulty: Average

Peer Responses

Session Progress

0Responses Correct:

207Responses Incorrect:

207Responses Total:

0%Responses - % Correct:

Blog (https://www.pastest.com/blog) About Pastest (https://www.pastest.com/about-us)Contact Us (https://www.pastest.com/contact-us) Help (https://www.pastest.com/help)

© Pastest 2016

Page 700: Back to Filters (/Secure/TestMe/Filter ... - 1 File Download

8/11/2016 MyPastest

https://mypastest.pastest.com/Secure/TestMe/Browser/429893#Top 1/2

Back to Filters (/Secure/TestMe/Filter/429893/QA)

Question 171 of 207

A 24-year-old woman who is 16 weeks’ pregnant presents to her GP complaining of frequencyand dysuria. She has been relatively well in her pregnancy so far and has no significant pastmedical history. On examination she is pyrexial 37.6°C and has some suprapubic tenderness.

Investigations:

Hb 12.1 g/dl

White cell count 10.4 × 10 /l

Platelets 201 × 10 /l

Na 140 mmol/l

K 4.0 mmol/l

Creatinine 90 μmol/l

Urine dipstick blood +, protein ++, leukocytes ++

Which one of the following antibiotics should be avoided if at all possible in this patient?

Explanation

Antibiotic prescribing in pregnancy

Data from ciprofloxacin exposure to pregnant women do not indicate any specific risk

of malformations

9

9

+

+

AmoxicillinA

Co-amoxiclavB

CiprofloxacinC

TrimethoprimD

CephalexinE

Page 701: Back to Filters (/Secure/TestMe/Filter ... - 1 File Download

8/11/2016 MyPastest

https://mypastest.pastest.com/Secure/TestMe/Browser/429893#Top 2/2

20936

That being said, in animal studies on juvenile and prenatal animals, the potential for

damage to articular cartilage was indicated

For this reason, as a precaution, use of ciprofloxacin in pregnancy is not recommended

Similarly, because of secretion in breast milk, ciprofloxacin should not be used by

breast-feeding women

Most data exist for amoxicillin, trimethoprim and cephalexin use in this population;

therefore, according to sensitivities, one of these three agents would be the most

appropriate choice

Trimethoprim is a folate antagonist and is therefore avoided if possible in early

pregnancy, but at 16 weeks it is a reasonable option for therapy

Next Question

Previous Question Tag Question Feedback End Review

Difficulty: Average

Peer Responses

Session Progress

0Responses Correct:

207Responses Incorrect:

207Responses Total:

0%Responses - % Correct:

Blog (https://www.pastest.com/blog) About Pastest (https://www.pastest.com/about-us)Contact Us (https://www.pastest.com/contact-us) Help (https://www.pastest.com/help)

© Pastest 2016

Page 702: Back to Filters (/Secure/TestMe/Filter ... - 1 File Download

8/11/2016 MyPastest

https://mypastest.pastest.com/Secure/TestMe/Browser/429893#Top 1/2

Back to Filters (/Secure/TestMe/Filter/429893/QA)

Question 172 of 207

A 65-year-old chronic alcoholic is receiving warfarin therapy for left ventricular aneurysmfollowing a previous acute myocardial infarction 6 months ago. He now presents withderanged liver function tests and ultrasound scan appearances are suggestive of cirrhosis.

What is the most appropriate change that should be made to his anticoagulation?

Explanation

The answer is Decrease or stop the warfarin according to INR –

There are increased risks of bleeding associated with cirrhosis, and the potential for INRto increase owing to impaired vitamin K metabolism and clotting factor synthesis. Ideally,warfarin therapy should be continued to afford protection against thromboembolismrelated to the left ventricular aneurysm. However, close INR monitoring is required tominimise bleeding risk. He may be anticoagulated by liver failure to the extent that he nolonger needs warfarin therapy.

Increase the dose of warfarin (Option B) is incorrect. This would be hazardous and unhelpful.

Stop all anticoagulant treatment (Option C) is incorrect. Although this would reduce the risksof bleeding complications, it would leave the patient exposed to the risks ofthromboembolism associated with the left ventricular aneurysm.

Stop warfarin and start aspirin (Option D) is incorrect. Switching to an antiplatelet mightreduce risks of bleeding, but would not offer effective protection against thromboembolism.

Stop warfarin and start subcutaneous heparin injections (Option E) is incorrect. Subcutanousheparin might be more risky because it would be more difficult to measure the extent of

Decrease or stop the warfarin according to INRA

Increase the dose of warfarinB

Stop all anticoagulant treatmentC

Stop warfarin and start aspirinD

Stop warfarin and start subcutaneous heparin injectionsE

Page 703: Back to Filters (/Secure/TestMe/Filter ... - 1 File Download

8/11/2016 MyPastest

https://mypastest.pastest.com/Secure/TestMe/Browser/429893#Top 2/2

46774

anticoagulation, whereas warfarin may be monitored using INR.

Next Question

Previous Question Tag Question Feedback End Review

Difficulty: Average

Peer Responses

Session Progress

0Responses Correct:

207Responses Incorrect:

207Responses Total:

0%Responses - % Correct:

Blog (https://www.pastest.com/blog) About Pastest (https://www.pastest.com/about-us)Contact Us (https://www.pastest.com/contact-us) Help (https://www.pastest.com/help)

© Pastest 2016

Page 704: Back to Filters (/Secure/TestMe/Filter ... - 1 File Download

8/11/2016 MyPastest

https://mypastest.pastest.com/Secure/TestMe/Browser/429893#Top 1/2

Back to Filters (/Secure/TestMe/Filter/429893/QA)

Question 173 of 207

A 34-year-old patient who is 28 weeks’ pregnant with her first baby is referred to you as anemergency with newly diagnosed symptomatic hyperthyroidism.

Which of the following statements best describes the management of maternalthyrotoxicosis?

Explanation

The answer is Neonatal goitre is a recognised complication of propylthiouracil –

Management of thyrotoxicosis in pregnancy is similar to that for other patients, althoughsurgery should normally be avoided if possible, and radioactive iodine is contraindicatedin view of the radiation hazard to the fetus. Carbimazole and propylthiouracil are capableof crossing the placenta and may cause neonatal hypothyroidism and goitre. Goitre mayinterfere with childbirth and normally resolves after delivery. Both carbimazole andpropylthiouracil are excreted in maternal breast milk, theoretically leading to neonatalhypothyroidism.

‘Block and replace’ combination carbimazole and thyroxine is preferred in pregnancy (OptionA) is incorrect. Both carbimazole and propylthiouracil cross the placenta, although thyroxinedoes not. Therefore, the ‘block and replace’ regimen is not appropriate as this creates agreater risk of neonatal hypothyroidism than using carbimazole alone.

Carbimazole causes neonatal goitre in 75% of babies of treated mothers (Option B) isincorrect. Neonatal hypothyroidism occurs in approximately 10% of babies.

Carbimazole does not cross the placenta (Option C) is incorrect. Both carbimazole andpropylthiouracil may cross the placenta and cause neonatal hypothyroidism and goitre.

‘Block and replace’ combination carbimazole and thyroxine is preferred in pregnancyA

Carbimazole causes neonatal goitre in 75% of babies of treated mothersB

Carbimazole does not cross the placentaC

Neonatal goitre is a recognised complication of propylthiouracilD

Propylthiouracil does not cross the placentaE

Page 705: Back to Filters (/Secure/TestMe/Filter ... - 1 File Download

8/11/2016 MyPastest

https://mypastest.pastest.com/Secure/TestMe/Browser/429893#Top 2/2

46486

Propylthiouracil does not cross the placenta (Option E) is incorrect. Both carbimazole andpropylthiouracil may cross the placenta and cause neonatal hypothyroidism and goitre.

Next Question

Previous Question Tag Question Feedback End Review

Difficulty: Average

Peer Responses

Session Progress

0Responses Correct:

207Responses Incorrect:

207Responses Total:

0%Responses - % Correct:

Blog (https://www.pastest.com/blog) About Pastest (https://www.pastest.com/about-us)Contact Us (https://www.pastest.com/contact-us) Help (https://www.pastest.com/help)

© Pastest 2016

Page 706: Back to Filters (/Secure/TestMe/Filter ... - 1 File Download

8/11/2016 MyPastest

https://mypastest.pastest.com/Secure/TestMe/Browser/429893#Top 1/2

Back to Filters (/Secure/TestMe/Filter/429893/QA)

Question 174 of 207

A 71-year-old man is admitted with a left-sided hemiplegia. He has a history of type-2diabetes, which is managed with metformin 500 mg bd. He is poorly compliant with othermedications and smokes 20 cigarettes per day. On examination he has a blood pressure of167/87 mmHg some 3 h after admission. His left-sided hemiplegia has undergone slightimprovement. A CT scan is reported as showing normal appearances.

What is the optimal timing of antihypertensive therapy?

Explanation

The answer is Within 2 weeks –

There is significant controversy concerning the benefits of early antihypertensivetherapy. Treatment should normally be started prior to discharge from hospital or 2weeks after the stroke (which ever is the soonest). Treatment should only be introducedurgently if there is accelerated hypertension or hypertensive emergency. Long-termtarget blood pressure is 140/90 mmHg or less, and preferably to 130/80 mmHg.

Immediate treatment (Option A) is incorrect. The potential hazard of immediate therapy isworsened cerebral ischaemia, and should only be contemplated with acceleratedhypertension or very high systolic blood pressure values.

More than 1 month (Option B) is incorrect. Treatment should be started within 2 weeks or atdischarge from hospital, whichever occurs soonest.

Should be avoided due to risk of hypotension (Option C) is incorrect. Hypertension is thesingle largest risk factor for stroke, and optimal blood pressure control will reduce recurrentevents.

Immediate treatmentA

More than 1 monthB

Should be avoided due to risk of hypotensionC

Within 1 monthD

Within 2 weeksE

Page 707: Back to Filters (/Secure/TestMe/Filter ... - 1 File Download

8/11/2016 MyPastest

https://mypastest.pastest.com/Secure/TestMe/Browser/429893#Top 2/2

46985

Within 1 month (Option D) is incorrect. Treatment should be started within 2 weeks or atdischarge from hospital, whichever occurs soonest.

Next Question

Previous Question Tag Question Feedback End Review

Difficulty: Average

Peer Responses

Session Progress

0Responses Correct:

207Responses Incorrect:

207Responses Total:

0%Responses - % Correct:

Blog (https://www.pastest.com/blog) About Pastest (https://www.pastest.com/about-us)Contact Us (https://www.pastest.com/contact-us) Help (https://www.pastest.com/help)

© Pastest 2016

Page 708: Back to Filters (/Secure/TestMe/Filter ... - 1 File Download

8/11/2016 MyPastest

https://mypastest.pastest.com/Secure/TestMe/Browser/429893#Top 1/2

Back to Filters (/Secure/TestMe/Filter/429893/QA)

Question 175 of 207

46622

You see a 39-year-old man in the endocrinology clinic. He has a history of hyperthyroidismand is currently receiving treatment with carbimazole.

What mechanism best explains the pharmacological actions of carbimazole?

Explanation

The answer is Inhibition of the iodination of tyrosine –

Carbimazole is converted to its active metabolite, methimazole, which inhibits iodinationof tyrosine, thereby preventing coupling of thyroxine precursors. It also has animmunosuppressive action, leading to a reduction in serum thyroid-stimulating hormone(TSH) receptor antibody (TRAb) concentrations.

Conversion to methimazole, which blocks the TSH receptor (Option A) is incorrect. It isconverted to methimazole, but its actions are unrelated to the TSH receptor.

Destruction of functioning thyroid cells (Option B) is incorrect. Carbimazole does not destroyfunctioning thyroid cells, but disrupts the intracellular iodination of tyrosine and coupling,therefore preventing formation of thyroxine and tri-iodothyronine.

Inhibition of 5'-deiodinase (Option C) is incorrect. Propylthiouracil not carbimazole inhibits5'-deiodinase and prevents the release of free tri-iodothyronine and thyroxine from thethyroid cell.

Inhibition of thyroglobulin proteolysis (Option E) is incorrect. Iodine but not carbimazoleinhibits thyroglobulin proteolysis.

Conversion to methimazole, which blocks the TSH receptorA

Destruction of functioning thyroid cellsB

Inhibition of 5'-deiodinaseC

Inhibition of the iodination of tyrosineD

Inhibition of thyroglobulin proteolysisE

Page 709: Back to Filters (/Secure/TestMe/Filter ... - 1 File Download

8/11/2016 MyPastest

https://mypastest.pastest.com/Secure/TestMe/Browser/429893#Top 2/2

Next Question

Previous Question Tag Question Feedback End Review

Difficulty: Average

Peer Responses

Session Progress

0Responses Correct:

207Responses Incorrect:

207Responses Total:

0%Responses - % Correct:

Blog (https://www.pastest.com/blog) About Pastest (https://www.pastest.com/about-us)Contact Us (https://www.pastest.com/contact-us) Help (https://www.pastest.com/help)

© Pastest 2016

Page 710: Back to Filters (/Secure/TestMe/Filter ... - 1 File Download

8/11/2016 MyPastest

https://mypastest.pastest.com/Secure/TestMe/Browser/429893#Top 1/2

Back to Filters (/Secure/TestMe/Filter/429893/QA)

Question 176 of 207

In a diabetes specialist clinic you are about to review a 59-year-old patient with type-2diabetes mellitus who is not responding to dietary advice and weight reduction. You havedecided to prescribe an oral hypoglycaemic agent.

Which of the following statements most accurately expresses the mechanism of action ofhypoglycaemic agents used in the management of diabetes mellitus?

Explanation

The answer is Acarbose inhibits α-glucosidase enzymes within the gastrointestinal tract –

Acarbose inhibits intestinal α-glucosidase, which therefore delays the digestion andabsorption of starch and sucrose, and minimises the peak glucose concentrations arisingafter a meal. Its main unwanted effects are flatus and gastrointestinal disturbance.Biguanides (e.g. metformin) primarily decrease hepatic glucose secretion and do notcause hypoglycaemia, even in overdose; the main unwanted effect is lactic acidosis, andthey are contraindicated in renal failure. Sulfonylureas (e.g. gliclazide, glimepiride)increase insulin secretion in response to a given glucose load, and may cause weight gainand hypoglycaemia. Pioglitazone reduces peripheral insulin resistance (increases insulinsensitivity), and adverse effects include weight gain, hypoglycaemia, fluid retention andcongestive heart failure.

Metformin may cause hypoglycaemia (Option B) is incorrect. Metformin and biguanides donot cause hypoglycaemia. Other medications may cause hypoglycaemia, particularlysulfonylureas.

Acarbose inhibits α-glucosidase enzymes within the gastrointestinal tractA

Metformin may cause hypoglycaemiaB

Metformin stimulates pancreatic insulin secretionC

Pioglitazone causes hepatotoxicity in 15–20% of patientsD

Sulfonylureas reduce peripheral insulin sensitivityE

Page 711: Back to Filters (/Secure/TestMe/Filter ... - 1 File Download

8/11/2016 MyPastest

https://mypastest.pastest.com/Secure/TestMe/Browser/429893#Top 2/2

46382

Metformin stimulates pancreatic insulin secretion (Option C) is incorrect. Metformin does notincrease insulin secretion; biguanides decrease hepatic glucose secretion.

Pioglitazone causes hepatotoxicity in 15–20% of patients (Option D) is incorrect. Pioglitazoneand rosiglitazone may cause fluid retention and congestive heart failure. Idiosyncratic liverfailure was caused by troglitazone and it was withdrawn.

Sulfonylureas reduce peripheral insulin sensitivity (Option E) is incorrect. The mainmechanism of sulfonylureas is enhanced pancreatic insulin secretion.

Next Question

Previous Question Tag Question Feedback End Review

Difficulty: Average

Peer Responses

Session Progress

0Responses Correct:

207Responses Incorrect:

207Responses Total:

0%Responses - % Correct:

Blog (https://www.pastest.com/blog) About Pastest (https://www.pastest.com/about-us)Contact Us (https://www.pastest.com/contact-us) Help (https://www.pastest.com/help)

© Pastest 2016

Page 712: Back to Filters (/Secure/TestMe/Filter ... - 1 File Download

8/11/2016 MyPastest

https://mypastest.pastest.com/Secure/TestMe/Browser/429893#Top 1/2

Back to Filters (/Secure/TestMe/Filter/429893/QA)

Question 177 of 207

A 70-year-old man comes to the clinic complaining of blue vision. He has chronic atrialfibrillation and hypertension but has been passed fit to take sildenafil by his doctor. Onexamination he looks well, his pulse is 74 beats per minute, he has atrial fibrillation and hisblood pressure is 142/78 mmHg.

Investigations:

Hb 13.1 g/dl

White cell count 4.9 × 10 /l

Platelets 182 × 10 /l

Na 142 mmol/l

K 4.5 mmol/l

Creatinine 105 μmol/l

Which one of the following is the most likely cause of his blue vision amongst themedications he has been taking?

Explanation

Drug side-effects

Digoxin is associated with yellow vision in overdose

Bisoprolol, like all β-blockers, is associated with increased dreams/possible night terrors

Sildenafil

9

9

+

+

TemazepamA

SildenafilB

DigoxinC

BisoprololD

AmlodipineE

Page 713: Back to Filters (/Secure/TestMe/Filter ... - 1 File Download

8/11/2016 MyPastest

https://mypastest.pastest.com/Secure/TestMe/Browser/429893#Top 2/2

21325

is a phosphodiesterase type-5 (PDE-5) inhibitor

also has some activity against PDE-6, which is involved in the functioning of

retinal photoreceptors

At high sildenafil doses, the PDE-6 inhibitory effect becomes clinically significant and

patients complain of blue vision

In total over half of men taking 200 mg or more of sildenafil experience some kind of

visual side-effects

Next Question

Previous Question Tag Question Feedback End Review

Difficulty: Average

Peer Responses

Session Progress

0Responses Correct:

207Responses Incorrect:

207Responses Total:

0%Responses - % Correct:

Blog (https://www.pastest.com/blog) About Pastest (https://www.pastest.com/about-us)Contact Us (https://www.pastest.com/contact-us) Help (https://www.pastest.com/help)

© Pastest 2016

Page 714: Back to Filters (/Secure/TestMe/Filter ... - 1 File Download

8/11/2016 MyPastest

https://mypastest.pastest.com/Secure/TestMe/Browser/429893#Top 1/2

Back to Filters (/Secure/TestMe/Filter/429893/QA)

Question 178 of 207

A 22-year-old woman who is 5+ weeks’ pregnant presents with dysuria to the EmergencyDepartment. She has felt feverish for the past 24 h and has been to pass urine two to threetimes per hour. On examination she is pyrexial at 37.8°C and has suprapubic tenderness. Shehas suffered a rash on penicillin in the past.

Investigations:

Hb 11.9 g/dl

White cell count 11.2 × 10 /l

Platelets 191 × 10 /l

Na 139 mmol/l

K 4.9 mmol/l

Creatinine 95 μmol/l

Which one of the following is the most appropriate antibiotic choice for her?

Explanation

Managing urinary tract infection in pregnancy

Appropriate options for managing urinary tract infection in pregnancy include both

cephalexin and trimethoprim

9

9

+

+

ClarithromycinA

CiprofloxacinB

DoxycyclineC

CephalexinD

TrimethoprimE

Page 715: Back to Filters (/Secure/TestMe/Filter ... - 1 File Download

8/11/2016 MyPastest

https://mypastest.pastest.com/Secure/TestMe/Browser/429893#Top 2/2

22464

However, given her stage in pregnancy, the theoretical action of trimethoprim in

interfering with folate metabolism drives cephalexin as the most appropriate option

Both tetracyclines and quinolones are expressly contraindicated for use in pregnancy

Clarithromycin is unlikely to be effective for UTI

There is up to 20% crossover with sensitivity to cephalosporins in patients with

penicillin allergy, so she should be observed for signs of allergy

Next Question

Previous Question Tag Question Feedback End Review

Difficulty: Average

Peer Responses

Session Progress

0Responses Correct:

207Responses Incorrect:

207Responses Total:

0%Responses - % Correct:

Blog (https://www.pastest.com/blog) About Pastest (https://www.pastest.com/about-us)Contact Us (https://www.pastest.com/contact-us) Help (https://www.pastest.com/help)

© Pastest 2016

Page 716: Back to Filters (/Secure/TestMe/Filter ... - 1 File Download

8/11/2016 MyPastest

https://mypastest.pastest.com/Secure/TestMe/Browser/429893#Top 1/2

Back to Filters (/Secure/TestMe/Filter/429893/QA)

Question 179 of 207

46910

A 37-year-old woman is referred to the Acute Medical Unit for investigation of breathlessness.She is 3 weeks post-partum and is solely breast feeding her infant.

Which of the following drugs, in normal therapeutic doses, should be avoided from the pointof view of breast-feeding?

Explanation

The answer is Aspirin –

Aspirin, when ingested by the mother, can cause neonatal bleeding problems, and thereis a risk of Reye’s syndrome in the infant even after exposure to low quantities.

Erythromycin (Option B) is incorrect. Erythromycin, like many antibiotics, is water soluble,and only small amounts cross into breast milk; it is unlikely to cause adverse effects in thebaby.

Heparin (Option C) is incorrect. Heparin crosses into breast milk in amounts too small toproduce significant anticoagulation in the neonate.

Thyroxine (Option D) is incorrect. Thyroxine crosses into breast milk in amounts too small toappreciably affect thyroid function or to affect tests for neonatal hypothyroidism.

Warfarin (Option E) is incorrect. Warfarin crosses into breast milk in amounts too small toproduce significant anticoagulation in the neonate.

AspirinA

ErythromycinB

HeparinC

ThyroxineD

WarfarinE

Next Question

Page 717: Back to Filters (/Secure/TestMe/Filter ... - 1 File Download

8/11/2016 MyPastest

https://mypastest.pastest.com/Secure/TestMe/Browser/429893#Top 2/2

Previous Question Tag Question Feedback End Review

Difficulty: Average

Peer Responses

Session Progress

0Responses Correct:

207Responses Incorrect:

207Responses Total:

0%Responses - % Correct:

Blog (https://www.pastest.com/blog) About Pastest (https://www.pastest.com/about-us)Contact Us (https://www.pastest.com/contact-us) Help (https://www.pastest.com/help)

© Pastest 2016

Page 718: Back to Filters (/Secure/TestMe/Filter ... - 1 File Download

8/11/2016 MyPastest

https://mypastest.pastest.com/Secure/TestMe/Browser/429893#Top 1/2

Back to Filters (/Secure/TestMe/Filter/429893/QA)

Question 180 of 207

46803

A 54-year-old man has been diagnosed with type-2 diabetes and started on metformin inaddition to appropriate dietary and lifestyle advice. He asks you about possible adverseeffects of metformin treatment.

Which of the following adverse effects is most likely to be attributable to metformintreatment?

Explanation

The answer is Lactic acidosis –

Metformin is a biguanide that is known to cause lactic acidosis, which may be a severe oreven fatal complication, particularly in the setting of intentional metformin overdose.Metformin acts mainly by inhibiting hepatic gluconeogenesis, and inhibits lactatedehydrogenase in the gut and liver. Haemodialysis may clear lactate and allow correctionof acidosis, although mortality remains high (over 30%), partly because of the organfailures that commonly coexist.

Anaphylaxis (Option A) is incorrect. This is not a recognised feature of metformin therapy.

Cardiotoxicity (Option B) is incorrect. Fluid retention and cardiac failure may be seen inpatients receiving glitazone therapy.

Pulmonary fibrosis (Option D) is incorrect. Pulmonary fibrosis is not a recognised feature ofmetformin therapy.

Rhabdomyolysis (Option E) is incorrect. Rhabdomyolysis is more likely to occur inconjunction with statin or fibrate therapy, or especially with the combination of the two.

AnaphylaxisA

CardiotoxicityB

Lactic acidosisC

Pulmonary fibrosisD

RhabdomyolysisE

Page 719: Back to Filters (/Secure/TestMe/Filter ... - 1 File Download

8/11/2016 MyPastest

https://mypastest.pastest.com/Secure/TestMe/Browser/429893#Top 2/2

46803

Next Question

Previous Question Tag Question Feedback End Review

Difficulty: Average

Peer Responses

Session Progress

0Responses Correct:

207Responses Incorrect:

207Responses Total:

0%Responses - % Correct:

Blog (https://www.pastest.com/blog) About Pastest (https://www.pastest.com/about-us)Contact Us (https://www.pastest.com/contact-us) Help (https://www.pastest.com/help)

© Pastest 2016

Page 720: Back to Filters (/Secure/TestMe/Filter ... - 1 File Download

8/11/2016 MyPastest

https://mypastest.pastest.com/Secure/TestMe/Browser/429893#Top 1/2

Back to Filters (/Secure/TestMe/Filter/429893/QA)

Question 181 of 207

36464

A 72-year-old woman with multiple medical problems is admitted to the EmergencyDepartment with progressive confusion over the past 6 weeks. She has a history ofdepression, diabetes and hypertension. On examination her BP is 135/72 mmHg, pulse is70/min and regular. Her BMI is 25. Venous blood gas sampling reveals a sodium of 118 mmol/l.

Which of the following agents is most likely to be the cause of her low sodium?

Explanation

The answer is Bendroflumethiazide

The drugs most commonly implicated in SIADH are:

• Thiazide diuretics• Vincristine• Vinblastine• Cyclophosphamide

Others include:

• Chlorpromamide• Carbamazepine• Phenothiazines• Tricyclic antidepressants• Clofibrate• Oxytocin• Vasopressin• Morphine• Barbiturates• Nicotine

AtorvastatinA

BendroflumethiazideB

FluoxetineC

GlicalzideD

RamiprilE

Page 721: Back to Filters (/Secure/TestMe/Filter ... - 1 File Download

8/11/2016 MyPastest

https://mypastest.pastest.com/Secure/TestMe/Browser/429893#Top 2/2

36464

Next Question

Previous Question Tag Question Feedback End Review

Difficulty: Average

Peer Responses

Session Progress

0Responses Correct:

207Responses Incorrect:

207Responses Total:

0%Responses - % Correct:

Blog (https://www.pastest.com/blog) About Pastest (https://www.pastest.com/about-us)Contact Us (https://www.pastest.com/contact-us) Help (https://www.pastest.com/help)

© Pastest 2016

Page 722: Back to Filters (/Secure/TestMe/Filter ... - 1 File Download

8/11/2016 MyPastest

https://mypastest.pastest.com/Secure/TestMe/Browser/429893#Top 1/2

Back to Filters (/Secure/TestMe/Filter/429893/QA)

Question 182 of 207

46776

A 52-year-old alcoholic was diagnosed with respiratory tuberculosis, and commenced onantituberculous treatment. After receiving treatment for 3 months, he has complained oftingling and numbness affecting both feet.

Which of the following drugs is most likely to be responsible for his symptoms?

Explanation

The answer is Isoniazid –

Peripheral neuropathy is rare with the standard 300 mg adult dose of isoniazid; however,it is more likely to occur with (1) higher doses, (2) slow acetylator status, or (3) thepresence of predisposing conditions including malnutrition, alcoholism, diabetes, AIDSand uraemia.

Ethambutol (Option A) is incorrect. The most common serious adverse event withethambutol is retrobulbar neuritis, causing loss of visual acuity and red–green colourblindness; regular ophthalmology review is required.

Pyrazinamide (Option C) is incorrect. Pyrazinamide causes hepatotoxicity (1–5% of cases),drug fever and hyperuricaemia.

Rifampicin (Option D) is incorrect. Rifampicin is associated with thrombocytopaenia, orange–red staining of body fluids and hepatitis.

Streptomycin (Option E) is incorrect. Streptomycin has been used to treat tuberculosis and isusually administered by intramuscular injection.

EthambutolA

IsoniazidB

PyrazinamideC

RifampicinD

StreptomycinE

Page 723: Back to Filters (/Secure/TestMe/Filter ... - 1 File Download

8/11/2016 MyPastest

https://mypastest.pastest.com/Secure/TestMe/Browser/429893#Top 2/2

Next Question

Previous Question Tag Question Feedback End Review

Difficulty: Average

Peer Responses

Session Progress

0Responses Correct:

207Responses Incorrect:

207Responses Total:

0%Responses - % Correct:

Blog (https://www.pastest.com/blog) About Pastest (https://www.pastest.com/about-us)Contact Us (https://www.pastest.com/contact-us) Help (https://www.pastest.com/help)

© Pastest 2016

Page 724: Back to Filters (/Secure/TestMe/Filter ... - 1 File Download

8/11/2016 MyPastest

https://mypastest.pastest.com/Secure/TestMe/Browser/429893#Top 1/2

Back to Filters (/Secure/TestMe/Filter/429893/QA)

Question 183 of 207

46940

A 65-year-old man has been receiving bendroflumethiazide for more than 15 years fortreatment of hypertension.

Which of the following mechanisms best explains the renal effect of thiazide diuretics?

Explanation

The answer is Proximal segment of the distal convoluted tubule –

Most diuretics act by inhibiting sodium and water reabsorption in the renal tubules.Thiazide diuretics inhibit Na /Cl cotransport in the proximal segment of the distalconvoluted tubule.

Ascending limb of Henle’s loop (Option A) is incorrect. The loop diuretics furosemide andbumetanide inhibit Na /K /Cl co-transport in the ascending limb of Henle’s loop.

Collecting duct (Option B) is incorrect. Aldosterone causes sodium absorption andpotassium secretion from the collecting ducts; aldosterone antagonists including eplerenoneand spironolactone inhibit salt and water absorption within the collecting ducts.

Descending limb of Henle’s loop (Option C) is incorrect. This is not a recognised target fordiuretics.

Distal segment of the distal convoluted tubule (Option D) is incorrect. The potassium-sparingdiuretic amiloride inhibits sodium channels in the distal segment of the distal convolutedtubule.

Ascending limb of Henle’s loopA

Collecting ductB

Descending limb of Henle’s loopC

Distal segment of the distal convoluted tubuleD

Proximal segment of the distal convoluted tubuleE

+ –

+ + –

Page 725: Back to Filters (/Secure/TestMe/Filter ... - 1 File Download

8/11/2016 MyPastest

https://mypastest.pastest.com/Secure/TestMe/Browser/429893#Top 2/2

Next Question

Previous Question Tag Question Feedback End Review

Difficulty: Average

Peer Responses

Session Progress

0Responses Correct:

207Responses Incorrect:

207Responses Total:

0%Responses - % Correct:

Blog (https://www.pastest.com/blog) About Pastest (https://www.pastest.com/about-us)Contact Us (https://www.pastest.com/contact-us) Help (https://www.pastest.com/help)

© Pastest 2016

Page 726: Back to Filters (/Secure/TestMe/Filter ... - 1 File Download

8/11/2016 MyPastest

https://mypastest.pastest.com/Secure/TestMe/Browser/429893#Top 1/2

Back to Filters (/Secure/TestMe/Filter/429893/QA)

Question 184 of 207

46846

A 28-year-old woman is taking a course of ciprofloxacin for pyelonephritis.

Which of the following best describes the mode of action of ciprofloxacin?

Explanation

The answer is Inhibition of DNA gyrase –

Quinolones such as ciprofloxacin primarily act by inhibiting bacterial DNA gyrase.

Inhibition of bacterial cell wall synthesis (Option A) is incorrect. Bacterial cell wall synthesis isinhibited by penicillins, cephalosporins and vancomycin.

Inhibition of bacterial ribosomal protein synthesis (Option B) is incorrect. Aminoglycosides,erythromycin and tetracyclines act via inhibition of bacterial ribosomal synthesis.

Inhibition of folate synthesis (Option D) is incorrect. Sulfonamides and trimethoprim areinhibitors of folate synthesis.

Production of β-lactamase (Option E) is incorrect. β-lactamase production is one commonmechanism of antibiotic resistance, which is often conferred via transfer of plasmids betweenbacteria.

Inhibition of bacterial cell wall synthesisA

Inhibition of bacterial ribosomal protein synthesisB

Inhibition of DNA gyraseC

Inhibition of folate synthesisD

Production of β-lactamaseE

Next Question

Page 727: Back to Filters (/Secure/TestMe/Filter ... - 1 File Download

8/11/2016 MyPastest

https://mypastest.pastest.com/Secure/TestMe/Browser/429893#Top 2/2

Previous QuestionTag Question Feedback End Review

Difficulty: Average

Peer Responses

Session Progress

0Responses Correct:

207Responses Incorrect:

207Responses Total:

0%Responses - % Correct:

Blog (https://www.pastest.com/blog) About Pastest (https://www.pastest.com/about-us)Contact Us (https://www.pastest.com/contact-us) Help (https://www.pastest.com/help)

© Pastest 2016

Page 728: Back to Filters (/Secure/TestMe/Filter ... - 1 File Download

8/11/2016 MyPastest

https://mypastest.pastest.com/Secure/TestMe/Browser/429893#Top 1/2

Back to Filters (/Secure/TestMe/Filter/429893/QA)

Question 185 of 207

Four patients present to the Emergency Department after exposure to an unusual vapour at alocal supermarket. Reports from the ambulance crew and first responders indicate they aredealing with a suspected chemical release, and you have been alerted that the patients mighthave been exposed to sarin.

Which of the following clinical features would most strongly support a diagnosis of nerveagent exposure?

Explanation

The answer is Hypersalivation –

Organophosphates and nerve agents are rapidly absorbed through the gastrointestinaland respiratory tract and skin. The principal action is inhibition of acetylcholinesterasesuch that there is accumulation of acetylcholine at muscarinic and nicotinic receptors.Muscarinic effects include miosis, hypersalivation, sweating, diarrhoea, excessivebronchial secretions and bradycardia. Nicotinic effects include muscle fasciculation andtremor. Central nervous system effects include anxiety, memory impairment, headacheand coma. Neuropathy may develop over several weeks, initially a flaccid paralysis, andlater hypertonia, hyperreflexia and a spastic paralysis. Treatments include supplementaloxygen, atropine and pralidoxime, an activator of cholinesterase.

Hypotension (Option B) is incorrect. This is not a significant feature.

Intense cough (Option C) is incorrect. Cough is not a typical feature, unlike exposure torespiratory irritants.

Mydriasis (Option D) is incorrect. Miosis is a characteristic clinical finding.

HypersalivationA

HypotensionB

Intense coughC

MydriasisD

TachycardiaE

Page 729: Back to Filters (/Secure/TestMe/Filter ... - 1 File Download

8/11/2016 MyPastest

https://mypastest.pastest.com/Secure/TestMe/Browser/429893#Top 2/2

45797

Tachycardia (Option E) is incorrect. Bradycardia may occur due to muscarinic cardiac action.

Next Question

Previous Question Tag Question Feedback End Review

Difficulty: Average

Peer Responses

Session Progress

0Responses Correct:

207Responses Incorrect:

207Responses Total:

0%Responses - % Correct:

Blog (https://www.pastest.com/blog) About Pastest (https://www.pastest.com/about-us)Contact Us (https://www.pastest.com/contact-us) Help (https://www.pastest.com/help)

© Pastest 2016

Page 730: Back to Filters (/Secure/TestMe/Filter ... - 1 File Download

8/11/2016 MyPastest

https://mypastest.pastest.com/Secure/TestMe/Browser/429893#Top 1/2

Back to Filters (/Secure/TestMe/Filter/429893/QA)

Question 186 of 207

A 38-year-old female patient has a history of brittle asthma and has received a number ofcourses of corticosteroid treatments over the years for flare ups of her asthma. She has nowbeen diagnosed with vertebral fractures secondary to osteoporosis.

Which of the following treatments would be the most appropriate treatment for minimisingthe risk of further fractures?

Explanation

The answer is Alendronate –

Glucocorticoid-induced osteoporosis (GIO) leads to fractures in 30–50% of patients.Bisphosphonates are currently the recommended first-line therapy for the preventionand treatment of GIO. They have been shown to increase bone mineral density at thespine and hip and to decrease the incidence of vertebral fractures (especially inpostmenopausal women). A broad range of therapies have been tested with less certainbenefits in terms of fracture risk, including selective oestrogen receptor modulators, oralcontraceptive pill, strontium, calcium, vitamin D, vitamin D analogues, sex hormones andanabolic agents.

Calcitonin (Option B) is incorrect. Calcitonin increases lumbar spine bone mineral density buthas no proven fracture efficacy.

Cyproterone acetate (Option C) is incorrect. Antiandrogen therapy is unlikely to be ofbenefit. Testosterone appears to increase bone mineral density, although the ability toprevent fractures is uncertain.

AlendronateA

CalcitoninB

Cyproterone acetateC

Oestradiol and norethisterone acetateD

Parathyroid hormoneE

Page 731: Back to Filters (/Secure/TestMe/Filter ... - 1 File Download

8/11/2016 MyPastest

https://mypastest.pastest.com/Secure/TestMe/Browser/429893#Top 2/2

46786

Oestradiol and norethisterone acetate (Option D) is incorrect. Female hormone replacementtherapy (HRT) has been found to increase lumbar spine bone mineral density in hypogonadalpatients on glucocorticoid therapy, but effects on the hip are inconsistent, and there are nodata concerning fracture prevention in GIO.

Parathyroid hormone (Option E) is incorrect. Parathyroid hormone treatment has recentlybeen shown to increase bone mineral density at the spine and hip, decreases the incidence ofnew vertebral fractures, and may be an alternative to bisphosphonates where these are nottolerated.

Next Question

Previous Question Tag Question Feedback End Review

Difficulty: Average

Peer Responses

Session Progress

0Responses Correct:

207Responses Incorrect:

207Responses Total:

0%Responses - % Correct:

Blog (https://www.pastest.com/blog) About Pastest (https://www.pastest.com/about-us)Contact Us (https://www.pastest.com/contact-us) Help (https://www.pastest.com/help)

© Pastest 2016

Page 732: Back to Filters (/Secure/TestMe/Filter ... - 1 File Download

8/11/2016 MyPastest

https://mypastest.pastest.com/Secure/TestMe/Browser/429893#Top 1/2

Back to Filters (/Secure/TestMe/Filter/429893/QA)

Question 187 of 207

A 32-year-old woman with long-term epilepsy was concerned about weight gain due totaking valproate. The only other medication she is taking is the oral contraceptive pill.

Which one of the following is the best alternative to valproate?

Explanation

The answer is Lamotrigine –

Lamotrigine is less likely to interfere with the oral contraceptive pill than otherantiepileptic agents. The overall risk of birth defects in women who take oneanticonvulsant is around 7%, compared with a background rate of 3% in the populationas a whole.

Carbamazepine (Option A) is incorrect. Carbamazepine is a powerful hepatic enzyme inducerand is associated with decreased effectiveness of the oral contraceptive pill owing toacceleration of the metabolism of oestrogens and progestogens.

Phenobarbital (Option C) is incorrect. Phenobarbital is a powerful hepatic enzyme inducerand is associated with decreased effectiveness of the oral contraceptive pill owing toacceleration of the metabolism of oestrogens and progestogens.

Phenytoin (Option D) is incorrect. Phenytoin is a powerful hepatic enzyme inducer and isassociated with decreased effectiveness of the oral contraceptive pill owing to accelerationof the metabolism of oestrogens and progestogens.

Topiramate (Option E) is incorrect. Topiramate is a hepatic enzyme inducer and is associatedwith decreased effectiveness of the oral contraceptive pill owing to acceleration of the

CarbamazepineA

LamotrigineB

PhenobarbitalC

PhenytoinD

TopiramateE

Page 733: Back to Filters (/Secure/TestMe/Filter ... - 1 File Download

8/11/2016 MyPastest

https://mypastest.pastest.com/Secure/TestMe/Browser/429893#Top 2/2

46930

metabolism of oestrogens and progestogens.

Next Question

Previous Question Tag Question Feedback End Review

Difficulty: Average

Peer Responses

Session Progress

0Responses Correct:

207Responses Incorrect:

207Responses Total:

0%Responses - % Correct:

Blog (https://www.pastest.com/blog) About Pastest (https://www.pastest.com/about-us)Contact Us (https://www.pastest.com/contact-us) Help (https://www.pastest.com/help)

© Pastest 2016

Page 734: Back to Filters (/Secure/TestMe/Filter ... - 1 File Download

8/11/2016 MyPastest

https://mypastest.pastest.com/Secure/TestMe/Browser/429893#Top 1/2

Back to Filters (/Secure/TestMe/Filter/429893/QA)

Question 188 of 207

What is the most common cause of paranoid psychosis with visual hallucination?

Explanation

The answer is Alcohol withdrawal –

Delirium tremens occurs in less than 5% of individuals withdrawing from alcohol andhappens around 2–4 days after cessation of alcohol intake. Untreated it carries a highmortality of up to 15%. Features include coarse tremor, agitation, confusion, delusion andvisual hallucinations. Fever, sweating and tachycardia may also occur; rarely there isassociated ketoacidosis. Co-existent hypoglycaemia and Wernicke–Korsakoff psychosisare considerations. Sedation with chlordiazepoxide is often necessary to preventseizures. Vitamin B supplements given intravenously (Pabrinex) should be considered.

Amphetamine withdrawal (Option B) is incorrect. Amphetamines are comparatively shortlived so that withdrawal features are less common.

Benzodiazepine withdrawal (Option C) is incorrect. Benzodiazepine withdrawal may provokeagitation and seizures, particularly after abrupt cessation in patients that have been treatedwith prolonged therapy at high doses.

Opiate withdrawal (Option D) is incorrect. Opioid withdrawal features may appear similar toalcohol withdrawal, although physical symptoms usually predominate, including sweating,tachycardia, cravings and diarrhoea.

Selective serotonin re-uptake inhibitor (SSRI) withdrawal (Option E) is incorrect. SSRIwithdrawal is associated with dysphoria and suicidal ideation, so guidelines advocate gradual

Alcohol withdrawalA

Amphetamine withdrawalB

Benzodiazepine withdrawalC

Opiate withdrawalD

Selective serotonin re-uptake inhibitor withdrawalE

Page 735: Back to Filters (/Secure/TestMe/Filter ... - 1 File Download

8/11/2016 MyPastest

https://mypastest.pastest.com/Secure/TestMe/Browser/429893#Top 2/2

46931

dose reduction over several weeks.

Next Question

Previous Question Tag Question Feedback End Review

Difficulty: Average

Peer Responses

Session Progress

0Responses Correct:

207Responses Incorrect:

207Responses Total:

0%Responses - % Correct:

Blog (https://www.pastest.com/blog) About Pastest (https://www.pastest.com/about-us)Contact Us (https://www.pastest.com/contact-us) Help (https://www.pastest.com/help)

© Pastest 2016

Page 736: Back to Filters (/Secure/TestMe/Filter ... - 1 File Download

8/11/2016 MyPastest

https://mypastest.pastest.com/Secure/TestMe/Browser/429893#Top 1/2

Back to Filters (/Secure/TestMe/Filter/429893/QA)

Question 189 of 207

46822

You are called to the endoscopy suite by a ‘2222’ emergency call. An 82-year-old woman isbeing prepared for an upper gastrointestinal endoscopy to investigate a recent coffee-groundvomit and melaena. Soon after receiving a dose of intravenous midazolam, she suffers acardiorespiratory arrest.

Which is the most appropriate choice of emergency treatment?

Explanation

The answer is Flumazenil –

Flumazenil, a benzodiazepine antagonist, is used to reverse the central sedative effectsof benzodiazepines after anaesthetic and similar procedures. Flumazenil has a shorterhalf-life than that of diazepam and midazolam and there is a risk that patients mayrequire a repeated dose.

Dantrolene (Option A) is incorrect. Dantrolene is used in the treatment of malignanthyperthermia such as occurs in the setting of serotonergic syndrome.

Diazepam (Option B) is incorrect. Diazepam would cause addive benzodiazepine effects andworsen respiratory depression.

Naloxone (Option D) is incorrect. Naloxone is used for the treatment of opiate overdose.

Neostigmine (Option E) is incorrect. Neostigmine is used in the treatment of myastheniagravis.

DantroleneA

DiazepamB

FlumazenilC

NaloxoneD

NeostigmineE

Page 737: Back to Filters (/Secure/TestMe/Filter ... - 1 File Download

8/11/2016 MyPastest

https://mypastest.pastest.com/Secure/TestMe/Browser/429893#Top 2/2

Next Question

Previous Question Tag Question Feedback End Review

Difficulty: Average

Peer Responses

Session Progress

0Responses Correct:

207Responses Incorrect:

207Responses Total:

0%Responses - % Correct:

Blog (https://www.pastest.com/blog) About Pastest (https://www.pastest.com/about-us)Contact Us (https://www.pastest.com/contact-us) Help (https://www.pastest.com/help)

© Pastest 2016

Page 738: Back to Filters (/Secure/TestMe/Filter ... - 1 File Download

8/11/2016 MyPastest

https://mypastest.pastest.com/Secure/TestMe/Browser/429893#Top 1/2

Back to Filters (/Secure/TestMe/Filter/429893/QA)

Question 190 of 207

A 40-year-old woman is brought unconscious to the Emergency Department. She apparentlyhas swallowed a large number of pills belonging to her friend who is being treated for apsychiatric disorder. On examination, she is apyrexial, her pulse is 130 bpm and her bloodpressure is 90/60 mmHg. Her pupils are dilated. ECG shows sinus tachycardia and occasionalventricular ectopics.

Which of the following drugs is she most likely to have taken?

Explanation

The answer is Imipramine –

Overdose with tricyclic antidepressants can produce anticholinergic effects (dilatedpupils, tachycardia) and sodium channel blockade (QRS prolongation, arrhythmia andseizures). Tricyclic overdose may also be associated with loss of consciousness, grosslyabnormal reflexes and divergent eye movements.

Chlordiazepoxide (Option A) is incorrect. Chlordiazepoxide overdose would cause centralsedation, respiratory depression and coma; it would not be expected to alter pupil size orcause tachycardia.

Chlorpromazine (Option B) is incorrect. Phenothiazine (chlorpromazine) poisoning causesdystonia (torticollis, opisthotonos).

Fluoxetine (Option C) is incorrect. Fluoxetine is a selective serotonin-reuptake inhibitor(SSRI) and overdose can cause blurring of vision, fits, headache, tremors and, rarely,serotonin syndrome (agitation, dysarthria, tremor, myoclonus and restlessness).

ChlordiazepoxideA

ChlorpromazineB

FluoxetineC

ImipramineD

LithiumE

Page 739: Back to Filters (/Secure/TestMe/Filter ... - 1 File Download

8/11/2016 MyPastest

https://mypastest.pastest.com/Secure/TestMe/Browser/429893#Top 2/2

46764

Lithium (Option E) is incorrect. Lithium causes blurring of vision, diarrhoea, vomiting,hypokalaemia, drowsiness, ataxia, coarse tremors, dysarthria, seizures, psychosis and coma.Dilated pupils are not a recognised feature of lithium toxicity.

Next Question

Previous Question Tag Question Feedback End Review

Difficulty: Average

Peer Responses

Session Progress

0Responses Correct:

207Responses Incorrect:

207Responses Total:

0%Responses - % Correct:

Blog (https://www.pastest.com/blog) About Pastest (https://www.pastest.com/about-us)Contact Us (https://www.pastest.com/contact-us) Help (https://www.pastest.com/help)

© Pastest 2016

Page 740: Back to Filters (/Secure/TestMe/Filter ... - 1 File Download

8/11/2016 MyPastest

https://mypastest.pastest.com/Secure/TestMe/Browser/429893#Top 1/2

Back to Filters (/Secure/TestMe/Filter/429893/QA)

Question 191 of 207

You admit a 65-year-old man, via the Emergency Department, who has suffered an acutemyocardial infarction. On examination of his medication history you note that he is taking anewly licensed antihypertensive agent and you suspect an adverse drug reaction.

Which of the following is most accurate concerning the Yellow Card recording systemcurrently employed in the UK?

Explanation

The answer is Only 10% of serious adverse drug reactions are identified by Yellow Cards –

The Yellow Card scheme is administered by the Medicines Healthcare ProductsRegulatory Agency, a governmental organisation. It was set up over 40 years ago as ameans of identifying important adverse effects at an earlier stage than would be allowedwithout systematic national data collection. It requires voluntary reporting; however, onlyaround 10% of adverse drug reactions (ADRs) are thought to be reported. When a newproduct is launched there may be significant over-reporting compared to other agents inthe same established drug class.

Approximately 50% of serious adverse drug reactions are identified through the system(Option A) is incorrect. Reporting rates are estimated to be around 10% overall.

Follow-up of serious adverse events is voluntary for pharmaceutical companies (Option B) isincorrect. Pharmaceutical manufacturers are bound by very strict guidelines around rapidreporting of ADRs and are obliged to follow up reported ADRs and notify the regulatoryauthorities.

Approximately 50% of serious adverse drug reactions are identified through thesystem

A

Follow-up of serious adverse events is voluntary for pharmaceutical companiesB

Only 10% of serious adverse drug reactions are identified by Yellow CardsC

Reporting of adverse drug reactions is compulsory for health professionalsD

The scheme is administered by the ABPIE

Page 741: Back to Filters (/Secure/TestMe/Filter ... - 1 File Download

8/11/2016 MyPastest

https://mypastest.pastest.com/Secure/TestMe/Browser/429893#Top 2/2

46876

Reporting of adverse drug reactions is compulsory for health professionals (Option D) isincorrect. The Yellow Card reporting system is voluntary.

The scheme is administered by the ABPI (Option E) is incorrect. The Association for theBritish Pharmaceutical Industry (ABPI) is a voluntary professional body that sets standardsand regulates performance in the different aspects of drug development, marketing andsafety reporting. The Yellow Card scheme is administered by the Medicines and Healthcareproducts Regulatory Authority (MHRA).

Next Question

Previous Question Tag Question Feedback End Review

Difficulty: Average

Peer Responses

Session Progress

0Responses Correct:

207Responses Incorrect:

207Responses Total:

0%Responses - % Correct:

Blog (https://www.pastest.com/blog) About Pastest (https://www.pastest.com/about-us)Contact Us (https://www.pastest.com/contact-us) Help (https://www.pastest.com/help)

© Pastest 2016

Page 742: Back to Filters (/Secure/TestMe/Filter ... - 1 File Download

8/11/2016 MyPastest

https://mypastest.pastest.com/Secure/TestMe/Browser/429893#Top 1/2

Back to Filters (/Secure/TestMe/Filter/429893/QA)

Question 192 of 207

A 62-year-old man is brought in by a friend who found him collapsed. In his pockets you finda card from the Psychiatry Outpatients Department, along with a full box of haloperidoltablets. On examination his temperature is 38.9 C, and he is alert but not responsive tocommands. He has widespread muscular rigidity.

Which one of the following is the most likely diagnosis?

Explanation

The answer is Neuroleptic malignant syndrome –

Neuroleptic malignant syndrome occurs in around 1–2% of patients taking phenothiazinesor butyrophenones (eg. haloperidol). It is less commonly recognised as a complication oftreatment with tricyclic antidepressants, selective serotonin re-uptake inhibitors (SSRIs)and metoclopramide. There is inhibition of central dopamine receptors in thehypothalamus and abnormal thermoregulation. Clinical features include hyperthermia,rigidity, altered mental status, variable blood pressure and autonomic dysfunction. Ifmuscle rigidity is severe, there may also be metabolic acidosis and myoglobinuria. Onsetis most common within the first 30 days of starting treatment, but can occur at any time;it typically lasts 2–3 weeks after stopping the causal drug, and mortality is 5–20%.Treatment is symptomatic and supportive with active cooling and rehydration.

Bacterial meningitis (Option A) is incorrect. Bacterial meningitis may present with purpuricrash, with symptoms of meningism (neck stiffness, headache, photophobia); generalisedmuscle rigidity is not usually a presenting feature.

ο

Bacterial meningitisA

Catatonic schizophreniaB

Cerebral malariaC

Haloperidol overdoseD

Neuroleptic malignant syndromeE

Page 743: Back to Filters (/Secure/TestMe/Filter ... - 1 File Download

8/11/2016 MyPastest

https://mypastest.pastest.com/Secure/TestMe/Browser/429893#Top 2/2

46964

Catatonic schizophrenia (Option B) is incorrect. Catatonic schizophrenia involves awithdrawn state and would not typically be associated with high temperature or rigidity.

Cerebral malaria (Option C) is incorrect. Cerebral malaria caused by Plasmodium falciparumis characterised by fever and reduced consciousness, and may also cause shock, metabolicacidosis, hypoglycaemia, renal failure and adult respiratory distress syndrome.

Haloperidol overdose (Option D) is incorrect. Haloperidol may cause significant musclerigidity and extra-pyramidal features, but would be less likely to cause a high temperatureand collapse.

Next Question

Previous Question Tag Question Feedback End Review

Difficulty: Average

Peer Responses

Session Progress

0Responses Correct:

207Responses Incorrect:

207Responses Total:

0%Responses - % Correct:

Blog (https://www.pastest.com/blog) About Pastest (https://www.pastest.com/about-us)Contact Us (https://www.pastest.com/contact-us) Help (https://www.pastest.com/help)

© Pastest 2016

Page 744: Back to Filters (/Secure/TestMe/Filter ... - 1 File Download

8/11/2016 MyPastest

https://mypastest.pastest.com/Secure/TestMe/Browser/429893#Top 1/2

Back to Filters (/Secure/TestMe/Filter/429893/QA)

Question 193 of 207

You review a 58-year-old man with tuberculosis and commence him on triple antituberculoustherapy. He has a past history of alcoholism. You are aware of the problem of peripheralneuropathy with isoniazid therapy.

Which of the following is most characteristic of isoniazid-induced peripheral neuropathy?

Explanation

The answer is Those with an N-acetyltransferase type-2 gene defect are predisposed toneuropathy –

Thankfully, isoniazid-associated peripheral neuropathy occurs in less than 1% of thosewho take the drug, although the incidence is increased at higher drug doses. It is relatedto a defect in the gene coding for N-acetyltransferase type 2, resulting in abnormalisoniazid metabolism and ‘slow acetylator’ status. Isoniazid is also associated with raisedtransaminases in 10–20% of those patients for whom it is prescribed, and rarely hepaticnecrosis and jaundice (liver toxicity is more likely to occur in ‘fast acetylator’ status.Rifampicin and pyrazinamide are two other antituberculous agents that may also beassociated with abnormal liver enzymes.

It occurs because of a defect in butyrylcholinesterase (Option A) is incorrect.Butyrylcholinesterase defects are associated with suxamethonium-induced apnoea.Butyrylcholinesterase is not relevant to isoniazid metabolism.

It only occurs in alcoholics (Option B) is incorrect. It is more common in alcoholics, but notunique to this patient group.

It occurs because of a defect in butyrylcholinesteraseA

It only occurs in alcoholicsB

Thiamine prophylaxis is usually used to prevent its occurrenceC

Those with an N-acetyltransferase type-1 gene defect are predisposed to neuropathyD

Those with an N-acetyltransferase type-2 gene defect are predisposed toneuropathy

E

Page 745: Back to Filters (/Secure/TestMe/Filter ... - 1 File Download

8/11/2016 MyPastest

https://mypastest.pastest.com/Secure/TestMe/Browser/429893#Top 2/2

46877

Thiamine prophylaxis is usually used to prevent its occurrence (Option C) is incorrect.Pyridoxine (vitamin B6) is given as prophylaxis against peripheral neuropathy occurring, notthiamine (vitamin B1).

Those with an N-acetyltransferase type-1 gene defect are predisposed to neuropathy (OptionD) is incorrect. NAT-1 mutations may be associated with increased risk of colorectaladenomas.

Next Question

Previous Question Tag Question Feedback End Review

Difficulty: Average

Peer Responses

Session Progress

0Responses Correct:

207Responses Incorrect:

207Responses Total:

0%Responses - % Correct:

Blog (https://www.pastest.com/blog) About Pastest (https://www.pastest.com/about-us)Contact Us (https://www.pastest.com/contact-us) Help (https://www.pastest.com/help)

© Pastest 2016

Page 746: Back to Filters (/Secure/TestMe/Filter ... - 1 File Download

8/11/2016 MyPastest

https://mypastest.pastest.com/Secure/TestMe/Browser/429893#Top 1/2

Back to Filters (/Secure/TestMe/Filter/429893/QA)

Question 194 of 207

A man had an argument with his girlfriend and drank anti-freeze in an impulsive attempt tocommit suicide. On admission his consciousness level is decreased, he is retching andcomplaining of abdominal pain. Biochemistry tests show pH 7.1, HCO 2 mmol/l, large basedeficit, calcium 1.98 mmol/l.

What would be the next immediate management after replacing calcium?

Explanation

The answer is Start sodium bicarbonate 8.4% infusion via central line –

As little as 30 ml of ethylene glycol may be fatal in adults This patient, who has takenethylene glycol in an attempt to commit suicide, has very low HCO levels, severeacidosis and a base deficit. In this situation the next immediate treatment is with sodiumbicarbonate; the 8.4% concentration should be given via central line over 30–40 min. It isalso important to commence treatment with fomepizole to inhibit alcoholdehydrogenase promptly.

Fomepizole or alcohol may be given as inhibitors of alcohol dehydrogenase.

Fomepizole has the advantage of not decreasing the level of consciousness.

It is likely that haemodialysis will be required; in patients requiring haemodialysis

either the alcohol infusion rate or the dose interval between doses of fomepizole

requires alteration.

3

HaemodialysisA

Start ethanol infusionB

Start oral ethanolC

Start peripheral sodium bicarbonate 8.4% infusionD

Start sodium bicarbonate 8.4% infusion via central lineE

3–

Page 747: Back to Filters (/Secure/TestMe/Filter ... - 1 File Download

8/11/2016 MyPastest

https://mypastest.pastest.com/Secure/TestMe/Browser/429893#Top 2/2

46928

Haemodialysis (Option A) is incorrect. Conventional indications for dialysis include pH < 7.25,acute renal failure or ethylene glycol level > 500 mg/l. In the first instance sodiumbicarbonate administration should be used to correct pH.

Start ethanol infusion (Option B) is incorrect. Intravenous ethanol infusion may beconsidered, as a competitive inhibitor of alcohol dehydrogenase, but this is not an immediatepriority until pH is corrected.

Start oral ethanol (Option C) is incorrect. Oral ethanol is a lower priority than correction ofpH to prevent seizures.

Start peripheral sodium bicarbonate 8.4% infusion (Option D) is incorrect. Peripheral sodiumbicarbonate is venotoxic and should be given via a central line where possible.

Next Question

Previous Question Tag Question Feedback End Review

Difficulty: Average

Peer Responses

Session Progress

0Responses Correct:

207Responses Incorrect:

207Responses Total:

0%Responses - % Correct:

Blog (https://www.pastest.com/blog) About Pastest (https://www.pastest.com/about-us)Contact Us (https://www.pastest.com/contact-us) Help (https://www.pastest.com/help)

© Pastest 2016

Page 748: Back to Filters (/Secure/TestMe/Filter ... - 1 File Download

8/11/2016 MyPastest

https://mypastest.pastest.com/Secure/TestMe/Browser/429893#Top 1/2

Back to Filters (/Secure/TestMe/Filter/429893/QA)

Question 195 of 207

A 59-year-old woman presents to hospital with features of altered behaviour withaggressiveness and increased libido. Her husband says that prior to this she had beenmarkedly withdrawn and had low mood and suicidal thoughts for several weeks. A liaisonpsychiatry review has indicated a suspected diagnosis of bipolar affective disorder and isarranging transfer to a psychiatry unit. Her partner is asking for advice regarding long-termtherapies that might be considered.

Which of the following drugs would offer most effective long-term improvement in patientswith bipolar affective disorder?

Explanation

The answer is Lithium –

Sodium valproate has been demonstrated to have antimanic effects and is now beingwidely used for this indication. Lithium therapy is effective in treatment and preventionof mania and depression, but is generally regarded as second-line therapy in patientsthat fail to tolerate or responde to sodium valproate. Lithium is potentially a very toxicdrug and therapeutic monitoring is to minimise the risks of toxicity (polyuria, polydipsia,diarrhoea, vomiting, tremor, confusion, ataxia, dizziness). Long-term use is associatedwith development of nephrogenic diabetes insipidus, hypothyroidism and renal failure,and it is not normally used for more than 5 years.

Carbamazepine (Option A) is incorrect. Carbamazepine may be considered for patients thathave failed to respond to sodium valproate or lithium.

Diazepam (Option B) is incorrect. Diazepam would be useful only in the manic phase.

CarbamazepineA

DiazepamB

FluoxetineC

LithiumD

PhenytoinE

Page 749: Back to Filters (/Secure/TestMe/Filter ... - 1 File Download

8/11/2016 MyPastest

https://mypastest.pastest.com/Secure/TestMe/Browser/429893#Top 2/2

46783

Fluoxetine (Option C) is incorrect. Fluoxetine may be effective in treating symptoms of thedepression phase but less effective overall than sodium valproate or lithium for bipolardisorder.

Phenytoin (Option E) is incorrect. Phenytoin may have some role as a mood stabiliser butefficacy is less well established than for other agents.

Next Question

Previous Question Tag Question Feedback End Review

Difficulty: Average

Peer Responses

Session Progress

0Responses Correct:

207Responses Incorrect:

207Responses Total:

0%Responses - % Correct:

Blog (https://www.pastest.com/blog) About Pastest (https://www.pastest.com/about-us)Contact Us (https://www.pastest.com/contact-us) Help (https://www.pastest.com/help)

© Pastest 2016

Page 750: Back to Filters (/Secure/TestMe/Filter ... - 1 File Download

8/11/2016 MyPastest

https://mypastest.pastest.com/Secure/TestMe/Browser/429893#Top 1/2

Back to Filters (/Secure/TestMe/Filter/429893/QA)

Question 196 of 207

A 54-year-old man has suffered a myocardial infarction managed in a Coronary Care Unit. Heis thought to be in a stable condition and is moved to a general ward area. He is in mild leftventricular failure (LVF), and echocardiography estimates his ejection fraction at around 34%.You commence eplerenone therapy at a dose of 25 mg. His BP is 135/75 mmHg, pulse is80/min and regular.

Which of the following adverse effects would be most important to observe for?

Explanation

The answer is Hyperkalaemia –

Eplerenone is an aldosterone antagonist that improves survival when added to existingtherapy after myocardial infarction in stable patients with an ejection fraction of less than40% and clinical evidence of heart failure; this benefit is in addition to existing treatmentwith ß-blockers, ACE inhibitors and antiplatelet medications, and eplerenone may reduceall-cause mortality by up to 15%.

Common adverse effects are hyperkalaemia, dizziness, hypotension, diarrhoea, nauseaand prerenal renal dysfunction. Eplerenone is metabolised via the CYP3A4 system, sothat inducers and inhibitors of the 3A4 enzyme subtype may precipitate druginteractions.

Constipation (Option A) is incorrect. Diarrhoea is an uncommon but recognised adverseeffect.

Hypertension (Option C) is incorrect. Eplerenone may cause hypotension including posturalhypotension.

ConstipationA

HyperkalaemiaB

HypertensionC

HypocalcaemiaD

HyponatraemiaE

Page 751: Back to Filters (/Secure/TestMe/Filter ... - 1 File Download

8/11/2016 MyPastest

https://mypastest.pastest.com/Secure/TestMe/Browser/429893#Top 2/2

46891

Hypocalcaemia (Option D) is incorrect. Hypocalcaemia is a recognised complication of loopdiuretics, including furosemide and bumetanide.

Hyponatraemia (Option E) is incorrect. Eplerenone and spironolactone have only a modesteffect on sodium excretion, and are less likely to cause hyponatraemia than loop or thiazidetype diuretics.

Next Question

Previous Question Tag Question Feedback End Review

Difficulty: Average

Peer Responses

Session Progress

0Responses Correct:

207Responses Incorrect:

207Responses Total:

0%Responses - % Correct:

Blog (https://www.pastest.com/blog) About Pastest (https://www.pastest.com/about-us)Contact Us (https://www.pastest.com/contact-us) Help (https://www.pastest.com/help)

© Pastest 2016

Page 752: Back to Filters (/Secure/TestMe/Filter ... - 1 File Download

8/11/2016 MyPastest

https://mypastest.pastest.com/Secure/TestMe/Browser/429893#Top 1/2

Back to Filters (/Secure/TestMe/Filter/429893/QA)

Question 197 of 207

46746

A patient undergoing cancer chemotherapy complains of increased urinary frequency andsuprapubic pain. Investigations reveal he has microcytic, hypochromic anaemia, leucopenia,thrombocytopaenia and haematuria.

Which of the following medications is the most likely cause of his symptoms?

Explanation

The answer is Cyclophosphamide –

Cyclophosphamide is inactive unless metabolised by the liver to 4-hydroxylcyclophosphamide, which is metabolised into alkylating species as well as tochloroacetaldehyde and acrolein. Acrolein causes chemical cystitis and thereforehydration must be maintained during cyclophosphamide therapy.

Cisplatin (Option A) is incorrect. Cisplatin is associated with nephrotoxicity, hypokalaemia,haemolysis, ototoxicity and peripheral neuropathy.

Chlorambucil (Option B) is incorrect. Chlorambucil causes myelosuppression, hepatotoxicityand azoospermia.

Melphalan (Option D) is incorrect. Melphalan suppresses the bone marrow and has othereffects, including inappropriate secretion of ADH, amenorrhoea, sterility and pulmonaryinfiltrates.

Methotrexate (Option E) is incorrect. Methotrexate blocks folic acid synthesis and may causemacrocytic anaemia, stomatitis, alopecia, hepatotoxicity and acute pneumonitis.

CisplatinA

ChlorambucilB

CyclophosphamideC

MelphalanD

MethotrexateE

Page 753: Back to Filters (/Secure/TestMe/Filter ... - 1 File Download

8/11/2016 MyPastest

https://mypastest.pastest.com/Secure/TestMe/Browser/429893#Top 2/2

Next Question

Previous Question Tag Question Feedback End Review

Difficulty: Average

Peer Responses

Session Progress

0Responses Correct:

207Responses Incorrect:

207Responses Total:

0%Responses - % Correct:

Blog (https://www.pastest.com/blog) About Pastest (https://www.pastest.com/about-us)Contact Us (https://www.pastest.com/contact-us) Help (https://www.pastest.com/help)

© Pastest 2016

Page 754: Back to Filters (/Secure/TestMe/Filter ... - 1 File Download

8/11/2016 MyPastest

https://mypastest.pastest.com/Secure/TestMe/Browser/429893#Top 1/2

Back to Filters (/Secure/TestMe/Filter/429893/QA)

Question 198 of 207

46896

A 40-year-old woman has been established on warfarin therapy following a deep venousthrombosis, with INR between 2.0 and 3.0. However, at her last clinic visit the INR was notedto be 4.9, without any change in warfarin dose. On questioning, she admits to having recentlyself-medicated with some of her partner’s medications.

Which of the following drugs is most likely to account for the change in INR?

Explanation

The answer is Cimetidine –

Cimetidine inhibits breakdown of warfarin by the hepatic cytochrome P450 system, sothat INR and bleeding risk increase.

Amoxicillin (Option A) is incorrect. Amoxicillin is neither a p450 enzyme inducer, norinhibitor, and as such is less likely to affect warfarin metabolism. It may interfere with gutflora and interfere with warfarin bsorption so that there may be a small fall in INR.

Aspirin (Option B) is incorrect. Aspirin may increase bleeding by inhibiting platelet function,but this will not affect the INR.

Diazepam (Option D) is incorrect. Diazepam does not significantly interact with warfarin.

Indometacin (Option E) is incorrect. Indometacin may increase free warfarin and INR owingto a protein-binding interaction; however, the effect on INR is less obvious than the additionof cimetidine, making cimetidine the preferred answer.

AmoxicillinA

AspirinB

CimetidineC

DiazepamD

IndometacinE

Page 755: Back to Filters (/Secure/TestMe/Filter ... - 1 File Download

8/11/2016 MyPastest

https://mypastest.pastest.com/Secure/TestMe/Browser/429893#Top 2/2

Next Question

Previous Question Tag Question Feedback End Review

Difficulty: Average

Peer Responses

Session Progress

0Responses Correct:

207Responses Incorrect:

207Responses Total:

0%Responses - % Correct:

Blog (https://www.pastest.com/blog) About Pastest (https://www.pastest.com/about-us)Contact Us (https://www.pastest.com/contact-us) Help (https://www.pastest.com/help)

© Pastest 2016

Page 756: Back to Filters (/Secure/TestMe/Filter ... - 1 File Download

8/11/2016 MyPastest

https://mypastest.pastest.com/Secure/TestMe/Browser/429893#Top 1/2

Back to Filters (/Secure/TestMe/Filter/429893/QA)

Question 199 of 207

46908

A 63-year-old lady is found to be jaundiced. Biochemical results are as follows: bilirubin 62μmol/l (1–22), γ-glutamyl transferase 400 IU (4–35), alanine aminotransferase 42 IU (5–35)and alkaline phosphatase 386 IU (45–105). She is taking many different drugs and youconsider the possibility of drug-induced jaundice.

Which of the following is most likely to be responsible for the biochemical findings?

Explanation

The answer is Chlorpromazine –

The biochemical picture is one of cholestasis or obstructive jaundice, rather thanhepatitis. A number of drugs can cause cholestasis, including chlorpromazine,chlorpropamide, erythromycin, nitrofurantoin, co-amoxiclav, nifedipine and statins(statins and nifedipine may also cause hepatitis).

Amiodarone (Option A) is incorrect. Amiodarone may cause hepatitis, but not cholestasis.

Isoniazid (Option C) is incorrect. Isoniazide may cause hepatitis (via its key metabolite), andthis is more likely to occur in slow acetylators.

Methotrexate (Option D) is incorrect. Methotrexate may provoke a fibrotic reaction within theliver.

Methyldopa (Option E) is incorrect. Methyldopa may cause an autoimmune hepatitis, butdoes not cause cholestasis.

AmiodaroneA

ChlorpromazineB

IsoniazidC

MethotrexateD

MethyldopaE

Page 757: Back to Filters (/Secure/TestMe/Filter ... - 1 File Download

8/11/2016 MyPastest

https://mypastest.pastest.com/Secure/TestMe/Browser/429893#Top 2/2

Next Question

Previous Question Tag Question Feedback End Review

Difficulty: Average

Peer Responses

Session Progress

0Responses Correct:

207Responses Incorrect:

207Responses Total:

0%Responses - % Correct:

Blog (https://www.pastest.com/blog) About Pastest (https://www.pastest.com/about-us)Contact Us (https://www.pastest.com/contact-us) Help (https://www.pastest.com/help)

© Pastest 2016

Page 758: Back to Filters (/Secure/TestMe/Filter ... - 1 File Download

8/11/2016 MyPastest

https://mypastest.pastest.com/Secure/TestMe/Browser/429893#Top 1/2

Back to Filters (/Secure/TestMe/Filter/429893/QA)

Question 200 of 207

46625

You are responsible for initiating anticoagulation therapy for a 56-year-old woman with newonset deep vein thrombosis.

Which one of the following is most strongly associated with warfarin therapy?

Explanation

The answer is Reduces protein C levels in the blood –

Warfarin competitively inhibits carboxylation of vitamin K-dependent factors: factor II,factor VII, factor IX, factor X and protein C. The half-life of warfarin is approximately 44 h.It is highly protein bound, and the quantities expressed in breast milk are too low to beof any clinical significance.

An initial loading dose is given because it has a short half-life (3 h) (Option A) is incorrect.The initial loading dose affords a more rapid onset of action, unrelated to the drug half-life.

Chronic use gives rise to osteoporosis in 10% of patients (Option B) is incorrect.Osteoporosis is an important adverse effect of long-term heparin therapy, as might be usedin pregnancy, cancer patients, or patients who are intolerant of oral anticoagulants.

May induce autoimmune thrombocytopaenia (Option C) is incorrect. Autoimmunethrombocytopaenia is a well characterised adverse effect of heparin treatment.

Should be avoided in lactating women (Option E) is incorrect. Warfarin does not penetratebreast milk in sufficiently large quantities to pose a significant risk to the infant.

An initial loading dose is given because it has a short half-life (3 h)A

Chronic use gives rise to osteoporosis in 10% of patientsB

May induce autoimmune thrombocytopaeniaC

Reduces protein C levels in the bloodD

Should be avoided in lactating womenE

Page 759: Back to Filters (/Secure/TestMe/Filter ... - 1 File Download

8/11/2016 MyPastest

https://mypastest.pastest.com/Secure/TestMe/Browser/429893#Top 2/2

Next Question

Previous Question Tag Question Feedback End Review

Difficulty: Average

Peer Responses

Session Progress

0Responses Correct:

207Responses Incorrect:

207Responses Total:

0%Responses - % Correct:

Blog (https://www.pastest.com/blog) About Pastest (https://www.pastest.com/about-us)Contact Us (https://www.pastest.com/contact-us) Help (https://www.pastest.com/help)

© Pastest 2016

Page 760: Back to Filters (/Secure/TestMe/Filter ... - 1 File Download

8/11/2016 MyPastest

https://mypastest.pastest.com/Secure/TestMe/Browser/429893#Top 1/2

Back to Filters (/Secure/TestMe/Filter/429893/QA)

Question 201 of 207

A 21-year-old student suffers from hay fever and acne. She has been receiving long-termerythromycin therapy for acne, and has attended clinic for advice about her hay fever, whichhas been much worse recently. She is about to take her final examinations and is worried thatcertain antihistamines may cause drowsiness.

Which antihistamine would be most suitable in these circumstances?

Explanation

The answer is Desloratadine –

Desloratadine is a long-acting selective H1-receptor antagonist that has little or noclinically significant interaction with erythromycin. It has comparatively poor penetrationinto the central nervous system and is not sedating. Cetirizine, desloratadine andfexofenadine may be prescribed for allergic rhinitis (hay fever) and all three are equallyeffective, but cetirizine and fexofenadine effects may be increased by erythromycin andother macrolides.

Chlormethiazole (Option A) is incorrect. Chlormethiazole is particularly sedating and shouldbe avoided where this may be undesirable.

Chlorphenamine maleate (Option B) is incorrect. Chlorpheniramine maleate is a sedatingantihistamine.

Loratadine (Option D) is incorrect. Loratadine effects may be increased by concomitanterythromycin.

ChlormethiazoleA

Chlorphenamine maleateB

DesloratadineC

LoratadineD

PromethiazineE

Page 761: Back to Filters (/Secure/TestMe/Filter ... - 1 File Download

8/11/2016 MyPastest

https://mypastest.pastest.com/Secure/TestMe/Browser/429893#Top 2/2

46761

Promethiazine (Option E) is incorrect. Promethiazine is particularly sedating and should beavoided where this may be undesirable.

Next Question

Previous Question Tag Question Feedback End Review

Difficulty: Average

Peer Responses

Session Progress

0Responses Correct:

207Responses Incorrect:

207Responses Total:

0%Responses - % Correct:

Blog (https://www.pastest.com/blog) About Pastest (https://www.pastest.com/about-us)Contact Us (https://www.pastest.com/contact-us) Help (https://www.pastest.com/help)

© Pastest 2016

Page 762: Back to Filters (/Secure/TestMe/Filter ... - 1 File Download

8/11/2016 MyPastest

https://mypastest.pastest.com/Secure/TestMe/Browser/429893#Top 1/2

Back to Filters (/Secure/TestMe/Filter/429893/QA)

Question 202 of 207

A 65-year-old woman with type 2 diabetes of 11 years’ duration presents with poorlycontrolled blood glucose levels. She was overweight and initially started on metformintherapy. Her diabetes was well controlled until the last 12 months. Despite strict adherence todiet, exercise and maximum daily doses of metformin, satisfactory blood glucose control hasproved difficult to achieve and the last Hb A1c was at 13%. You consider adding pioglitazone.

Which of the following best describes the pharmacological properties of pioglitazone?

Explanation

The answer is An insulin sensitiser, which decreases peripheral insulin resistance –

Pioglitazone exerts its effects by activating the peroxisome proliferation-activatedreceptor-gamma (PPARγ), which is a nuclear receptor that influences the differentiationof fibroblasts into adipocytes and lowers free fatty acid levels. Its major therapeuticaction is to decrease peripheral insulin resistance, although at higher doses it may alsodecrease hepatic glucose production. Pioglitazone appears to act as an insulin sensitiser,and its full hypoglycaemic potency may take up to 12 weeks. Adverse events ofpioglitazone include fluid retention, weight gain and decreased bone mineral density.

A benzoic acid derivative (Option A) is incorrect. Benzoic acid derivatives have notherapeutic action relevant to treatment of diabetes.

A sulfonylurea (Option B) is incorrect. Sulfonlyureas (eg gliclazide) stimulate insulinsecretion, and may provoke hypoglycaemia.

An α-glucosidase inhibitor (Option C) is incorrect. Acarbose is an α-glucosidase inhibitor thatdelays glucose absorption from the gut. Acarbose acts locally in the small intestine by

A benzoic acid derivativeA

A sulfonylureaB

An α-glucosidase inhibitorC

An insulin secretagogue, which stimulates β-cell insulin secretionD

An insulin sensitiser, which decreases peripheral insulin resistanceE

Page 763: Back to Filters (/Secure/TestMe/Filter ... - 1 File Download

8/11/2016 MyPastest

https://mypastest.pastest.com/Secure/TestMe/Browser/429893#Top 2/2

46633

inhibiting α-glucosidase enzymes, an action that slows digestion of ingested carbohydrates,delays glucose absorption and reduces the increase in postprandial blood glucose.

An insulin secretagogue, which stimulates β-cell insulin secretion (Option D), is incorrect.Glitazones modulate the response to insulin, but do not directly stimulate or inhibit insulinsecretion.

Next Question

Previous Question Tag Question Feedback End Review

Difficulty: Average

Peer Responses

Session Progress

0Responses Correct:

207Responses Incorrect:

207Responses Total:

0%Responses - % Correct:

Blog (https://www.pastest.com/blog) About Pastest (https://www.pastest.com/about-us)Contact Us (https://www.pastest.com/contact-us) Help (https://www.pastest.com/help)

© Pastest 2016

Page 764: Back to Filters (/Secure/TestMe/Filter ... - 1 File Download

8/11/2016 MyPastest

https://mypastest.pastest.com/Secure/TestMe/Browser/429893#Top 1/2

Back to Filters (/Secure/TestMe/Filter/429893/QA)

Question 203 of 207

A 54-year-old man attends the cardiology clinic for review. He has a history of myocardialinfarction 5 years ago and his current angina symptoms are stable. However, he mentions thathe has been suffering impotence recently, and is keen to start sildenafil.

Which one of the following medications is most likely to interact with sildenafil and should beavoided?

Explanation

The answer is Isosorbide mononitrate –

Interactions with phosphodiesterase type-5 inhibitors Glyceryl trinitrate, nicorandil andisosorbide all act as nitric oxide donors, which can lead to profound hypotension whenthey are combined with any of the phosphodiesterase type-5 (PDE5) inhibitors. Commonside-effects associated with sildenafil include headache, flushing, indigestion, nasalcongestion, dizziness and visual colour distortion.

Bisoprolol (Option A) is incorrect. Patients that have stable symptoms on a β-blocker cannormally take sildenafil without difficulty.

Doxazosin (Option B) is incorrect. Alpha-blockers may precipitate hypotension when used inconjunction with medications such as sildenafil and should be avoided if possible. Tominimise the hypotensive effect of α-blockade, sildenafil is introduced at the lowest dosefirst. The risks of nitrates are higher, hence isosorbide nitrate is the preferred answer.

Furosemide (Option C) is incorrect. Furosemide is unlikely to cause problems.

BisoprololA

DoxazosinB

FurosemideC

Isosorbide mononitrateD

NifedipineE

Page 765: Back to Filters (/Secure/TestMe/Filter ... - 1 File Download

8/11/2016 MyPastest

https://mypastest.pastest.com/Secure/TestMe/Browser/429893#Top 2/2

46991

Nifedipine (Option E) is incorrect. Patients that have stable symptoms on nifedipine cannormally take sildenafil without difficulty.

Next Question

Previous Question Tag Question Feedback End Review

Difficulty: Average

Peer Responses

Session Progress

0Responses Correct:

207Responses Incorrect:

207Responses Total:

0%Responses - % Correct:

Blog (https://www.pastest.com/blog) About Pastest (https://www.pastest.com/about-us)Contact Us (https://www.pastest.com/contact-us) Help (https://www.pastest.com/help)

© Pastest 2016

Page 766: Back to Filters (/Secure/TestMe/Filter ... - 1 File Download

8/11/2016 MyPastest

https://mypastest.pastest.com/Secure/TestMe/Browser/429893#Top 1/2

Back to Filters (/Secure/TestMe/Filter/429893/QA)

Question 204 of 207

46980

You are asked to see a 17-year-old woman in the Emergency Department. She has beenbrought to the unit by her boyfriend, who is concerned that she may have taken an overdoseafter being depressed about her mother, who is dying of carcinoma of the breast.

Which one of the following features would most strongly indicate that she has taken some ofher mother’s morphine sulfate pills?

Explanation

The answer is Sweating –

Psychological and physical dependence on morphine may occur. Common adverseeffects include nausea, vomiting, constipation, drowsiness, small pupils and confusion.Less common effects include sweating, rash, flushing, oedema, headache and raisedintracranial pressure. Large doses typically cause respiratory depression, circulatoryfailure and coma.

Dilated pupils (Option A) is incorrect. Pupils are small and may be very constricted ‘pinpoint’.

Hypertension (Option B) is incorrect. Flushing and hypotension may occur.

Increased libido (Option C) is incorrect. Libido is typically reduced.

Urinary incontinence (Option E) is incorrect. Constipation and urinary retention may occur.

Dilated pupilsA

HypertensionB

Increased libidoC

SweatingD

Urinary incontinenceE

Next Question

Page 767: Back to Filters (/Secure/TestMe/Filter ... - 1 File Download

8/11/2016 MyPastest

https://mypastest.pastest.com/Secure/TestMe/Browser/429893#Top 2/2

Previous Question Tag Question Feedback End Review

Difficulty: Average

Peer Responses

Session Progress

0Responses Correct:

207Responses Incorrect:

207Responses Total:

0%Responses - % Correct:

Blog (https://www.pastest.com/blog) About Pastest (https://www.pastest.com/about-us)Contact Us (https://www.pastest.com/contact-us) Help (https://www.pastest.com/help)

© Pastest 2016

Page 768: Back to Filters (/Secure/TestMe/Filter ... - 1 File Download

8/11/2016 MyPastest

https://mypastest.pastest.com/Secure/TestMe/Browser/429893#Top 1/2

Back to Filters (/Secure/TestMe/Filter/429893/QA)

Question 205 of 207

You are asked to review a pregnant patient with HIV disease.

Which of the following measures would be expected to be most effective in reducing the riskof HIV infection in the newborn child?

Explanation

The answer is Postnatal administration of zidovudine to the baby –

Oral zidovudine from 28 weeks’ gestation onwards can reduce the risk of transmission. Arange of studies suggest that zidovudine is effective in reducing the risk of HIVtransmission from mother to neonate by 8.3% to 18%. The optimal regimen is tocommence zidovudine as an intravenous infusion at the onset of labour and continuetreatment in the neonate up to 6 weeks of age. There is insufficient evidence thatintravenous zidovudine during labour is of any benefit, unless followed by treatment ofthe child in the postnatal period.

Elective caesarean section (Option A) is incorrect. Elective caesarean section may reduce therisk compared with vaginal delivery, but the effectiveness of this strategy is much lesscertain, hence option C is the preferred answer.

Oral zidovudine at the time of conception (Option B) is incorrect. There is no evidence thatoral administration of zidovudine at the time of conception reduces the risk of HIVtransmission.

Vaginal delivery with shortening of the second stage of labour (Option D) is incorrect.Shortening the second stage by operative delivery may increase the risk of transmission.

Elective caesarean sectionA

Oral zidovudine at the time of conceptionB

Postnatal administration of zidovudine to the babyC

Vaginal delivery with shortening of the second stage of labourD

Zidovudine administration to the mother at the time of breast-feedingE

Page 769: Back to Filters (/Secure/TestMe/Filter ... - 1 File Download

8/11/2016 MyPastest

https://mypastest.pastest.com/Secure/TestMe/Browser/429893#Top 2/2

46751

Zidovudine administration to the mother at the time of breast-feeding (Option E) isincorrect. Breast-feeding is generally avoided as this increases the risk of HIV transmission.

Next Question

Previous Question Tag Question Feedback End Review

Difficulty: Average

Peer Responses

Session Progress

0Responses Correct:

207Responses Incorrect:

207Responses Total:

0%Responses - % Correct:

Blog (https://www.pastest.com/blog) About Pastest (https://www.pastest.com/about-us)Contact Us (https://www.pastest.com/contact-us) Help (https://www.pastest.com/help)

© Pastest 2016

Page 770: Back to Filters (/Secure/TestMe/Filter ... - 1 File Download

8/11/2016 MyPastest

https://mypastest.pastest.com/Secure/TestMe/Browser/429893#Top 1/2

Back to Filters (/Secure/TestMe/Filter/429893/QA)

Question 206 of 207

46899

On admission examination, a 60-year-old man is noted to have reduced facial expression,rigidity and bradykinesia. He has been taking a long-term medication and a diagnosis ofpossible drug-induced parkinsonism is suspected.

Which of the following treatments is most likely to have caused these clinical features?

Explanation

The answer is Droperidol –

Butyrophenones (eg haloperidol, droperidol) and phenothiazines are the most commoncause of drug-induced parkinsonism.

Domperidone (Option A) is incorrect. Domperidone (unlike metoclopramide) does not crossthe blood–brain barrier and acts on peripheral dopaminergic receptors. It is much less likelyto cause extrapyramidal effects than metoclopramide.

Imipramine (Option C) is incorrect. Imipramine is a tricyclic antidepressant with someanticholinergic effects within the central nervous system, but it is not expected to causeparkinsonism.

Orphenadrine (Option D) is incorrect. Orphenadrine is an anticholinergic treatment that maybe effective for some symptoms of drug-induced parkinsonism.

Selegiline (Option E) is incorrect. Selegiline is a type B monoamine oxidase inhibitor (MAOI)used to treat Parkinson’s disease.

DomperidoneA

DroperidolB

ImipramineC

OrphenadrineD

SelegilineE

Page 771: Back to Filters (/Secure/TestMe/Filter ... - 1 File Download

8/11/2016 MyPastest

https://mypastest.pastest.com/Secure/TestMe/Browser/429893#Top 2/2

Next Question

Previous Question Tag Question Feedback End Review

Difficulty: Average

Peer Responses

Session Progress

0Responses Correct:

207Responses Incorrect:

207Responses Total:

0%Responses - % Correct:

Blog (https://www.pastest.com/blog) About Pastest (https://www.pastest.com/about-us)Contact Us (https://www.pastest.com/contact-us) Help (https://www.pastest.com/help)

© Pastest 2016

Page 772: Back to Filters (/Secure/TestMe/Filter ... - 1 File Download

8/11/2016 MyPastest

https://mypastest.pastest.com/Secure/TestMe/Browser/429893#Top 1/2

Back to Filters (/Secure/TestMe/Filter/429893/QA)

Question 207 of 207

46816

A 20-year-old patient presents after taking an overdose involving her grandmother’s aspirintablets. She has been complaing of nausea and vomiting, and has developed tinnitus,sweating and dizziness.

Which one of the following is the best initial step in her management?

Explanation

The answer is Oral activated charcoal –

Oral activated charcoal should be considered if the patient presents within 1 h ofingestion of the salicylate overdose to minimise drug absorption, and may be effectiveeven if administered beyond 1 h by increasing aspirin clearance.

Acetylcysteine (Option A) is incorrect. Acetylcysteine is effective for paracetamol toxicity.

Haemodialysis (Option B) is incorrect. Haemodialysis may be necessary for severe aspirinpoisoning, but would not be appropriate initial management. Haemodialysis is the treatmentof choice for severely poisoned patients, particularly those with features of central nervoussystem toxicity and metabolic acidosis that fails to correct after intravenous sodiumbicarbonate.

Intravenous H2-receptor blocker (Option C) is incorrect. An intravenous H2-receptor blockermay be helpful in preventing gastric irritation, but would do little to reduce systemic toxicity.

Intubation (Option D) is incorrect. Intubation is not indicated at this stage. Pulmonaryoedema occasionally complicates salicylate toxicity.

AcetylcysteineA

HaemodialysisB

Intravenous H2-receptor blockerC

IntubationD

Oral activated charcoalE

Page 773: Back to Filters (/Secure/TestMe/Filter ... - 1 File Download

8/11/2016 MyPastest

https://mypastest.pastest.com/Secure/TestMe/Browser/429893#Top 2/2

End Session

Previous Question Tag Question Feedback End Review

Difficulty: Average

Peer Responses

Session Progress

0Responses Correct:

207Responses Incorrect:

207Responses Total:

0%Responses - % Correct:

Blog (https://www.pastest.com/blog) About Pastest (https://www.pastest.com/about-us)Contact Us (https://www.pastest.com/contact-us) Help (https://www.pastest.com/help)

© Pastest 2016